Вы находитесь на странице: 1из 485

SAN BEDA COLLEGE OF LAW

AY 2016-2017

FIRST SEMESTER

COMPILED CASE DIGESTS

SALES AND LEASE

SUBMITTED BY

2B 2016-2017

SUBMITTED TO

ATTY MANUEL P. CASIO

2B 16-17 SALES AND LEASE Page 1


2B 2016-2017 CLASS LIST

AGUILAR, CHARMAINE GAFFUD, ADRIAN JARED

ALMODAL, NIA ANDREA GUMILAB, RAFIE

AVENDANO, JAMES LEE JARETA, JIRAH

AZIS, SITTIE NAJLIAH LACADIN, ANNA RAEZA

BASILLA, ACE JOMARI LEE, JOHN RALEIGH

BERNARDEZ, LORETO PLACIDO LIBERATO, SUZETTE RIA


RUDOV
MAGNO, CHRIST DENN
BERNARDINO, ELLAINE ANNE
MENESES, JUSTIN LEANNE
BORDADOR, MARIA REBECCA
MERCADER, JEREMY
CLAIRE
NUEZ, CAESAR IAN
BULFANGO, AUDREY LOUISE
PALMA, KAREN CLARISSE
CABURAL, ALYSSA MARIE
PEREZ, JETHRO
CHUA, JAY HANSON
POJAS, KRISTOFER ABE
CRUZ, JEMIMA RACHEL
RAMOS, ANGELA LORRAINE
DE GUZMAN, SARRAH
SAMANIEGO, HONEY
DELOS REYES, BELYNJON
SANGKAL, ANNA CLARITA
DEVESA, ROY
SEAR, MARIA MARUJA
DULDULAO, PAULINE ROSE
SORIANO, FEDERICO
EUROPA, LOU ZENITH

FERNANDO, AIRA ABBYGALE TORRES, NIO

FULGENCIO, SALVADOR

2B 16-17 SALES AND LEASE Page 2


SALES

ABSOLUTE AND CONDITIONAL SALES

1. PEOPLE'S HOMESITE & HOUSING CORPORATION vs. COURT OF APPEALS, RIZALINO L.


MENDOZA and ADELAIDA R. MENDOZA.

G.R. No. L-61623 December 26, 1984

Aquino, J

FACTS:

On February 18, 1960, the People's Homesite & Housing Corporation (PHHC) board of directors passed Resolution
No. 513 wherein it stated "that subject to the approval of the Quezon City Council of the above-mentioned
Consolidation Subdivision Plan, Lot 4. containing4,182.2 square meters be, as it is hereby awarded to Spouses
Rizalino Mendoza and Adelaida Mendoza, at a price of twenty -one pesos (P21.00) per square meter" and "that this
award shall be subject to the approval of the OEC (PHHC) Valuation Committee and higher authorities" .

The city council disapproved the proposed consolidation subdivision plan. Another subdivision plan was prepared
and submitted to the city council for approval. The revised plan, which included Lot 4, with a reduced area of
2,608.7, was approved by the city council.

The PHHC board of directors passed a resolution recalling all awards of lots to persons who failed to pay the
deposit or down payment for the lots awarded to them. The Mendozas never paid the price of the lot nor made the
20% initial deposit.

The PHHC board of directors passed Resolution No. 218, withdrawing the tentative award of Lot 4 to the Mendoza -
spouses under Resolution No. 513 and re-awarding said lot jointly and in equal shares to Miguela Sto. Domingo,
Enrique Esteban, Virgilio Pinzon, Leonardo Redublo and Jose Fernandez.

The five awardees made the initial deposit. The corresponding deeds of sale were executed in their favor. The
subdivision of Lot 4 into five lots was approved by the city council and the Bureau of Lands.

The Mendoza spouses asked for reconsideration of the withdrawal of the previous award to them of Lot 4 and for
the cancellation of the re-award of said lot to Sto. Domingo and four others.

2B 16-17 SALES AND LEASE Page 3


ISSUE:

Whether or not there was a perfected sale of Lot 4, with the reduced area, to the Mendozas which they can enforce
against the PHHC by an action for specific performance.

HELD:

NO. There was no perfected sale of Lot 4.

It was conditionally or contingently awarded to the Mendozas subject to the approval by the city council of the
proposed consolidation subdivision plan and the approval of the award by the valuation committee and higher
authorities. The city council did not approve the subdivision plan.

When the plan with the area of Lot 4 reduced to 2,608.7 square meters was approved, the Mendozas should have
manifested in writing their acceptance of the award for the purchase of Lot 4 just t o show that they were still
interested in its purchase although the area was reduced and to obviate ally doubt on the matter. They did not do so.
The PHHC board of directors acted within its rights in withdrawing the tentative award. We cannot say there wa s a
meeting of minds on the purchase of Lot 4 with an area of 2,608.7 square meters at P21 a square meter.

"The contract of sale is perfected at the moment there is a meeting of minds upon the thing which is the object of the
contract and upon the price. From that moment, the parties may reciprocally demand performance, subject to the law
governing the form of contracts." (Art. 1475, Civil Code).

"In conditional obligations, the acquisition of rights, as well as the extinguishment or loss of those already acquired,
shall depend upon the happening of the event which constitutes the condition. (Art. 1181, Civil Code).

2B 16-17 SALES AND LEASE Page 4


2. DELTA MOTORS VS GENUINO

GR NO. 5565, FEBRUARY 8, 1989

FACTS:
Private Respondents are owners of an iceplant and cold storage who ordered black iron pipes to Delta Motors
(herein petitioner) for which the latter provided two letter quotations indicating the selling price and delivery of said
pipes.
The terms of payment are also included in the letter quotations which must be complied with by the respondents.
Private respondents made initial payments on both contracts but delivery of the pipes was not made by Delta Motors
so that the Genuinos are not willing to give subsequent payments notwithstanding the agreed terms of payment
requiring them of such.
In July 1972 Delta offered to deliver the iron pipes but the Genuinos did not accept the offer because the
construction of the ice plant building where the pipes were to be installed was not yet finished.
Three years later, on April 15, 1975, Hector Genuino, in behalf of Espaa Extension Ice Plant and Cold Storage,
asked Delta to deliver the iron pipes within thirty (30) days from its receipt of the request.
But petitioner Delta is unwilling to deliver said iron pipes unless the Genuin os agree to a new quotation price set by
the former. Private Respondents rejected the new quoted prices and instead filed a complaint for specific
performance with damages seeking to compel Delta to deliver the pipes. Meanwhile, Delta, in its answer prayed for
rescission of the contracts pursuant to Art. 1191 of the New Civil Code.
TC ruled in favor of Delta.
On appeal, the Court of Appeals reversed and ordered private respondents to make the payments specified in "Terms
of Payment (b)" of the contracts and to execute the promissory note required in the first contract and thereafter,
Delta should immediately commence delivery of the black iron pipes.

ISSUE

Whether or not Delta is entitled for rescission of contract as the latter is subject to suspensive conditions and only
upon their performance or compliance would its obligation to deliver the pipes arise?

HELD

No. While there is merit in Delta's claim that the sale is subject to suspensive conditions, the Court finds that it has,
nevertheless, waived performance of these conditions and opted to go on with the contracts although at a much
higher price. Art. 1545 of the Civil Code provides:

Art. 1545. Where the obligation of either party to a contract of sale is subject to any condition which is not
performed, such party may refuse to proceed with the contract or he may waived performance of the condition. . .

It would be highly inequitable for petitioner Delta to rescind the two (2) contracts considering the fact that not only
does it have in its possession and ownership the black iron pipes, but also the down payments private respondents
have paid. Delta cannot ask for increased prices based on the price offer stipulation in the contracts and in the
increase in the cost of goods. Reliance by Delta on the price offer stipulation is misplaced. The moment private

2B 16-17 SALES AND LEASE Page 5


respondents accepted the offer of Delta, the contract of sale between them was perfected and neither party could
change the terms thereof. Neither could petitioner Delta rely on the fluctuation in the market price of goods to
support its claim for rescission.

Finally, Delta cannot ask for increased prices based on the price offer stipulation in the contracts and in the increase
in the cost of goods. Reliance by Delta on the price offer stipulation is misplaced. Said stipulation makes reference
to Delta's price offer as remaining firm for thirty (30) days and thereafter, will be subject to its review and
confirmation. The offers of Delta, however, were accepted by the private respondents within the thirty (30)-day
period. And as stipulated in the two (2) letter-quotations, acceptance of the offer gives rise to a contract between the
parties:

In the event that this proposal is acceptable to you, please indicate your conformity by signing the spac e provided
herein below which also serves as a contract of this proposal. [Exhs. "A" and "C"; Exhs. "1" and "2".]

And as further provided by the Civil Code:

Art. 1319. Consent is manifested by the meeting of the offer and the acceptance upon the thing and the cause which
are to constitute the contract.

Art. 1475. The contract of sale is perfected at the moment there is a meeting of minds upon thing which is the object
of the contract and upon the price.

Thus, the moment private respondents accepted the offer of Delta, the contract of sale between them was perfected
and neither party could change the terms thereof.

Neither could petitioner Delta rely on the fluctuation in the market price of goods to support its claim for rescission.
The stipulation in the two (2) contracts as to delivery, ex-stock subject to prior sales, means that "the goods have not
been delivered and that there are no prior commitments other than the sale covered by the contracts.. . once the offer
is accepted, the company has no more option to change the price." [CFI Decision, p. 5; Rollo, p. 45; Emphasis
supplied.] Thus, petitioner cannot claim for higher prices for the black iron pipes due to the increase in the cost of
goods. Based on the foregoing, petitioner Delta and private respon dents Genuinos should comply with the original
terms of their contracts.

2B 16-17 SALES AND LEASE Page 6


3.DIGNOS vs CA

158 SCRA 375

FACTS:

Facts: Before Us is a petition for review on certiorari seeking the reversal of the decision of the CA affirming the
decision of the CFI in declaring that the second sale of the disputed land null and void.

Petitioner spouses were owners of a land located in Cebu City. On June 7, 1965, petitioners sold the said land to the
respondents for the sum of P28,000.00, payable in two installments, with an assumption of indebtedness with the
First Insular Bank of Cebu in the sum of P12,000.00, which was paid and acknowledged by the vendors in the deed
of sale executed in favor of the petitioners, and the next installment in the sum of P4,000.00 to be paid on or before
September 15, 1965.

On November 25, 1965, the petitioners sold the same land in favor of spouses, Luciano Cabigas and Jovita L. De
Cabigas, who were then U.S. citizens, for the price of P35,000.00. A deed of absolute sale was executed by the
petitioners in favor of the Cabigas spouses, and which was registered in the Office of the Register of Deeds pursuant
to the provisions of Act No. 3344.

As the petitioners refused to accept from respondent the balance of the purchase price of the land, and as the latter
discovered the second sale made by the former to the Cabigas spouses, respondents brought the present suit.

RTC: Declared the second sale null and void and ordered the respondents to pay P16,000.00 upon the execution of
the sale.

CA: Affirmed RTCs decision.

ISSUE

Is the first sale an absolute sale which effectively transfers the ownership over the property in question to the
respondent and not merely a contract to sell or promise to sell?

HELD

The first sale is an ABSOLUTE SALE and NOT a contract to sell.

2B 16-17 SALES AND LEASE Page 7


Jurisprudence dictates that deed of sale is absolute in nature although denominated as a "Deed of Conditional Sale"
where nowhere in the contract in question is a proviso or stipulation to the effect that title to the prope rty sold is
reserved in the vendor until full payment of the purchase price, nor is there a stipulation giving the vendor the right
to unilaterally rescind the contract the moment the vendee fails to pay within a fixed period.

In the case at bar, a careful examination of the contract shows that there is no such stipulation reserving the title of
the property on the vendors nor does it give them the right to unilaterally rescind the contract upon non -payment of
the balance thereof within a fixed period. On the contrary, all the elements of a valid contract of sale under Article
1458.

Moreover, "where time is not of the essence of the agreement, a slight delay on the part of one party in the
performance of his obligation is not a sufficient ground for the rescission of the agreement." Considering that
respondent has only a balance of P4,000.00 and was delayed in payment only for one month, equity and justice
mandate that he be given an additional period within which to complete payment of the purchase price .

2B 16-17 SALES AND LEASE Page 8


4. VIRGILIO ROMERO VS CA

G.R. No. 107207 November 23, 1995

Vitug, J.

FACTS:

Petitioner Virgilio R. Romero was engaged in the business of production, manufacture and exportation of
perlite filter aids, permalite insulation and processed perlite ore. In 1988, petitioner and his foreign partners decided
to put up a central warehouse in Metro Manila on a land area of approximately 2,000 square meters. The project was
made known to several freelance real estate brokers.

A day or so after the announcement, Alfonso Flores and his wife, accompanied by a broker, offered a
parcel of land measuring 1,952 square meters. in the name of private respondent Enriqueta Chua vda. de Ongsiong.
Petitioner visited the property and, except for the presence of squatters in the area, he found the place suitable for a
central warehouse.

Later, the Flores spouses called on petitioner with a proposal that should he advance the amount of
P50,000.00 which could be used in taking up an ejectment case against the sq uatters, private respondent would agree
to sell the property for only P800.00 per square meter. Petitioner expressed his concurrence. On 09 June 1988, a
contract, denominated "Deed of Conditional Sale," was executed between petitioner and private responden t. It was
agreed that the vendor would take care of the informal settlers.

Pursuant to the agreement, private respondent filed a complaint for ejectment against Melchor Musa and 29
other squatter families with the Metropolitan Trial Court of Paraaque. A few months later, a judgment was
rendered ordering the defendants to vacate the premises. The decision was handed down beyond the 60dayperiod
(expiring 09 August 1988) stipulated in the contract.

In a letter, dated 07 April 1989, private respondent sought to return the P50,000.00 she received from
petitioner as downpayment since, she said, she could not "get rid of the squatters" on the lot. They contend that the
Deed of Conditional Sale had been rendered null and void by virtue of his client's failure to evict the squatters from
the premises within the agreed 60day period. He added that private respondent had decided to retain the property.

Private respondent, prompted by petitioner's continued refusal to accept the return of the P50,000.00
advance payment, filed with the Regional Trial Court of Makati, for rescission of the deed of "conditional" sale, plus
damages, and for the consignation of P50,000.00 cash.

Regional Trial Court of Makati rendered decision holding that private respondent had no right to re scind
the contract since it was she who "violated her obligation to eject the squatters from the subject property" and that
petitioner, being the injured party, was the party who could, under Article 1191 of the Civil Code, rescind the
agreement. The court ruled that the provisions in the contract relating to (a) the return/reimbursement of the
P50,000.00 if the vendor were to fail in her obligation to free the property from squatters within the stipulated period
or(b), upon the other hand, the sum's forfeiture by the vendor if the vendee were to fail in paying the agreed
purchase price, amounted to "penalty clauses".

The lower court, accordingly, dismissed the complaint and ordered, instead, private respondent to eject or
cause the ejectment of the squatters from the property and to execute the absolute deed of conveyance upon payment
of the full purchase price by petitioner.

CA reversed the decision and opined opined that the contract entered into by the parties was subject to a resolutory
condition, i.e., the ejectment of the squatters from the land, the non-occurrence of which resulted in the failure of the
object of the contract

2B 16-17 SALES AND LEASE Page 9


ISSUE: WON private respondents has the right to rescind the contract

HELD: NO! A perfected contract of sale may either be absolute or conditional depending on whether the agreement
is devoid of, or subject to, any condition imposed on the passing of title of the thing to be conveyed or on the
obligation of a party thereto. When ownership is retained until the fulfilment of a posit ive condition the breach of
the condition will simply prevent the duty to convey title from acquiring an obligatory force. If the condition is
imposed on an obligation of a party which is not complied with, the other party may either refuse to proceed or
waive said condition (Art. 1545, Civil Code). Where, of course, the condition is imposed upon the perfection of the
contract itself, the failure of such condition would prevent the juridical relation itself from coming into existence.

In any case, private respondent's action for rescission is not warranted. She is not the injured party. The
right of resolution of a party to an obligation under Article 1191 of the Civil Code is predicated on a breach of faith
by the other party that violates the reciprocity between them. It is private respondent who has failed in her obligation
under the contract.

Petitioner did not breach the agreement. He has agreed, in fact, to shoulder the expenses of the execution of
the judgment in the ejectment case and to make arrangements with the sheriff to effect such execution. Petitionerhas
tendered payment and demanded forthwith the execution of the deed of absolute sale. Parenthetically, this offer to
pay, having been made prior to the demand for rescission, assuming for the sake of argument that such a demand is
proper under Article 1592 of the Civil Code, would likewise suffice to defeat private respondent's prerogative to
rescind thereunder.

2B 16-17 SALES AND LEASE Page 10


5. CORONEL vs CA

G.R. No. 103577. October 7, 1996

Melo, J.:

FACTS:

On January 19, 1985Coronel executed a document entitled "Receipt of Down Payment" in favor of plaintiff
Alcaraz with the following conditions appurtenant to the sale: 1) Ramona will make a down payment of P50,000.00
upon execution of the document aforestated; 2) The Coronels will cause the transfer in their names of the title of the
property registered in the name of their deceased father upon receipt of the P50,000.00 Pesos down payment; and
3)Upon the transfer in their names of the subject property, the Coronels will execute the deed of absolute sale in
favor of Ramona and the latter will pay the former the whole balance of P1,190,000.00. On February 18, 1985, the
Coronels sold the property to Mabanag; Coronels cancelled and rescinded the contract with Alcaraz by depositing
the down payment paid by Concepcion in the bank in trust for Ramona Patricia Alcaraz. As such, Concepcion, et
al., filed a complaint for specific performance against the Coronels. Coronels executed a Deed of Absolute Sale over
the subject property in favor of Catalina

ISSUE:

Whether or not there is perfected contract of sale

HELD: No.

It is essential to distinguish between a contract to sell and a conditional contract of sale specially in cases where the
subject property is sold by the owner not to the party the seller contracted with, but to a third person, as in the case at
bench. In a contract to sell, there being no previous sale of the property, a third person buying such property despite
the fulfillment of the suspensive condition such as the full payment of the purchase price, for instance, cannot be
deemed a buyer in bad faith and the prospective buyer cannot seek the relief of reconveyance of the property. There
is no double sale in such case. Title to the property will transfer to the buyer after registration because there is no
defect in the owner-seller's title per se, but the latter, of course, may be used for damages by the intending buyer.

In a conditional contract of sale, however, upon the fulfillment of the suspensive condition, the sale becomes
absolute and this will definitely affect the seller's title thereto. In fact, if there had been previous delivery of the
subject property, the seller's ownership or title to the property is automatically transferred to the buyer such that, the
seller will no longer have any title to transfer to any third person. Applying Article 1544 of the Civil Code, such
second buyer of the property who may have had actual or constructive knowledge of such defect in the seller's title,
or at least was charged with the obligation to discover such defect, cannot be a registrant in good faith. Such second

2B 16-17 SALES AND LEASE Page 11


buyer cannot defeat the first buyer's title. In case a title is issued to the second buyer, the first buyer may seek
reconveyance of the property subject of the sale.

With the foregoing conclusions, the sale to the other petitioner, Catalina B. Mabanag, gave rise to a case of double
sale where Article 1544 of the Civil Code will apply.

In a case of double sale, what finds relevance and materiality is not whether or not the second buyer was a buyer in
good faith but whether or not said second buyer registers such second sale in good faith, that is, without knowledge
of any defect in the title of the property sold.

As clearly borne out by the evidence in this case, petitioner Mabanag could not have in good faith, registered the
sale entered into on February 18, 1985 because as early as February 22, 1985, a notice of lis pendens had been
annotated on the transfer certificate of title in the names of petitioners, whereas petitioner Mabanag registered the
said sale sometime in April, 1985. At the time of registration, therefore, petitioner Mabanag knew that the same
property had already been previously sold to private respondents, or, at least, she was charged with knowledge that a
previous buyer is claiming title to the same property. Petitioner Mabanag cannot close her eyes to the defect in
petitioners' title to the property at the time of the registration of the property.

2B 16-17 SALES AND LEASE Page 12


6. UNITED MUSLIM AND CHRISTIAN URBAN POOR ASSOCIATION, INC. VS.

BRYC-V DEVELOPMENT CORPORATION

G.R. No. 179653, July 31, 2009

NACHURA, J.:

FACTS:

Respondent Sea Foods Corporation (SFC) is the registered owner of Lot No. 300 located in Lower Calainan,
Zamboanga City and covered by Transfer Certificate of Title (TCT) No. 3182. Sometime in 1991, petitioner United
Muslim and Christian Urban Poor Association, Inc. (UMCUPAI), an organization of squatters occupying Lot No.
300, through its President, Carmen T. Diola, initiated negotiations with SFC for the purchase thereof. UMCUPAI
expressed its intention to buy the subject property using the proceeds of its pending loan application with National
Home Mortgage Finance Corporation (NHMF). The parties executed a Letter of Int ent. However, the intended sale
was derailed due to UMCUPAIs inability to secure the loan from NHMF as not all its members occupying Lot No.
300 were willing to join the undertaking. Intent on buying the subject property, UMCUPAI, in a series of
conferences with SFC, proposed the subdivision of Lot No. 300 to allow the squatter-occupants to purchase a
smaller portion thereof.

Consequently, sometime in December 1994, Lot No. 300 was subdivided into three (3) parts covered by separate
titles. UMCUPAI failed to acquire Lot No. 300-C for lack of funds. On March 5, 1995, UMCUPAI negotiated anew
with SFC and was given by the latter another three months to purchase Lot No. 300-C. However, despite the
extension, the three-month period lapsed with the sale not consummated because UMCUPAI still failed to obtain a
loan from NHMF. Thus, on July 20, 1995, SFC sold Lot No. 300-C for P2,547,585.00 to respondent BRYC-V
Development Corporation (BRYC).

UMCUPAI filed with the RTC a complaint against respondents SFC and BRYC s eeking to annul the sale of Lot
No. 300-C, alleging that the sale between the respondents violated its valid and subsisting agreement with SFC
embodied in the Letter of Intent. According to UMCUPAI, the Letter of Intent granted it a prior, better, and
preferred right over BRYC in the purchase of Lot No. 300-C. According to the RTC, the Letter of Intent was simply
SFCs declaration of intention to sell, and not a promise to sell, the subject lot. On the whole, the RTC concluded that
the Letter of Intent was neither a promise, nor an option contract, nor an offer contemplated under Article 1319 of
the Civil Code, or a bilateral contract to sell and buy. CA affirmed.

ISSUE

Whether or not UMCUPA ask for the annulment of the sale?

HELD:

NO. Nowhere in the Letter of Intent does it state that SFC relinquishes its title over the subject property,
subject only to the condition of complete payment of the purchase price; nor, at the least, that SFC, although
expressly retaining ownership thereof, binds itself to sell the property exclusively to UMCUPAI. The Letter of

2B 16-17 SALES AND LEASE Page 13


Intent to Buy and Sell is just that a manifestation of SFCs intention to sell the property and UMCUPAIs intention to
acquire the same.

The Letter of Intent/Agreement between SFC and UMCUPAI is merely a written preliminary understanding of the
parties wherein they declared their intention to enter into a contract of sale. It is subject to the condition that
UMCUPAI will apply with the Home Mortgage and Finance Corporation for a loan to pay the ac quisition price of
said land. One of the requirements for such loan is a formal manifestation of Intent to Sell from SFC. Thus, the
Letter of Intent to Sell fell short of an offer contemplated in Article 1319 of the Civil Code because it is not a certain
and definite proposal to make a contract but merely a declaration of SFCs intention to enter into a contract.
UMCUPAIs declaration of intention to buy is also not certain and definite as it is subject to the condition that
UMCUPAI shall endeavor to raise funds to acquire subject land. The acceptance of the offer must be absolute; it
must be plain and unconditional. Moreover, the Letter of Intent/Agreement does not contain a promise or
commitment to enter into a contract of sale as it merely declared the intention of the parties to enter into a contract
of sale upon fulfillment of a condition that UMCUPAI could secure a loan to pay for the price of a land.

The failure of UMCUPAI to acquire Lot No. 300-C before it was sold to BRYC-V cannot be blamed on SFC
because all that UMCUPAI had to do was to raise funds to pay for Lot No. 300-C which it did with respect to Lot
No. 300-A.

2B 16-17 SALES AND LEASE Page 14


7.DELFIN TAN VS ERLINDA C. BENOLIRAO, ANDREW C. BENOLIRAO, ROMANO C. BENOLIRAO,
DION C. BENOLIRAO, SPS. REYNALDO TANINGCO and NORMA D. BENOLIRAO, EVELYN T.
MONREAL, and
ANN KARINA TANINGCO

GR NO. 153820, October 16, 2009

BRION, J

FACTS
Spouses Lamberto and Erlinda Benolirao and the Spouses Reynaldo and Norma Taningco were the coowners of a
689square meter parcel of land (property) located in Tagaytay City and covered by Transfer Certificate of Title
(TCT) No. 26423. On October 6, 1992, the co-owners executed a Deed of Conditional Sale over the property in
favor of Tan for the price of P1,378,000.00. Pursuant to such, Tan issued and delivered to the co-owners/vendors a
check for P20,000 as downpayment for the property, for which a receipt was issued to him.
Later, Lamberto Benorilao died intestate. Erlinda, his widow and one of the vendors of the property, and their
children, executed an extrajudicial settlement of the decedents property. An annotation was made thereon to the
tenor that x x x any liability to credirots (sic), excluded heirs and other persons having right to the property, for a
period of two (2) years, with respect only to the share of Erlinda, Andrew, Romano and Dion, all surnamed
Benolirao On the basis of such settlement, a new certificate of title over the property was issued in the names of the
Spouses Reynaldo and Norma Taningco and Erlinda Benolirao and her children.
As stated in the Deed of Conditional Sale, Tan had until March 15, 1993 to pay the balance of the purchase price. By
agreement of the parties, this period was extended by two months, so Tan had until May 15, 1993 to pay the balance.
Tan however failed to pay, despite further extensions given to him, prompting the vendors to make a written demand
within 5 days of notice, otherwise, they would rescind the sale.
Tan refused to comply with such, alleging that by the annotation in the TCT, he is prevented from getting a clean
title, and therefore he could no longer be required to pay the balance of the purchase price. He then demanded the
return of his down payment. When the vendors refused to comply with his demand, Tan filed a complaint with t he
RTC for specific performance, alleging that there there was a novation of the Deed of Conditional Sale done without
his consent since the annotation on the title created an encumbrance over the property. Tan prayed for the refund of
the down payment and the rescission of the contract. Tan later on amended his complaint, contending that if the
respondents insist on forfeiting the down payment, he would be willing to pay the balance of the purchase price
provided there is reformation of the Deed of Conditional Sale. In the meantime, Tan caused the annotation on the
title of a notice of lis pendens .
However, later, the respondents issued a Deed of Absolute Sale in favor ofHector De Guzman. Thereafter, the
respondents moved for the cancellation of the notice of lis pendens on the ground that it was inappropriate since the
case that Tan filed was a personal action which did not involve either title to, or possession of, real property. The
RTC issued an order dated October 22, 1993 granting the respondents mo tion to cancel the lis pendens annotation on
the title. Meanwhile, based on the Deed of Absolute Sale in his favor, de Guzman registered the property and TCT
No. 28104 was issued in his name.
Tan then filed a motion to carry over the lis pendens annotatio n to TCT No. 28104 registered in de Guzmans name,
but the RTC denied the motion. The RTC ruled that the forfeiture of the downpayment was in accordance with the
conditions in the Deed of Conditional Sale.The CA affirmed the RTC ruling, hence this petition .

ISSUE
Whether or not the forfeiture of the downpayment was correctly upheld by the RTC and the CA

2B 16-17 SALES AND LEASE Page 15


HELD
NO. The contract to sell was terminated when the vendors could no longer legally compel Tan to pay the balance of
the purchase price as a result of the legal encumbrance which attached to the title of the property. Since Tans refusal
to pay was due to the supervening event of a legal encumbrance on the property and not through his own fault or
negligence, we find and so hold that the forfeiture of Tans down payment was clearly unwarranted.
The true nature of the contract in question is revealed in stipulation (d) That in case, BUYER has complied with
the terms and conditions of this contract, then the SELLERS shall execute and deliver to the BUYER the appropriate
Deed of Absolute Salethat is, the contract here is a contract to sell, where the seller promises to execute a deed
of absolute sale upon the completion by the buyer of the payment of the price.
In the present case, and as held in a number of cases decided by the Supreme Court, rescission cannot be had as a
remedy in contracts to sell. The rationale has been enunciated in Santos vs CA:
[I]n a contract to sell, title remains with the vendor and does not pass on to the vendee until the
purchase price is paid in full. Thus, in a contract to sell, the payment of the purchase price is a
positive suspensive condition. Failure to pay the price agreed upon is not a mere breach, casual or serious, but a
situation that prevents the obligation of the vendor to convey title from acquiring an obligatory force. This is entirely
different from the situation in a contract of sale, where nonpayment of the price is a negative resolutory condition.
The effects in law are not identical. In a contract of sale, the vendor has lost ownership of the thing sold and cannot
recover it, unless the contract of sale is rescinded and set aside. In a contract to sell, however, the vendor remains the
owner for as long as the vendee has not complied fully with the condition of paying the purchase price. If the vendor
should eject the vendee for failure to meet
the condition precedent, he is enforcing the contract and not rescinding it. x x x Article 1592 speaks of nonpayment
of the purchase price as a resolutory condition. It does not apply to a contract to sell. As to Article 1191, it is
subordinated to the provisions of Article 1592 when applied to sales of immovable property. Neither provision is
applicable [to a contract to sell].

NOTES:
Among the conditions stated in the Deed are the following:
a) An initial downpayment of TWO HUNDRED (P200,000.00) THOUSAND PESOS, Philippine Currency, upon
signing of this contract then the remaining balance of ONE MILLION ONE HUNDRED SEVENTY EIGHT
THOUSAND (P1,178,000.00) PESOS, shall be payable within a period of one hundred fifty (150) days from date
hereof without interest
b) That for any reason, BUYER fails to pay the remaining balance within above mentioned period, the BUYER shall
have a grace period of sixty (60) days within which to make the payment, provided that there shall be an interest of
15% per annum on the balance amount due from the SELLERS
c) That should in case (sic) the BUYER fails to comply with the terms and conditions within the above stated grace
period, then the SELLERS shall have the right to forfeit the down payment, and to rescind this conditional sale
without need of judicial action
d) That in case, BUYER have complied with the terms and conditions of this contract, then the
SELLERS shall execute and deliver to the BUYER the appropriate Deed of Absolute Sale

2B 16-17 SALES AND LEASE Page 16


8. RAYMUNDO DE LEON VS BENITA ONG

G.R. No. 170405, February 2, 2010

FACTS:
On March 10, 1993, petitioner Raymundo S. de Leon sold three parcels of land with improvements situated in
Antipolo, Rizal to respondent Benita T. Ong. Parcels of land were subject to a mortgage in favor of Real Savings
and Loan Association, Incorporated. A Notarized Deed of Absolute Sale with Assumption of Mortgage was
executed by the parties.
The following are found in the deed:
- that petitioner does hereby sell, transfer and convey in a manner absolute and irrevocable, unto said respondent...
the subject parcels of land
- upon payment by Ong of Php415,000 petitioner will execute and sign a Deed of Absolute Sale with Assumption
of Mortgage
- Ong to assume payment of outstanding loan with RSLAI in the amount of Php684,500
Ong gave the Php415,000, de Leon handed keys to properties and sent a letter to the bank informing the latter of the
sale and authorizing it to accept payment from respondent and release the certificates of title.
Ong introduced improvements and likewise informed the bank about the sale. The bank said that Ong shall undergo
credit investigation.
Respondent learned that petitioner again sold the same properties to one Leona Viloria after March 10, 1993 and
changed the locks, rendering the keys he gave her useless. Respondent thus proceeded to RSLAI to inquire about the
credit investigation. However, she was informed that petitioner had already paid the a mount due and had taken back
the certificates of title.
Respondent filed a complaint for specific performance, declaration of nullity of the second sale and damages against
petitioner and Viloria in the Regional Trial Court (RTC) of Antipolo, Rizal, Branch 74. She claimed that since
petitioner had previously sold the properties to her on March 10, 1993, he no longer had the right to sell the same to
Viloria. Thus, petitioner fraudulently deprived her of the properties.

ISSUES

(1) Whether the contract entered into is a Contract of Sale or Contract to Sell.
(2) Whether there was a Void Subsequent Sale or Double Sale.
HELD
(1) The contract entered into was a CONTRACT OF SALE
In a Contract of Sale the seller conveys ownership of the property to the buyer upon the perfe ction of the contract.
Should the buyer default in the payment of the purchase price, the seller may either sue for the collection thereof or
have the contract judicially resolved and set aside. The non -payment of the price is therefore a negative resolutory
condition.
In a Contract to Sell, it is subject to a positive suspensive condition. The buyer does not acquire ownership of the
property until he fully pays the purchase price. For this reason, if the buyer defaults in the payment thereof, the seller
can only sue for damages.
*Nothing in said instrument implied that petitioner reserved ownership of the properties until the full payment of the
purchase price. Terms and conditions pertained to the performance of the contract, not the perfection thereof nor the
transfer of ownership.
* Article 1498 of the Civil Code provides that, as a rule, the execution of a notarized deed of sale is equivalent to the

2B 16-17 SALES AND LEASE Page 17


delivery of a thing sold.
Indicators that petitioner had unqualifiedly delivered and transferred ownership of properties to respondent:
- notarized deed of absolute sale was executed
- keys to the properties were tuned over
- RSLAI was authorized to received payment from Ong and release certificates of title to her
* Article 1186. The condition shall be deemed fulfilled when the obligor voluntarily prevents its fulfillment.
Since de Leon himself prevented approval of the bank with regard to assumption of mortgage by Ong by paying his
outstanding obligation and taking bank certificates of title without notifying Ong, it is deemed fulfilled.
(2)There was DOUBLE SALE
Ong is a purchaser in good faith.
- A purchaser in good faith is one who buys the property of another without notice that some other person has a right
to, or an interest in, such property and pays a full and fair price for the same at the time of such purchase, or before
he has notice of some other persons claim or interest in the property.
Ong is deemed released from payment of the remainder of purchase price.
- de Leon voluntarily prevented the fulfillment of this condition and the same has become legally impossible
Ong is also considered in possession of the property already for the same reasons indicated for the transfer of
ownership.
Nonetheless, while the condition as to the payment of the balance of the purchase price was deemed fulfilled,
respondents obligation to pay it subsisted. Otherwise, she would be unjustly enriched at the expense of petitioner.
Petitioner to transfer titles to respondent.

2B 16-17 SALES AND LEASE Page 18


CONTRACT TO SELL

1. LUZON DEVELOPMENT BANK VS ANGELES CATHERINE ENRIQUEZ

G.R. No. 168646 January 12, 2011

DEL CASTILLO, J.

FACTS

Petitioner DELTA is engaged in the business of developing and selling real estate properties, particularly Delta
Homes I in Cavite. In 1995, for the purpose of developing Delta Homes 1, DELTA obtained a 4 million loan from
the Luzon Development Bank (BANK), a domestic financial corporation that extends loans to subdivision
developers/owners. To secure the loan, DELTA executed real estate mortgage on their properties, including Lot 4 of
Delta Homes I, which is the subject matter of the case.

In 1997, DELTA executed a Contract to Sell with respondent Angeles Catherine Enriquez (Enriquez) for Lot 4 of
Delta Homes I . The contract indicated that (1) upon failure to pay 3 successive monthly installment payments,
DELTA may consider the Contract to Sell null and void ab initio without need for further court action and all
payments shall be forfeited, and (2) DELTA shall execute a final deed of sale in favor of the BANK only upon full
payment of the total consideration.

DELTA defaulted in its loan and the BANK agreed to dacion en pago instead of foreclosing the REM; a Deed of
Assignment in Payment of Debt was executed in favor of the BANK.

In 1999, Enriquez filed a complaint against the BANK and DELTA, and sought a full refund of the amount paid to
DELTA, award of damages, and the imposition of administrative fines on DELTA and the BANK. The HLURB
Board ruled that the Contract to Sell was valid and the ownership over Lot 4 was already conveyed to Enriquez.

DELTA's contention was that the Contract to Sell contained a condition that ownership shall only be transferred
to Enriquez upon the latters full payment of the purchase price to DELTA. Since Enriquez has yet to comply with
this suspensive condition, ownership is retained by DELTA.

ISSUE

Whether or not the Contract to Sell conveyed ownership to Enriquez

HELD

NO, in a Contract to Sell the seller obliges himself to sell the subject propert y only upon the happening of a
suspensive condition, such as full payment of the purchase price; ownership is retained by the prospective seller
without further remedies by the prospective buyer. The terms of their Contract to Sell expressly provided that
DELTA has reserved ownership until full payment, and shall have the right to unilaterally void the contract upon

2B 16-17 SALES AND LEASE Page 19


Enriquez's failure to pay installment thrice successively.

Since the Contract to Sell did not transfer ownership to Enriquez, DELTA can valid ly transfer such ownership to the
BANK. However, because what is involved is a subdivision lot, the transferee BANK is bound by the Contract to
Sell and has to respect Enriquez's rights as per PD 957. Although the Contract to Sell was not registered, the BANK
is not deemed a purchaser in good faith because (1) the property was a subdivision lot and therefore within the
purview of PD 957, and (2) it knew that the loaned amounts were to be used for the development of DELTAs
subdivision project, for this was indicated in the corresponding promissory notes.

"Bound by the terms of the Contract to Sell, the BANK is obliged to respect the same and honor the payments
already made by Enriquez for the purchase price of Lot 4. Thus, the BANK can only collect the balan ce of the
purchase price from Enriquez and has the obligation, upon full payment, to deliver to Enriquez a clean title over the
subject property."

Special note: Section 17 of PD 957 - Registration. All contracts to sell, deeds of sale, and other similar i nstruments
relative to the sale or conveyance of the subdivision lots and condominium units, whether or not the purchase price
is paid in full, shall be registered by the seller in the Office of the Register of Deeds of the province or city where the
property is situated.

2B 16-17 SALES AND LEASE Page 20


2. SPS ONNIE SERRANO AND AMPARO HERRERA VS GODOFREDO CAGUIAT

G.R. No. 139173, February 28, 2007

SANDOVAL-GUTIERREZ, J.:

FACTS:

Spouses Onnie and Amparo Herrera are the registered owners of a lot located in Las Pias, Metro Manila.
Sometime in March 1990, Godofredo Caguiat, offered to buy the lot. Spouses Serrano agreed to sell it at P1,500.00
per square meter. Caguiat then gave them P100,000.00 as partial payment. In turn, Spouses Serrano gave him the
corresponding receipt stating that Caguiat promised to pay the balance of the purchase price on or before March 23,
1990. On March 28, 1990, Caguiat, then wrote Spouses Serrano informing them of his readiness to pay the balance.
Spouses Serrano, however, informed him that they are leaving for abroad and that they are cancelling the
transaction. He may likewise recover the earnest money of P100, 000. anytime. Thereafter, Spouses Serrano sent a
check in the amount of P100,000 to Caguiats counsel. Caguiat then filed a complaint ag ainst Spouses Serrano for
specific performance and damages.

ISSUE

Whether or not the document receipt that was signed by both and is issued to Caguiat is a contract of sale since there
was already an earnest money given.

HELD

sNO, the receipt is that of a contract to sell. A contract to sell is akin to a conditional sale where the efficacy or
obligatory force of the vendors obligation to transfer title is subordinated to the happening of a future and uncertain
event, so that if the suspensive condition does not take place, the parties would stand as if the conditional obligation
had never existed. The suspensive condition is commonly full payment of the purchase price. In a contract to sell,
ownership is retained by the seller and is not to pass to the b uyer until full payment of the price. In this case, the
Receipt for Partial Payment shows that the true agreement between the parties is a contract to sell. First,
ownership over the property was retained by petitioners and was not to pass to respondent until full payment
of the purchase price. Second, the agreement between the parties was not embodied in a deed of sale. Third,
petitioners retained possession of the certificate of title of the lot.

2B 16-17 SALES AND LEASE Page 21


3.DARREL CORDERO, EGMEDIO BAUTISTA, ROSEMAY BAUTISTA, MARION BAUTISTA, DANNY
BOY CORDERO, LADYLYN CORDERO and BELEN CORDERO, vs F.S. MANAGEMENT &
DEVELOPMENT CORPORATION

G.R. No. 167213 October 31, 2006

FACTS:

On or about October 27, 1994, petitioner entered into a contract to sell with re spondent, over five (5) parcels of land
located in Nasugbu, Batangas. The contract to sell contained the following terms and conditions:

1. That the BUYER will buy the whole lots above described from the OWNER consisting of 50 hectares
more or less at P25/sq.m. or with a total price of P12,500,000.00;

2. That the BUYER will pay the OWNER the sum of P500,000.00 as earnest money which will entitle the
latter to enter the property and relocate the same, construct the necessary paths and roads with the help of
the necessary parties in the area;

3. The BUYER will pay the OWNER the sum of THREE MILLION FIVE HUNDRED THOUSAND
PESOS ONLY (P3,500,000.00) on or before April 30, 1995 and the remaining balance will be paid within
18 mons. (sic) from the date of payment of P3.5 Million pesos in 6 equal quarterly payments or
P1,411,000.00 every quarter;

4. The title will be transferred by the OWNER to the BUYER upon complete payment of the agreed
purchase price. Provided that any obligation by the OWNER brought about by enc umbrance or mortgage
with any bank shall be settled by the OWNER or by the BUYER which shall be deducted the total purchase
price;

5. Provided, the OWNER shall transfer the titles to the BUYER even before the complete payment if the
BUYER can provide post dated checks which shall be in accordance with the time frame of payments as
above stated and which shall be guaranteed by a reputable bank;

6. Upon the payment of the earnest money and the down payment of 3.5 Million pesos the BUYER can
occupy and introduce improvements in the properties as owner while owner is guaranteeing that the
properties will have no tenants or squatters in the properties and cooperate in the development of any
project or exercise of ownerships by the BUYER;

7. Delay in the payment by the BUYER in the agreed due date will entitle the SELLER for the legal
interest.

Pursuant to the terms and conditions of the contract to sell, respondent paid earnest money in the amount of
P500,000 on October 27, 1994. She likewise paid P1,000,000 on June 30, 1995 and another P1,000,000 on July 6,
1995. No further payments were made thereafter.

Petitioners thus sent respondent a demand letter, however Respondent ignored the demand.

2B 16-17 SALES AND LEASE Page 22


The Petitioners, filed before the RTC of Paraaque a complaint for rescission of contract with damages alleging
that respondent failed to comply with its obligations under the contract to sell, specifically its obligation to pay the
downpayment of P3.5 Million by April 30, 1995, and the balance within 18 months thereafter; and that consequently
petitioners are entitled to rescind the contract to sell as well as demand the payment of damages.

The Respondent, alleged that petitioners have no cause of action considering that they were the first to violate the
contract to sell by preventing access to the properties despite payment of P2.5 Million Pesos; petitioners prevented it
from complying with its obligation to pay in full by refusing to execute the final contract of sale unless additional
payment of legal interest is made; and petitioners refusal to execute the final contract of sale was due to the
willingness of another buyer to pay a higher price.

The TRIAL COURT, issued its decision on November 18, 1997, finding for petitioners and ordering respondent to
pay damages and attorneys fees.

The COURT OF APPEALS set aside the contract to sell, it finding that petitioners obligation thereunder did not
arise for failure of respondent to pay the full purchase price. It also set aside the award to petitioners of damages for
not being duly proven. And it ordered petitioners to return "the amount received from [respondent]."

ISSUE:

Whether or not a CONTRACT TO SELL may be subject to rescission under Article 1191 of the Civil Code.

HELD:

NO. Under a contract to sell, the seller retains title to the thing to be sold until the purchaser fully pays the
agreed purchase price. The full payment is a positive suspensive condition, the non-fulfillment of which is not
a breach of contract but merely an event that prevents the seller from conveying title to the purchaser. The
non-payment of the purchase price renders the contract to sell ineffective and without force and effect.

The petitioners retained ownership without further remedies by the respondents until the payment of the purc hase
price of the property in full. Such payment is a positive suspensive condition, failure of which is not really a
breach, serious or otherwise, but an event that prevents the obligations of the petitioners to convey title from
arising.

The non-fulfillment by the respondent of his obligation to pay, which is a suspensive condition to the
obligation of the petitioners to sell and deliver the title to the property, rendered the contract to sell
ineffective and without force and effect.

Article 1191 of the New Civil Code will not apply because it presupposes an obligation already extant. There
can be no rescission of an obligation that is still non-existing, the suspensive condition not having happened.

2B 16-17 SALES AND LEASE Page 23


Articles 1191 and 1592 of the Civil Code are applicable to contracts of sale. In contracts to sell, RA 6552 applies.

x x x Known as the Maceda Law, R.A. No. 6552 recognizes in conditional sales of all kinds of real estate
(industrial, commercial, residential) the right of the seller to cancel the contract upon non-payment of an
installment by the buyer, which is simply an event that prevents the obligation of the vendor to convey title
from acquiring binding force. It also provides the right of the buyer on installments in case he defaults in the
payment of succeeding installments x x x.

The properties subject of the contract having been intended for commercial, and not for residential, purposes,
petitioners are entitled to retain the payments already made by respondent. RA 6552 expressly recognizes the
vendors right to cancel contracts to sell on installment basis industrial and commercial properties with full retention
of previous payments.

WHEREFORE, the assailed Court of Appeals Decision is AFFIRMED with the MODIFICATION that
petitioners are entitled to retain the payments already received from respondent.

2B 16-17 SALES AND LEASE Page 24


DATION IN PAYMENT

1. DAO HENG BANK VS SPS LILIA AND REYNALDO LAIGO

G.R. No. 173856

November 20, 2008

Ponente: Carpio Morales

FACTS:

Sps. Laigo obtained loans from Dao Heng Bank to secure payment of which they forged 3 real estate
mortgages. They failed to settle their outstanding obligations, drawing them to verbally offer to cede to Dao Heng
one of 2 mortgage lots by way of dacion en pago. Dao Heng had the properties appraised. Bu t there appears to have
been no further action after the appraisal.

Dao Heng was later to demand the settlement of respondents obligation which they failed to heed. Dao
Heng filed an application to foreclose the REM and the properties were then sold to pet itioner at a public auction.

Respondents negotiated for redemption a year after the sale to which the petitioner stated the redemption
price. But nothing was heard from the respondents thereafter. The petitioner then advised the respondents that it
would have the titles consolidated after the expiration of the redemption period.

Six days before the expiration of the redemption period, the respondents filed a complaint for annulment of
the foreclosure of the properties and for them to deliver by way of dacion en pago one of the mortgaged properties
as full payment of [their] mortgaged obligation. Respondents claim that Dao Heng verbally agreed to enter into a
dacion en pago.

Petitioner claimed that there was no meeting of the minds between the parties on t he settlement of
respondents loan via dacion en pago and that the claim of respondents are unenforceable under the Statute of
Frauds. Respondents opposed and argued that their delivery of the titles to the mortgaged properties constituted
partial performance of their obligation under the dacion en pago.

ISSUE:

Whether or not there was a perfected dacion en pago (No)

HELD:

Dacion en pago as a mode of extinguishing an existing obligation partakes of the nature of sale whereby property is
alienated to the creditor in satisfaction of a debt in money. It is an objective novation of the obligation, hence,
common consent of the parties is required in order to extinguish the obligation.

. . . In dacion en pago, as a special mode of payment, the debtor offers another thing to the creditor who accepts it as
equivalent of payment of an outstanding debt. The undertaking really partakes in one sense of the nature of sale, that

2B 16-17 SALES AND LEASE Page 25


is, the creditor is really buying the thing or property of the debtor, payment for which is t o be charged against the
debtors debt. As such the elements of a contract of sale, namely, consent, object certain, and cause or consideration
must be present. In its modern concept, what actually takes place in dacion en pago is an objective novation of the
obligation where the thing offered as an accepted equivalent of the performance of an obligation is considered as the
object of the contract of sale, while the debt is considered the purchase price. In any case, common consent is an
essential prerequisite, be it sale or novation, to have the effect of totally extinguishing the debt or obligation.

Being likened to that of a contract of sale, dacion en pago is governed by the law on sales. The partial
execution of a contract of sale takes the transaction out of the provisions of the Statute of Frauds so long as the
essential requisites of consent of the contracting parties, object and cause of the obligation concur and are clearly
established to be present.

Respondents claim that petitioners commissioning of an appraiser to appraise the value of the mortgaged properties,
his services for which they and petitioner paid, and their delivery to petitioner of the titles to the properties constitute
partial performance of their agreement to take the case out of the provisions on the Statute of Frauds.

There is no concrete showing, however, that after the appraisal of the properties, petitioner approved
respondents proposal to settle their obligation via dacion en pago. The delivery to petitioner of the titles to the
properties is a usual condition sine qua non to the execution of the mortgage, both for security and registration
purposes. For if the title to a property is not delivered to the mortgagee, what will prevent the mortgagor from again
encumbering it also by mortgage or even by sale to a third party.

Finally, that respondents did not deny proposing to redeem the mortgages, as reflected in petitioners June
29, 2001 letter to them, dooms their claim of the existence of a perfected dacion en pago

2B 16-17 SALES AND LEASE Page 26


PAYMENT BY CESSION

1. DBP VS CA AND LYDIA CUBA

GR NO. 118342 January 5, 1998

FACTS

Private respondent Lydia Cuba is a grantee of a fishpond lease agreement from the Government. She later obtained a
loan from DBP in the amounts of P109, 000, P109, 000, and P98, 700 under the terms stated in the three promissory
notes. As a security for the said loan Cuba executed a two Deed of Assignment of her Leasehold Rights. Then she
failed to pay her loan when it became due in accordance with the terms of the promis sory notes. DBP in turn
appropriated the leasehold rights of Cuba over the fishpond, without foreclosure proceedings, whether judicial or
extrajudicial. After appropriating the said leasehold rights DBP executed a Deed of Conditional Sale of the
Leasehold Rights in favor of respondent Cuba over the same fishpond, to which Cuba agreed. Respondent Cuba
failed to pay the amortizations stipulated in the Deed of Conditional Sale, however she was able enter with DBP a
temporary arrangement with DBP for the Deferment Notarial Rescission of Deed of Conditional Sale. However, a
Notice of Rescission thru Notarial Act was sent the DBP to Cuba, then it took possession of the fishpond in
question. After it took possession of the said fishpond, DBP disposed the property in favor of Agripina Caperal
through a deed of conditional sale. Then a new fishpond lease agreement was awarded by the Government to
Caperal.

Lydia Cuba filed an action with the Regional Trial Court of Pangasinan for the declaration of nullity of DBPs
appropriation of her leaseholds over the subject fishpond, for the annulment of the Deed of Conditional Sale
executed in her favor by DBP, the annulment of DBPs sale of the fishpond to Caperal, and the restoration of her
rights over the said fishpond and for damages. The RTC ruled in favor of Cuba, declaring that DBPs taking
possession and ownership of the subject property without foreclosure was violative of Art. 2088 of the Civil Code,
and that condition No.12 of the Assignment of the Leasehold Rights was void for being a clear case of
pactumcommissorium.

DBP claims that the was in the nature of a cession under Art. 1255.

Both Cuba and DBP elevated the case to the CA, with Cuba seeking an increase in the amount of damages, while
DBP questioned the findings of fact and law of the RTC. The CA reversed the ruling of the RTC with regards to the
validity of the acts of DBP.

ISSUE:

Whether or not the Assignment of the Leasehold Rights constitute payment by cession under Art. 1255 of the Civil
code.

HELD:

2B 16-17 SALES AND LEASE Page 27


No. The Supreme court held that the assignment did not amount to payment by cession under Article 1255 of the
Civil Code for the plain and simple reason that there was only one creditor, the DBP. Article 1255 contemplates the
existence of two or more creditors and involves the assignment of all the debtors property.

2B 16-17 SALES AND LEASE Page 28


CONTRACT OF AGENCY TO SELL

1. QUIROGA VS PARSONS HARDWARE CO

GR No. L-11491 Aug. 23, 1918

Avancena, J.:

FACTS

On Jan. 4, 1911, petitioner and respondent entered into a contract, in which Don Andres Quiroga grants Parsons
Hardware exclusive rights to sell his beds in the Visayas, furnishing beds of his manufacture for respondent's stores
in Iloilo, with a 25% discount. In his complaint, plaintiff avers that the respondent fa iled to comply with the
obligation of ordering by the dozen, and other obligations implied in a contract of agency.

ISSUE

Whether or not the contract was of agency or a sale

HELD

It was a contract of sale. None of the words in the contract indicated an agency. The words commission on sales
meant nothing more than a discount on the invoice price. In order to classify a contract, due regard must be given to
its essential clauses. In the contract in question, what was essential, as constituting its cause a nd subject matter, is
that the plaintiff was to furnish the defendant with the beds which the latter might order, at the price stipulated, and
that the defendant was to pay the price in the manner stipulated. The price agreed upon was the one determined by
the plaintiff for the sale of these beds in Manila, with a discount of from 20 to 25 per cent, according to their class.
Payment was to be made at the end of sixty days, or before, at the plaintiff's request, or in cash, if the defendant so
preferred, and in these last two cases an additional discount was to be allowed for prompt payment. These are
precisely the essential features of a contract of purchase and sale. There was the obligation on the part of the plaintiff
to supply the beds, and, on the part of the defendant, to pay their price. These features exclude the legal conception
of an agency or order to sell whereby the mandatory or agent received the thing to sell it, and does not pay its price,
but delivers to the principal the price he obtains from the sale of the thing to a third person, and if he does not
succeed in selling it, he returns it. By virtue of the contract between the plaintiff and the defendant, the latter, on
receiving the beds, was necessarily obliged to pay their price within the term fixed, without any other consideration
and regardless as to whether he had or had not sold the beds.

2B 16-17 SALES AND LEASE Page 29


2. KER & CO., LTD. vs. JOSE B. LINGAD, as Acting Commissioner of Internal Revenue

G.R. No. L-20871, April 30, 1971

FERNANDO, J.:

FACTS

It was shown that petitioner was assessed by the then Commissioner of Internal Revenue Melecio R. Domingo the
sum of P20,272.33 as the commercial broker's percentage tax, surcharge, and compromise penalty for the period
from July 1, 1949 to December 31, 1953. There was a request on the part of petitioner for the cancellation of such
assessment, which request was turned down. As a result, it filed a petition for review with the Court of Tax Appeals.
In its answer, the then Commissioner Domingo maintained his stand that petitioner should be taxed in such amount
as a commercial broker.

Such liability arose from a contract of petitioner with the United States Rubber International, the former being
referred to as the Distributor and the latter specifically designated as the Company. The contract was to apply to
transactions between the former and petitioner, as Distributor, from July 1, 1948 to continue in force until
terminated by either party giving to the other sixty days' notice.

Then came this crucial stipulation: "The Company shall from time to time consign to the Distributor and the
Distributor will receive, accept and/or hold upon consignment the products specified under the terms of this
agreement in such quantities as in the judgment of the Company may be nece ssary for the successful solicitation and
maintenance of business in the territory, and the Distributor agrees that responsibility for the final sole of all goods
delivered shall rest with him. All goods on consignment shall remain the property of the Comp any until sold by the
Distributor to the purchaser or purchasers, but all sales made by the Distributor shall be in his name, in which the
sale price of all goods sold less the discount given to the Distributor by the Company in accordance with the
provision of paragraph 13 of this agreement, whether or not such sale price shall have been collected by the
Distributor from the purchaser or purchasers, shall immediately be paid and remitted by the Distributor to the
Company. It is further agreed that this agreement does not constitute Distributor the agent or legal representative 4
of the Company for any purpose whatsoever. Distributor is not granted any right or authority to assume or to create
any obligation or responsibility, express or implied, in behalf o f or in the name of the Company, or to bind the
Company in any manner or thing whatsoever." Issue: whether the relationship thus created is one of vendor and
vendee or of broker and principal.

ISSUE:

Whether the relationship thus created is one of vendor and vendee or of broker and principal.

HELD:

2B 16-17 SALES AND LEASE Page 30


Broker-principal. The relationship of Ker and Co and US rubber was that of a principal-broker/ agency. Ker and Co
is only an agent of the US rubber because it can dispose of the products of the Company only to certain persons or
entities and within stipulated limits, unless excepted by the contract or by the Rubber Company, it merely receives,
accepts and/or holds upon consignment the products, which remain properties of the latter company, every effort
shall be made by petitioner to promote in every way the sale of the products and that sales made by petitioner are
subject to approval by the company. Since the company retained ownership of the goods, even as it delivered
possession unto the dealer for resale to customers, the price and terms of which were subject to the companys
control, the relationship between the company and the dealer is one of agency.

2B 16-17 SALES AND LEASE Page 31


3. LOURDES VALERIO LIM VS PEOPLE

GR NO. L-34338 November 21, 1984

FACTS

On January 10, 1966, the Lim went to the house of Maria Ayroso and proposed to sell Ayrosos tobacco. Ayroso
agreed to the proposition of the ap- pellant to sell her tobacco consisting of 615 kilos at P1.30 a kilo. The appellant
was to receive the overprice for which she could sell the tobacco. An agreement was made that said tobacco was to
be sold by Lim and that the proceed will be given to Ayroso as soon as it was sold. This was signed by the appellant
and witnessed by the complainants sister, Salud Bantug, and t he latters maid, Genoveva Ruiz. Of the total value of
P799.50, the appellant had paid to Ayroso only P240.00, and this was paid on three different times. Demands for the
payment of the balance of the value of the tobacco were made upon the appellant by Ay roso, and particularly by her
sister, Salud Bantug. Although the appellant denied that demands for payment were made upon her, it is a fact that
on October 19, 1966, she wrote a letter to Salud Bantug that she was apologizing for the delay of the giving of
proceeds and that she has some of the money already.

ISSUE Whether or not Lims role constitutes as an agent with respect to Valerio.

HELD

Yes. Aside from the fact that Maria Ayroso testified that the appellant asked her to be her agent in selling Ayros os
tobacco, the appellant herself admitted that there was an agreement that upon the sale of the tobacco she would be
given something. The appellant is a businesswoman, and it is, unbelievable that she would go to the extent of going
to Ayrosos house if she did not intend to make a profit out of the transaction. Certainly, if she was doing a favor to
Maria Ayroso and it was Ayroso who had requested her to sell her tobacco, it would not have been the appellant
who would have gone to the house of Ayroso, but it would have been Ayroso who would have gone to the house of
the appellant and deliver the tobacco to the appellant. The fact that appellant received the tobacco to be sold at P1.30
per kilo and the proceeds to be given to complainant as soon as it was sold, strongly negates transfer of ownership of
the goods to the petitioner.

2B 16-17 SALES AND LEASE Page 32


4. SPS FERNANDO AND LOURDES VILORIA VS CONTINENTAL AIRLINES

GR No. 188288 January 16, 2012

Reyes, J.:

FACTS:

On or about July 21, 1997 and while in the United States, Fernando purchased for himself and his wife, Lourdes,
two (2) round trip airline tickets from San Diego, California to Newark, New Jersey on board Continental Airlines.
Fernando purchased the tickets at US$400.00 each from a travel agency called Holiday Travel and was attended to
by a certain Margaret Mager (Mager). According to Spouses Viloria, Fernando agreed to buy the said tickets after
Mager informed them that there were no available seats at Amtrak, an intercity passenger train service provider in
the United States.

Fernando opted to request for a refund. Mager, however, denied his request as the subject tickets are non -
refundable and the only option that Continental Airlines can offer is the re -issuance of new tickets. Fernando
decided to reserve two (2) seats with Frontier Air. Upon inquiry in Amtrak, Fernando was told that there are seats
available and he can travel on Amtrak anytime and any day he pleased. Fernando then purchased two (2) tickets for
Washington, D.C. In a letter dated March 24, 1998, Continental Micronesia denied Fernandos request for a refund
and advised him that he may take the subject tickets to any Continental ticketing location for the re -issuance of new
tickets. Continental Micronesia informed Fernando that the subject tickets may be used as a form of payment for the
purchase of another Continental ticket, albeit with a re-issuance fee. Thereafter, In a letter dated June 21, 1999,
Fernando demanded for the refund of the subject tickets as he no longer wished to have them rep laced. Fernando
claimed that CAIs acts constitute a breach of its undertaking under its March 24, 1998 letter.

Hence, a complaint was filed against the respondents. Among other things, Fernando asserts that Mager
was a sub-agent of Holiday Travel who was in turn a ticketing agent of Holiday Travel who was in turn a ticketing
agent of Continental Airlines. Proceeding from this premise, they contend that Continental Airlines should be held
liable for the acts of Mager. The Regional Trial Court rendered a judgment in favor of the plaintiff ordering CAI to
refund the money they used to purchase the ticket with legal interest from July 21, 1997 and for damages. On
appeal, the Court of Appeals reversed the lower courts judgment. CA held that the elements of ag ency are not
attendant under the circumstances stated. It ruled that. As the basis of agency is representation, there must be, on
the part of the principal, an actual intention to appoint, an intention naturally inferable from the principals words or
actions. In the same manner, there must be an intention on the part of the agent to accept the appointment and act
upon it. Absent such mutual intent, there is generally no agency We have perused the evidence and documents so
far presented. We find nothing except bare allegations of plaintiffs -appellees that Mager/Holiday Travel was acting
in behalf of Continental Airlines. From all sides of legal prism, the transaction in issue was simply a contract of sale,
wherein Holiday Travel buys airline tickets from Continental Airlines and then, through its employees, Mager
included, sells it at a premium to clients.

ISSUE

Whether or not it is a contract of agency to sell

2B 16-17 SALES AND LEASE Page 33


HELD

Contrary to the findings of the CA, all the elements of an agency exist in this case. Tha t CAI admits the authority of
Holiday Travel to enter into contracts of carriage on its behalf is easily discernible from its February 24, 1998 and
March 24, 1998 letters. Further, the Court held that, the decisions say the transfer of title or agreement to transfer it
for a price paid or promised is the essence of sale. If such transfer puts the transferee in the attitude or position of an
owner and makes him liable to the transferor as a debtor for the agreed price, and not merely as an agent who must
account for the proceeds of a resale, the transaction is a sale; while the essence of an agency to sell is the delivery to
an agent, not as his property, but as the property of the principal, who remains the owner and has the right to control
sales, fix the price, and terms, demand and receive the proceeds less the agent's commission upon sales made.

CAI is the one bound by the contracts of carriage embodied by the tickets being sold by Holiday Travel on its
behalf. It is undisputed that CAI and not Holiday Travel who is the party to the contracts of carriage executed by
Holiday Travel with third persons who desire to travel via Continental Airlines, and this conclusively indicates the
existence of a principal-agent relationship. That the principal is bound by all the obligations contracted by the agent
within the scope of the authority granted to him is clearly provided under Article 1910 of the Civil Code and this
constitutes the very notion of agency.

2B 16-17 SALES AND LEASE Page 34


CONTRACT FOR A PIECE OF WORK

1. CIR VS ENGINEERING AND SUPPLY CO

64 SCRA 590

FACTS

Engineering Equipment and Supply Co. (Engineering for short), a domestic corporation, is an engineering and
machinery firm. As operator of an integrated engineering shop, it is engaged, among others, in the de sign and
installation of central type air conditioning system.
Engineering equipment was accused of tax evasion by misdeclaring its imported articles and failing to pay the
correct percentage tax due.
The court of tax appeals ruled that respondent is as a contractor and not a manufacturer and as such, respondent is
declared exempt from the deficiency manufacturers sales tax covering the period of xxx.

Commissioner of internal revenue appealed.

ISSUE

Whether respondent is a contractor or a manufacturer as would make it liable for manufacturers sales tax.

HELD

The arguments of both the Engineering and the Commissioner call for a clarification of the term contractor as well
as the distinction between a contract of sale and contract for furnishing services, labor and materials. The distinction
between a contract of sale and one for work, labor and materials is tested by the inquiry whether the thing
transferred is one not in existence and which never would have existed but for the order of the party desiring to
acquire it, or a thing which would have existed and has been the subject of sale to some other persons even if the
order had not been given.If the article ordered by the purchaser is exactly such as the plaintiff makes and keeps on
hand for sale to anyone, and no change or modification of it is made at defendant's request, it is a contract of sale,
even though it may be entirely made after, and in consequence of, the defendants order for it.

Our New Civil Code, likewise distinguishes a contract of sale from a contract for a piece of work in Art. 1467.The
word "contractor" has come to be used with special reference to a person who, in the pursuit of the independent
business, undertakes to do a specific job or piece of work for other persons, using his own means and methods
without submitting himself to control as to the petty details.The true test of a contractor as was held in the cases of
Luzon Stevedoring Co., vs. Trinidad, 43, Phil. 803, 807-808, and La Carlota Sugar Central vs. Trinidad, 43, Phil.
816, 819, would seem to be that he renders service in the course of an independent occupation, representing the will
of his employer only as to the result of his work, and not as to the means by which it is accomplished.

Respondent, fabricates, assembles, s upplies and installs in the buildings of its various customers the central type air
conditioning system; prepares the plans and specifications therefor which are distinct and different from each other;
the air conditioning units and spare parts or accessories thereof used by petitioner are not the window type of air

2B 16-17 SALES AND LEASE Page 35


conditioner which are manufactured, assembled and produced locally for sale to the general market; and the
imported air conditioning units and spare parts or accessories thereof are supplied and installed by petitioner upon
previous orders of its customers conformably with their needs and requirements.The facts and circumstances
aforequoted support the theory that Engineering is a contractor rather than a manufacturer.

2B 16-17 SALES AND LEASE Page 36


2. DEL MONTE PHILS VS NAPOLEON ARAGONES

GR No. 153033 June 23, 2005

FACTS: On September 18, 1988, herein petitioner Del Monte Philippines Inc. (DMPI) entered into an Agreement
with MEGA-WAFF, represented by Managing Principal Edilberto Garcia (Garcia), whereby the latter undertook

the supply and installation of modular pavement at DMPIs condiments warehouse at Cagayan de Oro City within
60 calendar days from signing of the agreement. To source its supply of concrete blocks to be installed on the
pavement of the DMPI warehouse, MEGA-WAFF, as CONTRACTOR represented by Garcia, entered into a
Supply Agreement with Dynablock Enterprises, represented by herein respondent Aragones, as SUPPLIER.

After the installation of the pavement in the warehouse, Arago nes later on demand from MEGA-WAFFthe full
payment of the concrete blocks on which he failed to collect. Aragones later failed to collect from MEGA -WAFF
the full payment of the concrete blocks. He thus sent DMPI a letter dated March 10, 1989, received

by the latter on March 13, 1989, advising it of MEGA-WAFFs unpaid obligation and requesting it to earmark and
withhold the amount of P188,652.65 from [MEGA-WAFFs] billing to be paid directly to him lest Garcia
collects and fails to pay him.

ISSUE: Whether or not it was a sale or piece of work.

HELD: Under Art. 1467 then of the Civil Code which provides:

ART. 1467. A contract for the delivery at a certain price of an article which the vendor in the ordinary
course of his business manufactures or procures for the general market, whether the same is on hand at the
time or not, is a

contract of sale, but if the goods are to be manufactured specially for the customer and upon his special
order, and not for the general market, it is a contract for a piece o f work.

The Supply Agreement was in the nature of a contract for a piece of work. Following Art. 1729 of the Civil Code
which provides:

ART. 1729. Those who put their labor upon or furnish materials for a piece of work undertaken by the
contractor have an action against the owner up to the amount owing from the latter to the contractor at the
time the claim is made.

2B 16-17 SALES AND LEASE Page 37


PERFECTION OF CONTRACT OF SALE

1. ANTONIO S. LIM, JR., represented by his attorney-in-fact, PAZ S. LIM vs. VICTOR K. SAN and
ELINDO LO

G.R. No. 159723. September 9, 2004

Ynares-Santiago, J.

FACTS:

Petitioner Antonio S. Lim, Jr., represented by his mother, Paz S. Lim, as attorney -in-fact, filed a
complaintbefore the Regional Trial Court of Davao City seeking the annulment of a Deed of Abs olute Sale
involving a parcel of land purportedly executed by Paz S. Lim in favor of her brother, respondent Victor K. San.
Plaintiff is an owner of a parcel of land situated at Bajada, Davao City. Constructed on the afore -cited parcel of land
is a fourteen (14) doors commercial building, and that defendant is paying an annual lease of ONE HUNDRED
THOUSAND (P100,000.00) PESOS to the herein plaintiff.

On May 29, 1991, the herein defendant taking undue advantage of the depressed mental state of plaintiffs
Attorney-in-Fact, brought about by the demise of her late husband, Dr. Antonio A. Lim Sr., caused some papers for
her to sign, which later turn out to be an Absolute Deed of Sale. Said document was notarized despite non -
appearance of the Attorney-in-Fact before the Notary Public.

That no consideration was ever paid, much less received by the plaintiff or by his Attorney -in-Fact. Simply
put, the Deed of Absolute Sale was void ab initio for lack of consideration and for lack of a valid consent. After
the signing of the aforecited Deed of Sale with its attendant legal flaws and infirmities, plaintiffs Title was
transferred in the name of the defendant, Victor K. San. Knowing that he is holding an infirmed Title, defendant,
Victor K. San is now in the process of selling the aforecited property including the commercial building erected
thereon to any third person; and that the defendant had already caused the cancellation of the Mother Title.

Respondent Victor K. San denied all the allegations of the petitioner. He alleged that the parcel of land
covered by TCT No. T-165010 of the Registry of Deeds of Davao City and registered in his name was validly and
regularly issued. He further claimed that he does not have any lease contract with the petitioner with respect to the
contested property and does not pay any monthly rental over the same. Moreover, respondent claimed that there was
full payment of the consideration of P264,450.00 for the subject property.

After trial on the merits, the Regional Trial Court of Davao City rendered a decision dismissing the
complaint. Petitioner appealed to the Court of Appeals which affirmed the judgment of the trial court in toto.

ISSUE:

Whether or not there was a valid and perfected contract of sale.

2B 16-17 SALES AND LEASE Page 38


HELD

YES.

A contract is a meeting of minds between two persons whereby one binds himself, with respect to the other, to give
something or to render some service. It has three essential elements, or those without which there can be no contract
consent, subject matter and cause. Knowledge of these essential elements is material because the perfection stage or
the birth of the contract only occurs when the parties to a contract agree upon the essential elements of the same. A
contract of sale is consensual; as such it is perfected by mere consent. Consent is essential for the existence of a
contract, and where it is wanting, the contract is non -existent. Consent in contracts presupposes the following
requisites: (1) it should be intelligent or with an exact notion of the matter to which it refers; (2) it should be free;
and (3) it should be spontaneous. Intelligence in consent is vitiated by error; freedom by violence, intimidation or
undue influence; and spontaneity by fraud. Thus, a contract where consent is given through mistake, violenc e,
intimidation, undue influence or fraud is voidable.

Contrary to the allegations of the petitioner that the consent of his attorney -in-fact to the deed of sale was vitiated, a
perusal of the records of this case showed that the petitioner failed to estab lish that violence, intimidation and undue
influence vitiated the consent of Paz S. Lim to the deed of sale pertaining to the subject property.

2B 16-17 SALES AND LEASE Page 39


2. SWEDISH MATCH AB VS CA (ALSO: RE: FORM)

GR NO. 128120 October 20, 2004

PARTIES:

STORA - parent company of SMAB

SMAB - Swedish match SB PHIMCO - Phimco Industries PHILS- Provident Tree Farms, Inc., and OTT/Louie

SMNV- Swedish Match NV of Netherlands

SMSA - Swedish Match Sociedad Anonimas with ENRIQUEZ as VP

SMSA - the management company of the Swedish Match group

Litonjua - ALS Management & Development Corporation and respondent Antonio Litonjua the president and
general manager of ALS.

Rossi - Chief Executive Officer of SMAB

FACTS
- Swedish Match, AB (hereinafter SMAB) is a corporation organized under the laws of Sweden not doing business
in the Philippines.

- SMAB, however, had three subsidiary corporations in the Philippines, all organized under Philippine laws, to wit:.
Phimco and Phils.

- Sometime in 1988, STORA, decided to sell SMAB of Sweden and the latters worldwide match, lighter and shaving
products operation to SMNV, a corporation organized and existing under the laws of Netherlands.

- SMNV initiated steps to sell the worldwide match and lighter b usinesses while retaining for itself the shaving
business.

- SMNV adopted a two-pronged strategy,

- the first being to sell its shares in Phimco Industries, Inc. and a match company in Brazil, which proposed sale
would stave-off defaults in the loan covenants of SMNV with its syndicate of lenders.

- The other move was to sell at once or in one package all the SMNV companies worldwide which were engaged in
match and lighter operations thru a global deal (hereinafter, global deal).

- SMSA was commissioned and granted full powers to negotiate by SMNV, with the resulting transaction, however,
made subject to final approval by the board.

- Enriquez was held under strict instructions that the sale of Phimco shares should be executed on or before 30 June
1990, in view of the tight loan covenants of SMNV.

2B 16-17 SALES AND LEASE Page 40


- Enriquez came to the Philippines in November 1989 and informed the Philippine financial and business circles that
the Phimco shares were for sale.

- Among those interested were AFP Retirement and Separation Benefits System and Litonjua

- Litonjua submitted to SMAB a firm offer to buy all of the latters shares in Phimco and all of Phimcos shares in
Provident Tree Farm, Inc. and OTT/Louie (Phils.), Inc. for the sum of P750,000,000.00.

- Rossi, in its letter dated 1 December 1989, thanked respondents for their interest in the Phimco shares.

- Rossi informed respondents that their price offer was below their expectations but urged them to undertake a
comprehensive review and analysis of the value and profit potentials o f the Phimco shares, with the assurance that
respondents would enjoy a certain priority although several parties had indicated their interest to buy the shares.

- Thereafter, an exchange of correspondence ensued between petitioners and respondents regarding the projected
sale of the Phimco shares.

- Litonjua offered to buy the disputed shares, excluding the lighter division for US$30.6 million, which per another
letter of the same date was increased to US$36 million.

- Litonjua stressed that the bid amount could be adjusted subject to availability of additional information and audit
verification of the company finances.

- Responding to Litonjuas offer, Rossi sent his letter, informing the former that;

1. ALS should undertake a due diligence process or pre-acquisition audit and review of the draft contract for the
Match and Forestry activities of Phimco at ALS convenience.

2. at the completion of the due diligence process, ALS should submit its final offer in US dollar terms not later than
30 June 1990, for the shares of SMAB corresponding to ninety-six percent (96%) of the Match and Forestry
activities of Phimco.

3. in case the global deal presently under negotiation for the Swedish Match Lights Group would materialize, SMAB
would reimburse up to US$20,000.00 of ALS costs related to the due diligence process.

- Litonjua expressed disappointment at the apparent change in SMABs approach to the bidding process.

- He pointed out that in their 4 June 1990 meeting, he was advised that one final bidder would be selec ted from
among the four contending groups as of that date and that the decision would be made by 6 June 1990.

- He criticized SMABs decision to accept a new bidder who was not among those who participated in the 25 May
1990 bidding.

- He informed Rossi that it may not be possible for them to submit their final bid on 30 June 1990, citing the advice
to him of the auditing firm that the financial statements would not be completed until the end of July.

- Litonjua added that he would indicate in their final offer more specific details of the payment mechanics and
consider the possibility of signing a conditional sale at that time.

- Two days prior to the deadline for submission of the final bid, Litonjua again advised Rossi that they would be
unable to submit the final offer by 30 June 1990, considering that the acquisition audit of Phimco and the review of
the draft agreements had not yet been completed.

2B 16-17 SALES AND LEASE Page 41


- He said, however, that they would be able to finalize their bid on 17 July 1990 and that in case their bid would turn
out better than any other proponent, they would remit payment within ten (10) days from the execution of the
contracts.

- Enriquez sent notice to Litonjua that they would be constrained to entertain bids from other parties in view of
Litonjuas failure to make a firm commitment for the shares of Swedish Match in Phimco by 30 June 1990.

- Rossi informed Litonjua that on 2 July 1990, they signed a conditional contract with a local group for the disposal
of Phimco. He told Litonjua that his bid would no longer be considered unless the local group would fail to
consummate the transaction on or before 15 September1990.

- Apparently irked by SMABs decision to junk his bid, Litonjua promptly responded by letter dated 4 July 1990.
Contrary to his prior manifestations, he asserted that, for all intents and purposes, the US$36 million bid which he
submitted on 21 May 1990 was their final bid based on the financial statements for the year 1989.

- He pointed out that they submitted the best bid and they were already finalizing the terms of the sale. He stressed
that they were firmly committed to their bid of US$36 million and if ever there would be adjustments in the bid
amount, the adjustments were brought about by SMABs subsequent disclosures and validated a ccounts, such as the
aspect that only ninety-six percent (96%) of Phimco shares was actually being sold and not one -hundred percent
(100%).

- More than two months from receipt of Litonjuas last letter, Enriquez sent a fax communication to the former,
advising him that the proposed sale of SMABs shares in Phimco with local buyers did not materialize. Enriquez then
invited Litonjua to:

A. resume negotiations with SMAB for the sale of Phimco shares. He indicated that SMAB would be prepared to
negotiate with ALS on an exclusive basis for a period of fifteen (15) days from 26 September 1990 subject to the
terms contained in the letter.

B. Additionally, Enriquez clarified that if the sale would not be completed at the end of the fifteen (15)-day period,
SMAB would enter into negotiations with other buyers.

- Shortly thereafter, Litonjua sent a letter expressing his objections to the totally new set of terms and conditions for
the sale of the Phimco shares.

- He emphasized that the new offer constituted an attempt to reopen the already perfected contract of sale of the
shares in his favor. He intimated that he could not accept the new terms and conditions contained therein.

- Respondents filed before RTC complaint for SPECIFIC PERFORMANCE with DAMAGES with
PRELIMINA RY INJUNCTION

- Respondents prayed that petitioners be enjoined from selling or transferring the Phimco shares, or otherwise
implementing the sale or transfer thereof, in favor of any person or entity other than respondents, and that any such
sale to third parties be annulled and set aside.

- Respondents also asked that petitioners be ordered to execute all documents or instruments and perform all acts
necessary to consummate the sales agreement in their favor.

- RTC dismissed respondents complaint.

2B 16-17 SALES AND LEASE Page 42


RTC RULING

- It ruled that there was no perfected contract of sale between petitioners and respondents.

- Petitioners did not accept the bid offer of respondents as the letter was a mere invitation for respondents to conduct
a due diligence process or pre-acquisition audit of Phimcos match and forestry operations to enable them to submit
their final offer on 30 June 1990.

- Assuming that respondents bid was favored by an oral acceptance made in private by officers of SMAB, the trial
court noted, such acceptance was merely preparatory to a formal acceptance by the SMAB the acceptance that
would eventually lead to the execution and signing of the contract of sale. Moreover, the court noted that
respondents failed to submit their final bid on the deadline set by petitioners.

CA RULING:

- Reversed ruling of RTC

- The Court of Appeals clarified, however, that by reversing the appealed decision it was not thereby declaring that
respondents are entitled to the reliefs prayed for in their complaint, but only that the cas e should not have been
dismissed on the ground of unenforceability under the Statute of Frauds. It ordered the remand of the case to the trial
court for further proceedings.

ISSUE:

Whether or not there was a perfected contract of sale between petitioners and respondents with respect to the
Phimco shares.

HELD:

PETITION WAS GRANTED

- A negotiation is formally initiated by an offer. A perfected promise merely tends to insure and pave the way for the
celebration of a future contract. An imperfect promise (policitacion), on the other hand, is a mere unaccepted offer.

- Public advertisements or solicitations and the like are ordinarily construed as mere invitations to make offers or
only as proposals. At any time prior to the perfection of the contract, either negotiating party may stop the
negotiation.

- The offer, at this stage, may be withdrawn; the withdrawal is effective immediately after its manifestation, such as
by its mailing and not necessarily when the offeree learns of the withdrawal.

- An offer would require, among other things, a clear certainty on both the object and the cause or consideration of
the envisioned contract.

- Consent in a contract of sale should be manifested by the meeting of the offer and the acceptance upon the thing
and the cause which are to constitute the contract. The offer must be certain and the acceptance absolute. A qualified
acceptance constitutes a counter-offer.

2B 16-17 SALES AND LEASE Page 43


- Quite obviously, Litonjuas letter, proposing the acquisition of the Phimco shares for US$36 million was merely an
offer.

- This offer, however, in Litonjuas own words, is understood to be subject to adjustment on the basis of an audit of
the assets, liabilities and net worth of Phimco and its subsidiaries and on the final negotiation between ourselves.

- Litonjua repeatedly stressed in his letters that they would not be able to submit their final bid by 30 June 1990.

- With indubitable inconsistency, respondents later claimed that for all intents and purposes, the US$36 million was
their final bid.

- If this were so, it would be inane for Litonjua to state, as he did, in his letter that they would be in a position to
submit their final bid only on 17 July 1990.

- The lack of a definite offer on the part of respondents could not possibly serve as the basis of th eir claim that the
sale of the Phimco shares in their favor was perfected, for one essential element of a contract of sale was obviously
wanting the price certain in money or its equivalent. The price must be certain, otherwise there is no true consent
between the parties.

- There can be no sale without a price.

- Granting arguendo, that the amount of US$36 million was a definite offer, it would remain as a mere offer in the
absence of evidence of its acceptance.

- To produce a contract, there must be acceptance, which may be express or implied, but it must not qualify the
terms of the offer. The acceptance of an offer must be unqualified and absolute to perfect the contract.

- In other words, it must be identical in all respects with that of the offer so as to produce consent or meeting of the
minds.

- Respondents failure to submit their final bid on the deadline set by petitioners prevented the perfection of the
contract of sale. It was not perfected due to the absence of one essential element which was the price certain in
money or its equivalent.

2B 16-17 SALES AND LEASE Page 44


3. TRADERS ROYAL BANK VS CUISON LUMB ER CO INC

GR NO. 174286 June 5, 2009

FACTS

On July 14, 1978 and December 9, 1979, respectively, CLCI, through its then president, Roman Cuison Sr., obtained
two loans from the bank. The loans were secured by a real estate mortgage over a parcel of land. CLCI failed to pay
the loan, prompting the bank to extrajudicially foreclose the mortgage on the subject property. The bank was
declared the highest bidder at the public auction that followed, conducted on August 1, 1985.

CLCI manifested its intention to restructure its loan obligations and to repurchase the subject property. On July 31,
1986, Mrs. Cuison, the widow and administratrix of the estate of Roman Cuison Sr., wrote the banks Officer-in-
Charge, Remedios Calaguas, a letter indicating her offered terms of repurchase

CLCI paid the bank P50,000.00 (on August 8, 1986) and P85,000.00 (on September 3, 1986). The bank received and
regarded these amounts as earnest money for the repurchase of the subject property.

CLCI failed to comply with the conditions brought by the bank for the repurchase of the land notwithstanding the
extensions of time given by the latter. Nevertheless, CLCI tendered, on February 3, 1987, a check for P135,091.57
to cover fifty percent (50%) of the twenty percent (20%) bid price. The check, however, was returned for
insufficiency of funds. On May 13, 1987, CLCI tendered an additional P50,000.00. On May 29, 1987, the bank sent
Atty. Cuison a letter informing him that the P185,000.00 CLCI paid was not a deposit, but formed part of the earnest
money under the TRB Repurchase Agreement. On August 28, 1987, Atty. Cuison, by letter, requested that CLCIs
outstanding obligation of P1,221,075.61 (as of July 31, 1987) be reduced to P1 million, and the amount of
P221,075.61 be condoned by the bank. To show its commitment to the request, CLCI paid the bank P100,000.00
and P200,000.00 on August 28, 1987. The bank credited both payments as earnest money.

On September 30, 1988, the bank informed CLCI that it would resell the subject property at an offered price of P3
million, and gave CLCI 15 days to make a formal offer; otherwise, the bank would sell the subject property to third
parties. On October 26, 1988, CLCI offered to repurchase the subject property for P1.5 million, g iven that it had
already tendered the amount of P400,000.00 as earnest money.

CLCI subsequently claimed that the bank breached the terms of repurchase, as it had wrongly considered its
payments (in the amounts of P140,485.18, P200,000.00 and P100,000.00) as earnest money, instead of applying
them to the purchase price. Through its counsel, CLCI demanded that the bank rectify the repurchase agreement to
reflect the true consideration agreed upon for which the earnest money had been given. The bank did not act on the
demand. Instead, it informed CLCI that the amounts it received were not earnest money, and that the bank was
willing to return these sums, less the amounts forfeited to answer for the unremitted rentals on the subject property.

2B 16-17 SALES AND LEASE Page 45


ISSUE

Whether or not a perfected contract of repurchase existed and can be enforced between the parties

HELD

Petition granted. There was a meeting of the minds between the parties on the offer and acceptance for the
repurchase of the subject property.

Under the law, a contract is perfected by mere consent, that is, from the moment that there is a meeting of the offer
and the acceptance upon the thing and the cause that constitute the contract. The law requires that the offer must be
certain and the acceptance absolute and unqualified. An acceptance of an offer may be express and implied; a
qualified offer constitutes a counter-offer. Case law holds that an offer, to be considered certain, must be definite,
while an acceptance is considered absolute and unqualified when it is identical in all respects with that of the offer
so as to produce consent or a meeting of the minds. We have also previously held that the ascertainment of whether
there is a meeting of minds on the offer and acceptance depends on the circumstances su rrounding the case.

The clear and neat principle is that the offer must be certain and definite with respect to the cause or consideration
and object of the proposed contract, while the acceptance of this offer express or implied must be unmistakable,
unqualified, and identical in all respects to the offer. The required concurrence, however, may not always be
immediately clear and may have to be read from the attendant circumstances; in fact, a binding contract may exist
between the parties whose minds have met, although they did not affix their signatures to any written document.

The facts of the present case, although ambivalent in some respects, point on the whole to the conclusion that both
parties agreed to the repurchase of the subject property.

A reading of the petitioners letter of October 20, 1986 informing CLCI that the banks board of directors passed a
resolution for the repurchase of [your] property shows that the tenor of acceptance, except for the repurchase price,
was subject to conditions not identical in all respects with the CLCIs letter-offer of July 31, 1986. In this sense, the
banks October 20, 1986 letter was effectively a counter-offer that CLCI must be shown to have accepted absolutely
and unqualifiedly in order to give birth to a perfected contract. Evidence exists showing that CLCI did not sign any
document to show its conformity with the banks counter-offer. Testimony also exists explaining why CLCI did not
sign; Atty. Cuison testified that CLCI did not agree with the implementat ion of the repurchase transaction since the
bank made a wrong computation. These indicators notwithstanding, we find that CLCI accepted the terms of the
TRC Repurchase Agreement and thus unqualifiedly accepted the banks counter-offer under the TRB Repurchase
Agreement and, in fact, partially executed the agreement as shown from the following undisputed evidence:

2B 16-17 SALES AND LEASE Page 46


(a) The letter-reply dated November 29, 1986 of Atty. Cuison, as president and general manager of CLCI, to the
bank (in response to the banks demand letter dated November 27, 1986 to pay 20% of the bid price); CLCI
requested an extension of time, until the end of December 1986, to pay its due obligation;

(b) Mrs. Cuisons letter-reply of February 3, 1987 (to the banks letter of January 13, 1987) showed that she
acknowledged CLCIs failure to comply with its requested extension and proposed a new payment scheme that
would be reasonable given CLCIs critical economic difficulties; Mrs. Cuizon tendered a check for P135,091.57,
which represented 50% of the 20% bid price;

(c) The CLCIs continuous payments of the repurchase price after their receipt of the banks letter of October 20,
1986;

(d) CLCIs possession of the subject property pursuant to paragraph 5 of the TRB Repurchase Ag reement,
notwithstanding the absence of a signed contract to sell between the parties;

2B 16-17 SALES AND LEASE Page 47


4. ARTURO R. ABALOS vs. DR. GALICANO S. MACATANGAY, JR.

G.R. No. 155043. September 30, 2004

Tinga, J

FACTS:

Spouses Arturo and Esther Abalos are the registered owners of a parcel of land with improvements located
at Azucena St., Makati City. Armed with a Special Power of Attorney (SPA) dated June 2, 1988, purportedly issued
by his wife, Arturo executed a Receipt and Memorandum of Agreement (RMOA) dated October 17, 1989, in favor
of respondent, binding himself to sell to respondent the subject property and not to offer the same to any other
party within thirty (30) days from date.

Arturo acknowledged receipt of a check from respondent in the amo unt of Five Thousand Pesos
(P5,000.00), representing earnest money for the subject property, the amount of which would be deducted from the
purchase price of One Million Three Hundred Three Hundred Thousand Pesos (P1,300,000.00).

Further, the RMOA stated that full payment would be effected as soon as possession of the property shall
have been turned over to respondent.Arturos wife, Esther, executed a Special Power of Attorney dated October 25,
1989, appointing her sister, Bernadette Ramos, to act for and in her behalf relative to the transfer of the property to
respondent.

Then, respondent caused the annotation of his adverse claim on the title of the spouses to the property on
November 14, 1989.

On November 16, 1989, respondent sent a letter to Arturo and Esther informing them of his readiness and
willingness to pay the full amount of the purchase price.Respondent demanded upon the spouses to comply with
their obligation to turn over possession of the property to him.

On the same date, Esther, through her attorney-in-fact, executed in favor of respondent, a Contract to Sell
the property to the extent of her conjugal interest therein for the sum of six hundred fifty thousand pesos
(P650,000.00) less the sum already received by her and Arturo. Esther agreed to surrender possession of the
property to respondent within twenty (20) days from November 16, 1989, while the latter promised to pay the
balance of the purchase price in the amount of one million two hundred ninety thousand pesos (P1,290,000.00) after
being placed in possession of the property. Esther also obligated herself to execute and deliver to respondent a deed
of absolute sale upon full payment.

Dec 1989, resp. informed the spouses that he had set aside the amount (P1,290,000.00) as evidenced by
Citibank Check as full payment of the purchase price. Arturo and Esther failed to deliver the property which
prompted respondent to cause the annotation of another adverse claim.

Respondent filed a complaint for specific performance with damages against petitioners.

RTC dismissed the complaint for specific performance. It ruled that the Special Power of Attorney (SPA) ostensibly
issued by Esther in favor of Arturo was void as it was falsified. Hence, the court concluded that the SPA c ould not
have authorized Arturo to sell the property to respondent. The trial court also noted that the check issued by
respondent to cover the earnest money was dishonored due to insufficiency of funds and while it was replaced

2B 16-17 SALES AND LEASE Page 48


with another check by respondent, there is no showing that the second check was issued as payment for the earnest
money on the property.

CA: reversed it. It ruled that the SPA in favor of Arturo, assuming that it was void, cannot affect the transaction
between Esther and respondent.thus, the sale of the property to respondent was effected.

ISSUE:

Whether or not petitioner may be compelled to convey the prop to respondent under the terms of RMOA and the
Contract TO SELL

HELD:

NO .The petitioner cannot be compelled to convey the pro perty to respondent.

The nullity of the RMOA as a contract of sale emanates not only from lack of Esthers consent thereto but also from
want of consideration and absence of respondents signature thereon. Such nullity cannot be obliterated by
Esthers subsequent confirmation of the putative transaction as expressed in the Contract to Sell. Under the law, a
void contract cannot be ratified and the action or defense for the declaration of the inexistence of a contract does not
prescribe. A void contract produces no effect either against or in favor of anyoneit cannot create, modify or
extinguish the juridical relation to which it refers.

True, in the Contract to Sell, Esther made reference to the earlier RMOA executed by Arturo in favor of respondent.
However, the RMOA which Arturo signed is different from the deed which Esther executed through her attorney -in-
fact. For one, the first is sought to be enforced as a contract of sale while the second is purportedly a contract to
sell only. For another, the terms and conditions as to the issuance of title and delivery of possession are divergent.

The congruence of the wills of the spouses is essential for the valid disposition of conjugal property. Where the
conveyance is contained in the same document which bears the conformity of both husband and wife, there could be
no question on the validity of the transaction. However, in the case at bar the signatures of spouses appeared in two
different documents. however, the two documents do not contain the same terms and c onditions,therefore, a valid
transaction could not have arisen.

Arturo and Esther appear to have been married before the effectivity of the Family Code. There being no indication
that they have adopted a different property regime, their property relations would automatically be governed by
the regime of conjugal partnership of gains.

The subject land which had been admittedly acquired during the marriage of the spouses forms part of their conjugal
partnership.

Under the Civil Code, the husband is the administrator of the conjugal partnership.

Even if he is statutorily designated as administrator of the conjugal partnership, he cannot validly alienate or
encumber any real property of the conjugal partnership without the wifes consent. Similarly, the wife cannot
dispose of any property belonging to the conjugal partnership without the conformity of the husband. The law is

2B 16-17 SALES AND LEASE Page 49


explicit that the wife cannot bind the conjugal partnership without the husbands consent, except in cases provided by
law.

Since Article 166 of the Civil Code requires the consent of the wife before the husband may alienate or encumber
any real property of the conjugal partnership, it follows that acts or transactions executed against this mandatory
provision are void except when the law itself authorizes their validity.

In all instances, the present law specifically requires the written consent of the other spouse, or authority of the
court for the disposition or encumbrance of conjugal partnership property without which, the disposition o r
encumbrance shall be void.

2B 16-17 SALES AND LEASE Page 50


5. XYST Corporation vs. DMC Urban Properties Development Inc.

G.R. No. 171968 July 31, 2009

Quisumbing, J

FACTS:

DMC Urban Properties Development, Inc (hereinafter DMC) and Citibank N.A. entered into an agreement whereby
the former would take part in the construction of Citibank Tower. The said agreement allocated in favor of DMC the
18th floor of the building with the condition that DMC shall not transfer any portion of the floor or rights or interests
thereto prior to the completion of the building without the written consent of Citibank N.A. Later, DMC found a
prospective buyer, Saint Agen Et Fils Limited (hereinafter SAEFL), which is a foreign corporation represented by
one William Seitz. This was done despite the fact that construction was not yet completed. In a letter dated
September 14, 1994, SAEFL accepted DMCs offer to sell. The letter included a property description and terms of
payment. In September 16, 1994, SAEFL sent a letter obliging DMC to cause Citibank N.A. to give its consent and
enter into a contract to sell with SAEFL. Seitz was informed that the 18th floor is not available for foreign
acquisition and so XYST Corporation (hereinafter XYST), a domestic corporation was substituted. XYST then paid
reservation fee but was later advised by DMC that the signing of the formal contract will not take place since
Citibank N.A. opted to exercise its right of first refusal. XYST and DMC agreed that if Citibank N.A. fails to
purchase the 18th floor on the agreed date, the same should be sold to XYST. Citibank N.A. did not exercise its right
of first refusal but it reminded DMC that the sale of the floor must be consistent with the documents adopted by the
co-founders of the project. Hence a copy of a pro-forma Contract to Sell was given to DMC and a copy of this was
forwarded to XYST. XYST made amendments to the pro-forma contract and was allowed by DMC to directly
negotiate with Citibank N.A. to facilitate the transaction. But Citibank N.A. refused t o concur with the changes
imposed by XYST hence DMC decided to call off the deal and returned the reservation fee of P1,000,000.00 to
XYST. A complaint was filed in the RTC by XYST for specific performance but this was dismissed.

ISSUE

Whether there is a perfected contract between DMC and XYST.

HELD

No contract was perfected. A contract to sell was entered into by the parties. In their contract to sell, the element of
consent is lacking since the acceptance by XYST is not absolute, hence, no contract of s ale existed between the
parties. The terms, conditions, and amendments which XYST tried to impose were proof of qualified acceptance.
XYST and DMC were still in the negotiation stage when the latter called off the deal. There are three stages to a
contract: (1) preparation or negotiation, (2) perfection, and (3) consummation. Negotiation begins from the time the
prospective contracting parties manifest their interest in the contract and ends at the moment of agreement of the
parties. The perfection or birth of the contract takes place when the parties agree upon the essential elements of the
contract. The last stage is the consummation of the contract wherein the parties fulfil or perform the terms agreed
upon in the contract, culminating in the extinguishment thereof. Citibank N.A.s consent to the intended sale was not
obtained since it did not conform to the amendments introduced by XYST to the pro -forma contract. There was only
an offer and a counter-offer that did not sum up to a final arrangement and no meeting of the minds was established.

2B 16-17 SALES AND LEASE Page 51


Therefore, since the element of consent is absent, there is no contract to speak of. Where the parties merely
exchanged offers and counter-offers, no agreement or contract is perfected. On holding that the P1,000,000. 00
reservation fee is not earnest money, the Supreme Court held that earnest money applies (only) to perfected sale.
In this case, no contract was perfected since consent was lacking. Hence, the contention that the P1,000,000.00
reservation fee it pad is in the nature of earnest money or down payment and shall be considered part of the price
and proof of perfection of contract is bereft of merit.

2B 16-17 SALES AND LEASE Page 52


6. PEDRO ROMAN vs. ANDRES GRIMALT

GR No. 2412. April 11, 1906.

FACTS

On July 2, 1904, counsel for Pedro Roman filed a complaint in the Court of First Instance of this city against
Andres Grimalt, praying that judgment be entered in his favor and

against the defendant (1) for the purchase price of the schooner Santa Marina, payable by inst allments in the manner
stipulated and for legal interest on the installments due on the dates set forth in the complaint, among others.

Counsel for plaintiff amended his complaint and alleged that, both parties, through one Fernando Agusti Pastor,
verbally agreed upon the sale of the said schooner that the defendant in a letter had agreed to purchase the said
schooner and offered to pay therefor in three installments of 500 pesos each, adding in his letter that if the plaintiff
accepted the plan of payment suggested by him the sale would become effective on the following day that plaintiff
had notified the defendant through Agusti Pastor that he accepted the plan of payment suggested by him and that
from that date the vessel was at his disposal, and offered to deliver the same at once to defendant if he so desired
that the contract having been closed and the vessel being ready for delivery to the purchaser, it was sunk about 3
o'clock p. m., June 25, in the harbor of Manila and is a total loss, as a resu lt of a severe storm and that on the 30th of
the same month demand was made upon the defendant for the payment of the purchase price of the vessel in the
manner stipulated and defendant failed to pay.

Defendant in his answer asked that the complaint be dismissed alleging that both parties met in a public
establishment of this city and the plaintiff personally proposed to the defendant the sale of the said vessel, the
plaintiff stating that the vessel belonged to him and that it was then in a seaworthy co ndition that defendant
accepted the offer of sale on condition that the title papers were found to be satisfactory, also that the vessel was in a
seaworthy condition that both parties then called on Calixto Reyes, a notary public, who, after examining th e
documents, informed them that they were insufficient to show the ownership of the vessel and to transfer title
thereto that plaintiff then promised to perfect his title and about June 23 called on defendant to close the sale, and
the defendant, believing that plaintiff had perfected his title, wrote to him on the 23d of June and set the following
day for the execution of the contract, but, upon being informed that plaintiff had done nothing to perfect his title, he
insisted that he would buy the vessel only when the title papers were perfected and the vessel duly inspected.

ISSUE

Whether or not the sale of the schooner was perfected

HELD

NO. A sale shall be considered perfected and binding as between vendor and vendee when they have agreed as to
the thing which is the object of the contract and as to the price, even though neither has been actually delivered.

2B 16-17 SALES AND LEASE Page 53


Ownership is not considered transmitted until the property is actually delivered and the purchaser has taken
possession of the same and paid the price agreed upon, in which case the sale is considered perfected. The sale of the
schooner was not perfected and the purchaser did not consent to the execution of the deed of transfer for the reason
that the title of the vessel was in the name of one Paulina Giron and not in the name of Pedro Roman, the alleged
owner. Roman promised, however, to perfect his title to the vessel, but he failed to do so. The papers presented by
him did not show that he was the owner of the vessel. If no contract of sale was ac tually executed by the parties the
loss of the vessel must be borne by its owner and not by a party who only intended to purchase it and who was
unable to do so on account of failure on the part of the owner to show proper title to the vessel and thus enab le them
to draw up the contract of sale. The vessel was sunk in the bay on the afternoon of the 25th of June, 1904, during a
severe storm and before the owner had complied with the condition exacted by the proposed purchaser, to wit, the
production of the proper papers showing that the plaintiff was in fact the owner of the vessel in question. The
defendant was under no obligation to pay the price of the vessel, the purchase of which had not been concluded. The
conversations had between the parties and the letter written by defendant to plaintiff did not establish a contract
sufficient in itself to create reciprocal rights between the parties.

2B 16-17 SALES AND LEASE Page 54


7. CIRILO PAREDES VS ESPINO

G.R. No. L-23351, March 13, 1968

Reyes, JBL, J

FACTS

Parades filed an action to compel Espino to execute a deed of sale and to pay damages. The complaint alleged that:

1. Espino "had entered into the sale" to Paredes of Lot No. 67 of the Puerto Princesa Cadastre at P4.00 a
square meter

2. The deal had been "closed by letter and telegram" but the actual execution of the deed of sale and
payment of the price were deferred to the arrival of defendant at Puerto Princesa

3. Espino upon arrival had refused to execute the deed of sale although Paredes was able and willing to
pay the price, and continued to refuse despite written demands of plaintiff

4. As a result, Paredes had lost expected profits from a resale of the property, and caused plaintiff mental
anguish and suffering, for which reason the complaint prayed for specific performance and damages.

Espino filed a motion to dismiss upon the ground that the complaint stated no cause of action, and that the Paredes
claim was unenforceable under the Statute of Frauds.

Paredes opposed in writing and annexed to his opposition a copy of a letter purportedly signed by Espino and a
telegram apparently also from Espino advising him of his arrival by boat about the last week of May 1964, as well
as a previous letter of referring to the lot as the one covered by Certificate of Title No. 62.

Decision of CFI Palawan: Dismissed the complaint on the ground that there being no written contract, under
Article 1403 of the Civil Code of the Philippines Although the contract is valid in itself, the same can not be
enforced by virtue of the Statute of Frauds.

Hence, this petition

Contention of Espino: The authenticity of the letters has not been established

ISSUE

Whether there is a perfected contract of sale through letter or telegram?

2B 16-17 SALES AND LEASE Page 55


HELD

Yes. The Statute of Frauds, embodied in Article 1403 of the Civil Code of the Philippines, does not require that the
contract itself be in writing. The plain text of Article 1403, paragraph (2) is clear that a written note or
memorandum, embodying the essentials of the contract and sign ed by the party charged, or his agent, suffices to
make the verbal agreement enforceable, taking it out of the operation of the statute.

In this case, the letter and telegram constitute an adequate memorandum of the transaction. They are signed by the
defendant and all essential terms of the contract are present and they satisfy the requirements of the statute of frauds

Regarding Espinos contention, the Court ruled that its not necessary for the purpose of showing prima facie
that the contract is enforceable. For as ruled by the Court in Shaffer vs. Palma, L-24115, March 1, 1968, whether the
agreement is in writing or not, is a question of evidence; and the authenticity of the writing need not be established
until the trial is held. The plaintiff having alleged that the contract is backed by letter and telegram, and the same
being a sufficient memorandum, his cause of action is thereby established, especially since the defendant has not
denied the letters in question. At any rate, if the Court below entertained any doubts about the existence of the
written memorandum, it should have called for a preliminary hearing on that point, and not dismissed the complaint.

2B 16-17 SALES AND LEASE Page 56


8. REGINA DIZON VS CA

G.R. No. 122544 January 28, 1999

Martinez, J.

FACTS

Overland Express Lines, Inc. entered into a Contract of Lease with Option to Buy with petitioners involving a
1,755.80 square meter parcel of land situated at corner MacArthur Highway and SouthH Street, Diliman, Quezon
City. The term of the lease was for 1 year commencing from May 16,1974 up to May 15, 1975. During this period,
Overland Express Lines was granted an option topurchase for the amount of P3,000.00 per square meter. Thereafter,
the lease shall be on a permonth basis with a monthly rental of P3,000.00.

For failure of Overland Express Lines to pay the increased rental of P8,000.00 per month effective June 1976,
petitioners filed an action for ejectment against it. The lower court rendered judgment ordering Overland Expres s
Lines to vacate the leased premises and to pay the sum of P624,000.00 representing rentals in arrears and/or as
damages in the form of reasonable compensation for the use and occupation of the premises during the period of
illegal detainer from June 1976 to November 1982 at the monthly rental of P8,000.00, less payments made, plus
12% interest per annum from November 18, 1976, the date of filing of the complaint, until fully paid, the sum of
P8,000.00 a month starting December 1982, until Overland Express Lines fully vacates the premises, and to
payP20,000.00 as and by way of attorneys fees.

Overland Express Lines claims that it actually paid the alleged P300,000.00 to Fidela Dizon, as representative
(agent) of petitioners in consideration of the option.

ISSUE

Whether or not Overland Express Lines there is a valid sale through the option to buy in the contract of lease.

HELD

NO. Having failed to exercise the option within the stipulated one year period, private respondent cannot enforce its
option to purchase anymore. Moreover, even assuming arguendo that the right to exercise the option still subsists at
the time private respondent tendered the amount on June 20, 1975, the suit for specific performance to enforce the
option to purchase was filed only on October 7, 1985 or more than ten (10) years after accrual of the cause of action
as provided under Article 1144 of the New Civil Code.

In this case, there was a contract of lease for one (1) year with option to purchase. The contract of lease expired
without the private respondent, as lessee, purchasing the property but remained in possession thereof. Hence, there
was an implicit renewal of the contract of lease on a monthly basis. The other terms of the original contract of lease
which are revived in the implied new lease under Article 1670 of the New Civil Code are only those terms which are
germane to the lessees right of continued enjoyment of the property leased. Therefore, an implied new lease does
not ipso facto carry with it any implied revival of private respondents option to purchase (as lessee thereof) the
leased premises. The provision entitling the lessee the option to purchase the leased premises is not deemed
incorporated in the impliedly renewed contract because it is alien to the possessio n of the lessee.

2B 16-17 SALES AND LEASE Page 57


Private respondents right to exercise the option to purchase expired with the termination of the original contract of
lease for one year. The rationale of this Court is that: This is a reasonable construction of the provision, which is
based on the presumption that when the lessor allows the lessee to continue enjoying possession of the property for
fifteen days after the expiration of the contract he is willing that such enjoyment shall be for the entire period
corresponding to the rent which is customarily paidin this case up to the end of the month because the rent was
paid monthly. Necessarily, if the presumed will of the parties refers to the enjoyment of possession the presumption
covers the other terms of the contract related to such possession, such as the amount of rental, the date when it must
be paid, the care of the property, the responsibility for repairs, etc. But no such presumption may be indulged in with
respect to special agreements which by nature are foreign to the righ t of occupancy or enjoyment inherent in a
contract of lease.

Under Article 1475 of the New Civil Code, the contract of sale is perfected at the moment there is a meeting of
minds upon the thing which is the object of the contract and upon the price. From that moment, the parties may
reciprocally demand performance, subject to the provisions of the law governing the form of contracts. Thus, the
elements of a contract of sale are consent, object, and price in money or its equivalent. It bears stressing tha t the
absence of any of these essential elements negates the existence of a perfected contract of sale. Sale is a consensual
contract and he who alleges it must show its existence by competent proof.

In an attempt to resurrect the lapsed option, private respondent gave P300,000.00 to petitioners (thru Alice A.
Dizon) on the erroneous presumption that the said amount tendered would constitute a perfected contract of sale
pursuant to the contract of lease with option to buy. There was no valid consent by the petitioners (as coowners of
the leased premises) on the supposed sale entered into by Alice A. Dizon, as petitioners alleged agent, and private
respondent. The basis for agency is representation and a person dealing with an agent is put upon inquiry and must
discover upon his peril the authority of the agent. As provided in Article 1868 of the New Civil Code, there was no
showing that petitioners consented to the act of Alice A. Dizon nor authorized her to act on their behalf with regard
to her transaction with private respondent. The most prudent thing private respondent should have done was to
ascertain the extent of the authority of Alice A. Dizon. Being negligent in this regard, private respondent cannot seek
relief on the basis of a supposed agency.

2B 16-17 SALES AND LEASE Page 58


9. TOYOTA SHAW VS CA

G.R. No. L-116650 May 1995

FACTS:

On June 14, 1989, Luna L. Sosa and his son, Gilbert, went to Toyota office at Shaw Boulevard, Pasig
(Toyota) to purchase a yellow Toyota Lite Ace where they met Popong Bernardo who was a sales representative of
said branch. Sosa emphasized that he needed the car not later than June 17, 1989 because a balikbayan guest would
be using it on June 18 to go home to Marinduque where he will celebrate his birthday on the 19th. Bernardo assured
Sosa that a unit would be ready for pick up on June 17 at 10:00 in the morning. They signed a document,
"Agreements Between Mr. Sosa & Popong Bernardo of Toyota Shaw, Inc., which did not mention anything about
the full purchase price and the manner the instalments were to be paid.

On June 15, Sosa and Gilbert delivered the down payment of P100,000.00 and Bernardo accomplished a
printed Vehicle Sales Proposal (VSP) No. 928 which showed Sosas full name and home address, that payment is by
"installment," to be financed by "B.A.," and that the "BALANCE TO BE FINANCED" is "P274,137.00", but the
spaces provided for "Delivery Terms" were not filled-up.

On June 17, however, Toyota did not deliver the Lite Ace. Hence, Sosa asked that his down payment be
refunded and Toyota issued a Far East Bank check for the full amount of P100,000.00 , which Sosa signed with the
reservation, "without prejudice to our future claims for damages." Toyota contended that the B.A. Finance
disapproved Sosas the credit financing application and further alleged that a particular unit had already been
reserved and earmarked for Sosa but could not be released due to the uncertainty of payment of the balance of the
purchase price. Toyota then gave Sosa the option to purchase the unit by paying the full purchase price in cash but
Sosa refused.

The trial court found that there was a valid perfected contract of sale between Sosa and Toyota which
bound the latter to deliver the vehicle and that Toyota acted in bad faith in selling to another the unit already
reserved for Sosa, and the Court of Appeals affirmed the said decision.

ISSUE: Whether or not there was a perfected contract of sale between respondent Sosa and petitioner Toyota?

HELD:

The Supreme Court granted Toyotas petition and dismissed Sosas complaint for damages because the
document entitled Agreements Between Mr. Sosa & Popong Bernardo of Toyota Shaw, Inc., was not a perfected
contract of sale, but merely an agreement between Mr. Sosa and Bernardo as private individuals and not between
Mr. Sosa and Toyota as parties to a contract.

There was no indication in the said document of any obligation on the part of Toyota to transfer ownership
of a determinate thing to Sosa and neither was there a correlative obligation on the part of the latter to pay therefor a
price certain. The provision on the down payment of P100,000.00 made no specific reference to a sale of a vehicle.
If it was intended for a contract of sale, it could only refer to a sale on instalment basis, as VSP No.928 exe cuted on
June 15, 1989 confirmed. The VSP also created no demandable right in favor of Sosa for the delivery of the vehicle
to him, and its non-delivery did not cause any legally indemnifiable injury.

2B 16-17 SALES AND LEASE Page 59


10. VIRGINIA PAGCO VS CA

G.R. No. L-109236 March 18, 1994

FACTS:

Private respondent Peter Quimson is the owner of a parcel if land situated at San Isidro Street, Singalong,
Manila. When private respondent acquired the property on March 17, 1987 through sale at public auction, eleven
(11) occupants were in possession of the property with their respective residential houses built thereon, among
whom are herein petitioners.

Private respondent had earlier negotiated with petitioners for the latter to buy the portions they occupy but
petitioners backed off. Private respondent subsequently informed the lessees to pay their back rentals and to remove
their houses because he needed the property for his own use and that of the immediate member of his family. For
failure of petitioners to heed private respondent's demand, a complaint for ejectment was filed against petitioners
and the other occupants of the property.

The Metropolitan Trial Court rendered judgment dismissing the complaint for ejectment on the ground that
there was a perfected sale over the property between private respondent and its occupants and, consequently, said
court had no jurisdiction over the case because the rights of the parties should be governed not by the law on lease
but by the law on sales, more specifically Article 1475 of the Civil Co de.

Private respondent appealed to the Regional Trial Court, which reversed the decision of the Metropolitan
Trial Court. Not satisfied with the RTC's decision, petitioners filed a petition for review with the Court of Appeals
which dismissed the petition. Hence, this petition.

ISSUE:

Whether or not there is a perfected contract of sale.

HELD:

No. As correctly found by both the Court of Appeals and the Regional Trial Court on the basis of the
evidence, there was no meeting of the minds between the parties regarding the offer by private respondent to sell his
property to the occupants. Private respondent wanted P980.00 per square meter, but the occupants were willing to
pay only P850.00.

2B 16-17 SALES AND LEASE Page 60


The phrase "perfected contract" in paragraph 10 of the complaint is used in its loose sense and does not
connote that there was a meeting of the minds between the parties. Observe that after the statement in said paragraph
that "after approving the proposed subdivision plan, the defendants suddenly and abruptly changed th eir minds and
repudiated the agreement which is already a perfected contract," there immediately follows the qualifying allegation
that "[defendants] deliberately and maliciously refused to continue negotiating with the plaintiff as expressed in the
attached letter of their counsel marked as Annex 'E' and made an integral part thereof.

The words "refused to continue negotiating with the plaintiff" have no other meaning except that there was
a negotiation regarding the offer to sell, but the negotiation fell through because of the refusal of petitioners and the
other occupants to talk further as evidenced by the letter of their counsel, which is Annex "C" of the complaint.

Finally, even granting that there was a perfected contract of sale, it can be implied that there was
subsequently a mutual withdrawal or "mutual backing out" from the contract. 8 This conclusion may be drawn from
the fact of the filing by private respondent of the complaint for ejectment, in which he alleged ownership of the
property in question and from the averments in petitioners' answer wherein they never claimed ownership of the
property by purchase from private respondent.

2B 16-17 SALES AND LEASE Page 61


11. CESAR RAET VS CA

G.R. No. 128016, September 17, 1998

FACTS

Petitioners Cesar and Elvira Raet (the spouses Raet) and petitioners Rex and Edna Mitra (the spouses Mitra)
negotiated with Amparo Gatus concerning the possibility of buying the rights of the latter to certain units at the Las
Villas de Sto. Nio Subdivision in Meycauayan, Bulacan.

This subdivision was developed by private respondent Phil-Ville Development and Housing Corporation (PVDHC)
primarily for parties qualified to obtain loans from the Government Service Insurance System (GSIS).

The spouses Raet and the spouses Mitra paid Gatus the total amounts of P40,000.00 and P35,000.00,
respectively, for which they were issued receipts by Gatus in her own name.

The spouses Raet and the spouses Mitra applied directly with private respondent PVDHC for the purchase of
units in the said subdivision. As they were not GSIS members, they looked for members who could act as
accommodation parties by allowing them to use their policies. Private respondent PVDHC would process the
applications for the purchase of the units upon the approval by the GSIS of petitioners loan applications.

The spouses Raet and spouses Mitra presented the GSIS policy of certain GSIS members. The spouses Raet
paid P32,653.00, while the spouses Mitra paid P27,000.00, to private respondent PVDHC, on the
understanding that these amounts would be credited to the purchase prices of the units which will be
determined after the approval of their loan applications with the GSIS. Meanwhile, the spouses Raet and
spouses Mitra were allowed to occupy the units they are to purchase in the subdivision.

However, that the GSIS disapproved the loan applications of petitioners. For this reason, they were advised
by private respondent PVDHC to seek other sources of financing. In the meantime, they were allowed to
remain in the subject premises.

Owing to the failure of petitioners to raise money, private respondent PVDHC asked them, in separate demand
letters to vacate the units they were occupying. As petitioners refused to do so, it filed ejectment cases against them.

The spouses Raet and the spouses Mitra filed a complaint for specific performance and damages against Amparo
Gatus and private respondent PVDHC with the HLURB. HLURB ruled in favor of petitioners awarding damages
and for Gatus to remit the amounts paid by the petitioners to PVDHC.

2B 16-17 SALES AND LEASE Page 62


On appeal, the Board of Commissioners of the HLURB reversed. Petitioners elevated the case to the Office of the
President which sustained the ruling of the Housing and Land Use Arbiter.

PVDHC appealed to the Court of Appeals which set aside the decisio n of the Office of the President and dismissed
the petitioners action without prejudice to their right to proceed against Amparo Gatus.

Hence this petition for review on certiorari by the spouses Raet and the spouses Mitra. Petitioners contend,
among others that, the Court of Appeals committed a reversible error in concluding that there were no
perfected contracts of purchase and sale between the petitioners and the private respondents.

ISSUE

Whether or not there was a perfected contract of sale between petitioners and private respondents.

HELD

First, private respondent PVDHC pointed out that the figures referred to by petitioners were mere estimates given to
them by Amparo Gatus. The parties transactions, therefore, lacked the requisites essential for the perfection of
contracts.

Second, petitioners dealt with Gatus. But Gatus was not the agent of private respondent PVDHC. Art. 1874 of the
Civil Code requires for the validity of a sale involving land that the agent should have an authorization in writ ing,
which Gatus did not possess. Petitioners knew from the beginning that Gatus was negotiating with them in her own
behalf, and not as an agent of private respondent PVDHC.

Third, since private respondent PVDHC had no knowledge of the figures Amparo Gat us gave to petitioners as
estimates of the costs of the units in question, it could not have ratified the same at the time the latter applied for the
purchase of the units. At any rate, private respondent PVDHC was to enter into agreements concerning the s ubject
units with petitioners only upon the approval of the latters loan applications with the GSIS which, as mentioned
earlier, failed to materialize.

Fourth, there are no written contracts to evidence the alleged sales. If petitioners and private respo ndent PVDHC had
indeed entered into contracts involving the subject units, it is rather strange that contracts of such importance have
not been reduced to writing.

2B 16-17 SALES AND LEASE Page 63


There was no contract of sale perfected between the private parties over the said property, there being no meeting of
the minds as to terms, especially on the price thereof. At best, only a proposed contract to sell obtained which did
not even ripen into a perfected contract due at the first instance to private respondents inability to secure ap proval of
their GSIS housing loans.

As it were, petitioners and private respondents have not hurdled the negotiation phase of a contract, which is the
period from the time the prospective contracting parties indicate interest on the contract to the time the contract
comes into existence - the perfection stage - upon the concurrence of the essential elements thereof.

2B 16-17 SALES AND LEASE Page 64


13. ARTATES AND POJAS VS URBI ET AL

G.R. No. L-29421 January 30, 1971

Reyes, J. B. L., J.

FACTS

Spouses Lino Artates and Manuela Pojas, in an action filed in the Court of First Instance Cagayan sought the
annulment of the execution of a homestead issued to them and duly registered under their names. They also pray for
the deed of sale by the creditor winner be declared null and void; that defendants be ordered to deliver to plaintiffs
possession of the land; and to pay to plaintiffs compensatory damages until possession is finally restored to them.
The public sale, conducted by the Provincial Sheriff of Cagayan was made t o satisfy a judgment against Lino
Artates in the amount of P1,476.35, and awarded to Daniel Urbi in a civil case for physical injuries inflicted by
Artates upon Urbi. In the execution sale, the property was sold to the judgment creditor, the only bidder.

The spouses contend that such sale is violative of the Public Land law exempting said property from execution for
any debt contracted within five years from the date of the issuance of the patent. They also allege that Urbi, with the
intention of defrauding the plaintiffs, executed a deed for the sale of the same parcel of land to defendant Crisanto
Soliven, a minor. As a result of the aforementioned transactions, defendants Urbi and Soliven entered into the
possession of the land and deprived plaintiffs of the owners' share in the rice crops harvested during the agricultural
year 1961-1962.

Court upheld the regularity and validity of the execution; but found the sale to the minor Soliven to be simulated;
therefore null and void. Such sale was intended to p lace the property beyond the reach of the judgment debtor. The
spouses had in fact offered to redeem the land within the 5-year period allowed by Section 119 of the Public Land
law for reacquisition thereof by the grantee. Urbi was ordered reconvey the pro perty to the plaintiffs upon the
spouses payment of the sum of P1,476.35 plus other fees, with legal interest.

ISSUE

Plaintiff spouses appeal: Is the execution valid, given Sec 118 of the Public Land Law (CA 141)?

HELD

YES.

Sec 118 provides:Except in favor of the Government or any of its branches, units, or institution, or legally
constituted banking corporations, lands acquired under free patent or homestead provisions shall not be subject
to encumbrance or alienation from the date of the approval of the application and for a term of five years
from and after the date of issuance of the patent or grant, nor shall they become liable to the satisfaction of
any debt contracted prior to the expiration of said period, but the improvements or crops on the land may be
mortgaged or pledged to qualified persons, associations or corporations.

This provision against the alienation or encumbrance of public lands granted within five years from the issuance of
patent, it has been held, is mandatory. A sale in violation thereof is null and void and produces no effect whatsoever.
The salutary purpose of the provision is to preserve and keep for the homesteader or his family the land given to him
gratuitously by the State, so that being a property owner, he may become and remain a contented and useful member
of our society.

The homestead patent was issued to appellants on 23 September 1952, and it was sold at public auction to satisfy the
civil liability of appellant Lino Artates adjudged in a 14 March 1956 court decision. As such, the indebtedness

2B 16-17 SALES AND LEASE Page 65


commenced from the date that the civil obligation was decreed. Consequently, it cannot be enforced against, or
satisfied out of, the sale of the homestead lot acquired by appellants less than 5 years before the obligation ac crued.
The exemption should not be given restrictive application.

The word "debt" in exemption statutes includes all that is due to a man under any form or obligation or promise
like civil liability arising from a crime committed by the homesteader. This is in consonance with the definite public
policy underlying these grants. And this is true even if the sale involved here is not voluntary. For purposes of
complying with the law, it is immaterial that the satisfaction of the debt by the encumbrancing or a lienation of the
land grant made voluntarily, as in the case of an ordinary sale, or involuntarily, such as that effected through levy on
the property and consequent sale at public auction. In both instances, the spirit of the law would have been violated.

Decision appealed from is reversed, and appellants are declared entitled to the return and possession of the lot,
without prejudice to their continuing obligation to pay the judgment debt, and expenses connected therewith.

2B 16-17 SALES AND LEASE Page 66


14. CAVITE DEVT BANK VS CYRUS LIM

G.R. No. 131679 February 1, 2000

Mendoza, J.

FACTS

On or about June 15, 1983, Rodolfo Guansing obtained a loan in the amount of Php 90,000 from Cavite
Development Bank (CDB), to secure which he mortgage a parcel of land situated in La Loma, Quezon City and
covered by TCT No. 300809 registered in his name. As Guansing defaulted in the payment of his loan, CDB
foreclosed the mortgage. At the foreclosure sale held on March 15, 1984, the mortgaged property was sold to CDB
as the highest bidder. Guansing failed to redeem, and on March 2, 1987, CDB consolidated title to the property in its
name. TCT No. 300809 in the name of Guansing was cancelled and, in lieu thereof, TCT No. 355588 was issued in
the name of CDB. On June 16, 1988, private respondent Lolita Chan Lim offered to purchase the property from
CDB for Php 300,000 and under the ff conditions: 10% Option Money; Balance payable in cash provided that the
property shall be cleared of illegal occupants. Lim paid CDB P30,000.00 as Option Money. However, after some
time following up the sale, Lim discovered that the subject property was originally registered in the name of
Perfecto Guansing, father of mortgagor Rodolfo Guansing, under TCT No. 91148. It appears that the father,
Perfecto, instituted Civil Case No. Q-39732 in the Regional Trial Court, Branch 83, Quezon City, for the
cancellation of his son's title. On March 23, 1984, the trial court rendered a decision restoring Perfecto's previous
title (TCT No. 91148) and cancelling TCT No. 300809 on the ground that the latter was fraudulently secured by
Rodolfo. Aggrieved by what she considered a serious misrepresentation by CDB and its mother-company, FEBTC,
on their ability to sell the subject property, Lim, joined by her husband, file d on August 29, 1989 an action for
specific performance and damages against petitioners in the Regional Trial Court. On March 10, 1993, the trial court
rendered a decision in favor of the Lim spouses. CA affirmed in toto the decision.

ISSUE

Whether or not the sale between CDB and Lim was perfected.

HELD

Yes. The parties actually entered into a contract of sale, partially consummated as to the payment of the price, where
the offer to purchase provides that, after the payment of the payment of the option money, only the balance of the
purchase price need be paid, implying that the option money forms part of the purchase price.

In this case, after the payment of the 10% option money, the Offer to Purchase provides for the payment only of the
balance of the purchase price, implying that the "option money" forms part of the purchase price. This is precisely
the result of paying earnest money under Art. 1482 of the Civil Code. It is clear then that the parties in this case
actually entered into a contract of sale, partially consummated as to the payment of the price.

2B 16-17 SALES AND LEASE Page 67


Given CDB's acceptance of Lim's offer to purchase, it appears that a contract of sale was perfected and, indeed,
partially executed because of the partial payment of the purchase price. There is, however, a serious legal obstacle to
such sale, rendering it impossible for CDB to perform its obligation as seller to deliver and transfer ownership of the
property.

2B 16-17 SALES AND LEASE Page 68


15. CONCHITA NOOL and GAUDENCIO ALMOJERA VS COURT OF APPEALS , ANACLETO NOOL
and EMILIA NEBRE

G.R. No. 116635 July 24, 1997

FACTS

Two (2) parcels of land are in dispute and litigated upon here. The first was formerly owned by Victorino Nool and
covered by Transfer Certificate of Title No. T-74950. The other parcel was previously owned by Francisco Nool
under Transfer Certificate of Title No. T-100945. The plaintiff spouses, Conchita Nool and Gaudencio Almojera
seek recovery of the aforementioned parcels of land from Anacleto Nool, a younger brother of Conchita, and Emilia
Nebre.

Plaintiff-appellants alleged that they are the owners of subject parcels of land, and they bought the same from
Conchita's other brothers, Victorino Nool and Francisco Nool; that as plaintiffs were in dire need of money, they
obtained a loan from the Ilagan Branch of the Development Bank of the Philippines secured by a real estate
mortgage on said parcels of land, which were still registered in the names of Victorino Nool and Francisco Nool, at
the time, and for the failure of plaintiffs to pay the said loan, including interest and surcharges, the mortgage was
foreclosed; that within the period of redemption, plaintiffs contacted defendant Anacleto Nool for the latter to
redeem the foreclosed properties from DBP, which the latter did; and as a result, the titles of the two (2) parcels of
land in question were transferred to Anacleto Nool; that as part of their arrangement or understanding, Anacleto
Nool agreed to buy from plaintiff Conchita Nool the two (2) parcels of land under controversy, fo r a total price of
P100,000.00, P30,000.00 of which price was paid to Conchita, and upon payment of the balance of P14,000.00,
plaintiffs were to regain possession of the two (2) hectares of land, which amounts defendants failed to pay, and the
same day the said arrangement was made; another covenant was entered into by the parties, whereby defendants
agreed to return to plaintiffs the lands in question, at anytime the latter have the necessary amount; that plaintiffs
asked the defendants to return the same but despite the intervention of the Barangay Captain of their place,
defendants refused to return the said parcels of land to plaintiffs; thereby impelling them (plaintiffs) to come to court
for relief.

Defendants-appellees theorized that they acquired the lands in question from the DBP, through negotiated sale, and
were misled by plaintiffs when defendant Anacleto Nool signed the private writing, agreeing to return subject lands
when plaintiffs have the money to redeem the same; defendant Anacleto having been made to believe, then, that his
sister, Conchita, still had the right to redeem the said properties.

The pivot of inquiry here is the nature and significance of the private document that defendants gave advance
payment of P30,000.00 therefor, and acknowledged that they had a balance of P14,000.00 to complete their
payment.

The two parcels of land in dispute were mortgaged to the Development Bank of the Philippines, to secure a loan
obtained by plaintiffs from DBP. For the non-payment of said loan, the mortgage was foreclosed and in the process,
ownership of the mortgaged lands was consolidated in DBP. After DBP became the absolute owner of the two

2B 16-17 SALES AND LEASE Page 69


parcels of land, defendants negotiated with DBP and succeeded in buying the same. By virtue of such sale by DBP
in favor of defendants, the titles of DBP were cancelled and the corresponding Transfer Certificates of Title issued
to the defendants.

Manuel S. Mallorca, authorized officer of DBP, certified that the mortgagors' right of redemption was not exerc ised
within the one-year redemption period. Hence, DBP became the absolute owner of said parcels of land for which it
was issued new certificates of title. About two years thereafter DBP entered into a Deed of Conditional Sale
involving the same parcels of land with Private Respondent Anacleto Nool as vendee. Subsequently, the latter was
issued new certificates of title.

ISSUES:

(1) Whether or not the contract of sale between Conchita Nool and Anacleto Nool is valid
(2) Whether or not Conchita Nool may exercise the contract of repurchase auxiliary to the aforementioned contract
of sale

HELD

The Regional Trial Court and the Court of Appeals ruled that the principal contract of sale and the auxiliary contract
of repurchase are both void. This conclusion of the two lower courts appears to find support in Dignos vs. Court of
Appeals, where the Court held:

Be that as it may, it is evident that when petitioners sold said land to the Cabigas spouses, they were no
longer owners of the same and the sale is null and void.

The sellers (Conchita) no longer had any title to the parcels of land at the time of sale. Since the alleged contract of
repurchase was dependent on the validity of the principal contract of sale, it is itself void. A void contract cannot
give rise to a valid one. Verily, Article 1422 of the Civil Code provides that "(a) contract which is the direct result of
a previous illegal contract, is also void and inexistent."

(Important) In the present case, it is likewise clear that the sellers can no longe r deliver the object of the sale to the
buyers, as the buyers themselves have already acquired title and delivery thereof from the rightful owner, the DBP.
Thus, such contract may be deemed to be inoperative and may thus fall, by analogy, under item no. 5 of Article 1409
of the Civil Code: "Those which contemplate an impossible service." Article 1459 of the Civil Code provides that
"the vendor must have a right to transfer the ownership thereof [object of the sale] at the time it is delivered." Here,
delivery of ownership is no longer possible. It has become impossible.

2B 16-17 SALES AND LEASE Page 70


Furthermore, Article 1505 of the Civil Code provides that "where goods are sold by a person who is not the owner
thereof, and who does not sell them under authority or with consent of the owner, the buyer acquires no better title to
the goods than the seller had, unless the owner of the goods is by his conduct precluded from denying the seller's
authority to sell." Here, there is NO allegation at all that petitioners were authorized by DBP to sell the property to
the private respondents. Jurisprudence, on the other hand, teaches us that "a person can sell only what he owns or is
authorized to sell; the buyer can as a consequence acquire no more than what the seller can legally transfer." No on e
can give what he does not have nemo dat quod non habet. On the other hand, the principal contract of sale herein
presupposes that petitioners could repurchase the property that they "sold" to private respondents. As petitioners
"sold" nothing, it follows that they can also "repurchase" nothing. Nothing sold, nothing to repurchase. In this light,
the contract of repurchase is also inoperative and by the same analogy, void.

2B 16-17 SALES AND LEASE Page 71


16. HEIRS OF SEVERINA SAN MIGUEL, namely: MAGNO LAPINA, PACENCIA LAPINA, MARCELO
LAPINA, SEVERINO LAPINA, ROSARIO LAPINA, FRANCISCO LAPINA, CELIA LAPINA assisted by
husband RODOLFO TOLEDO, vs.THE HONORABLE COURT OF APPEALS, DOMINADOR SAN
MIGUEL, GUILLERMO F. SAN ARTEMIO F. SAN MIGUEL, PACIENCIA F. SAN MIGUEL,
CELESTINO, assisted by husband, ANTERO CELESTINO, represented by their Attorney-in-Fact ENRICO
CELESTINO, AUGUSTO SAN MIGUEL, ANTONIO SAN MIGUEL, RODOLFO SAN MIGUEL,
CONRADO SAN MIGUEL and LUCITA SAN MIGUEL, respondents.

FACTS

This case involves a parcel of land originally claimed by Severina San Miguel. The land is situated in Panapan,
Bacoor, Cavite. Without Severina's knowledge, Dominador managed to cause the subdivision of the land into three
(3) lots. September 25, 1974, Dominador, et al. filed a petition with the Court of First Instance, Cavite, as a land
registration court. Land Registration Commission (hereafter "LRC") rendered a decision directing the issuance of
Original Certificate of Title No. 0-1816 in the names of Dominador, et al. Severina filed with Court of First
Instance alleging that the land registration proceedings were fraudulently concealed by Dominador from her. CFI
declared as null and void and let the writ of possession previously issued in favor of petitioner Severina San Miguel
be implemented and alias writ of demolition be issued in favor of petitioners. Both writ were unsatisfied. Severina's
heirs, decided not to pursue the writs and entered into a compromise with Dominador, et al. According to the
compromise, Severina's heirs were to sell the subject lots to Dominador, et al. for one and a half million pesos (P1.5
M) conditioned upon the purchase of another lot 11 which was not yet titled at an additional sum of (P300,000.00)

August 6, 1993, pursuant to the kasunduan, Severina's heirs and Dominador, et al. executed a deed of sale
designated as "kasulatan sa bilihan ng lupa." Dominador, et al. filed a motion praying that Severina's heirs deliver
the owner's copy of the certificate of title to them. Severina's heirs opposed the mot ion stressing that under the
kasunduan, the certificate of title would only be surrendered upon Dominador, et al.'s payment of the amount of
three hundred thousand pesos. Dominador, et al. admitted non -payment of three hundred thousand pesos
(P300,000.00) for the reason that Severina's heirs have not presented any proof of ownership over the untitled parcel
of land covered by LRC-Psu-1312. Apparently, the parcel of land is declared in the name of a third party, a certain
Emiliano Eugenio. Severina's heirs countered that the arguments of Dominador, et al. were untenable in light of the
provision in the kasunduan where Dominador, et al. admitted their ownership over the parcel of land

ISSUE

Whether or not there is perfected contract of sale? Yes

HELD

True, under 1459, in contracts of sale, the vendor need not possess title to the thing sold at the perfection of the
contract. However, the vendor must possess title and must be able to transfer title at the time of delivery. In a
contract of sale, title only passes to the vendee upon full payment of the stipulated consideration, or upon delivery of
the thing sold.

2B 16-17 SALES AND LEASE Page 72


Under the facts of the case, Severina's heirs are not in a position to transfer title. Without passing on the question of
who actually owned the land covered by LRC Psu -1312, we note that there is no proof of ownership in favor of
Severina's heirs. In fact, it is a certain Emiliano Eugenio, who holds a tax declaration over the said land in his name.
Though tax declarations do not prove ownership of the property of the declarant, tax declarations and receipts can be
strong evidence of ownership of land when accompanied by possession for a period sufficient for prescription.
Severina's heirs have nothing to counter this document.

In Nool v. Court of Appeals, we held that if the sellers cannot deliver the object of the sale to the buyers, such
contract may be deemed to be inoperative. By analogy, such a contract may fall under Article 1405, No. 5: Those
which contemplate an impossible service.

NOTE

ARTICLE 1458. By the contract of sale one of the contracting parties obligates himself to transfer the ownership of
and to deliver a determinate thing, and the other to pay a price certain in money or its equivalent. . .

ARTICLE 1459. The thing must be licit and the vendor must have a right to transfer the ownership thereof at the
time it is delivered.

ARTICLE 1495. The vendor is bound to transfer the ownership of and deliver, as well as warrant the thing which is
the object of sale

Copy of kasunduan

"5. Na ang Lot 1 at Lot 2, plano LRC Psu-1313 na binabanggit sa itaas na ipinagkasundo ng mga tagapagmana
ni Severina San Miguel na kilala sa kasulatang ito sa taguring LAPINA (representing Severina's heirs), na ilipat sa
pangalan nina SAN MIGUEL (representing Dominador's heirs) alang alang sa halagang ISANG MILYON AT
LIMANG DAANG LIBONG PISO (P1,500,000.00) na babayaran nina SAN MIGUEL kina LAPINA;

"6. Na si LAPINA at SAN MIGUEL ay nagkakasundo na ang lote na sakop ng plano LRC-Psu-1312, may
sukat na 108 metro cuadrado ay ipagbibili na rin kina SAN MIGUEL sa halagang TATLONG DAANG LIBONG
PISO (P300,000.00);

"7. Na kinikilala ni SAN MIGUEL na ang tunay na may-ari ng nasabing lote na sakop ng plano LRC Psu-1312
ay sina LAPINA

2B 16-17 SALES AND LEASE Page 73


17. ANGEL CLEMENO, JR., MALYN CLEMENO, and NILUS SACRAMENTO vs.

ROMEO R. LOBREGAT

G.R. No. 137845 September 9, 2004

Callejo, J

FACTS

Spouses Nilus and Teresita Sacramento were the owners of a parcel of land covered by a Transfer Certificate of
Title (TCT) and the hous e constructed thereon in Novaliches, Quezon City. They mortgaged the property with the
Social Security System (SSS) as security for their housing loan and surrendered the owners and duplicate copies of
the certificate of title.

The spouses executed a Deed of Sale with Assumption of Mortgage in favor of Maria Linda Clemeno and her
husband Angel C. Clemeno, Jr., with the conformity of the SSS.

The Register of Deeds issued a TCT over the property in the name of the vendees, who, in turn, execu ted a
Real Estate Mortgage Contract over the property in favor of the SSS to secure the payment for the balance of the
loan. The Spouses Clemeno also surrendered the owners duplicate copy of the said title to the SSS.

On June 4, 1987, respondent Romeo R. Lobregat and petitioner Angel Clemeno, Jr., relatives by consanguinity,
entered into a verbal contract of sale over the property covered by the TCT under the following terms and
conditions: (a) the respondent would pay the purchase price of the prope rty in the amount of P270,000.00, inclusive
of the balance of the loan of the petitioners, the Spouses Clemeno with the SSS within two years from June 4, 1987;
(b) the respondent would pay the monthly amortizations of the vendors loan with the SSS; and (c) upon the payment
of the purchase price of the property, the Spouses Clemeno would execute a deed of sale in favor of the respondent.

The respondent made a down payment for which petitioner Clemeno, Jr. issued a receipt. He also made partial
payments, paid realty taxes due, and continued remitting to the SSS the monthly amortizations due for the account
of petitioner Clemeno, Jr.

The respondent offered to pay the said balance to petitioner Clemeno, Jr., but the latter told him to keep the
money because the owners duplicate copy of the title was still with the SSS and to instead continue paying the
monthly amortizations due.

On May 9, 1990, the SSS had executed a Release of Real Estate Mortgage in favor of petitioner Clemeno, Jr.
and released the owners duplicate of TCT.

The respondent offered to pay the balance of the purchase price of the property to petitioner Clemeno, Jr. and
asked the latter to execute the deed of sale over the property and deliver the title over the prope rty under his name,
but petitioner Clemeno, Jr. refused to do so unless the respondent agreed to buy the property at the price prevailing
in 1992. The respondent refused.

On June 12, 1992, respondents counsel wrote petitioner Clemeno, Jr., informing the latter that the respondent
had already paid part of the purchase price of the property and that he was ready to pay the balance thereof. He
demanded that petitioner Clemeno, Jr. execute a deed of absolute sale over the property and deliver the title thereto
in his name upon his receipt of the payment.

2B 16-17 SALES AND LEASE Page 74


In his reply-letter, petitioner Clemeno, Jr. stated that he never sold the property to the respondent; that he merely
tolerated the respondents possession of the property for one year or until 1987, after which the latter offered to buy
the property, which offer was rejected; and that he instead consented to lease the property to the respondent. The
petitioner also declared in the said letter that even if the respondent wanted to buy the property , the same was
unenforceable as there was no document executed by them to evince the sale. They further assert that even if they
had entered into an agreement with the respondent, such agreement was a mere contract to sell, not a contract of
sale.

The respondent filed a Complaint against the petitioners for breach of contract, specific performance with
damages with the RTC of Quezon City.

ISSUE

Whether or not there is a perfected contract of sale.

HELD

YES.

The contract between the parties was a perfected verbal contract of sale, not a contract to sell over the subject
property, with the petitioner as vendor and the respondent as vendee.

Sale is a consensual contract and is perfected by mere consent, which is manifested by a meeting of the minds as to
the offer and acceptance thereof on three elements: subject matter, price and terms of payment of the price.

The contract entered into by the parties was not a contract to sell because there was no agreement for the petitioners
to retain ownership over the property until after the respondent shall have paid the purchase price in full, nor an
agreement reserving to the petitioners the right to unilaterally resolve the contract upon the buyers failure to pay
within a fixed period. Unlike in a contract of sale, the payment of the price is a positive suspensive condition in a
contract to sell, failure of which is not a breach but an event that prevents the obligation of the vendor to convey the
title from becoming effective.

The contract of sale of the parties is enforceable notwithstanding the fact that it was an oral agreement and not
reduced in writing as required by Article 1403(2) of the New Civil Code. This is so because the provision applies
only to executory, and not to completed, executed or partially executed contracts. In this case, the contract of sale
had been partially executed by the parties, with the transfer of the possession of the property to the respondent and
the partial payments made by the latter of the purchase price thereof.

2B 16-17 SALES AND LEASE Page 75


18. ATKINS KROLL &CO VS B CUA HIAN TEK (ALSO: RE: OFFER AND ACCEPTANCE)
ATKINS, KROLL and CO., INC. VS
B. CUA HIAN TEK

G.R. No. L-9871 January 31, 1958

Bengzon, J

FACTS:

Petitioner is a supplier of Sardines. On September 13, 1951 - petitioner sent to respondent a letter offering:

- 400 Ctns. Luneta brand Sardines in Tomato Sauce 48/15-oz. Ovals at $8.25 Ctn.
- 300 Ctns. Luntea brand Sardines Natural 48/15 oz. talls at $6.25 Ct.
- 300 Ctns. Luneta brand Sardines in Tomato Sauce 100/5-oz. talls at $7.48 Ct.

The shipment is during September/October from US Ports. Cua Hian Tek accepted the offer unconditionally and
delivered his letter of acceptance on September 21, 1951. However, due to shortage of catch of sardines by the
packers in California, Atkins Kroll & Co., failed to deliver the commodities it had offered for sale. Petitioner does
not dispute such timely acceptance. It merely raises the point that the acceptance only created an option, which,
lacking consideration, had no obligatory force. Petitioner argues, "was a promise to sell a determinate thing for a
price certain. Upon its acceptance by respondent, the offer became an accepted unilateral promise to sell a
determinate thing for price certain. Inasmuch as there was no consideration to support the p romise to sell distinct
from the price, the promise is not binding on the petitioner even if it was accepted by respondent.

ISSUE

Was there was a perfected contract of sale?

HELD

YES. A bilateral contract to sell and to buy was created upon acceptance. So much so that B. Cua Hian Tek could
be sued, he had backed out after accepting, by refusing to get the sardines and/or to pay for their price. Indeed, the
word "option" is found neither in the offer nor in the acceptance.

ART. 1324. When the offerer has showed the offeree a certain period to accept, the offer may be
withdrawn at any time before acceptance by communicating such withdrawal, except when the
option is founded upon a consideration, as something paid or promised.

Ordinarily an offer to buy or sell may be withdrawn or countermanded before acceptance, even though the offer
provides that it will not be withdrawn or countermanded, or allows the offeree a certain time within which to accept
it, unless such provision or agreement is supported by an independent consideration. If the option is given without a
consideration, it is a mere offer of a contract of sale, which is not binding until accepted. If, however, acceptance is
made before a withdrawal, it constitutes a binding contract of sale, even though the option was not supported by a
sufficient consideration. It can be taken for granted, as contended by the defendants, that the option contract was not
valid for lack of consideration. But it was, at least, an offer to sell, which was accepted by letter, and of this

2B 16-17 SALES AND LEASE Page 76


acceptance the offerer had knowledge before said offer was withdrawn. The concurrence of both acts the offer and
the acceptancecould at all events have generated a contract.

2B 16-17 SALES AND LEASE Page 77


SALE BY AUCTION

1. DBP VS CA AND EMERALD RESORT (ON PROCEDURAL REQS UNDER SPECIAL LAWS ON
FORECLOS URE SALES)

GR NO. 125838 June 10, 2003

FACTS

Before Us is a petition for review on certiorari seeking to reverse the decision of the CA affirming the decision of
the RTC in declaring the foreclosure of the mortgaged properties void for failure to comply with the posting and
publication requirements under the applicable laws.

Respondent obtained a loan from the petitioner, to secure such, former mortgaged its personal and real properties to
the latter. On 5 June 1986, alleging that respondent failed to pay its loan, petitioner filed with the Office of the
Sheriff, RTC of Iriga City, an Application for Extra- judicial Foreclosure of Real Estate and Chattel Mortgages
Sheriffs issued the required notices of public auction sale of the personal and real properties. However, they failed to
execute the corresponding certificates of posting of the notices. The Office of the Sheriff scheduled on August 12,
1986 the public auction sale of the real properties. The first scheduled public auction was publish. However, the
Office of the Sheriff postponed the auction sale on August 12, 1986 to September 11, 1986 at the request of the
respondent. Petitioner did not republish the notice of the rescheduled auction sale because both parties signed an
agreement to postpone the August 12, 1986 auction sale.

ISSUE

Whether or not Petitioner complied with the posting and publication requirements under applicable laws for a valid
foreclosure.

HELD
Valid as to chattel mortgage. Void as to real estate mortgage.

There is no question that Petitioner published the notice of auction sale scheduled on Aug 12. 1986. However, no
auction sale took place on the said date because Petitioner,at the instance of the respondent, a greed to postpone the
same to Sept 11, 1986.

Publication, therefore, is required to give the foreclosure sale a reasonably wide publicity such that those interested
might attend the public sale. To allow the parties to waive this jurisdictional requireme nt would result in converting
into a private sale what ought to be a public auction.

2B 16-17 SALES AND LEASE Page 78


Petitioner, however, complied with the mandatory posting of the notices of the auction sale of the personal
properties. Under the Chattel Mortgage Law, the only requirement is posting of the notice of auction sale. There
was no postponement of the auction sale of the personal properties and the foreclosure took place as scheduled.
Thus, the extrajudicial foreclosure of the chattel mortgage in the instant case suffers from no procedural infirmity.

2B 16-17 SALES AND LEASE Page 79


2. PROVINCE OF CEBU VS HEIRS OF RUFINA MORALES

G.R. No. 170115 February 19, 2008

Ynares-Santiago, J

FACTS:

Province of Cebu leased in favor of Rufina Morales a 210 square meter lot which formed part of Lot No.
646A of the Banilad Estate. Subsequently petitioner donated several parcels of land to the City of Cebu. Among
those donated was Lot No. 646A which the City of Cebu divided into sublots. The area occupied by Morales was
denominated as Lot No. 646A3.

The city sold Lot No. 646A3 as well as the other donated lots at public auction in order to raise money for
infrastructure projects. The highest bidder for Lot No. 646A3 was Hever Bascon but Morales allowed to match the
highest bid since she had a preferential right to the lot as actual occupant thereof. Morales thus paid the required
deposit and partial payment for the lot.

In the meantime, petitioner filed an action for reversion of donation against the City of Cebu. Petitioner and
the City of Cebu entered into a compromise agreement which the court approved. The agreement provided for the
return of the donated lots to petitioner except those that have already been utilized by the City of Cebu.

Morales died during the pendency of the case. Apart from th e deposit and down payment, she was not able
to make any other payments on the balance of the purchase price for the lot.

One of the nieces of Morales, respondent Catalina V. Quesada, wrote to then Cebu Governor Eduardo R.
Gullas asking for the formal conveyance of Lot No. 646A3 to Morales surviving heirs, in accordance with the
award earlier made by the City of Cebu.

The requests remained unheeded thus, Quesada, together with the other nieces of Morales, filed an action
for specific performance and reconveyance of property against petitioner. They also consigned with the court the
amount of P13,450.00 representing the balance of the purchase price which petitioner allegedly refused to accept.

ISSUE

Whether or not the heirs of Morales have the right over the property?

HELD

YES! The appellate court correctly ruled that petitioner, as successor-in-interest of the City of Cebu, is bound to
respect the contract of sale entered into by the latter pertaining to Lot No. 646A3. The City of Cebu was the owner
of the lot when it awarded the same to respondents predecessor-in-interest, Morales, who later became its owner
before the same was erroneously returned to petitioner under the compromise judgment. The award is tantamount to
a perfected contract of sale between Morales and the City of Cebu, while partial payment of the purchase price and
actual occupation of the property by Morales and respondents effectively transferred ownership of the lot to the
latter. This is true notwithstanding the failure of Morales and respondents to pay the balance of the purchase price.

A sale by public auction is perfected "when the auctioneer announces its perfection by the fall of the
hammer or in other customary manner". It does not matter that Morales merely matched the bid of t he highest bidder
at the said auction sale. The contract of sale was perfected as to Morales, since she merely stepped into the shoes of
the highest bidder.

2B 16-17 SALES AND LEASE Page 80


Consequently, there was a meeting of minds between the City of Cebu and Morales as to the lot sold and its
price, such that each party could reciprocally demand performance of the contract from the other. A contract of sale
is a consensual contract and is perfected at the moment there is a meeting of minds upon the thing which is the
object of the contract and upon the price. From that moment, the parties may reciprocally demand performance
subject to the provisions of the law governing the form of contracts. Thus, the City of Cebu could no longer dispose
of the lot in question when it was included as among those returned to petitioner pursuant to the compromise
agreement in Civil Case No. 238BC.

2B 16-17 SALES AND LEASE Page 81


OFFER AND ACCEPTANCE

1. BEAUMONT VS PRIETO

41 PHIL 670

FACTS

Defendant Benito Legarda was one of the owners of fee simple of the Nagt ajan Hacienda, while defendant Benito
Valdes was his attorney-in-fact and acted as such by virtue of a power of attorney duly executed under notarial seal
presented in the office of the register of deeds.

On December 4, 1911, acting by virtue of the authority, Valdes sent this letter to Borck:

MANILA, December 4, 1911.

Mr. W. BORCK,

Real Estate Agent,

Manila, P.I.

SIR: In compliance with your request I herewith give you an option for three months to buy the property of Mr.
Benito Legarda known as the Nagtahan Hacienda, situated in the district of Sampaloc, Manila, and consisting of
about, 1,993,000 sq. meters of land, for the price of its assessed government valuation.

B. VALDES.

On January 19, 1911, Borck, in writing, accepted the terms of the offer and requested of Valdes to be allowed to
inspect the property, titles, and other documents pertaining to the property, and offered to pay the defendant
immediately as soon as a reasonable examination could be made of said documents.

MANILA, January 19, 1912.

DR. BENITO VALDES,

195 San Sebastian,

City.

2B 16-17 SALES AND LEASE Page 82


SIR: I hereby advise you that I am ready to purchase the Hacienda Nagtahan, situated in the district of Sampaloc
and Nagtahan, Manila, and in the Province of Rizal, consisting of about 1,993,000, square meters of land, property
of Mr. Benito Legarda, for the sum of three hundred and seven thousand (307,000) pesos Ph. c. the price quoted in
the option given my by you.

Full payment will be made on or before the third day of March 1912, provided all documents in connection with the
Hacienda Nagtahan, as Torrens title deed, contracts of leases and other matters be immediately placed at my
disposal for inspection and if such papers have been found in good order.

Very truly yours,

W. BORCK.

Defendants, however, refused to deliver to him the documents and to execute any instrument of conveyance in his
favor. Plaintiff avers that, by reason of such refusal, he incurred great expense and suffered great losses. He filed a
complaint first against Valdes, later amended to include Legarda praying that defendants be ordered to execute a
public instrument in his favor evidencing

the contract obligation, to convey in absolute sale to him the property (specific performance), to render an account
of rents profits collected, or damages in case of impossibility of specific performance.

ISSUE

Whether or not there is an option contract.

HELD

The letter of December 4, 1911 contained, as aforesaid, an offer of sale or a proposal of sale on the partof the
defendant Valdes to the plaintiff Borck, of the Nagtajan Hacienda, for the assessed valuation of the same, effective
during the period of three months counting from the said date. Such proposal or offer was an expression of the will
only of the defendant Valdes, manifested to the plaintiff Borck. In order that such a proposal might have the force of
a contract, it was necessary that the plaintiff Borck's will should have been expressed in harmony with all the terms
of the said proposal.

Consent is shown by the concurrence of the offer and the acceptance of the thing and the cause which are to
constitute the contract. (Art. 1262, Civil Code.)

There is no contract unless, among other requisites, there is consent of the contracting parties. (Art. 1261, par. 1, of
the same code.)

Contracts are perfected by mere consent, and from that time they are binding, not only with regard to the fulfillment
of what has been expressly stipulated, but also with regard to all the consequences which, according to their
character, are in accordance with good faith, use, and law. (Art. 1258, Civil Code.)

2B 16-17 SALES AND LEASE Page 83


Promises are binding in just so far as they are accepted in the explicit terms in which they are made; it not being
lawful to alter, against the will of the promisor, the conditions imposed by him (Decision of the supreme court of
Spain, of November 25, 1858); for only thus may the indispensable consent of the parties exist for the perfection of
the contract. (Decision of the same court, of September 26, 1871.)

An option is an unaccepted offer. It states the terms and conditions on which the owner is willing to sell or lease his
land, if the holder elects to accept them withinthe time limited. If the holder does so elect, he must give notice to
the other party, and the accepted offer thereupon becomes a valid and binding contract. If an acceptance is not made
within the time fixed, the owner is no longer bound by his offer, and the option is at an end. (words and Phrases, vol.
6, p. 5000, citing McMillan vs. Philadelphia Co., 28 Atl., 220; 159 Pa., 142.)

An offer of a bargain by one person to another, imposes no obligation upon the former, unless it be accepted by the
latter, according to the terms in which the offer was made. Any qualification or, or departure from, those terms,
invalidates the offer, unless the same be agreed to by the person who made it. (Eliason et al. vs. Henshaw, 4
Wheaton, 225.)

In order that an acceptance of proposition may be operative it must be unequivocal, unconditional, and without
variance of any sort between it and the proposal, . . . . An absolute acceptance of a proposal, coupled with any
qualification or condition, will not be regarded as a complete contract, because there at no time exists the requisite
mutual assent to the same thing in the same senses. (Bruner et al. vs. Wheaton, 46 Mo., 363.)

As already seen, the plaintiff Borck accepted the offer of sale made to him, or the option of purchase given him by
the defendant Valdes, of the Nagtajan Hacienda, for the assessed valuation of the same, but his acceptance was not
in accordance with the condition with regard to the payment of the price of the property, under which the offer or the
option was made for, while this payment was to be paid in cash, as the plaintiff Borck himself admitted and the
defendant Valdes positively stated in his testimony, and also a provided by law, for the reason that the time was not
fixed in said offer or option when the payment should be made, the plaintiff Borck made the offer to pay the said
price, in the first of them, within the period of five months from December 14, 1911; in the second, within the period
of three months from the same date, and, finally, in the other two documents, within an indefinite period which
could as well be ten days as twenty or thirty or more, counting from t he date when the muniments of title relative to
the said hacienda should have been placed at his disposal to be inspected and he should have found them satisfactory
and, in consequence thereof, the deed of conveyance should have been executed in his favor by the defendant
Valdes.

So that there was no concurrence of the offer and the acceptance as to one of the conditions related to the cause of
the contract, to wit, the form in which the payment should be made. The expression of Borck's will was not in
accordance with all the terms of Valdes' proposal, or, what amounts to the same thing, the latter's promise was not
accepted by the former in the specific terms, in which it was made, and finally, the acceptance of the said proposal
on Borck's part was not unequivocal and without variance of any sort between it and the proposal, because, in view
of the terms in which the payment was offered by Borck, there was variance from the moment in which according to
said terms, in the first two letters, the payment of the price should be made on or before the 1st of May and on or
before the 3d of March, 1912, respectively, that is, within a period limited in those letters, and the offer of payment
was equivocal inasmuch as, by the last two letters, it was made to depend on certain acts as a basis for fixing the
period in which the said payment should have to be made; finally, there was no mutual conformity between the
person who made the proposal or offer, Valdes, and the person who accepted it, Borck, in the same sense with
respect to the form of payment, and Borck deviated from the terms of the proposition with regard to the form of

2B 16-17 SALES AND LEASE Page 84


payment and the record does not show that Valdes assented to such variance.
efendant Benito Legarda was one of the owners of fee simple of t he Nagtajan Hacienda, while defendant Benito
Valdes was his attorney-in-fact and acted as such by virtue of a power of attorney duly executed under notarial seal
presented in the office of the register of deeds.

On December 4, 1911, acting by virtue of the authority, Valdes sent this letter to Borck:

On January 19, 1911, Borck, in writing, accepted the terms of the offer and requested of Valdes to be allowed to
inspect the property, titles, and other documents pertaining to the property, and offered to pay the defendant
immediately as soon as a reasonable examination could be made of said documents.

Defendants, however, refused to deliver to him the documents and to execute any instrument of conveyance in his
favor. Plaintiff avers that, by reason of such refusal, he incurred great expense and suffered great losses. He filed a
complaint first against Valdes, later amended to include Legarda praying that defendants be ordered to execute a
public instrument in his favor evidencing

the contract obligation, to convey in absolute sale to him the property (specific performance), to render an account
of rents profits collected, or damages in case of impossibility of specific performance.

Whether or not there is an option contract.

The letter of December 4, 1911, Exhibit E, contained, as aforesaid, an offer of sale or a proposal of sale on the partof
the defendant Valdes to the plaintiff Borck, of the Nagtajan Hacienda, for the assessed valuation of the same,
effective during the period of three months counting from the said date. Such proposal or offer was an expression of
the will only of the defendant Valdes, manifested to the plaintiff Borck. In order that such a proposal might have the
force of a contract, it was necessary that the plaintiff Borck's will sh ould have been expressed in harmony with all
the terms of the said proposal.

Consent is shown by the concurrence of the offer and the acceptance of the thing and the cause which are to
constitute the contract. (Art. 1262, Civil Code.)

There is no contract unless, among other requisites, there is consent of the contracting parties. (Art. 1261, par. 1, of
the same code.)

Contracts are perfected by mere consent, and from that time they are binding, not only with regard to the fulfillment
of what has been expressly stipulated, but also with regard to all the consequences which, according to their
character, are in accordance with good faith, use, and law. (Art. 1258, Civil Code.)

2B 16-17 SALES AND LEASE Page 85


Promises are binding in just so far as they are accepted in the explicit terms in which they are made; it not being
lawful to alter, against the will of the promisor, the conditions imposed by him (Decision of the supreme court of
Spain, of November 25, 1858); for only thus may the indispensable consent of the parties exist for t he perfection of
the contract. (Decision of the same court, of September 26, 1871.)

An option is an unaccepted offer. It states the terms and conditions on which the owner is willing to sell or lease his
land, if the holder elects to accept them withinthe time limited. If the holder does so elect, he must give notice to
the other party, and the accepted offer thereupon becomes a valid and binding contract. If an acceptance is not made
within the time fixed, the owner is no longer bound by his offer, and the option is at an end. (words and Phrases, vol.
6, p. 5000, citing McMillan vs. Philadelphia Co., 28 Atl., 220; 159 Pa., 142.)

An offer of a bargain by one person to another, imposes no obligation upon the former, unless it be accepted by the
latter, according to the terms in which the offer was made. Any qualification or, or departure from, those terms,
invalidates the offer, unless the same be agreed to by the person who made it. (Eliason et al. vs. Henshaw, 4
Wheaton, 225.)

In order that an acceptance of proposition may be operative it must be unequivocal, unconditional, and without
variance of any sort between it and the proposal, . . . . An absolute acceptance of a proposal, coupled with any
qualification or condition, will not be regarded as a complete contract, because there at no time exists the requisite
mutual assent to the same thing in the same senses. (Bruner et al. vs. Wheaton, 46 Mo., 363.)

As already seen, the plaintiff Borck accepted the offer of sale made to him, or the option of purchase given him by
the defendant Valdes, of the Nagtajan Hacienda, for the assessed valuation of the same, but his acceptance was not
in accordance with the condition with regard to the payment of the price of the property, under which the offer or the
option was made for, while this payment was to be paid in cash, as the plaintiff Borck himself admitted and the
defendant Valdes positively stated in his testimony, and also a provided by law, for the reason that the time was not
fixed in said offer or option when the payment should be made, the plaintiff Borck made the offer to pay the said
price, in the first of them, within the period of five months from December 14, 1911; in the second, within the period
of three months from the same date, and, finally, in the other two documents, within an indefinite period which
could as well be ten days as twenty or thirty or more, counting from the date when the muniments of title relative to
the said hacienda should have been placed at his disposal to be inspected and he should have found them
satisfactory and, in consequence thereof, the deed of conveyance should have been executed in his favor by the
defendant Valdes.

So that there was no concurrence of the offer and the acceptance as to one of the conditions relate d to the cause of
the contract, to wit, the form in which the payment should be made. The expression of Borck's will was not in
accordance with all the terms of Valdes' proposal, or, what amounts to the same thing, the latter's promise was not
accepted by the former in the specific terms, in which it was made, and finally, the acceptance of the said proposal
on Borck's part was not unequivocal and without variance of any sort between it and the proposal, because, in view
of the terms in which the payment was offered by Borck, there was variance from the moment in which according to
said terms, in the first two letters, the payment of the price should be made on or before the 1st of May and on or

2B 16-17 SALES AND LEASE Page 86


before the 3d of March, 1912, respectively, that is, within a p eriod limited in those letters, and the offer of payment
was equivocal inasmuch as, by the last two letters, it was made to depend on certain acts as a basis for fixing the
period in which the said payment should have to be made; finally, there was no mutu al conformity between the
person who made the proposal or offer, Valdes, and the person who accepted it, Borck, in the same sense with
respect to the form of payment, and Borck deviated from the terms of the proposition with regard to the form of
payment and the record does not show that Valdes assented to such variance.

2B 16-17 SALES AND LEASE Page 87


2. FLORENCIA CRONICO VS J. M. TUASON & CO., INC., and CLAUDIO R. RAMIREZ,

G.R. No. L-35272 August 26, 1977

Fernandez, J

FACTS:

Appellant J. M. Tuason & Co. Inc. was the registered owner of a parcel of land in Quezon City and embraced by
TCT No. 49235. In March, 1962, plaintiff Florencia Cronico, offered to buy the lot from the appellant company,
who personally talked to Benjamin F. Bautista, Manager of the Real Estate Department of Gregorio Araneta, Inc.,
the appellant company's attorney-in-fact, proposing to buy the lot. She was required to present proofs to show her
rights to the lot. On March 8, 1962, Florencia Cronico exhibited certain documents showing her priority rights to
buy the lot. In March 1962, defendant-appellant Claudio Ramirez also learned that the lot in question was being sold
by the appellant company. The occupants who also had priority rights to buy the land, who informed Ramirez of the
sale, waived their rights.

In the same month, March, 1962, plaintiff Cronico and defendant- appellant Ramirez sent separate individual letters
to appellant company wherein they expressed their desire to purchase the land and requested information concerning
the area, the price and other terms and conditions of the contract to sell. On March 20, 1962, the appellant company
sent separate reply letters to prospective buyers including plaintiff Cronies and defendant -appellant Ramirez. They
were dropped in the Manila Post Office at 11:00 in the morning of March 21, 1962 by registered mail. It so
happened that plaintiff Cronico went to the appellant company's office on March 21, 1962, and she was informed
that the reply letter of the appellant company to prospective buyers o f the same lot had been mailed. With this
information, plaintiff Cronies and Mary E. Venturanza went to the post office in Manila and she was able to get the
letter at about 3:30 in the afternoon of the same date. After she got the letter, plaintiff Cronie s and Mary E.
Venturanza went directly to the office of Gregorio Araneta Inc., Escolta, Manila, and presented the letter to
Benjamin Bautista, Head of the Real Estate Department of said company. Since she had no money, plaintiff Cronies
requested Mary E. Venturanza to issue a check in the amount of P33,572.00 to cover the down payment for the lot.
However, Benjamin Bautista did not accept the cheek. He advised plaintiff Cronies that it is Gregorio Araneta II
who would decide whose offer to buy may be accepts after the appellant company receives the registry return cards
attached to the registered letters sent to the offerors.

On March 22, 1962, between 10:00 and 11:00 a.m., appellant Ramirez received from the post office at San
Francisco del Monte, Quezon City, the reply letter of the appellant company dated March 20, 1962, wherein it stated
that Lot 22, Block 461, Sta. Mesa Heights Subdivision, was available for sale under the conditions therein set forth
and that the said lot was being offered for sale on a first come first serve basis. Appellant Ramirez proceeded to the
office of Benjamin Bautista in the same morning stating that he accepted the conditions stated in the appellant
company's letter. Benjamin Bautista advised appellant Ramirez to wait for the decision of Gregorio Araneta II. The
next day, March 23, 1962, appellant Ramirez presented his letter to the appellant company confirming his verbal
acceptance of the terms and conditions in connection with the sale. On March 31, 1962, Atty. Jose E. Patang co in
behalf of appellant Ramirez wrote the appellant company requesting the early execution of the proper contract to sell
over Lot No. 22. A check in the amount of P33,572 was enclosed in the letter to cover the down payment for said
lot. The request was favorably considered.

On April 2, 1962, the J. M. Tuason & Co. Inc., and Claudio R. Ramirez executed a contract to sell whereby the
appellant company agreed to sell to appellant Ramirez the lot in question for a total price of P167,896.00 subject to
the terms and conditions therein set forth. Meanwhile, on March 27, 1962, the appellant company received a letter in
behalf of Florencia Cronies requesting that the lot subject of litigation be 'sold to her. She tendered a check to cover

2B 16-17 SALES AND LEASE Page 88


the down payment which was, however, returned. On April 4, 1962, the appellant company sent a letter to the
plaintiff-appellee informing her that it had decided to sell the lot in question to appellant Ramirez. Florencia Cronico
filed a suit against the defendants -appellants J. M. Tuason & Co., Inc. and Claudio Ramirez to annul and set aside
the contract to sell executed by and between appellant company and appellant Ramirez.

ISSUE:

Whether or not Cronico can annul the contract?

HELD

No. It is a fact that the petitioner, Florencia Cronico upon being tipped by Benjamin Bautista, head of the Real
Estate Department of Gregorio Araneta Inc., that the reply letters of the appellant company were already placed in
the mails on March 21, 1962 at 11:00 o'clock in the morning, immediately went to the Manila post office and
claimed the registered letter addressed to her without waiting for the ordinary course for registered mails to be
delivered. The petitioner took delivery of the registered letter addressed to her at the entry section of the Manila post
office. While this procedure may be tolerated by the postal authorities, the act of the petitioner in taking delivery of
her letter at the entry section of the Manila post office without waiting for said letter to be delivered to her in due
course of mail is a violation of the "first come first served" condition imposed by the respondent J. M. Tuason & Co.
Inc., acting through Gregorio Araneta Inc.

The petitioner cannot claim that she had accepted the promise before it was withdrawn because, as stated above, she
had violated the condition of "first, come, first served" Moreover, it was only on March 27, 1962 that the respondent
company received a letter from counsel of the petitioner requesting that the lot subject of this litigation be sold to
her. The respondent, Claudio R. Ramirez, had on March 23, 1962, confirmed in writing his verbal acceptance of the
terms and

Thus, the contention of the petitioner that she has become the obligee or creditor of the respondent company because
she was the first to comply with the terms of the letter-offer has no merit. Her so-called acceptance has no effect
because she violated of the condition.

2B 16-17 SALES AND LEASE Page 89


3. SPOUSES TRINIDAD AND EPIFANIO NATINO vs. THE INTERMEDIATE APPELLATE COURT,
THE RURAL BANK OF AGUILAR, INC. AND THE PROVINCIAL SHERIFF EXOFFICIO OF
PANGASINAN
GR NO. 73573 May 23, 1991

Davide Jr, J

FACTS
On 12 October 1970 petitioners executed a real estate mortgage in favor of respondent bank as security for a loan of
P2,000.00. Petitioners failed to pay the loan on due date. The bank applied for the extrajudicial foreclosure of the
mortgage. At the foreclosure sale, the respondent bank was the highest and winning bidder. A certificate of sale was
executed in its favor by the sheriff and the same was reg istered with the Office of the Register of Deeds. The
certificate of sale, a copy of which was furnished the petitioners by registered mail, expressly provided that the
redemption period shall be two years from the registration thereof.
Since no redemption was made by petitioners within the twoyear period, the sheriff issued a Final Deed of
Sale.Petitioners, however, claimed that they were granted by respondent bank an extension of the redemption
period but the latter denied it.
Respondent bank filed a petition for a writ of possession, which petitioners later opposed on the ground that they
had consigned the redemption money of P4,000.00 on 12 December 1979. The court rejected the opposition and
issued the writ of possession. However, to prevent its execution, petitioners instituted with the then Court of First
Instance of Pangasinan a complaint against respondent bank and the ExOfficio Provincial Sheriff for the annulment
of the aforementioned final deed of sale and for the issuance of a writ of preliminary injunction. Petitioners alleged
that the final deed of sale was prematurely issued since they were granted an extension of time to redeem the
property. The trial court ruled that it was highly unlikely that the plaintiffs who are personally known to the
president and manager herself, and from whom she had to hire trucks, would not have made any move or offer to
redeem the property within the redemption period. The presumption is that they exercised ordinary care of their
concerns. Further, the trial court ruled that there was indeed an extension of the redemption period. there was an
agreement between them and the defendant bank through its personnel and its president and manager, acting as its
agents to extend the period for redemption for the plaintiffs. However, the plaintiffs were not given a specific time to
pay and redeem but were given by the President and Manager of the bank such time when their means permit them
to do so. The court deduced that since no time was specified but the fulfillment of t he obligation was not solely
reliant on the will of then plaintiffs, then the court may be asked to fix a period.
Upon appeal to the IAC, the appellate court reversed the trial court decision, holding that there was no sufficient
evidence to indicate that then appellees had tendered the redemption money within the redemption period which was
refused by the bank. The IAC denied motion for reconsideration, hence this petition.

ISSUE
Whether or not the petitioners may validly redeem the property in the case at bar, there being an extension of the
redemption period given to them

HELD
NO. The second paragraph of Article 1479 of the Civil Code provides that An accepted unilateral. promise to buy
or to sell a determinate thing for a price certain is binding u pon thevpromissory if the promise is supported by a
consideration distinct from the price.
The decision of the trial court is drawn from inferences which are not supported by adequate or
sufficient facts or is based on erroneous assumptions as it is remarkably silent as to the
dates when petitioner Epifanio Natino went to the respondent bank to talk with a bank personnel to offer to pay the
loan. If indeed the offer was made within the redemption period, but the Bank refused to accept the

2B 16-17 SALES AND LEASE Page 90


redemption money, petitioners should have made the tender to the sheriff who made the sale and who then had the
duty to accept the tender and execute the certificate of redemption. There was no such tender to the Sheriff. Again, if
indeed this occurred during the redemption period, then, as correctly pointed out by respondent IAC, it was not
necessary to ask for extension of the period to redeem.
In respect to the alleged assurance given by Mrs. Brodeth, the President and Manager of the Bank, sometime in May
of 1978 to the effect that petitioners can redeem the property as soon as they have the money, it is obvious that this
took place after the expiration of the redemption period. The IAC was correct in ruling that this could only relate to
the matter of resale of the property, not redemption. Furthermore, even assuming for the sake of argument that Mrs.
Brodeth gave the assurance, the same could bind the bank only if its Board of Directors approved or ratified it. No
evidence was offered to prove such action by the Board. M oreover, Mrs. Brodeth denied that during that meeting in
May 1978 she made the assurance according to her petitioner Epifanio neither mentioned the loan nor offered to
redeem, although earlier he was told that to 'redeem" the property he should pay P30,000.00. The latter statement
supports the conclusion of the IAC that this was the Bank's offer for the resell (not redemption of the property),
which, logically took place after the expiration of the redemption period.
Even if Mrs. Brodeth is to be understood to have promised to allow the petitioners to buy the property at any time
they have the money, the Bank was not bound by the promise not only because it was not approved or ratified by the
Board of Directors but also because, and more decisively, it was a promise unsupported by a consideration distinct
from the repurchase price.

2B 16-17 SALES AND LEASE Page 91


4. SOUTHWESTERN SUGAR AND MOLASSES VS ATLANTIC GULF
G.R. No. L-7382

June 29, 1955

FACTS
March 24, 1953: Atlantic Gulf granted Southwestern Sugar an option to bu y its barge no. 10 for Php30,000 to be
exercised within a period of 90 days.
May 11, 1953, SS wrote to AG advising the latter that it wanted "to exercise our option at your earliest convenience"
and requested that it be notified as soon as the barge was av ailable.
May 12, 1953, the AG replied stating that their understanding was that the "offer of option" is to be a cash
transaction and to be effected "at the time the lighter is available", and, on June 25, 1953, reiterating the
unavailability of the barge, it further advised the SS that since there is still further work for it, and as this situation
still applies" the barge could not be turned over to the latter company.
June 27, 1953 SS instituted action to compel AG to sell the barge, deposited with the c ourt a check in the amount of
Php30,000 but was later withdrawn.. June 29, 1953, AG withdrew the offer of option with due notices to SS.

ISSUE

Whether Ag may be compelled to proceed with the sale pursuant to the offer of option.

HELD
The general rule regarding offer and acceptance is found in Art. 1324 which states that when the offerer gives to the
offeree a certain period to accept, "the offer may be withdrawn at any time before acceptance" except when the
option is founded upon consideration. On the other hand, Art. 1479 provides that "an option to sell", or a "promise to
buy or to sell" to be valid must be "supported by a consideration distinct from the price." In other words, "an
accepted unilateral promise" can only have a binding effect if suppo rted by a consideration, which means that the
option can still be withdrawn, even if accepted, if the same is not supported by any consideration.
Since the case on hand is an option to sell or a promise to buy or sell, the specific provision which speaks o f the said
options should be applied. Here it is not disputed that the option is without consideration. It can thereforebe
withdrawn notwithstanding the acceptance made of it by appellee.

2B 16-17 SALES AND LEASE Page 92


5. NAVARRO VS SUGAR PRODUCERS

G.R. No. L-12888 April 29, 1961

Barrera, J

FACTS

September 19,1956, defendant offered plaintiff the sale of 15,000 to 20,000 metric tons of molasses at P50.00 per
metric ton, giving him up to noon of September 24th, 1956 within which to accept the offer, else he shall n egotiate
with other buyers. Five minutes before noon of September 24, 1956, plaintiff formally accepted the offer of sale and
informed the latter in writing that he binds himself to purchase 20,000 metric tons of Philippine molasses, 185-
degrees specific gravity, 60% sugar by invert, at P50.00 per metric ton.

Later that day, defendant advised plaintiff that it committed a typographical error indicating the specific gravity of
the molasses at 185-degrees which should be only 85-degrees, the latter being the high for molasses at 60% sugar by
invert. After such, not a single word was heard from plaintiff which caused the defendant to negotiate with foreign
buyers.Three days later, defendant informed plaintiff of additional terms and conditions for the sale, t o which
plaintiff expressed willingness and readiness to accept though with changes as to price (from P50 to P32 per metric
ton). Defendant turned down such and informed plaintiff that they would no longer continue with the sale, and he
shall negotiate with other buyers.

Plaintiff prayed that the defendant be ordered to comply with their obligation (to sell), else pay the plaintiff for
damages for non-compliance with contractual obligations.

Defendant claims that there is no cause of action as there is no binding contract between them because an accepted
unilateral promise can only have a binding effect if supported by a consideration.

ISSUE

Whether or not the defendant is bound to sell to the plaintiff as per the acceptance done five minutes before noo n of
September 24, 1956

HELD: NO.

The Supreme Court held that because defendant's promise to sell is not supported by any consideration distinct from
the price, said promise can, therefore, be withdrawn notwithstanding the acceptance. Plaintiff's complain t does not,
hence state a cause of action. Additionally, the Supreme Court noted that the offer as well as the acceptance lacked
an essential element: the manner of payment of the purchase price. Due to such, there was no complete meeting of
the minds of the parties necessary for the perfection of a contract of sale. Defendant was justified in withdrawing his
offer to sell the molasses.

2B 16-17 SALES AND LEASE Page 93


6. SANCHEZ VS RIGOS

45 SCRA 368, JUNE 14, 1972

Concepcion, CJ

FACTS:

On April 3, 1961, Nicolas Sanchez and Severina Rigos executed an instrument, entitled Option to Purchase,
whereby Mrs. Rigos agreed, promised and committed to sell to Sanchez, for the sum of P1,510.00, a parcel of land
situated in the barrios of Abar and Sibot, municipality of San Jose, within 2 years with the understanding that said
option shall be deemed terminated and elapsed. if Sanchez shall fail to exercise his right to buy the property
within the stipulated period. Several attempts to pay the amount was rejected by Mrs. Rigos. Sanchez then
deposited the P1,510.00 to the Court of First Instance and filed a case for specific performance and damages.

CFI RULED IN FAVOR OF SANCHEZ: There was a valid acceptance before the offer was withdrawn, therefore
there is a perfected contract of sale.

RIGOS CONTENTION: The option contract is a unilateral promise to sell, and the same being unsupported by
any valuable consideration, it is therefore null and void.

ISSUE

Whether or not the acceptance to the offer is valid even though the option contract is not supported by a
consideration.

HELD

YES, the acceptance is valid. It is true that under article 1324 of the new Civil Code, the general rule regarding
offer and acceptance is that, when the offerer gives to the offeree a certain period to accept, t he offer may be
withdrawn at any time before acceptance except when the option is founded upon consideration, but this general
rule must be interpreted as modified by the provision of article 1479 above referred to, which applies to a promise
to buy and sell specifically. As already stated, this rule requires that a promise to sell to be valid must be supported
by a consideration distinct from the price.

However, this Court itself, in the case of Atkins, Kroll and Co., Inc. v, Cua Hian Tek, saw no distin ction
between Articles 1324 and 1479 of the Civil Code and applied the former where a unilateral promise to sell similar
to the one sued upon here was involved, treating such promise as an option which, although not binding as a
contract in itself for lack of a separate consideration, nevertheless generated a bilateral contract of purchase
and sale upon acceptance. Speaking through Associate Justice, later Chief Justice, Cesar Bengzon, this Court said:

Furthermore, an option is unilateral: a promise to sell at the price fixed whenever the offeree should
decide to exercise his option within the specified time. After accepting the promise and before he exercises
his option, the holder of the option is not bound to buy. He is free either to buy or not to buy later. In this
case however, upon accepting herein petitioners offer a bilateral promise to sell and to buy ensued, and the

2B 16-17 SALES AND LEASE Page 94


respondent ipso facto assumed the obligation of a purchaser. He did not just get the right subsequently to
buy or not to buy. It was not a mere option then it was bilateral contract of sale.

In other words, since there may be no valid contract without a cause or consideration, the promisor is not bound by
his promise and may, accordingly, withdraw it. Pending notice of its withdrawal, his accepted promise partakes,
however, of the nature of an offer to sell which, if accepted, results in a perfected contract of sale.

2B 16-17 SALES AND LEASE Page 95


7. RURAL BANK OF PARANAQUE VS CA

No. L-62051. March 18, 1985.

FACTS:

Isidra Remolado, the owner of the lot located at 41 Molave Street, United Paraaque Subdivision, Barrio Ibayo,
Paraaque, Rizal, mortgaged it in 1996 to the Rural Bank of Paraaque, Inc. as security for a loan of P15,000. She
paid the loan. On April 17, 1971 she mortgaged it again to the bank She eventually secured loans totalling P18,000.
The loans become overdue. The bank foreclosed the mortgage on July 21, 1972 and bought the property at the
foreclosure sale for P22,192.70. The one-year period of redemption was to expire on Aug ust 21, 1973. On August 8,
1973 the bank advised Remolado that she had until August 23 to redeem the property . On August 9, 1973 before the
expiration of the one-year redemption period, the bank gave her a statement showing that she should pay P25,491.96
for the redemption of the property on August 23. No redemption was made on that date. On September 3, 1973 the
bank consolidated its ownership over the property. Remolados title was cancelled. A new title was issued to the
bank on September 5. On September 24, 1973, the bank gave Remolado up to ten oclock in the morning of October
31, 1973, or 37 days, within which to repurchase the property. On October 26, 1973 Remolado and her daughter,
promised to pay the bank P33,000 on October 31 for the repurchase o f the property. Even if it is assumed that the
banks commitment to resell the property was accepted by Remolado, that option was not supported by a
consideration distinct from the price (Art. 1479, Civil Code). Lacking such consideration, the option is vo id
(Southwestern Sugar & Molasses Co. vs. Atlantic Gulf & Pacific Company, 97 Phil. 249). Remolado did not
repurchase the property on October 31. Five days later, or on November 5, Remolado and her daughter delivered
P33,000 cash to the banks assistant manager as repurchase price, The amount was returned to them the next day,
the bank was no longer willing to allow the repurchase. November 6, Remolado filed an action to compel the
bank to reconvey the property to her for P25,491.96 plus interest and other charges and to pay P35,000 as damages.
November 15, the bank sold the property to Pilar Aysip for P50,000. Trial Court ruled in favor of Remolado,
ordering the bank to return the property. The CA affirmed. Hence this petition.

ISSUE

Whether or not there was binding agreement for the propertys repurchase

HELD

None. Even on the assumption that the bank should be bound by its commitment to allow repurchase on or before
October 31, 1973, still Remolado had no cause of action because she did not repurchase the property on that
date.

In the instant case, the bank acted within its legal rights when it refused to give Remolado any extension to
repurchase after October 31, 1973, It had given her about two years to liquidate her obligation. She failed to do so.

Wherefore, the Appellate Courts judgment is reversed and set aside.

2B 16-17 SALES AND LEASE Page 96


8. NIETES VS CA

G.R. No. L32873 August 18, 1972

Concepcion, J

FACTS

In 1959, petitioner and respondent Garcia entered into a Contract of Lease with Option to Buy. One of the
conditions of which is that should the lessor agrees to give the lessee an option to buy the land and the school
building for a price of P100,000 within the period of the Contract of Lease. Nietes had made payments totaling only
P24,757.

In 1964, respondents couns el wrote Nietes informing him that the contract is to be rescinded for his failure to
comply with the conditions set forth in the contract which for the respondent are the principal moving factors which
had induced the lessor in agreeing with the terms embodied in the contract of lease.

In response to this, the counsel of Nietes informed respondent that his client was going to exercise his OPTION to
buy the land and building subject matter of the lease. Nietes deposited with Agro -Industrial Bank checks amounting
to 84,860.50 as balance of the purchase price of the property. Thereafter, Nietes commenced an action for specific
performance against respondent.

The trial court ruled in favor of Nietes however, the Court of Appeals reversed the trial courts decision on the
ground that under the contract between the parties, the full purchase price must be paid before the option be
exercised.

ISSUE:

Whether or not the option to buy can be exercised despite the failure to pay for the full purchase price (yes)

HELD:

The contract does not say that Nietes had to pay the stipulated price of P100,000 before exercising his option to buy
the property in question. Accordingly, said option is governed by the general principles on obligations, pursuants to
which:

In reciprocal obligations, neither party incurs in delay if the other does not comply or is not ready to comply in a
proper manner with what is incumbent upon him. From the moment one of the parties fulfills his obligation, delay
by the other begins.

In the case of an option to buy, the creditor may validly and effectively exercise his right by merely advising the
debtor of the former's decision to buy and expressing his readiness to pay the stipulated price, provided that the same
is available and actually delivered to the debtor upon execution and delivery by him of the corresponding deed of
sale. Unless and until the debtor shall have done this the creditor is not and cannot be in default in the discharge of
his obligation to pay. In other words, notice of the creditor's decision to exercise his option to buy need not be
coupled with actual payment of the price, so long as this is delivered to the owner of the property upon performance
of his part of the agreement. Nietes need not have deposited, therefore, with th e Agro-Industrial Bank checks

2B 16-17 SALES AND LEASE Page 97


amounting altogether to P84,860.50 on July 26, 1965, and the withdrawal thereof soon after does not and cannot
affect his cause of action in the present case. In making such deposit, he may have had the intent to show his ability
to pay the balance of the sum due to Dr. Garcia as the sale price of his property. In short, said deposit and its
subsequent withdrawal cannot affect the result of the present case.

It is, consequently, Our considered opinion that Nietes had validly an d effectively exercised his option to buy the
property of Dr. Garcia, at least, on December 13, 1962, when he acknowledged receipt from Mrs. Nietes of the sum
of P2,200 then delivered by her "in partial payment on the purchase of the property" described in the "Contract of
Lease with Option to Buy"; that from the aggregate sum of P29,957.00 paid to him up to that time, the sum of
P12,708.33 should be deducted as rental for the period from June 1960 to December 13, 1962, or roughly thirty (30)
months and a half, thereby leaving a balance of P17,248.67, consisting of P12,291.67, representing the rentals for
the unused period of the lease, plus P4,957.00 paid in excess of said rental and advanced solely on account of the
purchase price; that deducting said sum of P17,248.67 from the agreed price of P100,000.00, there results a balance
of P82,751.33 which should be paid by Nietes to Dr. Garcia, upon execution by the latter of the corresponding deed
of absolute sale of the property in question, free from any lien or encumbrance whatsoever, in favor of Nietes, and
the delivery to him of said deed of sale, as well as of the owner's duplicate of the certificate of title to said property;
and that Dr. Garcia should indemnify Nietes in the sum of P2,500 as and for attorney's fees.

2B 16-17 SALES AND LEASE Page 98


9.JOSE MAS, administrator of the estate of FRANCISCA HILARIO, deceased, v. TIMOTEO LANUZA
AND WIFE, ANDREA FLORES, ET AL.,

5 PHIL 457

FACTS

Judgment was rendered in favor of the plaintiff for the possession of lot No. 120, Calle Clavel, Tondo, Manila, and
declaring said lot to be the property of the estate of which the plaintiff is administrator. Plaintiff introduced in
evidence an agreement in writing executed on the 4th of July, 1882, and signed by the appellants and by Fra ncisca
Hilario, whereby the latter, since deceased, gave the appellants permission to enter upon the land in question, and to
occupy it. Defendants admit the execution of the agreement; they discovered later that Francisca Hilario held the
property merely as administratrix for the true owner. On the 7th of December, 1892, they loaned the true owner, one
Joaquin Lao-Jico, 200 pesos, and took from him an agreement in writing whereby he promised to sell them the said
property for 500 pesos, an agreement which was never consummated however, because he died a short time
thereafter.

ISSUE
Whether or not the contract to sell constituted and offer and acceptance, which in turn passed title and dominion
over the property.

HELD
The agreement to sell did not pass title or dominion over the property, and only gave the defendants a right to
demand the fulfillment of the terms thereof, should it appear that the instrument is what it purports to be, and that the
title was in fact in the said Joaquin Lao-Jico.

2B 16-17 SALES AND LEASE Page 99


10. BARRETTO VS STA MARINA

GR No. L-8169 Dec. 29, 1913

Torres, J.:

FACTS

Respondent, a Spaniard, was owner and proprietor of La Insular Cigar and Cigarette Factory. Petitioner was his
agent in managing the business in the Philippines. His services were un der a contract, in which he was to be paid
P37,000 per annum. He alleges that on Jan. 8, 1910, he was terminated without justification by the respondent. The
facts show that on Jan. 2, 1909, petitioner wrote a letter, saying that he wishes to resign as he was unable to go after
a certain Uy Yan, who owes the company P97,000. Respondent took a long time before appointing a new agent and
manager, J. McGavin. Petitioner demands compensation for his services before Jan. 8, 1910.

ISSUE

Whether or not petitioner is entitled to payment of compensation

HELD

No. Even though the acceptance took a long time, petitioner himself resigned, through his letter. By placing his job
at the disposal of the owner, he has already given up his right to compensation.

2B 16-17 SALES AND LEASE Page 100


11. UP VS PHILAB (ALSO: RE: FORM)

GR NO. 152411 September 29, 2004

FACTS
UP decided to construct an integrated system of research organization known as the Research Complex. As part of
the project, laboratory equipment and furniture were purchased for the National Institute of Biotechnology and
Applied Microbiology (BIOTECH) at the UP Los Baos. Providentially, the Ferdinand E. Marcos Foundation
(FEMF) came forward and agreed to fund the acquisition of the laboratory furniture, including th e fabrication
thereof. Dr. Padolina of BIOTECH arranged for Philippine Laboratory Industries, Inc. (PHILAB), to fabricate the
laboratory furniture and deliver the same to BIOTECH for the BIOTECH Building Project, for the account of the
FEMF.

Padolina wrote Lirio (of FEMF) and requested for the issuance of the purchase order and downpayment for the
office and laboratory furniture for the project. Padolina assured Lirio that the contract would be prepared as soon as
possible before the issuance of the purchase orders and the downpayment for the goods, and would be transmitted to
the FEMF as soon as possible.

Padolina informed Hector Navasero, the President of PHILAB, to proceed with the fabrication of the laboratory
furniture, per the directive of FEMF Executive Assistant Lirio. Padolina also requested for copies of the shop
drawings and a sample contract for the project, and that such contract and drawings had to be finalized before
the down payment could be remitted to the PHILAB the following week. Howev er, PHILAB failed to forward any
sample contract. Subsequently, PHILAB made partial deliveries of office and laboratory furniture to BIOTECH.
FEMF remitted P600,000 to PHILAB as downpayment for the laboratory furniture for the BIOTECH project. Then
FEMF made another partial payment of P800,000 to PHILAB. The remittances were in the form of checks drawn by
FEMF and delivered to PHILAB, through Padolina.

On October 16, 1982, UP, represented by its Executive Officer, Rolando Gapud, executed a Memorandum of
Agreement (MOA) in which FEMF agreed to grant financial support and donate sums of money to UP for the
construction of buildings, installation of laboratory and other capitalization for the project, not to exceed
P29,000,000.00.

In the meantime, Navasero promised to submit the contract for the installation of laboratory furniture to BIOTECH
but it failed to do so. Instead of submitting the said contract, PHILAB submitted to BIOTECH an accomplishment
report on the project as of February 28, 1983, and requested payment thereon. PHILAB had completed 78% of the
project, amounting to P2,288,573.74 out of the total cost of P2,934,068.90. The FEMF had already paid forty
percent (40%) of the total cost of the project.

PHILAB submitted to BIOTECH Invoice No. 01643 in the amount of P702,939.40 for the final payment of
laboratory furniture. BIOTECH forwarded the invoice to Lirio on December 18, 1984 for its payment. Lirio, in turn,
forwarded the invoice to Gapud but FEMF failed to pay the bill. Biotech wrote again but FEMD again failed to pay
until BIOTECH demanded for interest.

President Marcos was ousted from office during the February 1986 EDSA Revolution. On March 26, 1986,
Navasero wrote BIOTECH requesting for its much-needed assistance for the payment of the balance already due
plus interest of P295,234.55 for its fabrication and supply of laboratory furniture. PHILAB wrote President Corazon
C. Aquino asking her help to secure the payment of the amount due from the FEMF.

2B 16-17 SALES AND LEASE Page 101


In the meantime, the PCGG wrote UP reques ting for a copy of the relevant contract and the MOA for its perusal.
But Navasero informed them that PHILAB and FEMF did not execute any contract regarding the fabrication and
delivery of laboratory furniture to BIOTECH. Exasperated, PHILAB filed a compla int for sum of money and
damages against UP.

In its answer, UP denied liability and alleged that PHILAB had no cause of action against it because it was merely
the donee/beneficiary of the laboratory furniture in the BIOTECH; and that the FEMF, which fund ed the project,
was liable to the PHILAB for the purchase price of the laboratory furniture. UP specifically denied obliging itself to
pay for the laboratory furniture supplied by PHILAB.

The CA reversed and set aside the decision of the RTC (which dismis sed the case) and held that there was never a
contract between FEMF and PHILAB. Consequently, PHILAB could not be bound by the MOA between the FEMF
and UP since it was never a party thereto. The appellate court ruled that, although UP did not bind itself t o pay for
the laboratory furniture; nevertheless, it is liable to PHILAB under the maxim: No one should unjustly enrich
himself at the expense of another.

ISSUE

Whether or not the Court of Appeals erred when it failed to apply the law on contracts between PHILAB and the
Marcos Foundation.

HELD Yes.
We agree with the petitioner that, based on the records, an implied -in-fact contract of sale was entered into between
the respondent and FEMF. A contract implied in fact is one implied from facts and circumstan ces showing a mutual
intention to contract. It arises where the intention of the parties is not expressed, but an agreement in fact creating an
obligation. It is a contract, the existence and terms of which are manifested by conduct and not by direct or explicit
words between parties but is to be deduced from conduct of the parties, language used, or things done by them, or
other pertinent circumstances attending the transaction. To create contracts implied in fact, circumstances must
warrant inference that one expected compensation and the other to pay. An implied -in-fact contract requires the
parties intent to enter into a contract; it is a true contract. The conduct of the parties is to be viewed as a reasonable
man would view it, to determine the existence or not of an implied-in-fact contract. The totality of the acts/conducts
of the parties must be considered to determine their intention. An implied -in-fact contract will not arise unless the
meeting of minds is indicated by some intelligent conduct, act or sign

In this case, the respondent was aware, from the time Padolina contacted it for the fabrication and supply of the
laboratory furniture until the go-signal was given to it to fabricate and deliver the furniture to BIOTECH as
beneficiary, that the FEMF was to pay for the same. Indeed, Padolina asked the respondent to prepare the draft of
the contract to be received by the FEMF prior to the execution of the parties (the respondent and FEMF), but
somehow, the respondent failed to prepare one. The resp ondent knew that the petitioner was merely the donee-
beneficiary of the laboratory furniture and not the buyer; nor was it liable for the payment of the purchase price
thereof. From the inception, the FEMF paid for the bills and statement of accounts of th e respondent, for which the
latter unconditionally issued receipts to and under the name of the FEMF.

Admittedly, the respondent sent to the petitioner its bills and statements of accounts for the payments of the
laboratory furniture it delivered to the petitioner which the petitioner, through Padolina, transmitted to the FEMF for
its payment. However, the FEMF failed to pay the last statement of account of the respondent because of the onset

2B 16-17 SALES AND LEASE Page 102


of the EDSA upheaval. It was only when the respondent lost all hope of collecting its claim from the government
and/or the PCGG did it file the complaint against the petitioner for the collection of the payment of its last delivery
of laboratory furniture.

2B 16-17 SALES AND LEASE Page 103


EARNEST MONEY

1. VILLANGCO REALTY VS BORMAHECO

65 SCRA 352

FACTS

Spouses Cervantes are owners of parcels of land located in Makati. The parcels of land were mortgaged to the
Development Bank of the Philippines (DPB), which was fully paid on July 10, 1969. Francisco Cervantes is the
President of Bormaheco Inc. The said company used the aforementioned lots for business purposes. The property
are situated adjacent to the property of herein petitioner company. In 1964, the lots were then negotiated for its sale
to the brothers Villongco, through the intervention of Edith Perez de Tagle, a real estate-broker. In the course of the
negotiation, Cervantes did not inform De Tagle and Villongco that the lots were mortgaged to DBP. After
negotiations, the sale was perfected and Villongco sent a check o f P100,000 as earnest money being subject to the
terms and conditions of Bormaheco. Cervantes then returned the earnest money having no interest to sell the
property. Petitioners filed an action for specific performance for the sale of the lots. Respondent argued that there
was no perfected sale because they returned the earnest money, thus there was no meeting of the minds.

ISSUE

Whether or not payment of earnest money presupposes a perfection of contract of sale.

HELD

Yes

The contract of sale is perfected at the moment there is meeting of minds upon the thing which is the object of the
contract and upon the price. From that moment, the parties may reciprocally demand performance, subject to the
provisions of the law governing the form of contracts Bormahecos acceptance of Villonco Realty Companys offer
to purchase the Buendia Avenue property, as shown in Teofilo Villoncos letter dated March 4, 1964, indubitably
proves that there was a meeting of minds upon the subject matter and consideration of the sale. Therefore, on that
date the sale was perfected. Earnest money provides to be the apparent manifestation of the intent of the buyer for
the contract's perfection as it constitutes the price.

2B 16-17 SALES AND LEASE Page 104


2. VELASCO VS CA

GR No 301018; June 29, 1973

Castro, J

FACTS:

On November 3, 1968, the Court of First Instance of Quezon City rendered a decision dismissing the complaint filed
by the petitioner against private respondent Magdalena Estate, Inc. for the purpose of compelling specific
performance by the respondent of an alleged deed of sale of a parcel of residential land for the total purchase price
of P100,000 in favor of the petitioners. The basis for the dismissal of the complaint was that the alleged purchase
and sale agreement "was not perfected".

It is alleged by the plaintiff (now referred to as petitioners) that the agreement was that the plaintiff was to
give a down payment of P10,000.00 to be followed by P20,000.00 and the balance would be paid in installments.
The plaintiff allegedly paid P10,000 and thereafter, tendered to the defendant the payment of the additional
P20,000.00 to complete the P30,000.00 the defendant refused to accept and that eventually it likewise refused to
execute a formal deed of sale obviously agreed upon. The plaintiff offered in evidence a receipt for the down
payment of P10,000 allegedly issued by Magdalena Estate, Inc. to prove that there was a perfected sale as
stipulated: earnest money for the purchase of Lot 15, Block 7, Psd-6129, Area 2,059 square meters including
improvements thereon P10,000.00." At the bottom of the receipt, the following appears: "Agreed price:
P100,000.00, P30,000.00 down payment, bal. in 10 years."

The defendant (now referred to as private respondents) alleged that a portion of the prope rty in question
was being leased by a certain Socorro Velasco indicated her desire to purchase the lot; that the defendant indicated
its willingness to sell the property to her at the price of P100,000.00 under the condition that a down payment of
P30,000.00 be made, P20,000.00 of which was to be paid on November 31, 1962; that on November 29, 1962
Socorro Velasco offered to pay P10,000.00 as initial payment instead of the agreed P20,000.00 but because the
amount was short of the alleged P20,000.00 the same was accepted merely as deposited; that Socorro Velasco
failed to complete the down payment of P30,000.00 and neither has she paid any installments on the balance of
P70,000.00 up to the present time; that it was only on January 8, 1964 that Socorro Velasc o tendered payment of
P20,000.00, which offer the defendant refused to accept because it had considered the offer to sell rescinded on
account of her failure to complete the down payment on or before December 31, 1962.

CA agreed with the respondent's contention that no contract of sale was perfected because the minds of the
parties did not meet "in regard to the manner of payment." CA ruled that there was a lack of complete "agreement in
regard to the manner of payment" of the lot in question.

ISSUE:

2B 16-17 SALES AND LEASE Page 105


Whether or not the P10,000 given by petitioner was an earnest money to make a consummated sale

HELD:

NO. The Court ruled that material averments show that the petitioners themselves admit that they and the respondent
still had to meet and agree on how and when the down-payment and the installment payments were to be paid. Such
being the situation, it cannot, therefore, be said that a definite and firm sales agreement between the parties had been
perfected over the lot in question. Indeed, the Court has already ruled before that a definite agreement on the manner
of payment of the purchase price is an essential element in the formation of a binding and unforceable contract of
sale. The fact, therefore, that the petitioners delivered to the respondent the sum of P10,000 as part of the down-
payment that they had to pay cannot be considered as sufficient proof of the perfection of any purchase and sale
agreement between the parties herein under article 1482 of the new Civil Code, as the petitioners themse lves admit
that some essential matter the terms of payment still had to be mutually covenanted.

2B 16-17 SALES AND LEASE Page 106


3. SPS DOROMAL VS CA

66 SCRA 575

FACTS

A lot in La Paz was originally decreed in the name of late Justice Horilleno, under an Original Certificate of Title
but before he died, on a date not particularized on the record, he executed a last will and testament attesting to the
fact that it was a co-ownership between himself and his brothers and sisters.
The heirs planned to sell their shares and if possible, if javellana agrees, the whole property to petitioner. Since they
are apart from eqch other, they each executed their powers of attorney.
Carlos, one of the heir was the one managing the dealings with petitioner. He received 5k for 5php per square meter
from petitioner as earnest money. He sent two letters to javellana with months interval stating on one, that the
price agreed upon was 5php per sq m, and on the other 4php per sq m.
Javellana declined to sell her share, all the others proceeded with the sale thereby making Javellana and petitioner
co-owners of the land 1/7 to the former and 6/7 to the latter.
Javellana subsequently exercised her right to repurchase the land sold by her relatives to petitioners with the price
The RTC ruled in favor of peritioners, javellana appealed to CA which ruled in her favor.

In gist: The problem revolves around whether or not the requirement of 1623-in connection w/ 1620- in terms of
redemption was satisfied. It was stated there that redemtion shou ld be made w/in 30 days of notice of the sale.
Petitioner argues that javellana was notified of the perfected contract when he she was informed of the earnest
money given to carlos on november 1967 and january 1968. The redemption letter was received by pe titioner only
on june 10, 1968 therefore, it was beyond the 30 day period

ISSUE

Whether or not the earnest money was really an earnest money in the sense that there was already a perfected
contract when it was given.

HELD No

There is no showing that s aid letters were in fact received by respondent and when they were actually received.
Besides, petitioners do not pinpoint which of these two letters, their dates being more than two months apart, is the
required notice.

Neither of said letters referred to a consummated sale. As may be observed, it was Carlos Horilleno alone who
signed them, and as of January 18, 1968, powers of attorney from the various co-owners were still to be secured.
Indeed, the later letter of January 18, 1968 mentioned that the price was P4.00 per square meter whereas in the
earlier letter of November 5, 1967 it was P5.00, as in fact, on that basis, as early as October 27, 1967, Carlos had
already received P5,000 from petitioners supposedly as earnest money, of which, however, mention was made by
him to his niece only in the later letter of January 18, 1968, the explanation being that at later negotiation it was
increased to P5.00 per square meter.

In other words, while the letters relied upon by petitioners could convey the idea that more or less some kind of
consensus had been arrived at among the other co-owners to sell the property in dispute to petitioners, it cannot be

2B 16-17 SALES AND LEASE Page 107


said definitely that such a sale had even been actually perfected.

The price differences on the two letters negates the possibility that there is already a price definite agreed upon.
While 5k might have been indeed paid to carlos, there is no showing that in the concept of earnest money
contemplated in 1482. We are more inclined to believe that the said P5,000 were paid in the concept of earnest
money as the term was understood under the Old Civil Code, that is, as a guarantee that the buyer would not back
out, considering that it is not clear that there was already a definite agreement as to the price.

2B 16-17 SALES AND LEASE Page 108


4. SALAS RODRIGUEZ VS LEUTERIO

47 PHIL 818

FACTS:

On September 24, 1920, the parties to this action entered into a contract by which the defendant agreed to sell, and
the plaintiff to buy, seven thousand square meters of land in the b arrio of Tuliahan, municipality of

Caloocan, Rizal, for the consideration of P5,600, which was paid by the plaintiff in the act of transfer. At the time of
this sale the particular lots contemplated as the subject of the sale had not been segregated, but t he seller

agreed to establish the lots with a special frontage on a principal thoroughfare as soon as the streets should be laid
out in a projected new subdivision of the city. As time passed the seller was unable to comply with this part of

the agreement and was therefore unable to place the purchaser in possession. The present action was accordingly
instituted by the purchaser in the Court of First Instance of the Province of Rizal for the resolution (in the complaint

improperly denominated rescission) of the contract and a return of double the amount delivered to the defendant as
the purchase price of the land. The trial court decreed a rescission (properly resolution) of the contract

and ordered the defense to return to the plaintiff the amount received, or the sum of P5,600, with legal interest from
the date of the filing of the complaint. From this judgment the plaintiff appealed.

ISSUE:

Whether or not the plaintiff is entitled to recover double the amount paid out by him as the purchase price of the
land

HELD

Article 1454 of the Civil Code is relied upon by plaintiff-appellant as authority for claiming double the amount paid
out by him. In this article it is declared that when earnest money or pledge is given to bind a contract of

purchase and sale, the contract may be rescinded if the vendee should be willing to forfeit the earnest money or
pledge or the vendor to return double the amount. This provision is clearly not pertinent to the case, for the reason

that where the purchase price is paid in whole or in part, the payment cannot be considered to be either earnest
money or pledge. In this connection the commentator Manresa observes that the delivery of part of the purchase

should not be understood as constituting earnest money unless it be shown that such was the intention of the parties

2B 16-17 SALES AND LEASE Page 109


MISREPRES ENTATION AS TO CAPACITY

1. MERCADO AND MERCADO v. ESPIRITU

G.R. No. L-11872 December 1, 1917

Torres, J.

FACTS:

The plaintiffs allege that they and their sisters Concepcion and Paz, all surnamed Mercado, were the
children and sole heirs of Margarita Espiritu, a sister of the deceased Luis Espiritu; that Margarita Espiritu died in
1897, leaving as her paraphernal property a tract of land in area situated in the barrio of Panducot, municipalit y of
Calumpit, Bulacan, and bounded as described in paragraph 4 of the amended complaint, which hereditary portion
had since then been held by the plaintiffs and their sisters, through their father Wenceslao Mercado, husband of
Margarita Espiritu.

That, about the year 1910, said Luis Espiritu, by means of cajolery, induced, and fraudulently succeeded in
getting the plaintiffs Domingo and Josefa Mercado to sign a deed of sale of the land left by their mother, for the sum
of P400, which amount was divided among the two plaintiffs and their sisters Concepcion and Paz, notwithstanding
the fact that said land, according to its assessment, was valued at P3,795.

Counsel therefore asked that judgment be rendered in plaintiffs' favor by holding to be null and void th e
sale they made of their respective shares of their land, to Luis Espiritu, and that the defendant be ordered to deliver
and restore to the plaintiffs the shares of the land that fell to the latter in the partition of the estate of their deceased
mother Margarita Espiritu, together with the products thereof, uncollected since 1901, or their equivalent, to wit,
P450 per annum, and to pay the costs of the suit.

In due season the defendant administrator answered the aforementioned complaint, denying each and all of
the allegations therein contained, that subsequently, on May 14, 1901, Wenceslao Mercado y Arnedo Cruz, the
plaintiffs' father, in his capacity as administrator of the property of his children sold under pacto de retro to the same
Luis Espiritu at the price of P375 the remainder of the said land, to wit, an area covered by six cavanes of seed to
meet the expenses of the maintenance of his (Wenceslao's) children, and this amount being still insufficient the
successively borrowed from said Luis Espiritu other sums of money aggregating a total of P600;

But that later, on May 17,1910, the plaintiffs, alleging themselves to be of legal age, executed, with their
sisters Maria del Consejo and Maria dela Paz, the notarial instrument ratifying said sale under pacto de retro of the
land that had belonged to their mother Margarita Espiritu, effected by their father Wenceslao Mercado in favor of
Luis Espiritu for the sum of P2,600, they sold absolutely and perpetually to said Luis Espiritu, in consideration of
P400, the property that had belonged to their deceased mother and which they acknowledged having received from
the aforementioned purchaser.

Plaintiffs maintain that the deed of sale executed by them was not valid because they were minors when
they executed it.

2B 16-17 SALES AND LEASE Page 110


ISSUE:

Whether a person who is really and truly a minor and, notwithstanding, attests that he is of legal age, can, after the
execution of the deed and within legal period, ask for the annulment of the instrument executed by him, because of
some defect that invalidates the contract, in accordance with the law.

HELD

NO.

The principal defect attributed by the plaintiffs to the document consists in that, on the date of May 17,
1910, when it was executed that they signed it, they were minors, that is, t hey had not yet attained the age of 21
years fixed by Act No. 1891, though no evidence appears in the record that the plaintiffs Josefa and Domingo
Mercado were in fact minors, for no certified copies were presented of their baptismal certificates, nor did the
plaintiffs adduce any supplemental evidence whatever to prove that Domingo was actually 19 and Josefa 18 years of
age when they signed the document Exhibit 3, on May 17, 1910, inasmuch as the copybook, notwithstanding the
testimony of the plaintiff Consejo Mercado, does not constitute sufficient proof of the dates of births of the said
Domingo and Josefa.

However, even in the doubt whether they certainly were of legal age on the date referred to, it cannot be
gainsaid that in the document they stated that they were of legal age at the time they executed and signed it, and on
that account the sale mentioned in said notarial deed Exhibit 3 is perfectly valid a sale that is considered as
limited solely to the parcel of land of 6 cavanes of seed, pledged by the deceased father of the plaintiffs in security
for P600 received by him as a loan from his brother-in-law Luis Espiritu, for the reason that the parcel of 15 cavanes
had been lawfully sold by its original owner, the plaintiffs' mother. The courts, in their interpretation of the law,
have laid down the rule that the sale of real estate, made by minors who pretend to be of legal age, when in fact they
are not, is valid, and they will not be permitted to excuse themselves from the fulfillment of the obligations
contracted by them, or to have them annulled in pursuance of the provisions of Law 6, title 19, of the 6th Partida;
and the judgment that holds such a sale to be valid and absolves the purchaser from the complaint filed against him
does not violate the laws relative to the sale of minors' property, nor the juridical rules established in consonance
therewith. (Decisions of the supreme court of Spain, of April 27, 1860, July 11, 1868, and March 1, 1875.)

2B 16-17 SALES AND LEASE Page 111


2. SIA SUAN and GAW CHIAO vs.RAMON ALCANTARA.

G.R. No. L-1720; March 4, 1950

Paras, J

FACTS

On August 3, 1931, a deed of sale was executed by Rufino Alcantara and his sons Damaso Alcantara and
Ramon Alcantara conveying to Sia Suan five parcels of land. (Ramon Alcantara was then 17 years, 10 mont hs and
22 days old.) On August 27, 1931, Gaw Chiao (husband of Sia Suan) received a letter from Francisco Alfonso,
attorney of Ramon Alcantara, informing Gaw Chiao that Ramon Alcantara was a minor and accordingly disavowing
the contract. After being contacted by Gaw Chiao, however, Ramon Alcantara executed an affidavit in the office of
Jose Gomez, attorney of Gaw Chiao, wherein Ramon Alcantara ratified the deed of sale.

On said occasion Ramon Alcantara received from Gaw Chiao the sum of P500. In the meantime, Sia Suan
sold one of the lots to Nicolas Azores from whom Antonio Azores inherited the same. On August 8, 1940, an action
was instituted by Ramon Alcantara in the Court of First Instance of Laguna for the annulment of the deed of sale as
regards his undivided share in the two parcels of land. Said action was against Sia Suan & Gaw Chiao, Antonio
Azores, Damaso Alcantara and Rufino Alcantara (the latter two being, respectively, the brother and father of Ramon
Alcantara)

CFI: absolved all the defendants. Ramon Alcantara appealed to the Court of Appeals which reversed the
decision of the trial court, on the ground that the deed of sale is not binding against Ramon Alcantara in view of his
minority on the date of its execution, and accordingly sentenced Sia Suan to pay to Ramon Alcantara the sum of
P1,750, with legal interest from December 17, 1931, in lieu of his share in the lot.

ISSUE:

Whether or not the execution sale made by ramon, then a minor, was valid.

HELD

YES. He even ratified it which made the contract binding.

It is undeniable that the deed of sale signed by the appellee, Ramon Alcantara, On August 3, 1931, showed
that he, like his co-signers (father and brother), was then of legal age. The courts, in their interpretation of the law,
have laid down the rule that the sale of real estate, made by minors who pretend to be of legal age, when in fact they
are not, is valid, and they will not be permitted to excuse themselves from the fulfillment of the obligations
contracted by them, and the judgment that holds such a sale to be valid and absolves the purchaser from the
complaint filed against him does not violate the laws relative to the sale of minors' property, nor the juridical rules
established in consonance therewith. The Court of Appeals has refused to apply this doctrine on the ground that the
appellants did not actually pay any amount in cash to the appellee and therefore did not suffer any detriment by
reason of the deed of sale, it being stipulated that the consideration therefor was a pre-existing indebtedness of
appellee's father, Rufino Alcantara.

2B 16-17 SALES AND LEASE Page 112


We are of the opinion that the Court of Appeals erred.

In the first place, in the case cited, the consideration for the sale consisted in greater part of a pre-existing
obligation. In the second place, under the doctrine, to bind a minor who represents himself to be of legal age, it is
not necessary for his vendee to actually part with cash, as long as the contract is supported by a valid consideration.
Since appellee's conveyance to the appellants was admittedly for and in virtue of a pre-existing indebtedness
(unquestionably a valid consideration), it should produce its full force and effect, in the absence of any other vice
that may legally invalidate the same. It is not here claimed that the deed of sale is nu ll and void on any ground other
than the appellee's minority. Appellee's contract has become fully efficacious as a contract executed by parties with
full legal capacity.

The circumstance that, about one month after the date of the conveyance, the appellee informed the
appellants of his minority, is of no moment, because appellee's previous misrepresentation had already estopped him
from disavowing the contract. Said belated information merely leads to the inference that the appellants in fact did
not know that the appellee was a minor on the date of the contract, and somewhat emphasizes appellee's bad faith,
when it is borne in mind that no sooner had he given said information than he ratified his deed of sale upon
receiving from the appellants the sum of P500. Counsel for the appellee argues that the appellants could not have
been misled as to the real age of the appellee because they were free to make the necessary investigation. The
suggestion, while perhaps practicable, is conspicuously unbusinesslike an d beside the point, because the findings of
the Court of Appeals do not show that the appellants knew or could have suspected appellee's minority. The Court of
Appeals seems to be of the opinion that the letter written by the appellee informing the appella nts of his minority
constituted an effective disaffirmance of the sale, and that although the choice to disaffirm will not by itself avoid
the contract until the courts adjudge the agreement to be invalid, said notice shielded the appellee from laches and
consequent estoppel.

This position is untenable since the effect of estoppel in proper cases is unaffected by the promptness with
which a notice to disaffirm is made. The appealed decision of the Court of Appeals is hereby reversed and the
appellants absolved from the complaint, with costs against the appellee, Ramon Alcantara. So ordered.

2B 16-17 SALES AND LEASE Page 113


AUTHORITY

1. MANUEL FUENTES AND LETICIA FUENTES VS ROCA

GR NO. 178902 April 21, 2010

FACTS

Sabina Tarroza owned a titled 358-square meter lot in Canelar, Zamboanga City. On October 11, 1982 she sold it to
her son, Tarciano T. Roca (Tarciano) under a deed of absolute sale. But Tarciano did not for the meantime have the
registered title transferred to his name.

Six years later in 1988, Tarciano offered to sell the lot to petitioners Manuel and Leticia Fuentes. They later signed
an agreement to sell dated April 29, 1988, which agreement expressly stated that it was to take effect in six months.

The agreement required the Fuentes spouses to pay Tarciano a down payment of P60,000.00 for the transfer of the
lots title to him. And, within six months, Tarciano was to clear the lot of structures and occupants and secure the
consent of his estranged wife, Rosario Gabriel Roca (Rosario), to the sale. Upon Tarcianos c ompliance with these
conditions, the Fuentes spouses were to take possession of the lot and pay him an additional P140,000.00 or
P160,000.00, depending on whether or not he succeeded in demolishing the house standing on it. If Tarciano was
unable to comply with these conditions, the Fuentes spouses would become owners of the lot without any further
formality and payment.

As soon as Tarciano met the other conditions, Atty. Plagata notarized Rosarios affidavit in Zamboanga City. On
January 11, 1989 Tarciano executed a deed of absolute sale in favor of the Fuentes spouses. They then paid him the
additional P140,000.00 mentioned in their agreement. A new title was issued in the name of the spouses who
immediately constructed a building on the lot.

Eight years later in 1997, the children of Tarciano and Rosario, filed an action for annulment of sale and
reconveyance of the land against the Fuentes spouses before the Regional Trial Court (RTC). The Rocas claimed
that the sale to the spouses was void since Tarcianos wife, Rosario, did not give her consent to it. Her signature on
the affidavit of consent had been forged.

The spouses denied the Rocas allegations. They presented Atty. Plagata who testified that he personally saw Rosario
sign the affidavit at her residence in Paco, Manila, on September 15, 1988. He admitted, however, that he notarized
the document in Zamboanga City four months later on January 11, 1989. All the same, the Fuentes spouses pointed
out that the claim of forgery was personal to Rosario and she alone could invoke it. Besides, the four-year
prescriptive period for nullifying the sale on ground of fraud had already lapsed.

2B 16-17 SALES AND LEASE Page 114


The Fuentes spouses point out that it was to Rosario, whose consent was not obtained, that the law gave the right to
bring an action to declare void her husbands sale of conjugal land. But here, Rosario died in 1990, the year after the
sale. Does this mean that the right to have the sale declared void is forever lost?

ISSUE

Whether or not only Rosario, the wife whose consent was not had, could bring the action to annul that sale

HELD

Petition denied.

Art. 124. (FC) x x x In the event that one spouse is incapacitated or otherwise unable to participate in the
administration of the conjugal properties, the other spouse may assume sole powers of administration. These powers
do not include the powers of disposition or encumbrance which must have the authority of the court or the written
consent of the other spouse. In the absence of such authority or consent, the disposition or encumbrance shall be
void. x x x

Under the provisions of the Civil Code governing contracts, a void or inexistent contract has no force and effect
from the very beginning. And this rule applies to contracts that are declared void by positive provision of law, as in
the case of a sale of conjugal property without the other spouses written consent. A void contract is equivalent to
nothing and is absolutely wanting in civil effects. It cannot be validated either by ratification or prescription.

But, although a void contract has no legal effects even if no action is taken to set it aside, when any of its terms have
been performed, an action to declare its inexistence is necessary to allow restitution of what has been given under it.
This action, according to Article 1410 of the Civil Code does not prescribe. Thus:

Art. 1410. The action or defense for the declaration of the inexistence of a contract does not prescribe.

Here, the Rocas filed an action against the Fuentes spouses in 1997 for annulment of sale and reconveyance of the
real property that Tarciano sold without their mothers (his wifes) written consent. The passage of time did not erode
the right to bring such an action.

Besides, even assuming that it is the Civil Code that applies to the transaction as the CA held, Article 173 provides
that the wife may bring an action for annulment of sale on the ground of lack of spousal consent during the marriage

2B 16-17 SALES AND LEASE Page 115


within 10 years from the transaction. Consequently, the action that the Rocas, her heirs, bro ught in 1997 fell within
10 years of the January 11, 1989 sale. It did not yet prescribe.

The Fuentes spouses of course argue that the RTC nullified the sale to them based on fraud and that, therefore, the
applicable prescriptive period should be that which applies to fraudulent transactions, namely, four years from its
discovery. Since notice of the sale may be deemed given to the Rocas when it was registered with the Registry of
Deeds in 1989, their right of action already prescribed in 1993.

But, if there had been a victim of fraud in this case, it would be the Fuentes spouses in that they appeared to have
agreed to buy the property upon an honest belief that Rosarios written consent to the sale was genuine. They had
four years then from the time they learned that her signature had been forged within which to file an action to annul
the sale and get back their money plus damages. They never exercised the right.

If, on the other hand, Rosario had agreed to sign the document of consent upon a false rep resentation that the
property would go to their children, not to strangers, and it turned out that this was not the case, then she would have
four years from the time she discovered the fraud within which to file an action to declare the sale void. But tha t is
not the case here. Rosario was not a victim of fraud or misrepresentation. Her consent was simply not obtained at all.
She lost nothing since the sale without her written consent was void. Ultimately, the Rocas ground for annulment is
not forgery but the lack of written consent of their mother to the sale. The forgery is merely evidence of lack of
consent.

Sale was void from the beginning. Consequently, the land remained the property of Tarciano and Rosario despite
that sale. When the two died, they passed on the ownership of the property to their heirs, namely, the Rocas. As
lawful owners, the Rocas had the right, under Article 429 of the Civil Code, to exclude any person from its
enjoyment and disposal.

In fairness to the Fuentes spouses, however, they should be entitled, among other things, to recover from Tarcianos
heirs, the Rocas, the P200,000.00 that they paid him, with legal interest until fully paid, chargeable against his
estate.

2B 16-17 SALES AND LEASE Page 116


2. TITAN CONSTRUCTION CORP VS MANUEL DAVID AND MARTINA DAVID

G.R. No. 169548; March 15, 2010

DEL CASTILLO, J.

FACTS

Manuel A. David, Sr. (Manuel) and Martha S. David (Martha) were married on March 25, 1957. In 1970, the
spouses acquired a 602 square meter lot located at White Plains, Quezon City, which wa s registered in the name of
MARTHA S. DAVID, of legal age, Filipino, married to Manuel A. David and covered by Transfer Certificate of
Title (TCT) No. 156043 issued by the Register of Deeds. In 1976, the spouses separated de facto, and no longer
communicated with each other.

Sometime in March 1995, Manuel discovered that Martha had previously sold the property to Titan Construction
Corporation (Titan) for P1,500,000.00 through a Deed of Sale dated April 24, 1995, and that TCT No. 156043 had
been cancelled and replaced by TCT No. 130129 in the name of Titan.

Thus, on March 13, 1996, Manuel filed a Complain t for Annulment of Contract and Recovenyance against Titan
before the RTC of Quezon City. Manuel alleged that the sale executed by Martha in favor of Tita n was without his
knowledge and consent, and therefore void. He prayed that the Deed of Sale and TCT No. 130129 be invalidated,
that the property be reconveyed to the spouses, and that a new title be issued in their names.

Titan claimed that it was a buyer ingood faith and for value because it relied on a Special Power of
Attorney (SPA) [11] dated January 4, 1995 signed by Manuel which authorized Marth a to dispose of the property on
behalf of the spouses. Titan thus prayed for the dismissal of the complaint.

Manuel claimed that the SPA was spurious, and that the signature purporting to be his was a forgery;
hence, Martha was wholly without authority to sell the property.

RTC and CA ruled that the Deed of Sale bet. Martha and Titan is VOID, ordered that the said property be
reconveryed in favor of the spouses and ordered RD to issue new cert of title.

Hence, this petition.

ISSUE

Whether or not the sale is VALID?

HELD

No. the Sale is Void because the property is part of the spouses conjugal partnership which requires the consent
of both spouses.

2B 16-17 SALES AND LEASE Page 117


The Civil Code of the Philippine, the law in force at the time of the celebration of the marriage be tween Martha and
Manuel in 1957, provides:

Article 160. All property of the marriage is presumed to belong to the conjugal partnership, unless it be proved that
it pertains exclusively to the husband or to the wife.

Article 116 of the Family Code is even more unequivocal in that [a]ll property acquired during the marriage,
whether the acquisition appears to have been made, contracted or registered in the name of one or both spouses ,
is presumed to be conjugal unless the contrary is proved.

Since the property was undoubtedly part of the conjugal partnership, the sale to Titan required the consent of both
spouses. Therefore, since the sale is without Manuels consent, the sale is VOID.

2B 16-17 SALES AND LEASE Page 118


3. SPS REX AND CONCEPCION AGGABAO VS DIONISIO PARULAN

G.R. No. 165803 September 1, 2010

Bersamin, J

FACTS

Involved in this action are two parcels of land and their improvements in Paraaque City and registered under the
name of Spouses Parulan, who have been estranged from one another. Real estate broker At anacio offered the
property to Spouses Aggabao who upon Atanacios insistence prevailed upon them, so that they and Atanacio met
with Ma. Elena (Parulans wife) at the site of the property. During their meeting, Spouses Aggabao paid Ma. Elena
earnest money amounting to P20,000.00 which she acknowledged with a handwritten receipt. Then and there, they
agreed on the terms of how the buyers will pay the price of the property. Spouses Aggabao complied with all the
terms with regard to the payment of the properties, but when Ma. Elena already needed to turn over the owners
duplicate copies for both lands, she was able to turn over only one (which was successfully transferred to the name
of spouses Aggabao). For the other one, she said that it is with a relative in HongKong but she promised to deliver it
to the spouses in a week. Needless to say, she failed to do so and by doing their own verification, the spouses found
out that said copy of title was in the hands of Dionisios brother. The spouses met with Dionis ios brother, Atty.
Parulan, who told them that he is the one with the power to sell the property. He demanded P800,000.00 for said
property and gave the spouses several days to decide. When Atty. Parulan did not hear back from the spouses, he
gave them a call, and was then informed that they have already paid the full amount to Ma. Elena. Subsequently,
Dionisio, through Atty. Parulan, commenced an action praying for the declaration of the nullity of the deed of
absolute sale executed by Ma. Elena, and the cancellation of the title issued to the petitioners by virtue thereof.

ISSUE

Whether or not the sale of conjugal property made by Ma. Elena, by presenting a special power of attorney to sell
(SPA) purportedly executed by respondent husband in her favor was validly made to the vendees

HELD

No, the Court ruled that the sale of conjugal property without the consent of the husband was not merely voidable
but void; hence, it could not be ratified. Spouses Aggabao also cannot use the defense that they are buyers in good
faith because they did not exercise the necessary prudence to inquire into the wifes authority to sell. The relevant
part of Article 124 of the Family Code provides that: In the event that one spouse is incapacitated or otherwise
unable to participate in the administration of the conjugal properties, the other spouse may assume sole powers of
administration. These powers do not include disposition or encumbrance without authority of the court or the written
consent of the other spouse.

In the absence of such authority or consent, the disposition or encumbrance shall be void.

2B 16-17 SALES AND LEASE Page 119


4. LILY ELIZABETH BRAVO-GUERRERO, BEN MAURICIO P. BRAVO, ROLAND P. BRAVO, JR.,
OFELIA BRAVO-QUIESTAS, HEIRS OF CORPUSINIA BRAVO-NIOR namely: GERSON U. NIOR,
MARK GERRY B. NIOR, CLIFF RICHARD B. NIOR, BRYAN B. NIOR,

WIDMARK B. NIOR, SHERRY ANNE B. NIOR, represented by LILY ELIZABETH BRAVO-GUERRERO


As their attorney-in-fact, and HONORABLE FLORENTINO A. TUASON, JR., Presiding Judge, Regional
Trial Court, Branch 139, Makati City, VS EDWARD P. BRAVO, represented by his attorney-in-fact
FATIMA C. BRAVO, respondent, and DAVID B. DIAZ, JR

GR NO. 152658 July 29, 2005

FACTS:

Spouses Mauricio Bravo (Mauricio) and Simona Andaya Bravo (Simona) owned two parcels of land
(Properties) measuring 287 and 291 square meters and located along Evangelista Street, Makati City, Metro
Manila. Mauricio and Simona had three childrenRoland, Cesar and Lily, all surnamed Bravo. Cesar died without
issue. Lily Bravo married David Diaz, and had a son, David B. Diaz, Jr. (David Jr.). Roland had six children,
namely, Lily Elizabeth Bravo-Guerrero (Elizabeth), Edward Bravo (Edward), Roland Bravo, Jr. (Roland Jr.),
Senia Bravo, Benjamin Mauricio Bravo, and their half-sister, Ofelia Bravo (Ofelia). Simona executed a General
Power of Attorney (GPA) on 17 June 1966 appointing Mauricio as her attorney -in-fact, authorizing him to
mortgage or otherwise hypothecate, sell, assign and dispose of any and all of my property, real, personal or mixed,
of any kind whatsoever and wheresoever situated, or any interest therein x x x. Mauricio subsequently mortgaged
the Properties to the Philippine National Bank (PNB) and Development Bank of the Philippines (DBP). On 25
October 1970, Mauricio executed a Deed of Sale with Assumption of Real Estate Mortgage (Deed of Sale)
conveying the Properties to Roland A. Bravo, Ofelia A. Bravo and Elizabeth Bravo8 (vendees), conditioned on
the payment of P1,000 and on the assumption by the vendees of the PNB and DBP mort gages over the Properties.
Despite being notarized, the Deed of Sale was not annotated on the TCTs. Neither was it presented to PNB and
DBP. The mortgage loans and the receipts for loan payments issued by PNB and DBP continued to be in Mauricios
name even after his death.

On 23 June 1997, Edward, represented by his wife, Fatima Bravo, filed an action for the judicial partition of the
Properties. Edward claimed that he and the other grandchildren of Mauricio and Simona are co -owners of the
Properties by succession. Despite this, petitioners refused to share with him the possession and rental income of the
Properties. Edward later amended his complaint to include a prayer to annul the Deed of Sale, which he claimed was
merely simulated to prejudice the other heirs. David Jr. also filed a complaint-in-intervention impugning the validity
of the Deed of Sale and praying for the partition of the Properties among the surviving heirs of Mauricio and
Simona. The trial court upheld Mauricios sale of the Properties to the vendees. The trial court ruled that the sale did
not prejudice the compulsory heirs, as the Properties were conveyed for valuable consideration. The trial court also
noted that the Deed of Sale was duly notarized and was in existence for many years without question about its
validity. Edward and David Jr. (respondents) then filed a joint appeal to the Court of Appeals. The Court of
Appeals declared the Deed of Sale void for lack of Simonas consent, and ordered the partition of the Properties.

ISSUE:

2B 16-17 SALES AND LEASE Page 120


Whether or not the CA erred in not upholding the validity of and enforcement of the Deed of Sale With Assumption
of Mortgage

HELD:

YES. The Supreme Court held that the Court of Appeals erred when it declared the Deed of Sale void based on
Article 166. Article 166 expressly applies only to properties acquired by the conjugal partnership after the effectivity
of the Civil Code of the Philippines (Civil Code). Although there is no dispute that the Properties were conjugal
properties of Mauricio and Simona, the records do not show, and the parties did not stipulate, when the Properties
were acquired. Under Article 1413 of the old Spanish Civil Code, the husband could alienate conjugal partnership
property for valuable consideration without the wifes consent. Even under the present Civil Code, however, the
Deed of Sale is not void. It is well-settled that contracts alienating conjugal real property without the wifes consent
are merely voidable under the Civil Codethat is, binding on the parties unless annulled by a competent courtand
not void ab initio. Under the Civil Code, only the wife can ask to annul a contract that disposes of conjugal real
property without her consent. The wife must file the action for annulment during the marriage and within t en years
from the questioned transaction. Article 173 is explicit on the remedies available if the wife fails to exercise this
right within the specified period. In such case, the wife or her heirs can only demand the value of the property
provided they prove that the husband fraudulently alienated the property. Respondents action to annul the Deed of
Sale based on Article 166 must fail for having been filed out of time. The marriage of Mauricio and Simona was
dissolved when

Mauricio died in 1973. More than ten years have passed since the execution of the Deed of Sale. Further,
respondents, who are Simonas heirs, are not the parties who can invoke Article 166. Article 173 reserves that
remedy to the wife alone. Only Simona had the right to have the sale of the Properties annulled on the ground that
Mauricio sold the Properties without her consent.

The SC also agreed with the trial court that Simona authorized Mauricio to dispose of the Properties when
she executed the GPA. True, Article 1878 requires a special power of attorney for an agent to execute a contract
that transfers the ownership of an immovable. However, the Court has clarified that Article 1878 refers to the nature
of the authorization, not to its form Even if a document is titled as a general power of attorney, the requirement of a
special power of attorney is met if there is a clear mandate from the principal specifically authorizing the
performance of the act. In this case, Simona expressly authorized Mauricio in the GPA to sell, assign and dispose
of any and all of my property, real, personal or mixed, of any kind whatsoever and wheresoever situated, or any
interest therein x x x as well as to act as my general representative and agent, with full authority to buy, sell,
negotiate and contract for me and in my behalf.25 Taken together, these provisions constitute a clear and specific
mandate to Mauricio to sell the Properties. Even if it is called a general power of attorney, the specific provisions
in the GPA are sufficient for the purposes of Article 1878. These provisions in the GPA likewise indicate that
Simona consented to the sale of the Properties.

2B 16-17 SALES AND LEASE Page 121


5. FRANCISCO A. VELOSO, petitioner, vs. COURT OF APPEALS, AGLALOMA B. ESCARIO, assisted by
her husband GREGORIO L. ESCARIO, the REGISTER OF DEEDS FOR THE CITY OF MANILA,
respondents.

G.R. No. 102737, August 21, 1996

Torres Jr, J

FACTS

Veloso was the owner of a parcel of land situated in the district of Tondo, Manila, covered by a Transfer Certificate
of Title issued by the Registry of Deeds of Manila and was registered in his name. However, the said title was
subsequently cancelled and a new one was issued in the name of Aglaloma B. Escario, married to Gregorio L.
Escario, on May 24, 1988.

On August 24, 1988, Veloso filed an action for annulment of documents, reconveyance of property with damages
and preliminary injunction and/or restraining order. He alleged that:

1. He was the absolute owner of the subject property and he never authorized anybody, not even his wife,
to sell it.

2. He was in possession of the title but when his wife, Irma, left for abroad, he found out that his copy was
missing. He then verified with the Registry of Deeds of Manila and there he discovered that his title was already
cancelled in favor of Aglaloma Escario. The transfer of property was supported by a General Power of Attorney and
Deed of Absolute Sale both executed by Irma Veloso and defendant Aglaloma Escario.

Veloso, however, denied having executed the power of attorney and alleged that his signature was falsified.
He also denied having seen or even known Rosemarie Reyes and Imelda Santos, the supposed witnesses in the
execution of the power of attorney. He vehemently denied having met or transacted with the defendant. Thus, he
contended that the sale of the property, and the subsequent transfer thereof, were null and void .

Defendant Aglaloma Escario in her answer alleged that:

1. She was a buyer in good faith and denied any knowledge of the alleged irregularity.

2. She allegedly relied on the general power of attorney of Irma Veloso which was sufficient in form and
substance and was duly notarized.

She contended that Veloso had no cause of action against her because the real party in interest was Irma
Veloso. Consequently, defendant Escario prayed for the dismissal of the complaint and the payment to her of
damages.

Regarding the property, Veloso testified that he acquired the subject property from the Philippine Building
Corporation on October 1, 1957. He married Irma Lazatin on January 20, 1962. Hence, the property did not belong
to their conjugal partnership. Veloso further asserted that he did not sign the power of attorney and as proof that his
signature was falsified, he presented Allied Bank Checks which allegedly bore his genuine signature .

2B 16-17 SALES AND LEASE Page 122


Decision of the RTC: Aglaloma Escario is the lawful owner of the property as she was deemed an innocent
purchaser for value. The assailed general power of attorney is valid and sufficient for the purpose. There is no need
for a special power of attorney when the special power was already mentioned in the general one. Veloso failed to
substantiate his allegation of fraud and he is not entirely blameless for although he admitted to be the only person
who had access to the title and other important documents, his wife was still able to possess the copy. Citing Section
55 of Act 496, the court held that Irma's possession and production of the certificate of title was deemed a
conclusive authority from the plaintiff to the Register of Deeds to enter a new cert ificate. Then applying the
principle of equitable estoppel, plaintiff was held to bear the loss for it was he who made the wrong possible.

Upon appeal with the Court of Appeals. The respondent court affirmed in toto the findings of the trial
court.

Hence, this petition for review.

Contention of Veloso: His signature on the power of attorney was falsified and that it was not duly notarized for as
testified by Atty. Tubig himself, he did not sign thereon nor was it ever recorded in his notarial register.

ISSUE

Whether or not the Court of Appeals committed a grave error in not finding that the forgery of the power of attorney
had been adequately proven, despite the preponderant evidence.

HELD

No, The Court finds petitioner's contentions not meritorious.

An examination of the records showed that the assailed power of attorney was valid and regular on its face. It was
notarized and as such, it carries the evidentiary weight conferred upon it with respect to its due execution. While it is
true that it was denominated as a general power of attorney, a perusal thereof revealed that it stated an authority to
sell, to wit:

2. To buy or sell, hire or lease, mortgage or otherwise hypothecate lands, tenements and hereditaments or
other forms of real property, more specifically TCT No. 49138, upon such terms and conditions and under
such covenants as my said attorney shall deem fit and proper.

Thus, there was no need to execute a separate and special power of attorney since the general power of attorney had
expressly authorized the agent or attorney in fact the power to sell the subject property . The special power of
attorney can be included in the general power when it is specified therein the act or transaction for which the special
power is required.

Regarding the petitioners contention, the Court ruled that the basis presented by the petitioner was
inadequate to sustain his allegation of forgery. Mere variance of the signatures cannot be considered as conclusive
proof that the same were forged. Forgery cannot be presumed. Petitioner, however, failed to prove his allegation and

2B 16-17 SALES AND LEASE Page 123


simply relied on the apparent difference of the signatures. His denial had not established that the signature on the
power of attorney was not his.

The Court agreed with the conclusion of the lower court that Escario was an innocent purchaser for value.
She relied on the power of attorney presented by Irma. Being the wife of the owner and having with her the title of
the property, there was no reason for Escario not to believe in her authority. Moreover, the power of attorney was
notarized and as such, carried with it the presumption of its due execution. Thus, having had no inkling on any
irregularity and having no participation thereof, Escario was a buyer in good faith.

It has been consistently held that a purchaser in good faith is one who buys property of another, without
notice that some other person has a right to, or interest in such property and pays a full and fair price for the same, at
the time of such purchase, or before he has notice of the claim or interest of some other person in the property.

Even granting for the sake of argument, that the petitioner's signature was falsified and consequently, the
power of attorney and the deed of sale were null and void, such fact would not rev oke the title subsequently issued
in favor of private respondent Aglaloma. In Tenio-Obsequio vs. Court of Appeals, it was held, viz:

The right of an innocent purchaser for value must be respected and protected, even if the seller obtained his
title through fraud. The remedy of the person prejudiced is to bring an action for damages against those
who caused or employed the fraud, and if the latter are insolvent, an action against the Treasurer of the
Philippines may be filed for recovery of damages against the Assurance Fund.

Finally, the trial court did not err in applying equitable estoppel in this case. The principle of equitable
estoppel states that where one or two innocent persons must suffer a loss, he who by his conduct made the loss
possible must bear it. From the evidence adduced, it should be the petitioner who should bear the loss.

2B 16-17 SALES AND LEASE Page 124


6. JOSEFINA VILLANUEVA-MIJARES VS CA

G.R. No. 108921. April 12, 2000.

FACTS

Felipe begot the following legitimate children: Simplicio, Benito, Leon, Nicolasa, Eustaqio, Camila, Fausta, and
Pedro. During his lifetime he acquired a parcel of land. Upon Felipes death, ownership of the land was passed on to
his children. In 1952, Pedro, one of the children of Felipe got his share equivalent to onesixth (1/6) o f the property
with an area of one thousand nine hundred five (1,905) square meters and had it declared under his name pursuant to
Tax Declaration No. 8085. The remaining portion of the land was left undivided. This was held in trust by Leon for
his coheirs. During Leons lifetime, his coheirs made several seasonable and lawful demands upon him to subdivide
and partition the property, but for one reason or another, no subdivision took place.

After the death of Leon in August 1972, private respondents discov ered that the shares of four of the heirs of Felipe,
namely, Simplicio, Nicolasa, Fausta and Maria Baltazar, spouse of Benito, was purchased by Leon as evidenced by
a Deed of Sale executed on August 25, 1946 but registered only in 1971. It also came to lig ht that Leon had,
sometime in July 1970, executed a sale and partition of the property in favor of his own children, herein petitioners.
By virtue of such Deed of Partition, private respondents had succeeded in obtaining Original Certificate of Title
(OCT) No. C256. On April 25, 1975, petitioners managed to secure separate and independent titles over their
proindiviso shares in their respective names.

Private respondents then filed a case for partition with annulment of documents and/or reconveyance and damages
with the Regional Trial Court of Kalibo, Aklan. Private respondents contended that the sale in favor of Leon was
fraudulently obtained through machinations and false pretenses. Thus, the subsequent sale of the lot by Leon to his
children was null and void despite the OCT in his favor Petitioners, for their part, claimed that the sale by Simplicio,
Fausta, Nicolasa, and Maria Baltazar was a valid sale; that private respondent Procerfina even signed as an
instrumental witness to the Deed of Sale; that Maria Baltazar, widow of Benito, as administrator of her husbands
estate, had the right to sell the undivided share of Benito; that the basis for the issuance of the OCT in Land
Registration Case No. K231 was the sale by his coheirs to Leon; that the order of default issued in Land Registration
Case No. K231 was against the whole world; that prescription had set in since they had been in possession of the
property in the concept of owners thereof since August 29, 1946, up to the present; and that private res pondents
were estopped since no trust relationship existed between the litigants.

On appeal, the private respondents conceded the right of Simplicio, Nicolasa, and Fausta to sell their respective
shares but disputed the authority of Maria Baltazar to convey any portion of her late husbands estate, since the latter
was his capital and did not form part of the conjugal property. The Court of Appeals ruled that under the Old Civil
Code and applicable jurisprudence, Maria Baltazar had no authority to sell the portion of her late husbands share
inherited by her then minor children since she had not been appointed their guardian. Respondent court likewise
declared that as far as private respondents Procerfina, Prosperidad, Ramon and Rosa, were concerned, the Dee d of
Sale of August 25, 1946 was unenforceable. Respondent appellate court also ruled that the prescription period had
not run in favor of Leon since private respondents had always known that Leon was the administrator of the estate. It
was only in 1975 when their suspicion were aroused and they inquired about the status of the land.

ISSUES

2B 16-17 SALES AND LEASE Page 125


1. Whether or not the appellate court erred in failing to declare action by the private respondents to recover the
property in question barred by laches, estoppel, prescription, and res judicata

2. Whether or not the appellate court erred in declaring the Deed of Sale of August 25, 1946 unenforceable against
the private respondents for being an unauthorized contract.

HELD

First Issue.

NO. Laches is negligence or omis sion to assert a right within a reasonable time, warranting the presumption that the
party entitled to assert it has either abandoned or declined to assert it. At the time of signing of the Deed of Sale of
August 26, 1948, private respondents Procerfina, Prosperidad, Ramon and Rosa were minors. They could not be
faulted for their failure to file a case to recover their inheritance from their uncle Leon, since up to the age of
majority, they believed and considered Leon their coheir and administrator. It wa s only in 1975, not in 1948, that
they became aware of the actionable betrayal by their uncle. Upon learning of their uncles actions, they filed an
action for recovery.

Petitioners aver that the failure of Maria Baltazars children to bringing their actio n in 1969 when they had reached
the age of majority meant that they had impliedly ratified the Deed of Sale and are now estopped to assail the same.
They erroneously relied on Asiatic Integrated Corporation v. Alikpala, 67 SCRA 60 (1975). In that case, pay ments
made by Asiatic pursuant to the terms of the contract accrued to the benefit of the City without protest on the part of
the municipal board, such that the Board already acquiesced to the validation of the contract. In the instant case,
there is no implied ratification, no benefit accruing to the children of Maria Baltazar.

Neither is the action barred by pre scription. In Vda. de Cabrera v. Court of Appeals and Sta. Ana, Jr. v. Court of
Appeals held that an action for reconveyance of a parcel of land based on implied or constructive trust prescribes in
10 years, the point of reference being the date of registration of the deed or the date of the issuance of the certificate
of title of the property. Here the questioned Deed of Sale was registered only in 1971. Private respondents filed their
complaint in 1975, hence well within the prescriptive period.

Petitioners assert that the disputed property is registered. Relying on Cachero v. Marzan, and Cureg v. Intermediate
Appellate Court, where we held that a land registration case is an action in rem binding upon the whole world, and
considering that the private respondents failed to object to the registration of the realty in question, then res judicata
had set in. True, but notwithstanding the binding effect of the land registration case upon the private respondents, the
latter are not deprived of a remedy. While a review of the decree of registration is no longer available after the
expiration of the one year period from entry thereof, an equitable remedy is still available. Those wrongfully
deprived of their property may initiate an action for reconveyance of the property.

Second Issue.

As to the second issue, we find no reversible error committed by the respondent appellate court in declaring the
Deed of Sale unenforceable on the children of Maria Baltazar. As correctly pointed out by the Court of Appeals,
there was no question as to the sale of the shares of Simplicio, Nicolasa, and Fausta, to their brother Leon. But not
so with Maria Baltazar concerning the share of her late husband, Benito, to Leon. Under the law then prevailing at
the time of the demise of her spouse, her husbands share in the common inheritance pertained to her minor children
who were her late husbands heirs and successors in interes t.

2B 16-17 SALES AND LEASE Page 126


7. NAPOLEON NERI VS HEIRS OF HADJI YUSOP UY AND JULPHA IBRAHIM UY

G.R. No. 194366 October 10, 2012

Perlas-Bernabe, J

FACTS:

Anunciacion Neri had seven children, two (2) from her first marriage with Gonzalo Illut, and five (5) from
her second marriage with Enrique Neri. Throughout the marriage of spouses Enrique and Anunciacion, they
acquired several homestead properties.

On September 21, 1977, Anunciacion died intestate. Her husband, Enrique, in his personal capacity and as
natural guardian of his minor children Rosa and Douglas, together with Napoleon, Alicia, and Visminda executed an
Extra-Judicial Settlement of the Estate with Absolute Deed of Sale on July 7, 1979, adjudicating among themselves
the said homestead properties, and thereafter, conveying them to the late spouses Hadji Yusop Uy and Julpha
Ibrahim Uy (spouses Uy) for a consideration of P 80,000.00.

On June 11, 1996, the children of Enrique filed a complaint for annulment of sale of the said homestead
properties against spouses Uy (later substituted by their heirs) before the RTC, assailing the validity of the sale for
having been sold within the prohibited period.

RTC rendered the sale void because Eutropia and Victoria were deprived of their hereditary rights and that Enrique
had no judicial authority to sell the shares of his minor children, Rosa and Douglas.

The CA reversed the RTC ruling and declared the extrajudicial settlement and sale valid. While
recognizing Rosa and Douglas to be minors at that time, they were deemed to have ratified the sale when they failed
to question it upon reaching the ae of majority. It also found laches to have set in because of their inaction for a long
period of time.

ISSUE:

Whether the father or mother, as the natural guardian of the minor un der parental authority, has the power
to dispose or encumber the property of the minor.

HELD:

All the petitioners herein are indisputably legitimate children of Anunciacion and consequently, are entitled
to inherit from her in equal shares. As such, upon the death of Anunciacion on September 21, 1977, her children and
Enrique acquired their respective inheritances.

While the settlement of the estate is null and void, the subsequent sale of the subject properties made by
Enrique and his children, Napoleon, Alicia and Visminda, in favor of the respondents is valid but only with respect
to their proportionate shares therein. As owners thereof, they can very well sell their undivided share in the estate.

With respect to Rosa and Douglas who were minors at the t ime of the execution of the settlement and sale,
their natural guardian and father, Enrique, represented them in the transaction. However, on the basis of the laws

2B 16-17 SALES AND LEASE Page 127


prevailing at that time, Enrique was merely clothed with powers of administration and bereft of any authority to
dispose of their 2/16 shares in the estate of their mother, Anunciacion. Consequently, the disputed sale entered into
by Enrique in behalf of his minor children without the proper judicial authority, unless ratified by them upon
reaching the age of majority, is unenforceable in accordance with Articles 1317 and 1403(1) of the Civil Code.

2B 16-17 SALES AND LEASE Page 128


DISQUALIFICATIONS
1. RUBIAS VS BATILLER

51 SCRA 120

FACTS:

Francisco Militante claimed ownership of a parcel of land located in the Barrio of General Luna,
municipality of Barotac Viejo province of Iloilo.

Before the war with Japan, Francisco Militante filed with the Court of First Instance of Iloilo an application
for the registration of the title of the land opposed by the Director of Lands, the Director of Forestry and other
oppositors. The Court of First Instance heard the land registration case on November 14, 1952, and after the trial this
court dismissed the application for registration. The appellant, Francisco Militante , appealed from the decision of
this Court to the Court of Appeals.

Pending the disposal of the appeal Francisco Militante sold to the plaintiff, Domingo Rubias the land

On August 17, 1965, defendant's counsel manifested in open court that before any trial on the merit of the case could
proceed he would file a motion to dismiss plaintiff's complaint which he did, alleging that plaintiff does not have
cause of action against him because the property in dispute which he (plaintiff) allegedly bought from his father-in-
law, Francisco Militante was the subject matter of LRC No. 695 filed in the CFI of Iloilo, which case was brought
on appeal to this Court and docketed as CA-G.R. No. 13497-R in which aforesaid case plaintiff was the counsel on
record of his father-in-law, Francisco Militante.

Hence, this appeal.

ISSUE:

Whether or not the contract of sale between appellant and his father-in-law, the late Francisco Militante over the
property subject was void because it was made when plaintiff was counsel of his fa ther-in-law in a land registration
case involving the property in dispute;

HELD:

Yes. No error could be attributed either to the lower court's holding that the purchase by a lawyer of the
property in litigation from his client is categorically prohibited by Article 1491, paragraph (5) of the Philippine Civil
Code, reproduced supra; and that consequently, plaintiff's purchase of the property in litigation from his client
(assuming that his client could sell the same since as already shown above, his clien t's claim to the property was
defeated and rejected) was void and could produce no legal effect, by virtue of Article 1409, paragraph (7) of our
Civil Code which provides that contracts "expressly prohibited or declared void by law' are "inexistent and tha t
"(T)hese contracts cannot be ratified. Neither can the right to set up the defense of illegality be waived."

2B 16-17 SALES AND LEASE Page 129


Article 1491 of our Civil Code (like Article 1459 of the Spanish Civil Code) prohibits in its six paragraphs
certain persons, by reason of the relation of trust or their peculiar control over the property, from acquiring such
property in their trust or control either directly or indirectly and "even at a public or judicial auction," as follows: (1)
guardians; (2) agents; (3) administrators; (4) public officers and employees; judicial officers and employees,
prosecuting attorneys, and lawyers; and (6) others especially disqualified by law.

The reason thus given by Manresa in considering such prohibited acquisitions under Article 1459 of the
Spanish Civil Code as merely voidable at the instance and option of the vendor and not void "that the Code does
not recognize such nullity de pleno derecho" is no longer true and applicable to our own Philippine Civil Code
which does recognize the absolute nullity of contracts "whose cause, object, or purpose is contrary to law, morals,
good customs, public order or public policy" or which are "expressly prohibited or declared void by law" and
declares such contracts "inexistent and void from the beginning."

2B 16-17 SALES AND LEASE Page 130


2. HULST VS PR BUILDERS

GR NO. 156364 September 25, 2008

FACTS

Jacobus Bernhard Hulst (petitioner) and his spouse Ida Johanna Hulst -Van Ijzeren (Ida), Dutch nationals, entered
into a Contract to Sell with PR Builders, Inc. (respondent), for the purchase of a 210-sq m residential unit in
respondent's townhouse project in Barangay Niyugan, Laurel, Batangas.

When respondent failed to comply with its verbal promise to complete the project by June 1995, the spouses Hulst
filed before the Housing and Land Use Regulatory Board (HLURB) a complaint for rescission of contract with
interest, damages and attorney's fees, docketed as HLRB Case No. IV6-071196-0618.

HLURB Arbiter Ma. Perpetua Y. Aquino (HLURB Arbiter) rendered a Decision in favor of spouses Hulst.

Meanwhile, spouses Hulst divorced. Ida assigned her rights over the purchased property to petitioner. From then
on, petitioner alone pursued the case.

A writ of execution was issued to satisfy the judgment. Thereafter, the public sale and other matters relating to the
case were raised on appeal.

Before resolving the question whether the CA erred in affirming the Order of the HLURB setting aside the levy
made by the sheriff, the Court is behooved to address a matter of public and national importance which completely
escaped the attention of the HLURB Arbiter and the CA: petitioner and his wife are foreign nationals who are
disqualified under the Constitution from owning real property in their names.

ISSUE

Whether or not petitioners are qualified to acquire the subject property

HELD

Section 7 of Article XII of the 1987 Constitution provides:

Sec. 7. Save in cases of hereditary succession, no private lands shall be transferred or conveyed except to
individuals, corporations, or associations qualified to acquire or hold lands of the public domain.

2B 16-17 SALES AND LEASE Page 131


The capacity to acquire private land is made dependent upon the capacity to acquire or hold lands of the public
domain. Private land may be transferred or conveyed only to individuals or entities "q ualified to acquire lands of the
public domain." The 1987 Constitution reserved the right to participate in the disposition, exploitation, development
and utilization of lands of the public domain for Filipino citizens or corporations at least 60 percent o f the capital of
which is owned by Filipinos. Aliens, whether individuals or corporations, have been disqualified from acquiring
public lands; hence, they have also been disqualified from acquiring private lands.

Since petitioner and his wife, being Dutch nationals, are proscribed under the Constitution from acquiring and
owning real property, it is unequivocal that the Contract to Sell entered into by petitioner together with his wife and
respondent is void. Under Article 1409 (1) and (7) of the Civil Cod e, all contracts whose cause, object or purpose is
contrary to law or public policy and those expressly prohibited or declared void by law are inexistent and void from
the beginning.

Article 1410 of the same Code provides that the action or defense for t he declaration of the inexistence of a contract
does not prescribe. A void contract is equivalent to nothing; it produces no civil effect. It does not create, modify or
extinguish a juridical relation.

2B 16-17 SALES AND LEASE Page 132


3. IN RE: SEPARATION OF PROPERTY MULLER VS MULLER

GR NO. 156364 August 29, 2006

FACTS

Petitioner Elena Buenaventura Muller and respondent Helmut Muller were married in Hamburg, Germany on
September 22, 1989. The couple resided in Germany at a house owned by respondents parents but decided to move
and reside permanently in the Philippines in 1992. By this time, respondent had inherited the house in Germany
from his parents which he sold and used the proceeds for the purchase of a parcel of land in Antipolo, Rizal at the
cost of P528,000.00 and the construction of a house amounting to P2,300,000.00. The Antipolo property was
registered in the name of petitioner under Transfer Certificate of Title No. 219438of the Register of Deeds of
Marikina, Metro Manila.

Due to incompatibilities and respondents alleged womanizing, drinking, and maltreatment, the spouses
eventually separated. On September 26, 1994, respondent filed a petition for separation of properties before the
Regional Trial Court of Quezon City. On August 12, 1996, the trial court rendered a decision which terminated the
regime of absolute community of property between the petitioner and respondent. It also decreed the separation of
properties between them and ordered the equal partition of personal properties located within the country, excluding
those acquired by gratuitous title during the marriage. With regard to the Antipolo property, the court held that it
was acquired using paraphernal funds of the respondent. However, it ruled that respondent cannot recover his funds
because the property was purchased in violation of Section 7, Article XII of the Constitution. Respondent appealed
to the Court of Appeals which rendered the assailed decision modifying the trial courts Decision. It held that
respondent merely prayed for reimbursement for the purchase of the Antipolo property, and not acquisition or
transfer of ownership to him. It also considered petitioners ownership over the property in trust for the respondent.
As regards the house, the Court of Appeals ruled that there is nothing in the Constitution which prohibits respondent
from acquiring the same.

ISSUE

Whether or not respondent is entitled to reimbursement of the funds used for the acquisition of the Antipolo
property.

HELD

No. Aliens, whether individuals or corporations, are disqualified from acquiring lands of the public domain. Hence,
they are also disqualified from acquiring private lands. The primary purpose of the constitutional provision is the
conservation of the national patrimony. Respondent was aware of the constitu tional prohibition and expressly
admitted his knowledge thereof to this Court. He declared that he had the Antipolo property titled in the name of
petitioner because of the said prohibition. His attempt at subsequently asserting or claiming a right on the said
property cannot be sustained.

The Court of Appeals erred in holding that an implied trust was created and resulted by operation of law in view
of petitioners marriage to respondent. Save for the exception provided in cases of hereditary succession,
respondents disqualification from owning lands in the Philippines is absolute.

2B 16-17 SALES AND LEASE Page 133


Further, the distinction made between transfer of ownership as opposed to recovery of funds is a futile exercise
on respondents part. To allow reimbursement would in effect permit respondent to enjoy the fruits of a property
which he is not allowed to own.

2B 16-17 SALES AND LEASE Page 134


4. CELSO HALILI AND ARTHUR HALILI VS CA

GR NO. 113539 March 12, 1998

Panganiban, J.

FACTS

Simeon de Guzman, an American citizen, died sometime in 1968, leaving real properties in the Philippines. His
forced heirs were his widow Helen Meyers Guzman, and his son, David Rey Guzman, both of whom are also
American citizens. Helen executed a deed of quitclaim assigning, transferring and conveying to David Rey all her
rights, titles and interests over six parcels of land which the two of them inherited from Simeon. David Rey Guzman
sold to Emiliano Cataniag one of the said parcels of land, upon which a new TCT was issued in the latters name.

Petitioners Halili, who are owners of the adjoining lot, filed a complaint before the Malolos, Bulacan RTC
questioning the constitutionality and validity of the two conveyances -- between Helen Guzman and David Rey
Guzman, and between the latter and Emiliano Cataniag -- and claiming ownership thereto based on their right of
legal redemption under NCC Art. 1621

RTC dismissed the complaint. It ruled that Helen Guzmans waiver of her inheritance in favor of her son was not
contrary to the constitutional prohibition against th e sale of land to an alien, since the purpose of the waiver was
simply to authorize David Rey Guzman to dispose of their properties in accordance with the Constitution and the
laws of the Philippines, and not to subvert them. On the second issue, it held t hat the subject land was urban; hence,
petitioners had no reason to invoke their right of redemption under NCC Art. 1621.

The CA also denied the appeal made by the petitioners. It further held that, although the transfer of the land to David
Rey may have been invalid for being contrary to the Constitution, there was no more point in allowing herein
petitioners to recover the property, since it has passed on to and was thus already owned by a qualified person.

ISSUE

Was the conveyance made to Cataniag null and void for being unconstitutional? What is the effect of a subsequent
sale by the disqualified alien vendee to a qualified Filipino citizen?

HELD

NO, the sale is valid.

True, Helen Guzmans deed of quitclaim-- collided with the Constitution, Article XII, Section 7 of which provides:
Save in cases of hereditary succession, no private lands shall be transferred or conveyed except to individuals,
corporations, or associations qualified to acquire or hold lands of the public domain. The landmark case of
Krivenko vs. Register of Deeds settled the issue as to who are qualified (and disqualified) to own public as well as
private lands in the Philippines. In there the Court maintained that the public agricultural lands mentioned in Section
1, Article XIII of the 1935 Constitution include residential, commercial and industrial lands. The constitutional
provision prohibiting the transfer or assignment of private agricultural lands to entities other than those qualified to
acquire or hold lands of the public domain in the Philippines (Section 5, Art XIII) closes the only remaining avenue
through which agricultural resources may leak into alien hands. It would certainly be futile to prohibit the alienation
of public agricultural lands to aliens if, after all, they may be freely so alienated upon their becoming private
agricultural lands in the hands of Filipino citizens. Undoubtedly, section 5 [now Sec. 7] is intended to insure the
policy of nationalization contained in section 1 [now Sec. 2]. Both sections must, t herefore, be read together.

2B 16-17 SALES AND LEASE Page 135


However, the transfer of an interest in a piece of land to an alien may no longer be assailed on constitutional grounds
after the entire parcel has been sold to a qualified citizen. Accordingly, since the disputed land is now o wned by
Cataniag, a Filipino citizen, the prior invalid transfer can no longer be assailed. The objective of the constitutional
provision -- to keep our land in Filipino hands -- has been served.

The rationale of this principle was explained in Vasquez vs. Li Seng Giap thus: If the ban on aliens from acquiring
not only agricultural but also urban lands, as construed by this Court in the Krivenko case, is to preserve the nations
lands for future generations of Filipinos, that aim or purpose would not be thwarted but achieved by making lawful
the acquisition of real estate by aliens who became Filipino citizens by naturalization.

2B 16-17 SALES AND LEASE Page 136


5. CAMILO BORROMEO VS ANTONIETTA DESCALLAR

G.R. No. 159310 February 24, 2009

Puno, C.J.

FACTS

Wilhelm Jambrich, an Austrian, met respondent Antonietta Opalla-Descallar, a separated mother of two boys who
was working as a waitress at St. Moritz Hotel. They fell in love and decided to live together. They transferred to
their own house and lots at Agro-Macro Subdivision, Cabancalan, Mandaue City. In the Contracts to Sell dated
November 18, 1985 and March 10, 1986 covering the properties, Jambrich and respondent were referred to as the
buyers. A Deed of Absolute Sale dated November 16, 1987 was likewise issued in their favor. However, when the
Deed of Absolute Sale was presented for registration before the Register of Deeds, registration was refused on the
ground that Jambrich was an alien and could not acquire alienable lands of the public domain. Consequently,
Jambrichs name was erased from the document but it could be noted that his signature remained . Transfer
Certificate of Title (TCT) Nos. 24790, 24791 and 24792 over the properties were issued in respondents name
alone.The idyll lasted only until April 1991. Jambrich met petitioner Camilo F. Borromeo sometime in 1986.
Petitioner was engaged in the real estate business. In 1989, Jambrich purchased an engine and some accessories for
his boat from petitioner, for which he became indebted to the latter for about P150,000.00. To pay for his debt, he
sold his rights and interests in the Agro-Macro properties to petitioner for P250,000, as evidenced by a "Deed of
Absolute Sale/Assignment." On July 26, 1991, when petitioner sought to register the deed of assignment, he
discovered that titles to the three lots have been transferred in the name of respondent, and that the subject property
has already been mortgaged. On August 2, 1991, petitioner filed a complaint against respondent for recovery of real
property before the Regional Trial Court of Mandaue City. RTC ruled in favor of the petitioner but the CA reversed
the decision.

ISSUE

Whether or not Jambrich has title to the properties in question and may transfer and assign any rights and interest in
favor of the petitioner

HELD

Yes. The transfer of land from Agro-Macro Development Corporation to Jambrich, who is an Austrian, would have
been declared invalid if challenged, had not Jambrich conveyed the properties to petitioner who is a Filipino citizen.
In United Church Board for World Ministries v. Sebastian, the Court reiterated the consistent ruling in a number of
cases that if land is invalidly transferred to an alien who subsequently becomes a Filipino citizen or transfers it to a
Filipino, the flaw in the original transaction is considered cured and the title of the transferee is rendered valid.
Applying United Church Board for World Ministries, the trial court ruled in favor of petitioner, viz.:

2B 16-17 SALES AND LEASE Page 137


[W]hile the acquisition and the purchase of (sic) Wilhelm Jambrich of the properties under litigation [were] void ab
initio since [they were] contrary to the Constitution of the Philippines, he being a foreigner, yet, the acquisition of
these properties by plaintiff who is a Filipino citizen from him, has cured the flaw in the original transaction and the
title of the transferee is valid.

The rationale behind the Courts ruling in United Church Board for World Ministries, as reiterated in subsequent
cases,is this since the ban on aliens is intended to preserve the nation s land for future generations of Filipinos, that
aim is achieved by making lawful the acquisition of real estate by aliens who became Filipino citizens by
naturalization or those transfers made by aliens to Filipino citizens. As the property in dispute is already in the hands
of a qualified person, a Filipino citizen, there would be no more public policy to be protected. The objective of the
constitutional provision to keep our lands in Filipino hands has been achieved.

2B 16-17 SALES AND LEASE Page 138


6. JG SUMMIT HOLDINGS VS CA

G.R. No. 124293 September 24, 2003

Puno, J

FACTS:

On January 27, 1977, the National Investment and Development Corporation (NIDC) entered into a Joint Venture
Agreement with Kawasaki Heavy Industries, Ltd. of Kobe, Japan (KAWASAKI) for the co nstruction, operation and
management of the Subic National Shipyard, Inc. (SNS) which subsequently became the Philippine Shipyard and
Engineering Corporation (PHILSECO). Under the JVA, the NIDC and KAWASAKI will contribute P330 million
for the capitalization of PHILSECO in the proportion of 60%-40% respectively with a right of first refusal should
either of them decide to sell, assign or transfer its interest.

NIDC transferred all its rights, title and interest in PHILSECO to the National Government pursua nt to
Administrative Order No. 14. President Corazon C. Aquino issued Proclamation No. 50 establishing the Committee
on Privatization (COP) and the Asset Privatization Trust (APT) to take title to, and possession of, conserve, manage
and dispose of non-performing assets of the National Government. Thereafter, on February 27, 1987, a trust
agreement was entered into between the National Government and the APT wherein the latter was named the trustee
of the National Governments share in PHILSECO. In 1989, as a result of a quasi-reorganization of PHILSECO to
settle its huge obligations to PNB, the National Governments shareholdings in PHILSECO increased to 97.41%
thereby reducing KAWASAKIs shareholdings to 2.59%

The COP and the APT deemed it best to sell the National Governments share in PHILSECO to private entities. APT
and KAWASAKI agreed that the latters right of first refusal under the JVA be exchanged for the right to top by five
percent (5%) the highest bid for the said shares. They further agreed that KA WASAKI would be entitled to name a
company in which it was a stockholder, which could exercise the right to top. On September 7, 1990, KAWASAKI
informed APT that Philyards Holdings, Inc. (PHI) would exercise its right to top.

At the pre-bidding conference held on September 18, 1993, interested bidders were given copies of the JVA between
NIDC and KAWASAKI, and of the Asset Specific Bidding Rules (ASBR) drafted for the National Governments
87.6% equity share in PHILSECO.The provisions of the ASBR were explained to the interested bidders who were
notified that bidding would be held on 2 December 1993. At the public bidding on said date, the consortium
composed of JG Summit Holdings, Inc. (JGSMI), Sembawang Shipyard Ltd. of Singapore (Sembawang), and
Jurong Shipyard Limited of Malaysia (Jurong), was declared the highest bidder at P2.03 billion. The following day,
the COP approved the sale of 87.67% National Government shares of stock in PHILSECO to said consortium. It
notified JGSMI of said approval "subject to the right of Kawasaki Heavy Industries, Inc./Philyards Holdings, Inc. to
top JGSMI's bid by 5% as specified in the bidding rules."

On December 29, 1993, petitioner informed APT that it was protesting the offer of PHI to top its bid on the grounds
that: (a) the KAWASAKI/PHI consortium composed of Kawasaki, Philyards, Mitsui, Keppel, SM Group, ICTSI and
Insular Life violated the ASBR because the last four (4) companies were the losing bidders thereby circumventing
the law and prejudicing the weak winning bidder; (b) only KAWASAKI could exercise the right to top; (c) giving
the same option to top to PHI constituted unwarranted benefit to a third party; (d) no right of first refusal can be

2B 16-17 SALES AND LEASE Page 139


exercised in a public bidding or auction sale; and (e) the JG Summit cons ortium was not estopped from questioning
the proceedings.

On 2 February 1994, JGSMI was notified that PHI had fully paid the balance of the purchase price of the subject
bidding. On 7 February 1994, the APT notified JGSMI that PHI had exercised its option to top the highest bid and
that the COP had approved the same on 6 January 1994. On 24 February 1994, the APT and PHI executed a Stock
Purchase Agreement. Consequently, JGSMI filed with the Supreme Court a petition for mandamus under GR
114057. On 11 May 1994, said petition was referred to the Court of Appeals. On 18 July 1995, the Court of Appeals
"denied" for lack of merit the petition for mandamus. JGSMI filed a motion for the reconsideration of said Decision
which was denied on 15 March 1996. JGSMI filed the petition for review on certiorari.

ISSUE

Whether the right of first refusal granted in the contract is violative of the Constitution

HELD

The agreement of co-shareholders to mutually grant the right of first refusal to each other, by itself, does n ot
constitute a violation of the provisions of the Constitution limiting land ownership to Filipinos and Filipino
corporations.As PHILYARDS correctly puts it, if PHILSECO still owns land, the right of first refusal can be validly
assigned to a qualified Filipino entity in order to maintain the 60:40 ratio. This transfer, by itself, does not amount to
a violation of the AntiDummy Laws, absent proof of any fraudulent intent. The transfer could be made either to a
nominee or such other party which the holder of the right of first refusal feels it can comfortably do business with.
Alternatively, PHILSECO may divest of its landholdings, in which case KAWASAKI, in exercising its right of first
refusal, can exceed 40% of PHILSECOs equity. In fact, it can even be s aid that if the foreign shareholdings of a
landholding corporation exceeds 40%, it is not the foreign stockholders ownership of the shares which is adversely
affected but the capacity of the corporation to own land that is, the corporation becomes disqualified to own land.
This finds support under the basic corporate law pri nciple that the corporation and its stockholders are separate
juridical entities. In this vein, the right of first refusal over shares pertains to the shareholders whereas the capacity
to own land pertains to the corporation. Hence, the fact that PHILSECO owns land cannot deprive stockholders of
their right of first refusal.

No law disqualifies a person from purchasing shares in a landholding corporation even if the latter will

exceed the allowed foreign equity, what the law disqualifies is the corporation from owning land . The

prohibition in the Constitution applies only to ownership of land . It does not extend to immovable or real property as
defined under Article 415 of the Civil Code. Otherwise, we would have a strange situation where the ownership of
immovable property such as trees, plants and growing fruit attached to the land would be limited to Filipinos and
Filipino corporations only.

2B 16-17 SALES AND LEASE Page 140


OBJECT

1. CLARIN VS RULONA

G.R. No. L-30786 February 20, 1984

Gutierrez Jr, J

FACTS

Respondent alleged that the petitioner sold ten hectares of his share of the disputed lot to him for P2,500.00. The
conditions of the sale were that a downpayment of P1,000.00 was to be made and then the balanc e of P1,500.00 was
to be paid in monthly installment of P100.00. P1100 was already paid but the petitioner declines to receive further
installments. Rulona filed a complaint for specific performance and recovery of improvements on the ground that
the petitioner and his wife violated the terms of the agreement of sale "by returning by their own volition and
without the consent of plaintiff, the amount of P1,100.00 in six postal money orders, covering the downpayment of
P1,000.00 and first installment of P100.00." Respondents alleged that such sale was subject to the following
conditions: (1) that the contract would be realized only if his co -heirs would give their consent to the sale of a
specific portion of their common inheritance from the late Aniceto Clarin before partition of the said common
property and (2) that should his co-heirs refuse to give their consent, the projected contract would be discontinued or
would not be realized. Trial court held that the sale is not subject to any condition nor is it vitiated by any flaw and
also although as co-owner the petitioner could not dispose of a specific portion of the land, nevertheless, his share
was bound by the effect of the sale. Court of Appeals sustained the findings of the trial court.

ISSUE

Whether or not the object(land) is valid considering it is co-owned by petitioners and his sisters?

HELD

Yes.

Sc held that we agree with the lower courts holding that although as a co -owner, the petitioner cannot dispose of a
specific portion of the land, his share shall be bound by the effect of the sale. This is anchored in Article 493 of the
Civil Code which provides:

Art. 493. Each co-owner shall have the full ownership of his part and the fruits and benefits pertaining thereto, and
he may therefore alienate, assign or mortgage it, and even substitute another person in its enjoyment, except when
personal rights are involved. But the effect of the alienation or the mortgage, with respect to the co -owners, shall be
limited to the portion which may be allotted to him in the division upon the termination of the co-ownership.

2B 16-17 SALES AND LEASE Page 141


2. DOMINGO CARABEO VS SPS NORBERTO AND SUSAN DINGCO

G.R. No. 190823 April 4, 2011

Carpio Morales, J.

FACTS

On July 10, 1990, petitioner entered into a contract denominated as "Kasunduan sa Bilihan ng Karapatan sa
Lupa" with respondents whereby petitioner agreed to sell his rights over a 648 square meter parcel of unregistered
land situated in Purok III, Tugatog, Orani, Bataan to respondents for P38,000.

The pertinent portion of the kasunduan reads:

Na ako ay may isang partial na lupa na matatagpuan sa Purok 111, Tugatog, Orani Bataan, na may sukat na 27 x 24
metro kuwadrado, ang nasabing lupa ay may sakop na dalawang punong santol at isang punong mangga, kayat ako
ay nakipagkasundo sa mag-asawang Norby Dingco at Susan Dingco na ipagbili sa kanila ang karapatan ng nasabing
lupa sa halagang P38,000.00.

Respondents tendered their initial payment upon signing of the contract, the remaining balance to be paid on
September 1990.

Respondents were later to claim that when they were about to hand in the balance of the purchase price,
petitioner requested them to keep it first as he was yet to settle an on -going "squabble" over the land.

Sometime in 1994, respondents learned that the alleged problem over the land had been settled and that
petitioner had caused its registration in his name under a Transfer Certificate of Title. They thereupon offered to pay
the balance but petitioner declined, drawing them to file a complaint before the Katarungan Pambarangay. No
settlement was reached, however, hence, respondent filed a complaint for specific performance before the Regional
Trial Court (RTC) of Balanga, Bataan.

Petitioner countered in his Answer to the Complaint that the sale was void for lack of object certain, the
kasunduan not having specified the metes and bounds of the land.

ISSUE

Whether or not the element of a contract, i.e., an object certain is present in this case.

HELD

YES.

That the kasunduan did not specify the technical boundaries of the property did not render the sale a nullity. The
requirement that a sale must have for its object a determinate thing is satisfied as long as, at the time the contract is
entered into, the object of the sale is capable of being made determinate without the necessity of a new or further
agreement between the parties. As the above-quoted portion of the kasunduan shows, there is no doubt that the
object of the sale is determinate.

2B 16-17 SALES AND LEASE Page 142


3. BELINDA TANEDO VS CA

G.R. No. 104482. January 22, 1996

Panganiban, J

FACTS

October 20, 1962, Lazardo Taedo executed a notarized deed of absolute sale in favor of his eldest brother, Ricardo
Taedo, and the latters wife private respondents herein, whereby he conveyed to the latte r in consideration of
P1,500.00, one hectare of whatever share he shall have over Lot No. 191, the said property being his future
inheritance from his parents. Upon the death of his father Matias, Lazaro executed an Affidavit of Conformity to re -
affirm sale he made in 1962. On January 13, 1981, Lazaro executed another notarized deed of sale in favor of
private respondents covering his undivided ONE TWELVE(1/12) of a parcel of land known as Lot 191. He
acknowledged therein his receipt of P 10,000.00 as consideration therefor. In February 1981, Ricardo learned that
Lazaro sold the same property to his children, petitioners herein, through a deed of sale dated December 29, 1980.

On June 7, 1982, private respondents recorded the Deed of Sale (Exh. 4) in their favor in the Registry of Deeds and
the corresponding entry was made in Transfer Certificate of Title No. 166451. Petitioners on July 16, 1982 filed a
complaint for rescission (plus damages) of the deeds of sale executed by Lazaro in favor of private respond ents
covering the property inherited by Lazaro from his father. TC decided in favor of private respondents, holding that
petitioners failed to adduce a preponderance of evidence to support (their) claim. CA affirmed RTC,ruling that the
Deed of Sale dated January 13, 1981 was valid and that its registration in good faith vested title in said respondents.

ISSUES:

what is on review in these proceedings by this Court is the Court of Appeals decision - which correctly identified the
subject matter of the January 13, 1981 sale to be the entire undivided 1/12 share of Lazaro in Lot No. 191 and which
is the same property disposed of on December 29, 1980 in favor of petitioners.

(1) Is the sale of a future inheritance valid?

(2) Was the subsequent execution on January 13, 1981 (and registration with the Registry of Property) of a deed of
sale covering the same property to the same buyers valid?

HELD:

- A contract of sale of anticipated future inheritance is null and void. 3

Consequently, said contract made in 1962 is not valid and cannot be the source of any right nor the creator of any
obligation between the parties.

- Hence, the affidavit of conformity dated February 28, 1980, insofar as it sought to validate or ratify the 1962
sale, is also useless and, in the words of the respondent Court, suffers from the same infirmity. Even private
respondents in their memorandum4 concede this.

2B 16-17 SALES AND LEASE Page 143


- With regard to whether there was a bad faith in the registration of the subject lot, It is well-settled that the
Supreme Court is not a trier of facts. In petitions for review under Rule 45 of the Revised Rules of Court, only
questions of law may be raised and passed upon. Absent any whimsical or capricious exercise of judgment, and
unless the lack of any basis for the conclusions made by the lower courts be amply demonstrated, the Supreme Court
will not disturb their findings.

- We see no valid reason to discard the factual conclusions of the appellate court. x x x (I)t is not the function of
this Court to assess and evaluate all over again the evidence, testimonial and d ocumentary, adduced by the parties,
particularly where, such as here, the findings of both the trial court and the appellate court on the matter coincide.
(italics supplied)

2B 16-17 SALES AND LEASE Page 144


4. ATTY PEDRO FERRER VS SPS ALFREDO DIAZ AND IMELDA DIAZ

GR NO. 165300 April 23, 2010

FACTS

Before Us is a petition for review of the decision of the CA modifying the summary judgment made by the RTC in
holding respondent spouses

Version of the Petitioner:

He claimed in his original complaint that the respondents, as represented by their daughter Comandante, thru an
SPA, obtained from him a loan on May 7,1999. This loan was secured by a real estate mortgage (REM) by way of
second mortgage over a title of a land and a PN payable within 6 mts or up to Nov 7, 1999. Petitioner further
claimed that on May 29, 1998 the daughter for P600,000, which formed part of the secured loan, executed in his
favor an instrument entitled Waiver of Hereditary Rights and Interests over a Real Property (sill undivided). On the
basis of this waiver, petitioner executed an affidavit of adverse claim which he caused to be annotated on the back of
the mortgaged title. On due date, respondents defaulted in their obligation as the checks issued by their daughter
were dishonored upon presentment. Despite repeated demands, respondents still failed and refused to pay the loan.
Petitioner then filed for the foreclosure of the mortgaged property.

Version of the Respondent:

Comandante affirmed the said loan however claimed that these loans were secured by chattel mortgages over her
taxi units in addition to several postdated checks she issued in favor of petitioner. Further, she asserted that said
complaint has no cause of action against her because the Real Estate Mortgage Contract and the waiver referred to
by petitioner in his complaint were not duly, knowingly and validly executed by her; that the Waiver of Hereditary
Rights and Interests Over a Real Property (Still Undivided) is a useless document as its execution is prohibited by
Article 1347 of the Civil Code, hence, it cannot be the source of any right or obligation in petitioners favor; that the
REM was of doubtful validity as she executed the same without valid authority from her parents; and, that the prayer
for collection and/or judicial foreclosure was irregular as petitioner cannot seek said remedies at the same time.

ISSUE

Is Comandantes waiver of hereditary rights a valid object of the contract?

HELD

No, this waiver of hereditary rights and interest over a real property is null and vo id for being violative of Art 1347
of the Civil Code.

Art 1347,para 2 CC, provides that no contract may be entered into upon a future inheritance except in cases
expressly authorized by law. For the inheritance to be considered future, the succession mus t not have been opened

2B 16-17 SALES AND LEASE Page 145


at the time of the contract. A contract may be classified as a contract upon future inheritance, prohibited under the
second paragraph of Article 1347, where the following requisites concur:

(1) That the succession has not yet been opened.

(2) That the object of the contract forms part of the inheritance; and

(3) That the promissor has, with respect to the object, an expectancy of a right which is purely hereditary in
nature.

2B 16-17 SALES AND LEASE Page 146


5. FILINVEST LAND INC VS ABDUL BACKY ET AL

G.R. No. 66641. March 6, 1992

Medialdea, J.:

FACTS:

Respondents were grantees of agricultural public lands located in Tambler, General Santos City through Homestead
and Fee patents sometime in 1986 and 1991 which are covered by and specifically described in Original Certificates
of Title issued by the Register of Deeds of General Santos City.

Negotiations were made by petitioner, represented by Lina de Guzman-Ferrer with the patriarch of the Ngilays,
Hadji Gulam Ngilay sometime in 1995. Eventually, a Deed of Conditional Sale of the properties in favor of
petitioner Filinvest Land, Inc. was executed. Upon its execution, respondents were asked to deliver to petitioner the
original owner's duplicate copy of the certificates of title of their respective properties. Respondents received the
downpayment for the properties on October 28, 1995.

A few days after the execution of the aforestated deeds and the delivery of the corresponding documents to
petitioner, respondents came to know that the sale of their properties was null and void, because it was done within
the period that they were not allowed to do so and that the sale did not have t he approval of the Secretary of the
Department of Environment and Natural Resources (DENR) prompting them to file a case for the declaration of
nullity of the deeds of conditional and absolute sale of the questioned properties and the grant of right of way with
the RTC, Las Pis, Branch 253.

On the other hand, petitioner claims that sometime in 1995, the representative of Hadji Ngilay approached petitioner
to propose the sale of a portion of his properties. Thereafter, representatives of petitioner flew to General Santos
City from Manila to conduct an ocular inspection of the subject properties. Petitioner was willing to purchase the
properties but seeing that some of the properties were registered as land grants through homestead patents,
representatives of petitioner informed Ngilay that they would return to General Santos City in a few months to
finalize the sale as ten (10) certificates of title were issued on November 24, 1991.

According to petitioner, Ngilay and his children prevailed upon the repres entatives of petitioner to make an advance
payment. To accommodate the Ngilays, petitioner acceded to making an advance with the understanding that
petitioner could demand anytime the return of the advance payment should Ngilay not be able to comply with t he
conditions of the sale. The Ngilays likewise undertook to secure the necessary approvals of the DENR before the
consummation of the sale.

ISSUE:

Whether or not he is a valid object of the contracts? NO.

2B 16-17 SALES AND LEASE Page 147


HELD

The five-year prohibitory period following the issuance of the homestead patent is provided under Section 118 of
Commonwealth Act No. 141, as amended by Commonwealth Act No. 456, otherwise known as the Public Land Act.
It bears stressing that the law was enacted to give the homesteader or patent ee every chance to preserve for himself
and his family the land that the State had gratuitously given to him as a reward for his labour in cleaning and
cultivating it. Its basic objective, as the Court had occasion to stress, is to promote public policy th at is to provide
home and decent living for destitute, aimed at providing a class of independent small landholders which is the
bulwark of peace and order. Hence, any act which would have the effect of removing the property subject of the
patent from the hands of a grantee will be struck down for being violative of the law.

To repeat, the conveyance of a homestead before the expiration of the five -year prohibitory period following the
issuance of the homestead patent is null and void and cannot be enforce d, for it is not within the competence of any
citizen to barter away what public policy by law seeks to preserve.

2B 16-17 SALES AND LEASE Page 148


6. JOSELITO BORROMEO VS. JUAN T. MINA

GR No. 193747 JUNE 5, 2013

PERLAS-B ERNAB E, J.:

FACTS:

Petitioner filed a Petition dated June 9, 2003 before the Provincial Agrarian Reform Office (PARO) of Isabela,
seeking that: (a) his landholding over the subject property (subject landholding) be exempted from the coverage of
the governments OLT program under PD 27 and (b) respondents emancipation patent over the subject property be
consequently revoked and cancelled. To this end, petitioner alleged that he purchased the aforesaid property from its
previous owner, one Serafin M. Garcia (Garcia), as evidenced by a deed of sale notarize d on February 19, 1982. For
various reasons, however, he was not able to effect the transfer of title in his name. Subsequently, to his surprise, he
learned that an emancipation patent was issued in respondents favor without any notice to him. He equally
maintained that his total agricultural landholdings was only 3.3635 hectares and thus, within the landowner's
retention limits under both PD 27 and Republic Act No. 6647, otherwise known as the Comprehensive Agrarian
Reform Law of 1988.

In this regard, he claimed that the subject landholding should have been excluded from the coverage of the
governments OLT program. After due investigation, the Municipal Agrarian Reform Officer (MARO) Joey
Rolando M. Unblas issued a Report dated September 29, 2003,11 finding that the subject property was erroneously
identified by the same office as the property of petitioners father, the late Cipriano Borromeo. In all actuality,
however, the subject property was never owned by Cipriano Borromeo as its true owner was Garcia notably, a
perennial PD 27 landowner who later sold the same to petitioner. PARO and DAR ruled in favor of petitioner. CA
found the said sale to be null and void for being a prohibited transaction under PD 27 which forbids the transfers or
alienation of covered agricultural lands after October 21, 1972 except to the tenant -beneficiaries thereof, of which
petitioner was not.

ISSUE:

Whether or not CA erred in declaring the sale null and void?

HELD:

NO. PD 27 prohibits the transfer of ownership over tenanted rice and/or corn lands after October 21, 1972 except
only in favor of the actual tenant tillers thereon.

Records reveal that the subject landholding fell under the coverage of PD 27 on October 21, 197238 and as such,
could have been subsequently s old only to the tenant thereof, i.e., the respondent. Notably, the status of respondent
as tenant is now beyond dispute considering petitioners admission of such fact.39 Likewise, as earlier discussed,
petitioner is tied down to his initial theory that his claim of ownership over the subject property was based on the
1982 deed of sale. Therefore, as Garcia sold the property in 1982 to the petitioner who is evidently not the tenant -
beneficiary of the same, the said transaction is null and void for being con trary to law.

2B 16-17 SALES AND LEASE Page 149


CAUSE

1. RP VS PHIL RESOURCES DEVT CORP

G.R. No. L10141 January 31, 1958

FACTS
The RP, in representation of the Bureau of Prisons, instituted against Macario Apostol and the Empire Insurance Co.
a complaint with the Court of First instance of Manila, alleging that defendant Apostol submitted the highest bid the
amount P450.00 per ton for the purchase of 100 tons of Palawan Almaciga from the Bureau of Prisons that a
contract therefor was drawn and by virtue of which, Apostol obtained goods from the Bureau of Prisons valued
P15,878.59 that of said account, Apostol paid only P691.10 leaving a balane obligation of P15,187.49. The
complaint further avers that Apostol submitted the best bid with the Bureau of Prisons for the purchase of three
million board feet of logs at P88.00 per 1,000 board feet that a contract was executed between the Director of
Prisons and Apostol pursuant to which contract Apostol obtained deliveries of logs valued at P65.830.00, and that
Apostol failed to pay a balance account Of P18,827.57. In his answer, Apostol interposed payment as a defense and
sought the dismissal of the complaint.
The Philippine Resources Development Corporation moved to intervene.The complaint recites that for sometime
prior to Apostol's transactions the corporate had some goods deposited in a warehouse at 1201 Herran, Manila that
Apostol, then the president of the corporation but without the knowledge or consent of the stockholders thereof,
disposed of said goods by delivering the same to the Bureau of Prisons of in an attempt to settle his personal debts
with the latter entity that upon discovery of Apodol's act , the corporation took steps to recover said goods by
demanding from the Bureau of Prisons the return thereof and that upon the refusal of the Bureau to return said
goods, the corporation sought leave to intervene.The trial court denied the motion for int ervention. A motion for the
reconsideration of said order was filed by the movant corporation and the same was likewise denied. Upon appeal to
the CA, the Court of Appeals set aside the order denying the motion to intervene and ordered the respondent court to
admit the herein respondent corporation's complaint in intervention with costs against Macario Apostol.
The present petition is filed, with the petitioner, contending that the intervenor has no legal interest in the matter in
litigation, because the action brought in the Court of First Instance of Manila against Macario Apostol and the
Empire Insurance Company is just for the collection from the defendant Apostol of a sum of money, the unpaid
balance of the purchase price of logs and almaciga bought by him from the Bureau of Prisons, whereas the
intervenor seeks to recover ownership and possession of G. I. sheets, black sheets, M. S. plates, round bars and G. I.
pipes that it claims its owns an intervention which would change a personal action into one a d rem and would
unduly delay the disposition of the case.

ISSUE
Whether or not the intervenor has no legal interest in the matter in litigation

HELD
NO. The CA correctly ruled that there is no merit in their contention that the subject matter of the original litigation
is a sum of money allegedly due to the Bureau of Prisons from Macario Apostol and not the goods or the materials
reportedly turned over by Apostol as payment of his private debts to the Bureau of Prisons which is now sought to
be recovered. It is true that the very subject matter of the original case is a sum of money. But it is likewise true as
borne out by the records, that the materials purportedly belonging to the petitioner corporation have been assessed
and evaluated and their price equivalent in terms of money have been determined and that said materials for

2B 16-17 SALES AND LEASE Page 150


whatever price they have been assigned by defendant now respondent Apostol as tokens of payment of his private
debts with the Bureau of Prisons.
The Government argues that "Price . . . is always paid in terms of money and the supposed payment beeing in kind,
it is no payment at all, "citing Article 1458 of the new Civil Code. However, the same Article provides that the
purschaser may pay "a price certain in money or its equivalent," which means that they meant of the price need not
be in money. Whether the G.I. sheets, black sheets, M. S. Plates, round bars and G. I. pipes claimed by the
respondent corporation to belong to it and delivered to the Bureau of Prison by Macario Apost ol in payment of his
account is sufficient payment therefore, is for the court to pass upon and decide after hearing all the parties in the
case. Should the trial court hold that it is as to credit Apostol with the value or price of the materials delivered by
him, certainly the herein respondent corporation would be affected adversely if its claim of ownership of such
sheets, plates, bars and pipes is true.

2B 16-17 SALES AND LEASE Page 151


2. BAGNAS VS CA

G.R. No. L-38498 August 10, 1989

FACTS

Hilario Mateum who died on March 11, 1964 was without any ascendants or descendants. He was survived only
with collateral relatives the nearest of whom are the petitioners, first cousins. The decedent left no will, no debts,
and an estate consisting of 29 parcels of land, 10 of which are involved in the case.
April 3, 1964, respondents who are more distant relatives of the deceased registered with RD deeds of sale
purportedly executed by the deceased. The consideration for the deeds was Php1.
Petitioners commenced suit against the respondents in the Court of First Instance of Cavite, seeking annulment of
the deeds of sale as fictitious, fraudulent or falsified, or, alternatively, as donations void for want of acceptance
embodied in a public instrument.

Assessed value of the properties amount to Php10,500.

ISSUE

Whether the sales were void for want of spuriousness of consideration.

HELD
Deeds purporting to be sales are VOID AB INITIO.
There is an apparent gross disproportion between the stipulated price (in each deed) Php1 plus unspecified and
unquantified services and the undisputably valuable real estate allegedly sold worth at least Php10,500 going only
by assessments for tax purposes (which are low indicators of the true value of a property). This demonstrate a false
and fictitious consideration.
Its validity can neither be sustained by the claim that it is a donation for the reason that the same is not accepted and
made in a public instrument.
Therefore, properties purportedly conveyed remained part of the estate of Hilario M ateum, and are recoverable by
his intestate heirs (petitioners) whose status as such is not challenged.

2B 16-17 SALES AND LEASE Page 152


3. GOQUILAY VS SYCIP

G.R. No. L-11840 July 26, 1960

Reyes, JBL J

FACTS

On May 29, 1940, Tan Sin An and Antonio C. Goquiolay entered into a general commercial partnership under the
partnership name "Tan Sin An and Antonio C. Goquiolay", for the purpose in dealing in real estate.

The plaintiff partnership purchased the 3 parcels of land from the Cadastral Survey of Davao, assuming th e payment
of a mortgage obligation of P25,000.00, payable to "La Urbana Sociedad Mutua de Construccion y Prestam.
Another 46 parcels were purchased by Tan Sin An in his individual capacity.

On September 25, 1940, the two separate obligations were consolid ated in an instrument executed by the partnership
and Tan Sin An, whereby the entire 49 lots were mortgaged in favor of the "Banco Hipotecario de Filipinas" (as
successor to "La Urbana") and they bound themselves to pay, jointly and severally, the remainin g balance of
P52,282.80 within eight 8 years.

Tan Sin died and defendant-wife Kong Chai Pin was appointed administratrix of the intestate estate of her deceased
husband.

The partnership failed to comply with payments, and Sing Yee and Cuan, Co., Inc. paid for the remaining balance of
the mortgage debt. In 1946, Sing Yee and Cuan Co., Inc. filed their claims in the intestate proceedings of Tan Sin.
Due to such, Kong Chai Pin sought the permission of the probate court to sell all 49 parcels of land to settle the
debts. Such was granted and thereafter a sale in favor of Washington Sycip and Betty Lee was executed. A
subsequent conveyance of 3 lots in favor of Insular Development Co., Inc., was also executed.

In 1949, surviving partner Antonio Goquiolay sought to set aside the order of the probate court approving the sale in
so far as his interest over the parcels of land sold was concerned. The orders were set aside and the case was
remanded for new trial.

In his complaint, Goquilay alleged among others that the sale was fraudulent due to the inadequate price paid for the
property (sold for P153,726.04 but a realtor asserted that the land was worth P312,000).

ISSUE

Whether or not the alleged inadequate price justifies the rescission of the sale

HELD

2B 16-17 SALES AND LEASE Page 153


No, the Supreme Court held that "the sale in question was practically a forced sale because the partnership had no
other means to pay its legitimate debts, this evidence certainly does not show such 'gross inadequacy' as to justify
rescission of the sale." The Court noted that the value asserted by the realtor was based on the value of the land 6
years after the sale, and there being a continued rise of real estate values since the liberation, such difference in
value was to be expected.

2B 16-17 SALES AND LEASE Page 154


4. LADANGA VS CA

G.R. No. L-55999 August 24, 1984

Aquino, J

FACTS

Clemencia was a spinster, retired at 65 in 1961. She had a nephew she legally adopted in 1961 named Bernardo S.
Aseneta, the child of her sister Gloria. She also had a niece named Salvacion , the daughter of her sister Flora. On
April 6, 1974 (when Clemencia was about 78 years old), she signed nine deeds of sale in favor of Salvacion for
various real properties. One deed of sale concerned the Paco property (administered by the Ladanga spouses ) which
purportedly was sold to Salvacion for P26,000.

On the witness stand, Clemencia denied having "received even one centavo" of the price of P26,000. This testimony
was corroborated by Soledad L. Maninang, 69, a dentist with whom Clemencia had lived for more than thirty years.
The notary testified that the deed of sale for the Paco property was signed in the office of the Quezon City registry
of deeds. He did not see Salvacion giving any money to Clemencia.

In May, 1975, Bernardo as guardian of Clemencia, filed an action for reconveyance of the Paco property, accounting
of the rentals and damages. Clemencia was not mentally incompetent but she was placed under guardianship
because she was an easy prey for exploitation and deceit.

Parenthetically, it should be stated that she died on May 21, 1977 at the age of 80. She allegedly bequeathed her
properties in a holographic will dated November 23, 1973 to Doctor Maninang. In that will she disinherited
Bernardo. The will was presented for probate.

In the instant case, the trial court and the Appellate Court declared void the sale of the Paco property. The Ladanga
spouses contend that the Appellate Court disregarded the rule on burden of proof. This contention is devoid of merit
because Clemencia herself testified that the price of P26,000 was not paid to her. The burden of the evidence shifted
to the Ladanga spouses. They were not able to prove the payment of that amount. The sale was fictitious.

The Ladanga spouses argue that the Appellate Court erred in no t considering that inadequacy of price may indicate a
donation or some other contract; in disregarding the presumption that the sale was fair and regular and for a
sufficient consideration; in overlooking important facts and in not holding that Bernardo ha d no right to file a
complaint to annul the sale.

ISSUE

2B 16-17 SALES AND LEASE Page 155


Whether or not there was a valid contract of sale notwithstanding lack of consideration? NO

HELD

A contract of sale is void and produces no effect whatsoever where the price, which appears therein a s paid, has in
fact never been paid by the purchaser to the vendor. Such a sale is inexistent and cannot be considered
consummated. It was not shown that Clemencia intended to donate the Paco property to the Ladangas. Her
testimony and the notary's testimony destroyed any presumption that the sale was fair and regular and for a true
consideration.

Judge Colayco concluded that the Ladangas abused Clemencia's confidence and defrauded her of properties with a
market value of P393,559.25 when she was already 78 years old.

The contention that Bernardo had no right to institute the instant action because he was not a compulsory heir of
Clemencia cannot be sustained. Bernardo was Clemencia's adopted son. Moreover, Clemencia, by testifying in this
case, tacitly approved the action brought in her behalf.

But the moral damages awarded by the trial court is not sanctioned by articles 2217 to 2220 of the Civil Code.
Clemencia's own signature in the deed brought about the mess within which she was entangled.

2B 16-17 SALES AND LEASE Page 156


5. GARDNER VS CA

G.R. No. L-59952 August 31, 1984

Melencio-Herrera, J

FACTS

Petitioners, the Gardner spouses, enter into an agreement with Respondent spouses, the Santoses to subdivide 2
parcels of land and they executed an absolute deed of sale in favor of the latter. The real truth is that what occurred
was a sale in trust since the petitioner obtained an amount of money from the respondents, who inturn promised to
improve the land.

Apparently, the Santoses transferred the properties to the Cuencas who in turn transferred it to the Verroyas who
executive a mortgage over the lot. Then Verroya executed a deed of transfers to the Natividads. Note that from the
titles of the Cuencas (the Second Transferees) to the titles of the Natividads (the Fourth Transferee), the Adverse
Claim of the Gardners continued to be carried, and that throughout the successive transfers, the petitioners continued
to remain in possession, cultivation and occupation of the disputed properties.

In their Answer, the Santoses claimed that the sale to them was conditional in the sense that the properties were to
be considered as the investment of the petitioners in the subdivision venture and that in the event that this did not
materialize they were to reconvey the lots to petitioners upon reimbursement by the latter of all sums advanced to
them; and that the deed of sale was to be registered for the protection of the Santoses considering the moneys that
the latter would be advancing.

Hence, the Gardners filed an action for declaration of Nullity, Rescission and damages against the 5 transferres and
mortgagees. The RTC ruled in favor of petitioners declaring the transfers null and void. The CA affirmed in toto the
RTC but reconsidered it decision and ruled that the sale of land to Natividads are valid.

ISSUE

Whether or not the admissions made by Santos in the pleadings are admissible? NO

HELD

The testimony of Ariosto Santos is at variance with the allegations in his Answer. As a general rule, facts alleged in
a party's pleading are deemed admissions of that party and binding upon it, but this is not an absolute and inflexible
rule. An Answer is a mere statement of fact which the party filing it expects to prove, but it is not evidence.

2B 16-17 SALES AND LEASE Page 157


1. Santos himself, in open Court, had repudiated the defenses raised in his answer and against his own interest, his
testimony is deserving of weight and credence. Both the Trial Court and the Appellate Court believed in his
credibility and we find no reason to overturn their findings thereon. Sa ntos likewise admitted against his own
interest that the petitioners did not receive from him any consideration, which corroborated the declarations of the
petitioners. The Subdivision Joint Venture Agreement and the Supplemental Agreement express that the true and
real nature of the agreement between the parties, which was for a subdivision and not a sale transaction.

2. All Five Transfers were absolutely simulated and fictitious and were, therefore, void ab initio and inexistent.
Contracts of sale are void and produce no effect whatsoever where the price, which appears therein as paid, has, in
fact, never been paid by the purchaser to the vendor.

2B 16-17 SALES AND LEASE Page 158


6. DE PORTUGAL VS IAC

G.R. No. 73564 March 25, 1988

Sarmiento, J

FACTS

Petitioner Cornelia Clanor and her late husband Pascual Portugal, during the lifetime of the latter, were able to
accumulate several parcels of real property. Among these were a parcel of residential land situated in Poblacion,
Gen. Trias, Cavite, designated as Lot No. 3201, covered by T.C.T. No. RT-9355, in their names, and an agricultural
land located at Pasong Kawayan, Gen. Trias, Cavite, known as Lot No. 2337, and also registered in their names
under T.C.T. No. RT-9356 of the Registry of Deeds for the Province of Cavite.

Sometime in January, 1967, the private respondent Hugo Portugal, a son of the spouses, borrowed from his mother,
Cornelia, the certificates of title to the above-mentioned parcels of land on the pretext that he had to use them in
securing a loan that he was negotiating. Cornelia, assented and delivered the titles to her son. The matter was never
again brought up until after Pascual Portugal died on November 17, 1974. (Cornelia herself died on November 12,
1987.) When the other heirs of the deceased Pascual Portugal, the petitioners herein, for the purposes of executing
an extra-judicial partition of Pascual's estate, wished to have all the properties of the spouses collated, Cornelia
asked the private respondent for the return of the two titles she previously loaned, Hugo manifested that the said
titles no longer exist. When further questioned, Hugo showed the petitioners Transfer Certificate of Title T.C.T. No.
23539 registered in his and his brother Emiliano Portugal's names, and which new T.C.T. cancelled the two previous
ones. This falsification was triggered by a deed of sale by which the spouses Pascual Portugal and Cornelia Clanor
purportedly sold for P8,000.00 the two parcels of land adverted earlier to their two sons, Hugo and Emiliano.
Confronted by his mother of this fraud, Emiliano denied any participation. And to show his good faith, Emiliano
caused the reconveyance of Lot No. 2337 previously covered by TCT No. RT-9356 and which was conveyed to him
in the void deed of sale. Hugo, on the other hand, refused to make the necessary restitution thus compelling the
petitioners, his mother and his other brothers and sisters, to institute an action for the annulment of the controversial
deed of sale and the reconveyance of the title over Lot No. 3201 (the residential land). After hearing, the trial court
rendered its decision declaring the deed of sale inoperative.

From this decision, Hugo Portugal appealed to the respondent appellate court which reversed, hence the present
petition.

ISSUES:

(1) Whether or not the present action has prescribed; NO

(2)Whether or not the appellate court could entertain the defense of prescription which was not raised by the private
respondents in their answer to the complaint nor in a motion to dismiss. Yes

2B 16-17 SALES AND LEASE Page 159


HELD:

(1) The trial court correctly ruled that the action instituted by the petitioners has not yet prescribed. Be that as it may,
the conclusion was reached through an erroneous rationalization, i.e., the case is purely for reconveyance based on
an implied or constructive trust. Obviously, the trial court failed to consider the lack of consideration or cause in the
purported deed of sale by which the residential lot was allegedly transferred to the private respondent by his parents.
On the other hand, the respondent Intermediate Appellate Court held that since the action for reconveyance was
fathered by a fraudulent deed of sale, Article 1391 of the Civil Code which lays down the rule that an action to annul
a contract based on fraud prescribes in four years, applies. Hence, according to the respondent court, as more than
four years had elapsed from January 23, 1967 when the assailed deed was registered and the petitioners' cause of
action supposedly accrued, the suit has already become stale when it was commenced on Oc tober 26, 1976, in the
Court of First Instance of Cavite.

The case at bar is not purely an action for reconveyance based on an implied or constructive trust. Neither is it one
for the annullment of a fraudulent contract. A closer scrutiny of the records o f the case readily supports a finding
that fraud and mistake are not the only vices present in the assailed contract of sale as held by the trial court. More
than these, the alleged contract of sale is vitiated by the total absence of a valid cause or cons ideration. The
petitioners in their complaint, assert that they, particularly Cornelia, never knew of the existence of the questioned
deed of sale. They claim that they came to know of the supposed sale only after the private respondent, showed to
them the controversial deed. And their claim was immeasurably bolstered when the private respondent's co -
defendant below, his brother Emiliano Portugal, who was allegedly his co -vendee in the transaction, disclaimed any
knowledge or participation therein. If this is so, and this is not contradicted by the decisions of the courts below, the
inevitable implication of the allegations is that contrary to the recitals found in the assailed deed, no consideration
was ever paid at all by the private respondent. Applying t he provisions of Articles 1350, 1352, and 1409 of the new
Civil Code in relation to the indispensable requisite of a valid cause or consideration in any contract, and what
constitutes a void or inexistent contract, we rule that the disputed deed of sale is void ab initio or inexistent, not
merely voidable. And it is provided in Article 1410 of the Civil Code, that '(T)he action or defense for the
declaration of the inexistence of a contract does not prescribe.

But even if the action of the petitioners is for reconveyance of the parcel of land based on an implied or constructive
trust, still it has been seasonably filed. For as heretofore stated, it is now settled that actions of this nature prescribe
in ten years, the point of reference being the date of registration of the deed or the date of the issuance of the
certificate of titIe over the property. In this case, the petitioner commenced the instant action for reconveyance in
the trial court on October 26, 1976, or less than ten years from January 23, 1967 when the deed of sale was
registered with the Register of Deeds. Clearly, even on this basis alone, the present action has not yet prescribed.

(2) We have already ruled that the defense of prescription although not raised by the defendant may neverthe less be
passed upon by the court when its presence is plainly apparent on the face of the complaint itself. At any rate, in
view of our earlier finding that the deed of sale in controversy is not simply fraudulent but void ab initio or
inexistent our ruling on this third issue would not have any material bearing on the overall outcome of this petition.
The petitioner's action remains to be seasonably instituted.

2B 16-17 SALES AND LEASE Page 160


7. GAUDENCIO VALERIO ET AL VS VICENTA REFRECA

G.R. NO. 163687 : March 28, 2006

Puno, J

FACTS

Narciso Valerio, owned two (2) adjacent agricultural lots in Calamba, Laguna. One of these lots, Lot 428, was a
four-hectare land. A portion thereof, consisting of 511 sq. m. and known as Lot 428-A, is the subject of the petition
in the case at bar.

Spouses Alejandro and Vicenta Refresca were tenants cultivating the land. On February 10, 1975, Narciso Valerio
and his wife Nieves, executed a Deed of Sale where he sold his landholding to his heirs. Narciso likewise conveyed
511 sq. m. of his landholding, known as Lot 428-A, in favor of his tenant Alejandro Refresca. On February 15,
1975, Narciso Valerio died.

On December 13, 1982, the parties to the Deed of Sale, the co-heirs and Alejandro, as co-owners, subdivided the
6.5-hectare land and executed a Deed of Agreement of Subdivision. The same 511 sq. m. of land was granted to
tenant Alejandro Refresca. On March 4, 1987, petitioners' mother, Nieves Valerio, died. After tenant Alejandro's
demise in 1994, his widow, respondent Vicenta Refresca, succeeded him by operation of law in tilling the land.

Thereafter, petitioners demanded that the respondents vacate the land. They alleged that the 511 sq. m. lot was given
to the respondents on the condition that they will surrender their tenancy rights over the entire land but respondents
failed to do so. In 1995, the Department of Agrarian Reform (DAR) issued a Resolution recognizing the right of
respondent Vicenta Refresca, widow of tenant Alejandro, to continue her peaceful possession and cultivation of the
6.5-hectare land.

In 1998, despite the DAR ruling, petitioners sent a demand letter to respondents to vacate the land. Respondents
refused. Petitioners filed a complaint before the Regional Trial Court (RTC) of Calamba, Laguna, against
respondents - - widow and children of Alejandro Refresca - - for the annulment of documents of transfer and title of
Alejandro.

Respondents maintained that the 511 sq. m. lot was granted by Narciso to tenant Alejandro as a homelot due to the
generosity of the Valerio spouses; that as co-owners, petitioners and Alejandro subdivided the land in order that
separate titles may be issued to them; that as tenant, she paid lease rentals to petitioners who initially accepted them;
and, that upon the death of petitioners' mother, Nieves Valerio, petitioners demanded the Refrescas to return the 511
sq. m. land as the former intended to sell the entire land which shall then be converted to commercial use.
Respondents likewise invoked prescription and estoppel in their defense.

2B 16-17 SALES AND LEASE Page 161


The RTC ruled in favor of petitioners. On appeal, the Court of Appeals reversed the decision of the RTC.

ISSUES

(1) Whether or not the sale of the subject land was simulated and fictitious? No

(2) Whether or not the the petitioners are estopped from impuging the validit y of the sale? Yes

HELD

(1)The main characteristic of an absolute simulation is that the apparent contract is not really desired or intended to
produce legal effect or in any way alter the juridical situation of the parties. As a result, an absolutely simulated or
fictitious contract is void. However, if the parties state a false cause in the contract to conceal their real agreement,
the contract is relatively simulated and the parties are still bound by their real agreement. Hence, where the essential
requisites of a contract are present and the simulation refers only to the content or terms of the contract, the
agreement is absolutely binding and enforceable between the parties and their successors in interest.

In the case at bar, the records reveal that the clear intent of Narciso Valerio in executing the 1975 Deed of Sale was
to transfer ownership of the apportioned areas of his 6.5-hectare land to petitioners as his heirs and to his tenant
Alejandro. Although no monetary consideration was received by landowner Narciso from any of the vendees, it
cannot be said that the contract was not supported by a cause or consideration or that Narciso never intended to
transfer ownership thereof.

The primary consideration in determining the true nature of a contract is the intention of the parties. If the words of a
contract appear to contravene the evident intention of the parties, the latter shall prevail. Such intention is
determined not only from the express terms of their agreement, but also from the contemporaneous and subsequent
acts of the parties.

In the case at bar, the circumstances reveal that when landowner Narciso executed the 1975 Deed of Sale, he
intended to transfer ownership of his entire 6.5-hectare landholding and apportion the area among Alejan dro and the
petitioners. Neither he nor his wife, during their lifetime, exerted effort to evict respondents when the latter allegedly
failed to comply with the condition to surrender their tenancy rights after the sale.

(2) We likewise agree with the findings of the Court of Appeals that petitioners are estopped in impugning the sale
as they overtly recognized the validity of the transfer of the apportioned lot to tenant Alejandro. Indeed, subsequent
to the execution of the Deed of Sale, petitioners and A lejandro, as co-owners, voluntarily partitioned the 6.5-hectare
lot which became the basis for the issuance of separate titles in their names. By this explicit act, petitioners clearly
intended to be bound by the 1975 Deed of Sale which transferred the sub divided lots to each of the parties.

2B 16-17 SALES AND LEASE Page 162


Thus, we rule that the 1975 Deed of Sale between the parties is a relatively simulated contract as the clear intent was
to transfer ownership over the land. Hence, the contract binds the parties to their true agreement , i.e., to cause the
transfer of the specific apportioned areas to Alejandro and petitioners. Petitioners failed to discharge the burden of
proving their allegation that the 1975 Deed of Sale is a void contract for being absolutely simulated.

As this Court has ruled on the validity of the 1975 Deed of Sale, we find no reason to pass upon the issue of
prescription raised by petitioners.

2B 16-17 SALES AND LEASE Page 163


8. SPS JOSE AND MILAGROS VILLACERAN VS JOSEPHINE DE GUZMAN

GR NO. 169055 February 22, 2012

Villarama, J

FACTS
Josephine De Guzman filed a Complaint with the RTC of Echague, Isabela against the spouses Jose and
Milagros Villaceran and Far East Bank & Trust Company (FEBTC), Santiago City Branch, for declaration of nullity
of sale, reconveyance, redemption of mortgage and damages with preliminary injunction. The complaint was later
amended to include annulment of foreclosure and Sheriffs Certificate of Sale. De Guzman alleged that she is the
registered owner of a parcel of land covered by Transfer Certificate of Title (TCT) No. T-236168. On April 17,
1995, she mortgaged the lot to the Philippine National Bank (PNB) of Santiago City to secure a loan of P600,000. In
order to secure a bigger loan to finance a business venture, De Guzman asked Milagros Villa ceran to obtain an
additional loan on her behalf. She executed a Special Power of Attorney in favor of Milagros. Considering De
Guzmans unsatisfactory loan record with the PNB, Milagros suggested that the title of the property be transferred to
her and Jose Villaceran and they would obtain a bigger loan as they have a credit line of up to P5,000,000 with the
bank.

On June 19, 1996, De Guzman executed a simulated Deed of Absolute Sale in favor of the spouses
Villaceran. On the same day, they went to the PNB and paid the amount of P721,891.67 using the money of the
spouses Villaceran. The spouses Villaceran registered the Deed of Sale and secured TCT No. T-257416 in their
names. Thereafter, they mortgaged the property with FEBTC Santiago City to secure a loan
of P1,485,000. However, the spouses Villaceran concealed the loan release from De Guzman. Later, when De
Guzman learned of the loan release, she asked for the loan proceeds less the amount advanced by the spouses
Villaceran to pay the PNB loan. However, the spouses Villaceran refused to give the money stating that they are
already the registered owners of the property and that they would reconvey the property to De Guzman once she
returns the P721,891.67 they paid to PNB.

De Guzman offered to pay P350,000 provided that the spouses Villaceran would execute a deed of
reconveyance of the property. In view of the simulated character of their transaction, the spouses Villaceran
executed a Deed of Absolute Sale dated September 6, 1996 in favor of De Guzman. They also promised to pay their
mortgage debt with FEBTC to avoid exposing the property to possible foreclosure and auction sale. However, the
spouses Villaceran failed to settle the loan and subsequently the property was extrajudicially foreclosed. A Sheriffs
Certificate of Sale was issued in favor of FEBTC for the amount of P3,594,000.De Guzman asserted that the spouses
Villaceran should be compelled to redeem their mortgage so as not to prejudice her as the real owner of the property.

The RTC ruled that the Deed of Sale dated June 19, 1996 executed by De Guzman in favor of the spouses
Villaceran covering the property located in Echague, Isabela was valid and binding on the parties. The RTC ruled
that the said contract was a relatively simulated contract, simu lated only as to the purchase price, but nonetheless
binding upon the parties insofar as their true agreement is concerned. The true consideration for the sale, according
to the RTC, was the P300,000 the spouses Villaceran gave to De Guzman plus the P721,891.67 they paid to PNB in
order that the title to the subject property may be releasedand used to secure a bigger loan in another bank. Hence,
according to the RTC, the spouses Villaceran should return to De Guzman (1) the P350,000 which she paid to them
in consideration of the September 6, 1996 Deed of Sale, which sale did not materialize because the title was in the
possession of FEBTC; and (2) the amount of P763,108.33 which is the net proceeds of the loan after deducting

2B 16-17 SALES AND LEASE Page 164


the P721,891.67 that the spouses paid to PNB. Thus, the sps Villaceran appealed to CA which affirmed the decision
of the trial court with modification. Hence, this petition.

ISSUE

Whether or not the CA erred in ruling that the Deed of Sale dated June 19, 1996 is a simulated contract and not a
true sale of the subject property.

HELD

The SC held that CA did not err in ruling that the Deed of Sale dated June 19, 1996 is a simulated contract
and not a true sale of the subject property. Article 1345 of the Civil Code provides that the simulation of a contract
may either be absolute or relative. In absolute simulation, there is a colorable contract but it has no substance as the
parties have no intention to be bound by it. The main characteristic of an absolute simulation is that the apparent
contract is not really desired or intended to produce legal effect or in any way alter the juridical situation of the
parties. As a result, an absolutely simulated or fictitious contract is void, and the parties may recover from each
other what they may have given under the contract. However, if the parties state a false cause in the contract to
conceal their real agreement, the contract is only relatively simulated and the parties are still bound by their real
agreement. Hence, where the essential requisites of a contract are present and the simulation refers only to the
content or terms of the contract, the agreement is absolutely binding and enforceable between the parties and their
successors in interest.

The primary consideration in determining the true nature of a contract is the intention of the parties. If the
words of a contract appear to contravene the evident intention of the parties, the latter shall prevail. Such intention is
determined not only from the express terms of their agreement, but als o from the contemporaneous and subsequent
acts of the parties. In the case at bar, there is a relative simulation of contract as the Deed of Absolute Sale dated
June 19, 1996 executed by De Guzman in favor of petitioners did not reflect the true intention of the parties.

It is worthy to note that both the RTC and the CA found that the evidence established that the aforesaid
document of sale was executed only to enable petitioners to use the property as collateral for a bigger loan, by way
of accommodating De Guzman. Thus, the parties have agreed to transfer title over the property in the name of
petitioners who had a good credit line with the bank. The CA found it inconceivable for De Guzman to sell the
property for P75,000 as stated in the June 19, 1996 Deed of Sale when petitioners were able to mortgage the
property with FEBTC for P1,485,000. Another indication of the lack of intention to sell the property is when a
few months later, on September 6, 1996, the same property, this time already registered in t he name of petitioners,
was reconveyed to De Guzman allegedly for P350,000.

2B 16-17 SALES AND LEASE Page 165


9. YU BUN GUAN VS ELVIRA ONG

GR NO. 144735 October 18, 2001

Panganiban, J

FACTS

Herein respondent said that she and petitioner are husband and wife, having been married acco rding to Chinese rites
on April 30, 1961. They lived together until she and her children were abandoned by [petitioner] on August 26,
1992, because of the latters incurable promiscuity, volcanic temper and other vicious vices out of the reunion were
born three (3) children, now living with her [respondent].

She purchased on March 20, 1968, out of her personal funds, a parcel of land, then referred to as the Rizal
property, from Aurora Seneris, and supported by Title No. 26795, then subsequently registered on April 17, 1968, in
her name.

Also during their marriage, they purchased, out of their conjugal funds, a house and lot, in 1983, thereafter,
registered in their names, under Title No. 118884.

Before their separation in 1992, she reluctantly agreed to execute a Deed of Sale of the Rizal Property on
the promise that Yu Bun Guan would construct a commerical building for the benefit of the children. He suggested
that the property should be in his name alone so that she would not be involved in any obligation . The consideration
for the sale was the execution of a Deed of Absolute Sale in favor their children and the payment of the loan he
obtained from Allied Bank. However, when the Deed of Sale was executed in favor of Yu Bun Guan, he did not pay
the consideration of P200K, supposedly the "ostensible" valuable consideration. Because of this, the new TCT
issued in his name was not delivered to him by Ong. Yu Bun Guan then filed for a Petition for Replacement of the
TCT, with an Affidavit of Loss attached. Ong, on the other hand, executed an Affidavit of Adverse Claim and asked
that the sale be declared null and void . RTC ruled in favor of Ong. CA affirmed.

ISSUE

Whether or not there was a valid contract of sale.

HELD

None. The SC held that there was no valid contract of sale. It is clear from the findings of the lower courts
that the Deed of Sale was completely simulated and thus, VOID without effect. No portion of the P200,000
consideration stated in the Deed was ever paid. And, from the facts of the case, it is clear that neither party had any
intention whatsoever to pay that amount. Instead, the Deed of Sale was executed merely to facilitate the transfer of
the property to petitioner pursuant to an agreement between them to enable him to construct a commercial building
and to sell the Juno property to their children. Being merely a subterfuge, that agreement cannot be taken as a
consideration for the sale.

2B 16-17 SALES AND LEASE Page 166


10. MOISES JOCSON VS CA

GR No. L-55322 February 16, 1989

Medialdea, J

FACTS

Spouses Emilio Jocson & Alejandra Poblete had two children: Moises Jocson & Agustina Jocson-Vasquez.
Agustina is married to Ernesto Vasquez. Alejandra died intestate. On April 1, 1972, Emilio died intestate. On Ju n e
20, 1973, M o is es filed co mp lain t , as s ailin g t h e v alid it y o f 3 d o cu men t s executed by Emilio during his
lifetime. He prays that the following be declared null & void and that the properties involved be partitioned between
him & his sister: 1 . ) D e e d o f S a l e e xe c u t e d J u l y 2 7 , 1 9 6 8 w h e r e i n Emi l i o s o l d t o A g u s t i n a
6 p arcels o f lan d in Naic, Cav it e fo r P10,000.00. Deed in clu d ed Emilio s man ifes t at io n t h at t h e
lan d s were s o ld at a lo w p rice b ecau s e it was h is loving, helpful & thoughtful daughter who bought the
property. He says his son possesses such qualities too. He further claims that the sale did not v i o l a t e a n y
l a w & t h a t h e d i d n o t t o u c h h i s w i f e s p r o p e r t i e s . H e acknowledged receipt of payment. 2.)
Deed o f Sale execu t ed Ju ly 27, 1968, s ellin g 2 rice mills & a camalig in Naic, Cavite to Agustina for
P5,000.00. Emilio acknowledged receipt too. 3.) Deed o f Ext raju d icial Part it io n & A d ju d icat io n w/ Sale
execu t ed M arch
9,1 9 6 9 w h e r e i n Emi l i o & A g u s t i n a , e xc l u d i n g M o i s e s , e xt r a j u d i c i a l l y partitioned the
unsettled estate of Alejandra dividing such into 3. Emilio sold his share to Agustina.

All documents were executed before a notary public. Nos. 1 & 2 were
registeredw/ t h e Reg is t er o f Deed s . Old cert ificat es were can celled & n ew cert ificat es issued in the name
of Agustina.

Moises alleges that # 1 i s n u l l & v o i d b e c a u s e h i s f a t h e r s c o n s e n t w a s o b t a i n e d b y


f r a u d , d eceit , u n d u e p res s u re, in flu en ce & o t h er illeg al mach in at io n s . He als o alleges that property
was sold for a simulated price considering that his s is t er h ad n o wo rk o r liv elih o o d o f h er o wn . A ls o , h e
claims t h at t h e contract is fictitious, simulated & fabricated.

The RTC d ecid ed in fav o r o f p et it io n er s t at in g t h at the


d o cu men t s were s imu lat ed & fict it io u s because: 1) no proof that Agustina did pay for the properties, 2) prices
were grossly inadequate tantamount to lack of consideration at all, 3) improbability of sale considering the
circumstances.

The CA reversed RTCs decision stating that Nos. 1 & 2 were barred by prescription because of the
annulment of contract based on fraud must be filed 4 years from discovery of such w/c begins on the d at e o f t h e
reg is t rat io n w/ t h e Reg is t er o f Deed s . A ll d o cu men t s were act u ally & intended to be binding & effective
against Emilio. Proof of such includes issuance of new titles.

2B 16-17 SALES AND LEASE Page 167


ISSUE

Whether or not the sales were w/o consideration.

HELD

NO. Since Moises alleges such, it is incumbent upon him to prove his allegations, especially
since documents show that his father (vendor) acknowledged receipt of price & they are notarized. He failed to do
so and thus he was not able to overcome the presumption that a contract is with consideration (CC Art. 1354). Even
his own wit n es s co n t rad ict ed h is claim t h at h is s is t er & h er h u s b an d h ad n o s o u rce o f income. Witness
Bagnas said that Agustina & Ernesto were into buy and sell of palay and rice. Even he himself said that he didnt
know if his sister had other businesses. Agustina testified that she was into buy & sell even prior to her marriage.

2B 16-17 SALES AND LEASE Page 168


11. RAFAEL SUNTAY SUBSTITUTED BY HIS HEIRS VS CA
GR No. 114950 December 19, 1995

Hermosisima, J

FACTS

Respondent Federico Suntay was the registered owner of a parcel of land situated in Sto. Nio, Hagonoy,
Bulacan. On the land may be found: a rice mill, a warehouse, and other improvements. A rice miller, Federico, in a
letter, dated September 30, 1960, applied as a miller-contractor of the then National Rice and Corn Corporation
(NARIC). He informed the NARIC that he had a daily rice mill output of 400 cavans of palay and warehouse
storage capacity of 150,000 cavans of palay. His application, although prepared by his nephew-lawyer, petitioner
Rafael Suntay, was disapproved, obviously because at that time he was tied up with several unpaid loans. For
purposes of circumvention, he had thought of allowing Rafael to make the application for him. Rafael prepared an
absolute deed of sale whereby Federico, for and in consideration of P20,000.00 conveyed to Rafael said parcel of
land with all its existing structures. Said deed was notarized. Less than three months after this conveyance, a counter
sale was prepared and signed by Rafael who also caused its delivery to Federico. Through this counter conveyance,
the same parcel of land with all its existing structures was sold by Rafael back to Federico for the same
consideration of P20,000.00. Although on its face, this second deed appears to have been notarized, an examination
thereof will show that it is not the said deed of sale but a certain real estate mortgage on a parcel of land.

Upon the execution and registration of the first deed, Certificate of Title No. 0-2015 in the name of
Federico was cancelled and in lieu thereof, TCT No. T-36714 was issued in the name of Rafael. In a letter, Federico,
through his new counsel, Agrava & Agrava, requested that Rafael deliver his copy of TCT No. T -36714 so that
Federico could have the counter deed of sale in his favor registered in his name. Rafael denied the same. Hence,
Federico filed a complaint for reconveyance and damages against Rafael. Rafael insisted that said property was
"absolutely sold and conveyed . . . for a consideration of P20,000.00, Philippine currency, and for other valuable
consideration".

The trial court upheld the validity and genuineness of the deed of sale executed by Federico in favor of
Rafael, which deed is referred to above as Exhibit A, it ruled that the counter-deed, referred to as Exhibit B,
executed by Rafael in favor of Federico, was simulated and without consideration, hence, null and void ab initio.
The CA reversed the decision of the trial court in its December 15, 1993 decision stating that the first deed of sale
was a mere accommodation arrangement executed without any consideration and therefore simulated contract of
sale.

ISSUE

Whether or not the deed of sale executed by Federico in favor of Rafael was simulated and wit hout consideration.

HELD

Yes. The SC held that the deed of sale executed by Federico in favor of his now deceased nephew, Rafael,
is absolutely simulated and fictitious and, hence, null and void, said parties having entered into a sale tran saction to
which they did not intend to be legally bound. As no property was validly conveyed under the deed, the second deed
of sale executed by the late Rafael in favor of his uncle, should be considered ineffective and unavailing.

2B 16-17 SALES AND LEASE Page 169


The history and relationship of trust, interdependence and intimacy between the late Rafael and
Federico is an unmistakable token of simulation. It has been observed that fraud is generally accompanied by
trust. Hardly is it inconsistent with practical experience, especially in the context of the Filipino family's way of life,
that Federico, the uncle, would almost naively lend his land title to his nephew and agree to its cancellation in his
nephew's favor because Federico, in the first place, trusted his nephew; was well aware of his power over him as
uncle, client, and patron; and was actually in possession of the land and rice mill. No one could even conceive of the
possibility of ejecting Federico therefrom on the basis of the sham transaction. The late Rafael never attempted to
physically dispossess his uncle or actually take over the rice mill during his lifetime.

Also, the most protuberant index of simulation is the complete absence of an attempt in any manner on
the part of the late Rafael to assert his rights of ownership over the land and rice mill in question. After the sale, he
should have entered the land and occupied the premises thereof. He did not even attempt to. If he stood as owner, he
would have collected rentals from Federico for the use and occupation of the land and its improvements. All that the
late Rafael had was a title in his name.

The SC also said that neither does the undisputed fact that the deed of sale executed by
Federico in favor of the late Rafael, is a notarized document, justify the conclusion that said sale is undoubtedly a
true conveyance to which the parties thereto are irrevocably and undeniably bound.

2B 16-17 SALES AND LEASE Page 170


ON FORMS OF CONTRACTS OF SALE

1. MARCIANA CONLU, ET AL. vs. PABLO ARANETA and ESPIRIDION GUANKO,


G.R. No. L-4508, March 4, 1910

Johnson, J

FACTS:
Plaintiffs commenced an action against the defendants to recover, as owners, certain parcels of land located
in the pueblo of Molo, Province of Iloilo, plus damages. The defendants alleg e that they are the owners of the
parcels of land in question. The question thus presented by the complaint and answer was simply, Who are the
owners of said parcels of land?
The lower court found that: The plaintiffs were the owners and were entitled to the possession of all of
the parcels of land. Evidence establishes the fact that the house in question, with the tile roof, was originally the
property of Catalina Tiongco, sister of Anselma, which was afterwards left to Anselma by virtue of the will
made by Catalina before her death. That after that time, Anselmas nephew, Vito Tiongco, was appointed
gobernadorcillo of Molo, whose appointment was contested due to the fact that he was not the owner of any realty.
That Anselma, who then possessed many properties, put Vito Tiongco into possession of the said tile-roofed house
as apparently his own property. He lived in the house from that date up to the time of his death in 1904, and, as it
appeared to everybody, he considered it as if he was the real owner thereof. He made many repairs as well as
alterations in the house on his account. The lower court found that, after being put into possession of the house in the
manner above mentioned, Anselma agreed that he (Vito) could have the house as his own if he would pay to her
P3,000 (which sum is alleged to be the amount paid by her sister Catalina for the erection of said house) And that
afterwards, and before the death of Anselma, he had paid this sum to the satisfaction of Anselma, and, while no
formal conveyance of the property on the part of Anselma can be adduced, the lower court found that some time
subsequent to that date he claimed it as his property and it was recognized as his own. Therefore, the lower court
found that the house with tile roof was, at the time of the death of Anselma, really the property of Vito Tiongco, her
nephew.
From this decision of the lower court the plaintiffs appealed and made the following assignments of error in
his court: First. In allowing the defendants to prove by means of o ral evidence, the ownership of the said realty.
Second. In declaring that Anselma Tiongco sold the realty in question to Vito Tiongco.

ISSUE
May the sale of real property made in 1887 be proved by oral testimony?

HELD
The Supreme Court ruled in the affirmative.
An oral contract for the sale of real estate, made prior to the enactment of the Code of Civil Procedure, is
binding between the parties thereto, although it may still be necessary for the parties seeking to enforce such
contract to take some action to secure the execution of proper documents, but this requirement will not render the
agreement invalid.
Section 335 of the Code of Procedure in Civil Actions, now in force, has established a rule relating to the
method of proving contracts of sale of real property, and an oral contract for the sale of real property can not now be
proven under said section 335 except "some note or memorandum thereof be in writing and subscribed by the party
charged or by his agent." However, said section (335) makes no attempt to render such contracts [oral contracts]
invalid. It simply provides that the contract shall not be enforced by an action, unless the same is evidenced by some
note or memorandum.

2B 16-17 SALES AND LEASE Page 171


It does not attempt to make contracts invalid which have not been e xecuted in writing. This provision does
not go to the existence of the contract, except when made by an agent. The contract exists and is valid, though it may
not be clothed with the necessary form and the effect of a noncompliance with the provisions of t he statute is simply
that no action can be proved unless the requirement is complied with; but a failure to except to the evidence because
it does not conform with the statute is a waiver of the provisions of the law.
If the parties to the action, during the trial make no objection to the admissibility of oral evidence to
support a contract of sale of real property, and thus permit the contract to be proved, it will be just as binding upon
the parties as if it had been reduced to writing. In the present case the defendants called thirteen witnesses, who each
testified concerning the sale of the parcel of land and the house in question by Anselma Tiongco to Vito Tiongco, in
or about the year 1887, and no objection was made by the plaintiffs to the admissibility of this testimony. The
plaintiffs did not invoke the provisions of section 335. They permitted the defendants to prove the oral contract of
sale. The contract of sale, therefore, being fully proven, and under the provisions of the law an oral contract for
the sale of real property being binding and valid between the parties, we see no escape from the conclusion that if
the evidence was sufficient to show the sale, that the contract was binding, even though it had not been
reduced to writing.

2B 16-17 SALES AND LEASE Page 172


2. JULIO TAPEC and PRISCA GALANO vs. COURT OF APPEALS and LORETO RAGUIRAG

G.R. No. 111952 October 26, 1994

FACTS:

On 4 December 1994, the petitioners, who are husband and wife, filed a complaint for recovery of ownership with
the Regional Trial Court at Batac, Ilocos Norte, against David Cabuyadao and herein private respondent Loreto
Raguirag. The petitioners alleged in their complaint that they are the owners of a parcel of land located at Oaiag -
Upay, Paoay, Ilocos Norte. They further averred that during the cadastral survey of Paoay, Ilocos Norte, unknown to
them and without their consent, the property was surveyed and subdivided into Lot Nos. 7452, 7444, and 7450, and
that under baseless claims of ownership, David Cabuyadao and Loreto Raguirag threatened to enter Lot No. 7452
and Lot No. 7444, respectively. The petitioners then prayed that they be declared the owners of Lot Nos. 7452 and
7444. The petitioners' claim of ownership is based on two deeds of absolute sale, one executed on 2 January 1950 by
Trinidad Gonzales in favor of petitioner Julio Tapec, and the other executed on 28 May 1949 by Rosario Gonzales
in favor of the petitioners, both acknowledged before the same notary public and duly registered with the Office of
the Register of Deeds under Act No. 3344. Petitioner further alleged that he has been in possession of the lots since
he purchased them and had them declared for taxation purposes in his name in 1950 and that before he bought the
property of Trinidad Gonzales, he had to first redeem it from Ireneo Raguirag to whom it was mortgaged by
Trinidad for P100.00 on 10 November 1947 and who (Trinidad) was in possession thereof. Private respondent
(defendant) Loreto Raguirag denied having knowledge of the property claimed by the petition ers but asserted that he
is the absolute owner of the parcel of land. He anchored his defense on a document, dated 15 May 1931 and
handwritten in Ilocano, wherein the brothers Victoriano, Gregorio, Matias, and Alejandro, all surnamed Gonzales,
sold to the spouses Manuel Raguirag and Clara Tapec, grandparents of respondent Raguirag, for a consideration of
P150.00 a pasture situated in Dumalaoing, Paoay, Ilocos Norte. Miguel Gonzales was a witness to the said
handwritten document. Private respondent Raguirag presented the 1931 private writing which, according to him,
was shown to him when he was a boy by his grandfather, Manuel Raguirag, who said, "all of these are our properties
of which I bought from Alejandro Gonzales." He claimed that his grandfather was in possession of the property until
his death during the Japanese occupation. Then his father, Ireneo Raguirag, continued such possession until he died
in 1967. Ireneo had the property declared for taxation purposes in 1962. After his father's death, Loreto took over the
possession of the property and during the cadastral survey of Paoay, it was claimed by Leoncia Raguirag, a sister of
Ireneo. The private respondent is merely possessing it as tenant-administrator.

ISSUE

Whether the deeds of sale to them, which were embodied in public instruments and registered under Act No. 3344,
should prevail over the alleged sale to the ancestors of respondent Raguirag executed much earlier in a private
instrument

HELD:

The deed of sale in a private writing executed on 15 May 1931 in favor of Manuel Raguirag and Clara Tapec,
private respondent's grandparents, is an ancient document whose proof of authenticity was no longer necessary
because of the concurrence of the requisites in Section 21, 24 Rule 132 of the Rules of Co urt. It was already more
than thirty years old at the time it was offered in evidence in 1986. It was produced from the custody of respondent

2B 16-17 SALES AND LEASE Page 173


Raguirag, an heir of the vendees in the said instrument. And it is unblemished by any alteration or circumstances of
suspicion. The said private instrument is a deed of sale in which all the requisites of a valid contract are present and
which is binding upon the parties. Article 1358 does not invalidate the acts or contracts enumerated therein if they
are not embodied in public documents. The sale in favor of the private respondent's grandparents was valid and
enforceable.

The Court cannot, however, accept that when the land was sold to the petitioners in 1950, the vendor had no right to
sell the subject property since at the time her family no longer owned the land and thus no legal right was transferred
by the vendor to the plaintiffs -appellees.

Firstly, the sale in 1950 was that executed on 2 January 1950 by Trinidad Gonzales and the property subject thereof
corresponds to Lot Nos. 7450 and 7452, while the sale executed on 28 May 1949 by Rosario Gonzales corresponds
to Lot No. 7444. It is the latter lot which is claimed by the private respondent. The original owner of the property
sold by Trinidad and Rosario was their father, Miguel Gonzales, and as indicated in the deeds of sale they executed,
the portion each sold was declared for taxation purposes in the name of their father. With respect to the 1931 sale,
Miguel Gonzales was not a vendor therein but a mere witn ess thereto. The vendors were Victoriano, Matias,
Alejandro, and Gregorio, all surnamed Gonzales. The Court of Appeals erred in finding that Trinidad had no more
right to sell the property.

Secondly, while the petitioners sufficiently established the iden tity of the property claimed by them, the private
respondent failed to prove the identity of the property. Since he specified that the property he claimed and asked the
court in his prayer that he be declared "the lawful owner and possessor" thereof, the b urden was on him to prove its
identity.

Thirdly, it was established that Trinidad Gonzales had mortgaged her property to the private respondent's father,
Ireneo Raguirag, on 10 November 1947. The mortgage was redeemed only shortly before its sale to the petitioners
in 1950. If Ireneo were its owner as heir of Manuel Raguirag, there was no reason for Ireneo to have accepted the
mortgage thereof.

Finally, the private respondent categorically admitted that he is only a tenant -administrator of Lot No. 7444. This
admission belies any claim of ownership. It was his aunt, Leoncia Raguirag, who claimed ownership over it during
the cadastral survey.

2B 16-17 SALES AND LEASE Page 174


3. LEONCIA T. ZAIDE and PRIMITIVO ZAIDE, substituted by SIMEON TOLENTINO, Guardian ad
litem of the Minors PACITA, ALEX, MARIA ZERLINA all surnamed ZAIDE, etc., vs. CA, ROBERTO DE
LEON and EDITA T. ZAIDE

G.R. No. L-46715-16 July 29, 1988

Narvasa, J

FACTS

Edita Zaide and her husband, Roberto de Leon, were the registered owners of a parcel of land situated in
Makati, Rizal, with an area of 201 square meters, covered by TCT No. 69088 of the Register of Deeds of Rizal.

Primitivo Zaide , Edita's brother, gave to Edita and her husband, Roberto de Leon, P2,000.00 as a loan, which the
latter used to redeem the land mortgaged by them to the Pasay Rural Bank. In addition, Primitive Zaide and his wife,
Leoncia, transferred ownership of a jitney owned by them, valued at P7,000.00, to Roberto de Leon.
Zaide Spouses claim that the vehicle was thus ceded as part of the purchase price of the de Leons' above described
land, which they had agreed to buy. In any case, neither the loan nor the transfer of the vehicle is disputed.
On January 11, 1965, Edita Zaide executed a public instrument denominated "Deed of Sale" by which, in
consideration of P5,000.00 paid to her, she sold the parcel of land covered by TCT No. 69088 to Leoncia T. Zaide.
The deed described both the vendor, Edita Zaide, and the vendee, Leoncia T. Zaide, as "married," but named neither
of their husbands. The document however did bear the signature of Edita's husba nd, Roberto de Leon, indicating his
"marital consent."The omission of the name of the vendee's husband in the deed of sale gave rise to a problem.
Precisely because of it, the Register of Deeds refused to accept it for registration. A second deed of sale couched
in the same terms as the first, acknowledge before the same Notary Public,but now the names of husbands are
presented and thus, it was promptly accepted for registration, by the Register of Deeds. The latter then cancelled
TCT No. 69088, and issued a new one, TCT No. 138606, in the name of "Leoncia T. Zaide, married to Primitivo
Zaide.. With this lot as collateral, the Zaide Spouses thereafter obtained a loan from the Government Service
Insurance System in the sum of P28,500.00. This was sometime in November, 1964. The proceeds were used to
construct a two-story apartment building on the land. On June 1, 1969, the house of the de Leons burned down.
They moved to one of the doors of the apartment built by the Zaide Spouses. They were asked to pay re ntals. They
refused. Litigation ensued.De Leon spouses filed a complaint against Zaide Spouses alleging that the second deed of
sale was forged and that they "could not possibly have sold their lot for the measly sum of P5,000.00 appearing in
the forged deed ..considering that the market price of the land ... cannot be less than P20,000.00."
CFI favored Zaide Spouses . (2nd deed of sale was valid). CA reversed the CFIs decision. Hence, this petition.

ISSUE

Whether or not first deed of sale is VALID even if defective in form

HELD

YES.

Under the circumstances, the genuineness and due execution of Exhibit 1(1 st deed of Sale) , which had been
formally offered and admitted by the Court, cannot but be conceded, not merely on the strength of the unrebutted

2B 16-17 SALES AND LEASE Page 175


presumptions of regularity of private transactions, but also and particularly, the admissions by the de Leons just
detailed.

However, although the first deed of sale (Exh. 1) was genuine, it was so far defective as to render it
unregistrable in the Registry of Property. As already pointed out, it did not set forth the name of the vendee's
husband and was for this reason refused registration by the Register of Deeds. The defect was unsubstantial. It did
not invalidate the deed. The legal dispositions are clear. Though defective in form, the sale was valid; and the
parties could compel each other to do what was needful to make the document of sale registrable.

The law generally allows a contract of sale to be entered into in any form, whether "in writing, or by word
of mouth, or partly in writing and partly by word or mouth, or (even) inferred from the conduct of the parties;" but if
the agreement concerns "the sale of land or of an interest therein," the law requires not only that "the same, or some
note or memorandum thereof, be in writing, and subscribed by the party charged" in order that it may be enforceable
by action, but also that the writing be in the form of a "public document." The law finally provides that "If the law
requires a document or other special form, as in the acts and contracts enumerated in .. (Article 1358), the
contracting parties may compel each other to observe that form, once the contract has been perfected .. (and such)
right may be exercised simultaneously with the action upon the contract."

In the case at bar, the Zaides thus had the right to compel the de Leons to observe the special form
prescribed by law; i.e., revised the public document by inserting the name of the vendee's husband. Indeed, this was
precisely what was done in the second deed of sale, Exhibit 2.

Hence, even if the second document of sale be invalidated as a forgery, and the de Leons' title to the land
restored to them, this would be inutile, an empty ceremony, since the de Leons could nevertheless still be compelled
by the Zaides to execute another deed, in proper form, to carry into effect the sale originally entered into.

2B 16-17 SALES AND LEASE Page 176


4. LIMKETKAI SONS MILLING, INC. vs. COURT OF APPEALS, ET AL.,

G.R. No. 118509 December 1, 1995

Melo, J

FACTS

Philippine Remnants was the owner of a piece of land which it then entrusted to BPI. Pedro Revilla was authorized
by BPI to sell the lot for P1000.00 per square meter. Revilla contacted Alfonso Lim who agreed to buy the land.
Alfonso Lim and Albino Limketkai went to BPI and were entertained by VP Albano and Asst. VP Aromin. BPI set
the price at P1,100 while Limketkai haggled to P900. They subsequently agreed on P1,000 per squ are meter on cash
basis. Alfonso Lim asked if it was possible to pay on terms and BPI officials said there was no harm in trying to ask
for payment in terms but if disapproved, the price would have to be paid in cash. Limketkai paid the initial 10% with
the remaining 90% to follow. Two or three days later, Alfonso Lim found out that their offer had been frozen and
then went to BPI to tender full payment of 33 million pesos to Albano but was refused by Albano.

ISSUE

Whether or not there was a perfected contract between petitioner Limketkai Sons Milling, Inc. and respondent Bank
of the Philippine Islands (BPI) covering the sale of a parcel of land.

HELD

Yes, there was a perfected contract of sale.

The contract of sale is perfected at the moment there is a meeting of minds upon the thing which is the object of the
contract and upon the price. From that moment, the parties may reciprocally demand performance, subject to the
provisions of the law governing the form of contracts. (Art. 1475, Civil Code)

Consent is manifested by the meeting of the offer and the acceptance upon the thing and the cause which are to
constitute the contract. The offer must be certain and the acceptance absolute. A qualified acceptance constitutes a
counter-offer (Art. 1319, Civil Code). An acceptance may be express or implied. (Art. 1320, Civil Code).

The perfection of the contract took place when Aromin and Albano, acting for BPI, agreed to sell and Alfonso Lim
with Albino Limketkai, acting for petitioner Limketkai, agreed to buy the disputed lot at P1,000.00 per square meter.
Aside from this there was the earlier agreement between petitioner and the authorized broker. There was a
concurrence of offer and acceptance, on the object, and on the cause thereof.

There was already a perfected contract of sale because both parties already agreed to the sale of P1,000.00 per
square meter. Even if Lim tried to negotiate for a payment in terms, it is clear that if it be disapproved, the payment
will be made in cash.

The phases that a contract goes through may be summarized as follows:

a. preparation, conception or generation, which is the period of negotiation and bargaining, ending at the moment of
agreement of the parties;

2B 16-17 SALES AND LEASE Page 177


b. perfection or birth of the contract, which is the moment when the part ies come to agree on the terms of the
contract; and

c. consummation or death, which is the fulfillment or performance of the terms agreed upon in the contract

2B 16-17 SALES AND LEASE Page 178


5. SWEDISH MATCH, AB, ET AL. vs. COURT OF APPEALS, ALS MANAGEMENT &
DEVELOPMENT CORPORATION AND ANTONIO K. LITONJUA

G.R. No. 128120, October 20, 2004

Tinga, J

FACTS:

STORA, the then parent company of Swedish Match AB (hereinafter SMAB) decided to sell SMAB of
Sweden and the latters worldwide match, lighter and shaving pro ducts operation to Eemland Management Services,
now known as Swedish Match NV of Netherlands, (SMNV). Ed Enriquez (Enriquez), Vice -President of Swedish
Match Sociedad Anonimas (SMSA)the management company of the Swedish Match group came to the
Philippines in November 1989 and informed the Philippine financial and business circles that its shares in Phimco
Indusctries, Inc. were for sale as part of its strategy to sell worldwide match and lighter businesses.

Respondent ALS Management & Development Corporation represented by its President Litonjua submitted
to SMAB a firm offer to buy all of the latters shares in Phimco and all of Phimcos shares in Provident Tree Farm,
Inc. and OTT/Louie (Phils.), Inc. for the sum of P750, 000,000.00.

Through its Chief Executive Officer, Massimo Rossi (Rossi), SMAB informed respondents that their offer
was below their expectations. Litonjua then offered to buy the disputed shares, which per another letter of the same
date was increased to US$36 million. Responding to Litonju as offer, Rossi informed the former that ALS should
undertake a due diligence process or pre-acquisition audit and review of the draft contract.

Since it took Respondent too long to submit the final offer, Enriquez sent notice to Litonjua that they would
be constrained to entertain bids from other parties. Rossi informed Litonjua tthat he signed a conditional contract
with a local group for the disposal of Phimco. Litonjua promptly responded asserting that the US$36 million bid
which he submitted was their final bid. Rossi informed Ltionjua that it entered into a conditional contract with a
local group, but when that did not materialize, Enrique invited Litonjua to resume negotiation, setting up new terms
and conditions. Litonjua rejected to the new terms and conditions.

Respondents, as plaintiffs filed a case for specific performance against defendants, herein petitioners.The
RTC dismissed respondents complaint. Upon appeal, the CA reversed the decision.

Now, petitioners contend that that the Court of Ap peals erred in failing to consider that the Statute of
Frauds requires not just the existence of any note or memorandum but that such note or memorandum should
evidence an agreement to sell and respondents contend that: the Court of Appeals correctly ruled that the Statute of
Frauds does not apply to the instant case and assert that the sale of the subject shares to them was perfected as
shown by the following circumstances.

ISSUE: WON there was a perfected contract of sale between petitioners and respondents with respect to the Phimco
shares.

HELD: NO.

2B 16-17 SALES AND LEASE Page 179


The Statute of Frauds embodied in Article 1403, paragraph (2), of the Civil Code requires certain contracts
enumerated therein to be evidenced by some note or memorandum in order to be enforceable. By law, contracts are
obligatory in whatever form they may have been entered into, provided all the essential requisites for their
validity are present. However, when the law requires that a contract be in some form in order that it may be
valid or enforceable, or that a contract be proved in a certain way, that requirement is absolute and
indispensable. Consequently, the effect of non-compliance with the requirement of the Statute is simply that no
action can be enforced unless the requirement is complied with. For a note or memorandum to satisfy the Statute, it
must be complete in itself and cannot rest partly in writing and partly in parol. The note or memorandum must
contain the names of the parties, the terms and conditions of the contract, and a description o f the property sufficient
to render it capable of identification.Such note or memorandum must contain the essential elements of the contract
expressed with certainty that may be ascertained from the note or memorandum itself, or some other writing to
which it refers or within which it is connected, without resorting to parol evidence

Contrary to the Court of Appeals conclusion, the exchange of correspondence between the parties hardly
constitutes the note or memorandum within the context of Article 1403 of the Civil Code. Rossis letter dated 11
June 1990, heavily relied upon by respondents, is not complete in itself. First, it does not indicate at what price the
shares were being sold. Second, the letter does not state the mode of payment of the price.

In a contract of sale, required is the concurrence of three elements, to wit: (a) consent or meeting of the
minds, that is, consent to transfer ownership in exchange for the price; (b) determinate subject matter, and (c) price
certain in money or its equivalent.35 Such contract is born from the moment there is a meeting of minds upon the
thing which is the object of the contract and upon the price. In general, contracts undergo three distinct stages, to
wit: negotiation; perfection or birth; and consummation. A negotiation is formally initiated by an offer.
Quite obviously, Litonjuas letter dated 21 May 1990, proposing the acquisition of the Phimco shares for
US$36 million was merely an offer. The lack of a definite offer on the part of respondents could no t possibly serve
as the basis of their claim that the sale of the Phimco shares in their favor was perfected, for one essential element of
a contract of sale was obviously wantingthe price certain in money or its equivalent. The price must be certain,
otherwise there is no true consent between the parties.

There can be no sale without a price. Respondents attempt to prove the alleged verbal acceptance of their
US$36 million bid becomes futile in the face of the overwhelming evidence on record that there was in the first
place no meeting of the minds with respect to the price. It is dramatically clear that the US$36 million was not the
actual price agreed upon but merely a preliminary offer which was subject to adjustment after the conclusion of the
audit of the company finances. Respondents failure to submit their final bid on the deadline set by petitioners
prevented the perfection of the contract of sale. It was not perfected due to the absence of one essential element
which was the price certain in money or its equivalent.

2B 16-17 SALES AND LEASE Page 180


6. UNIVERS ITY OF THE PHILIPPINES VS PHILAB INDUSTRIES, INC.

G.R. No. 152411, September 29, 2004

Callejo Sr, J

FACTS

In 1979, the UP decided to construct an integrated system of research organization known as the Research Complex.
As part of the project, laboratory equipment and furniture were purchased for the National Institute of
Biotechnology and Applied Microbiology (BIOTECH) at the UP Los Baos. Providentially, the Ferdinand E.
Marcos Foundation (FEMF) came forward and agreed to fund the acquisition of the laboratory furniture, including
the fabrication thereof.

Renato E. Lirio, the Executive Assistant of the FEMF, gave the go -signal to BIOTECH to contact a corporation to
accomplish the project.

July 23, 1982 - Dr. William Padolina, the Executive Deputy Director of BIOTECH, arranged for Philippine
Laboratory Industries, Inc. (PHILAB), to fabricate the laboratory furniture and deliver the same to BIOTECH for the
BIOTECH Building Project, for the account of the FEMF. Lirio directed Padolina to give the go-signal to PHILAB
to proceed with the fabrication of the laboratory furniture, and requested Padolina to forward the contract of the
project to FEMF for its approval.

July 13, 1982 - Padolina wrote Lirio and requested for the issuance of the purchase order and downpayment for the
office and laboratory furniture for the project. Padolina assured Lirio that the contract would be prepared as soon as
possible before the issuance of the purchase orders and the downpayment for t he goods, and would be transmitted to
the FEMF as soon as possible.

July 23, 1982 - In a Letter, Padolina informed Hector Navasero, the President of PHILAB, to proceed with the
fabrication of the laboratory furniture and also requested for copies of the shop drawings and a sample contract for
the project, and that such contract and drawings had to be finalized before the down payment could be remitted to
the PHILAB the following week. However, PHILAB failed to forward any sample contract. PHILAB made part ial
deliveries of office and laboratory furniture to BIOTECH after having been duly inspected by their representatives
and FEMF Executive Assistant Lirio.

August 24, 1982 - FEMF remitted P600,000 to PHILAB as downpayment for the laboratory furniture for the
BIOTECH project and thereafter made another partial payment of P800,000 on October 22, 1982. The remittances
were in the form of checks drawn by FEMF and delivered to PHILAB, through Padolina.

2B 16-17 SALES AND LEASE Page 181


October 16, 1982 - UP, through Emil Q. Javier, the Chancellor of UP Los Baos and FEMF, represented by its
Executive Officer, Rolando Gapud, executed a Memorandum of Agreement (MOA) in which FEMF agreed to grant
financial support and donate sums of money to UP for the construction of buildings, installation of laboratory and
other capitalization for the project, not to exceed P29,000,000.00.

The MOA, additionally states that:

(1) The foundation shall acquire and donate to the UNIVERSITY the site for the RESEARCH COMPLEX,

(2) Donate or cause to be donated to the UNIVERSITY the sum of P29,000,000.00, and

(3) Shall continue to support the activities of the RESEARCH COMPLEX.

In the meantime, Navasero promised to submit the contract for the installation of laboratory furniture to BIOTECH,
by January 12, 1983. However, Navasero failed to do so. In a Letter BIOTECH reminded Navasero of the need to
submit the contract so that it could be submitted to FEMF for its evaluation and approval. Instead of submitting the
said contract, PHILAB submitted to BIOTECH an accomplishment report on the project as of February 28, 1983,
and requested payment thereon. By May 1983, PHILAB had completed 78% of the project. The FEMF had already
paid forty percent (40%) of the total cost of the project.

May 12, 1983 - Padolina wrote Lirio and furnished him the progress billing from PHILAB. On August 11, 1983, the
FEMF made another partial payment representing the already delivered laboratory and office furniture after the
requisite inspection and verification thereof by representatives from the BIOTECH, FEMF, and PHILAB. The
payment was made in the form of a check, for which PHILAB issued Official Receipt No. 202 to FEMF through
Padolina.

July 1, 1984 - PHILAB submitted to BIOTECH Invoice No. 01643 in the amount of P702,939.40 for the final
payment of laboratory furniture. Representatives from BIOTECH, PHILAB, and Lirio for the FEMF, conducted a
verification of the accomplishment of the work and confirmed the same.

December 18, 1984 - BIOTECH forwarded the invoice to Lirio for its payment. Lirio, in turn, forwarded the invoice
to Gapud, presumably sometime in the early part of 1985. However, the FEMF failed to pay the bill. PHILAB
reiterated its request for payment through a letter on May 9, 1985.

March 21, 1985 - BIOTECH again wrote Lirio, requesting the payment of PHILABs bill. It sent another letter to
Gapud, on November 22, 1985, again appealing for the payment of PHILABs bill.

December 5, 1985 - In a Letter to BIOTECH, PHILAB requested payment of P702,939.40 plus interest thereon of
P224,940.61.There was, however, no response from the FEMF.

February 24, 1986 - PHILAB wrote BIOTECH, appealing for the payment of its bill even on installment basis.

2B 16-17 SALES AND LEASE Page 182


President Marcos was ousted from office during the February 1986 EDSA Revolution.

March 26, 1986 - Navasero wrote BIOTECH requesting for its much-needed assistance for the payment of the
balance already due plus interest of P295,234.55 for its fabrication and supply of laboratory furniture.

April 22, 1986 - PHILAB wrote President Corazon C. Aquino asking her help to secure the payment of the amount
due from the FEMF.19 The letter was referred to then Budget Minister Alberto Romulo, who referred the letter to
then UP President Edgardo Angara on June 9, 1986.

September 30, 1986 - Raul P. de Guzman, the Chancellor of UP Los Baos, wrote then Chairman of the
Presidential Commission on Good Government (PCGG) Jovito Salonga, submitting PHILABs claim to be officially
entered as "accounts payable" as soon as the assets of FEMF were liquidated b y the PCGG.

In the meantime, the PCGG wrote UP requesting for a copy of the relevant contract and the MOA for its perusal.

Chancellor De Guzman wrote Navasero requesting for a copy of the contract executed between PHILAB and FEMF.

October 20, 1987 - In a Letter, Navasero informed De Guzman that PHILAB and FEMF did not execute any
contract regarding the fabrication and delivery of laboratory furniture to BIOTECH.

Exasperated, PHILAB filed a complaint for sum of money and damages against UP. In its answer, UP denied
liability and alleged that PHILAB had no cause of action against it because it was merely the donee/beneficiary of
the laboratory furniture in the BIOTECH; and that the FEMF, which funded the project, was liable to the PHILAB
for the purchase price of the laboratory furniture. UP specifically denied obliging itself to pay for the laboratory
furniture supplied by PHILAB.

Decision of the RTC: Dismissed the complaint without prejudice to PHILABs recourse against the FEMF.

Appeal with the CA: The CA reversed and set aside the decision of the RTC and held that there was never a
contract between FEMF and PHILAB. Consequently, PHILAB could not be bound by the MOA between the FEMF
and UP since it was never a party thereto. The appellate court ruled that, although UP did not bind itself to pay for
the laboratory furniture; nevertheless, it is liable to PHILAB under the maxim: "No one should unjustly enrich
himself at the expense of another.

Hence, this petition.

2B 16-17 SALES AND LEASE Page 183


Argument of Petitioner: The CA overlooked the evidentiary effect and substance of the corresponding letters and
communications which support the statements of the witnesses showing affirmatively that an implied contract of
sale existed between PHILAB and the FEMF. Furthermore, there was no contract that existed between it and the
respondent as it could not have entered into any agreement without the requisite public bidding and a formal written
contract.

Argument of the Respondent: The CA did not err in not applying the law on contracts b etween the respondent and
the FEMF. It, likewise, attests that it was never privy to the MOA entered into between the petitioner and the FEMF.
The respondent adds that what the FEMF donated was a sum of money equivalent toP29,000,000, and not the
laboratory equipment supplied by it to the petitioner. The respondent submits that the petitioner, being the recipient
of the laboratory furniture, should not enrich itself at the expense of the respondent.

ISSUES

(1) Whether or not the Court of Appeals erred when it failed to apply the law on contracts between PHILAB
and the Marcos Foundation.
(2) Whether or not the Court of Appeals erred in applying the legal principle of unjust enrichment when it held
that the university, and not the Marcos Foundation, is liable to PHILAB.

HELD

The petition is meritorious.

It bears stressing that the respondents cause of action is one for sum of money predicated on the alleged promise of
the petitioner to pay for the purchase price of the furniture, which, despite demands, the petitioner failed to do.
However, the respondent failed to prove that the petitioner ever obliged itself to pay for the laboratory furniture
supplied by it. Hence, the respondent is not entitled to its claim against the petitioner.

PATIES to a CONTRACT

There is no dispute that the respondent is not privy to the MOA executed by the petitioner and FEMF; hence, it is
not bound by the said agreement. Contracts take effect only between the parties and their assigns. A contract cannot
be binding upon and cannot be enforced against one who is not a party to it, even if he is aware of such contract and
has acted with knowledge thereof. Likewise admitted by the parties, is the fact that there was no written contract
executed by the petitioner, the respondent and FEMF relating to the fabrication and delivery of office and laboratory
furniture to the BIOTECH. Even the CA failed to specifically declare that the petitioner and the respondent entered
into a contract of sale over the said laboratory furniture. The parties are in accord that the FEMF had remitted to the
respondent partial payments via checks drawn and issued by the FEMF to the respondent, through Padolina, in the
total amount of P2,288,573.74 out of the total cost of the project ofP2,934,068.90 and that the respo ndent received
the said checks and issued receipts therefor to the FEMF. There is also no controversy that the petitioner did not pay
a single centavo for the said furniture delivered by the respondent that the petitioner had been using ever since.

2B 16-17 SALES AND LEASE Page 184


IMPLIED-IN-FACT CONTACT OF SALE

The Court agreed with the petitioner that, based on the records, an implied-in-fact contract of sale was entered into
between the respondent and FEMF. A contract implied in fact is one implied from facts and circumstances showing
a mutual intention to contract. It arises where the intention of the parties is not expressed, but an agreement in fact
creating an obligation. It is a contract, the existence and terms of which are manifested by conduct and not by direct
or explicit words between parties but is to be deduced from conduct of the parties, language used, or things done by
them, or other pertinent circumstances attending the transaction. To create contracts implied in fact, circumstances
must warrant inference that one expected compensation and the other to pay. An implied -in-fact contract requires
the parties intent to enter into a contract it is a true contract. The conduct of the parties is to be viewed as a
reasonable man would view it, to determine the existence or not of an implied-in-fact contract. The totality of the
acts/conducts of the parties must be considered to determine their intention. An implied -in-fact contract will not
arise unless the meeting of minds is indicated by some intelligent conduct, act or sign.

In this case, the respondent was aware, from the time Padolina contacted it for the fabrication and supply of the
laboratory furniture until the go-signal was given to it to fabricate and deliver the furniture to BIOTECH as
beneficiary, that the FEMF was to pay for the same. Indeed, Padolina asked the respondent to prepare the draft of
the contract to be received by the FEMF prior to the execution of the parties (the respondent and FEMF), but
somehow, the respondent failed to prepare one. The respondent knew that the petitioner was merely the donee-
beneficiary of the laboratory furniture and not the buyer; nor was it liable for the payment of the purchase price
thereof. From the inception, the FEMF paid for the bills and statement of accounts of the responde nt, for which the
latter unconditionally issued receipts to and under the name of the FEMF.

The respondent, in its Letter dated March 26, 1986, informed the petitioner and sought its assistance for the
collection of the amount due from the FEMF. The respon dent even wrote former President Aquino seeking her
assistance for the payment of the amount due, in which the respondent admitted it tried to collect from her
predecessor, namely, the former President Ferdinand E. Marcos.

Admittedly, the respondent sent to the petitioner its bills and statements of accounts for the payments of the
laboratory furniture it delivered to the petitioner which the petitioner, through Padolina, transmitted to the FEMF for
its payment. However, the FEMF failed to pay the last statement of account of the respondent because of the onset
of the EDSA upheaval. It was only when the respondent lost all hope of collecting its claim from the government
and/or the PCGG did it file the complaint against the petitioner for the collection of t he payment of its last delivery
of laboratory furniture.

2B 16-17 SALES AND LEASE Page 185


DELIVERY

1. ABUAN VS GARCIA

G.R. No. L-20091, July 30, 1965

FACTS

On August 7, 1953, petitioners Perpetua Abuan et al. sold a parcel of rice land to defendants Eustaquio Garcia et al.
through a deed of Absolute Sale. A TCT was issued to defendants.
Later, petitioners filed an action to recover the land, alleging the sale was tainted with fraud and was without
consideration. Reaching an amicable settlement, the parties entered into an Agre ement dated Feb 38, 1995, under
which defendants paid P500 as partial payment of the purchase price of the land, and promised to pay the balance of
P1,500 on or before April 30, 1955, with a grace period of 30 days. The Agreement also stated that it shall
supersede all previous agreements or contracts herefore entered into
Palintiffs instituted the present action on March 4,1960. Defendants moved to dismiss, on the ground that plaintiffs
right of action was already barred, because the five-year redemption period had already expired. Section 119 of the
Public Land provides:
Every conveyance of land acquired under the free patent or homestead provisions, when proper, shall be
subject to re-purchase by the applicant, his widow, or legal heirs, for a period of five years from the date of
conveyance.
Plaintiff argues that the period should be counted from the date of full payment (May 1965) since it was on this date
that the contract was consummated.
CFI Nueva Vizcaya dismissed the complaint, fixing the starting date as February 28,1995, when the Agreement was
entered into. CA certified the case to SC.

ISSUE

Is the contention of the plaintiff where the five-year redemption period should be counted from the date of full
payment tenable?

HELD

No, plaintiffs argument is untenable, thus dismissal by CA is affirmed.

Conveyance means transfer of ownership. It means the date when the title to the land is transferred from
one person to another. The five-year period should, therefore, reckoned with from the date that defendants acquired
ownership upon execution of absolute sale on August 7,1953.

Furthermore, under Art 1498, NCC, when the sale is made through a public instrument, as in this case, the
execution thereof shall be equivalent to the delivery of the thing which is the object of the contract, if from the deed
the contrary does not appear or cannot be clearly inferred. This manner of delivery is common to personal as well as
real property. It is clear, therefore that defendants acquired ownership t o the land in question upon the execution of
the Deed of Absolute sale on August 7, 1953. The Agreement on Feb 28,1995, only superseded the deed as to the
terms and conditions of payment. The Agreement did not operate to revest the ownership of the land to the plaintiffs.

Assuming arguendo that the Deed is null and void as petitioner alleges, we can consider the date of the
Agreement at the latest, as the time with which ownership is vested in the defendants. While it is a private

2B 16-17 SALES AND LEASE Page 186


instrument, the execution of which could not be construed as constructive delivery under Art 1498, Art 1496
explicitly provides that ownership of the thing sold is acquired by the vendee form the moment it is delivered to him
in any other manner signifying an agreement that the possession is transferred from the vendor to the vendee. The
intention to give possession (and ownership) is manifest in the Agreement, especially considering the following
circumstances: (1) the payment of part of the purchase price, there being no stip ulation in the Agreement that
ownership will not vest in the vendees until full payment of the price; and (2) the fact that the Agreement was
entered into consideration of plaintiffs desistance, as in fact they did desist, in prosecuting their reivindicat ory
action, thereby leaving the property in the hands of the then and now defendants - as owners thereof, necessarily. In
the absence of an express stipulation to the contract, the payment of the price is not a condition to the transfer of
ownership, which passes by delivery of the thing to the buyer.

2B 16-17 SALES AND LEASE Page 187


2. FILINVEST CREDIT CORPORATION v. PHILIPPINE ACETYLENE, CO., INC.
G.R. No. L-50449 January 30, 1982

De Castro, J.

FACTS:
Philippine Acetylene, Co. (Phil Acetylene) purchased from certain Alexander Lim a Chevorlet car, 1969 model for
P55,247.80 with a down payment of P20,000.00 and the balance of P35,247.80 payable by installment with 12%
interest, as evidenced by a promissory note.

Phil Acetylene executed a chattel mortgage over the car as a security for the said promissory note. Such mortgage
was in favor of Lim. Subsequently, Lim assigned to Filinvest Finance Corporation (FFC) all his rights, title, and
interests in the promissory note and chattel mortgage by virtue of a Deed of Assignment

FFC has a merger with Credit and Development Corporation which created the new corporation, Filinvest Credit
Corporation (FCC), the plaintiff. FFC assigned all its rights, title, and interests on the aforesaid promissory note and
chattel mortgage to FCC which, in effect, the payment of the unpaid balance owed by Phil Acetylene to Alexander
Lim was financed by FCC. As a consequence Lim, in effect, has been fully paid despite the balance of Phil
Acetylene.

When Phil Acetylene defaulted in the payment of 9 successive installments, FCC sent a demand letter asking the
former to pay its balance and return the mortgaged car. In reply, Phil Acetylene returned the mortgaged car together
with the document "Voluntary Surrender with Special Power of Attorney To Sell"

FCC wrote a letter to Phil Acetylene stating that it cannot sell the car due to outstanding taxes. Also, FCC offered to
deliver back the car to Phil Acetylene but the latter refused to accept it, so the former instituted an action for
collection of a sum of money with damages in the CFI of Manila.

In its answer, Phil Acetylene, admitted the material allegations of the complaint but avers that FCC has no cause of
action against it since its obligation towards the appellee was extinguished when it returned the mortgaged car to
FCC

ISSUE

Whether or not there is a valid dacion en pago that would extinguish the obligation of Phil Acetylene?

HELD

NO. The mere return of the mortgaged car by the mortgagor does not constitute dacion en pago in the absence ,
express or implied of the true intention of the parties.

Dacion en pago is the transmission of the ownership of a thing by the debtor to the creditor as an accepted
equivalent of the performance of obligation. In dacion, the debtor offers another thing to the creditor who accepts it
as equivalent of payment of an outstanding debt. The essential elements of a contract of sale must be present,
consent, object certain, and cause or consideration.

In any case, common consent is an essential prerequisite, to have the effect of totally extinguishing the debt or

2B 16-17 SALES AND LEASE Page 188


obligation. The evidence on the record fails to show that the Filinvest consented, or at least intended, that the mere
delivery to, and acceptance by him, of the mortgaged car be construed as actual pa yment, more specifically dacion
en pago.

The fact that the mortgaged motor vehicle was delivered to FCC does not necessarily mean that ownership
transferred from Phil Acetylene to FCC. In the absence of clear consent of FCC to the proffered special mode of
payment, there can be no transfer of ownership of the mortgaged car.

As to the strength of the Voluntary Surrender with Special Power of Attorney To Sell, it only authorized FCC to
look for a buyer and sell the vehicle in behalf of the Phil Acetylen e who retains ownership thereof, and to apply the
proceeds of the sale to the mortgage indebtedness. FCC, in essence was constituted as a mere agent to sell the motor
vehicle which was delivered not as its property. If it were, he would have full power of disposition of the property,
not only to sell it.

2B 16-17 SALES AND LEASE Page 189


3. PERFECTO DY, JR. vs.COURT OF APPEALS, GELAC TRADING INC., and ANTONIO V.
GONZALES

G.R. No. 92989, July 8, 1991

FACTS:

Wilfredo Dy purchased a truck and a farm tractor through finan cing extended by Libra Finance and
Investment Corporation (Libra). Both truck and tractor were mortgaged to Libra as security for the loan.

Petitioner Perfecto Dy wrote a letter to Libra requesting that he be allowed to purchase from Wilfredo Dy
the said tractor and assume the mortgage debt of the latter.Libra thru its manager, Cipriano Ares approved the
petitioner's request. Thus, Wilfredo Dy executed a deed of absolute sale in favor of the petitioner over the tractor. At
this time, the subject tractor was in the possession of Libra Finance due to Wilfredo Dy's failure to pay the
amortizations. Despite the offer of full payment by the petitioner to Libra for the tractor, the immediate release could
not be effected because Wilfredo Dy had obtained financing not only for said tractor but also for a truck and Libra
insisted on full payment for both.A PNB check was issued by petitioners sister in favor of Libra, settling in full the
indebtedness of Wilfredo Dy with Libra. Payment having been effected through an out-of-town check, Libra insisted
that it be cleared first before Libra could release the chattels in question.

Meanwhile a collection case against Wilfredo Dy to recover the sum of P12,269.80 was pending in another
court in Cebu. The provincial sheriff was able to seize and levy on the tractor which was in the premises of Libra in
Carmen, Cebu. The tractor was subsequently sold at public auction where Gelac Trading was the lone bidder. Gelac
sold the tractor to one of its stockholders, Antonio Gonzales. Consequently, the petitioner filed an action to recover
the subject tractor against GELAC Trading with the Regional Trial Court of Cebu City.

The RTC rendered judgment in favor of the petitioner declaring him to be the owner of the tractor. The
Court of Appeals reversed the decision of the RTC holding that the tractor in question still belonged to Wilfredo Dy
when it was seized and levied by the sheriff by virtue of the alias writ of execution issued in the Civil Case.

ISSUE

Whether or not ownership of the farm tractor had already passed to herein petitioner when said tractor was levied on
by the sheriff pursuant to an alias writ of execution issued in another case in favor of respondent gelac trading inc.

HELD

YES.

The chattel mortgagor continues to be the owner of the property, and therefore, has the power to alienate
the same; however, he is obliged under pain of penal liability, to secure the written consent of the mortgagee.And
even if no consent was obtained from the mortgagee, the validity of the sale would still not be affected.

There is no reason why Wilfredo Dy, as the chattel mortgagor can not sell the subject tractor. The consent
of Libra Finance was obtained when it allowed the petitioner to purchase the tractor and assume the mortgage debt

2B 16-17 SALES AND LEASE Page 190


of his brother. The sale between the brothers was therefore valid and binding as between them and to the mortgagee,
as well.

Article 1496 of the Civil Code states that the ownership of the thing sold is acquired by the vendee
from the moment it is delivered to him in any of the ways specified in Articles 1497 to 1501 or in any other
manner signing an agreement that the possession is transferred from the vendor to the vendee. Articles 1498
and 1499 are applicable in the case at bar.

In the instant case, actual delivery of the subject tractor could not be made. However, there was
constructive delivery already upon the execution of the public instrument pursuant to Article 1498 and upon
the consent or agreement of the parties when the thing sold cannot be immediately transferred to the
possession of the vendee. (Art. 1499)

Where a third person purchases the mortgaged property, he automatically steps into the shoes of the
original mortgagor. His right of ownership shall be subject to the mortgage of the thing sold to him. In the case at
bar, the petitioner was fully aware of the existing mortgage of the subject tractor to Libra. In fact, when he was
obtaining Libra's consent to the sale, he volunteered to assume the remaining balance of the mortgage debt of
Wilfredo Dy which Libra undeniably agreed to.

The payment of the check was actually intended to extinguish the mortgage obligation so that the tractor
could be released to the petitioner. It was never intended nor could it be considered as payment of the purchas e price
because the relationship between Libra and the petitioner is not one of sale but still a mortgage. The transaction
between the brothers is distinct and apart from the transaction between Libra and the petitioner. The contention,
therefore, that the consummation of the sale depended upon the encashment of the check is untenable.

The sale of the subject tractor was consummated upon the execution of the public instrument on September
4, 1979. At this time constructive delivery was already effected. Hen ce, the subject tractor was no longer owned by
Wilfredo Dy when it was levied upon by the sheriff in December, 1979. Well settled is the rule that only properties
unquestionably owned by the judgment debtor and which are not exempt by law from execution sh ould be levied
upon or sought to be levied upon. For the power of the court in the execution of its judgment extends only over
properties belonging to the judgment debtor.

2B 16-17 SALES AND LEASE Page 191


4. VISAYAN SAWMILL VS CA

G.R. No. 83851. March 3, 1993

FACTS

Babasanta bought a piece of land from spouses Lu, for P15.00 per square meter. Petitioner sent a letter to spouses Lu
requesting for the issuance of the deed of sale and, as he alleges that there was a double sale, that the same be
cancelled. Spouses Lu, on the other hand, contended that when Babasanta failed to pay for the balance, she
requested for a discount which was not accommodated, which caused Babasanta to back out from the sale. San
Lorenzo Development Corporation (SLDC) filed a motion for intervention, alleging that as a buyer in good faith, it
has interest on the dispute.

ISSUE

Who has a better right over the property?

HELD

Babasanta and Spouses Lu entered into a contract to sell, and not a contract of sale. It shows on the receipt as
evidence that Spouses Lu received P50,000.00 as a downpayment. Although it was not stated that the seller reserves
ownership until full price has been paid, Babasantas subsequent acts show that the parties did not intend to convey
ownership until it is fully paid. Spouses Lu recognised his retained ownership by declining to execute a deed of sale
until full payment has been rendered. In a contract of sale, title passes to the vendee upon the delivery of the thing
sold; whereas in a contract to sell, by agreement the owners hip is reserved in the vendor and is not to pass until the
full payment of the price. In a contract of sale, the vendor has lost and cannot recover ownership until and unless the
contract is resolved or rescinded; whereas in a contract to sell, title is retained by the vendor until the full payment of
the price, such payment being a positive suspensive condition and failure of which is not a breach but an event that
prevents the obligation of the vendor to convey title from becoming effective

The mere acknowledgement of receipt of the partial payment does not transfer ownership. SLDC is a buyer in good
faith, because the notice of lis pendens was annotated after the deed of sale was executed in their favor.

2B 16-17 SALES AND LEASE Page 192


5. EUFRENIA BALATICO VDA DE AGATEP VS ROBERTA RODRIGUEZ

G.R. No. 170540, October 28, 2009

FACTS

Petition for review on certiorari, seeking the reversal of CA decision which affirmed decision of Aparri,
Cagayan RTC. Dispute involving 1377 sqm parcel of land located at Zinundungan, Lasam, Cagayan
Subject property previously owned by Natalia Aguinaldo Vda. de Lim.
July 18, 1975: Lim mortgaged lot to PNB Tuguegarao Branch, to secure P30,000 loan. Mortgage contract
was duly annotated on the TCT.
She defaulted, prompting foreclosure by PNB. Land sold at p ublic auction to PNB as highest
bidder. Lim failed to redeem the property.
After the expiration of 1 year redemption period allowed by law, PNB consolidated its ownership
over the disputed land. New TCT issued in the name of PNB
BUT while the mortgage was still in effect on August 18, 1976: Lim sold the property to herein petitioner's
husband, Isaac Agatep for a P18,000.
Said sale was NOT REGISTERED. Neither did Lim deliver the title to petitioner or her husband.
Nonetheless, Agatep took possession of the s ame, fenced it with barbed wire and introduced
improvements thereon. Agatep died in 1978. His heirs, including herein petitioner, continued to
possess the property.
1992: lot was included among PNB's acquired assets for sale. 1993: Lot sold to herein respo ndent Roberta
L. Rodriguez (Rodriguez), who is the daughter of respondent Lim. New TCT issued in name of Rodriguez
1995: Agatep filed complaint for reconveyance and/or damages with the RTC of Aparri, Cagayan against
herein respondents. RTC dismissed. CA affirmed RTC.
RTC: Lim enriched herself at the expense of petitioner and her husband by benefiting from the
proceeds of sale but failing to deliver the object of such sale Petitioner should be awarded
adequate compensation for the value of the loss suffered

ISSUE

Were the RTC and CA correct in finding that PNB was a mortgagee, buyer and seller in good faith?
Yes.
Court finds no error in the findings that PNB is indeed an innocent mortgagee for value. When the
lots were mortgaged to PNB by Lim, the titles thereto were in the latter's name, and they showed
neither vice nor infirmity.
In accepting the mortgage, PNB was not required to make any further investigation of the titles to
the properties being given as security, and could rely entirely on what was st ated in the aforesaid
title.
The public interest in upholding the indefeasibility of a certificate of title, as evidence of the
lawful ownership of the land or of any encumbrance thereon, protects a buyer or mortgagee who,
in good faith, relies upon what appears on the face of the certificate of title
Did PNB acquire ownership over the disputed lot even though the same was not delivered to it? Yes.

2B 16-17 SALES AND LEASE Page 193


Petitioner asserts that the execution of a public document does not constitute sufficient delivery to
PNB, considering that the subject property is in the adverse possession, under claim of ownership,
of petitioner and her predecessor-in-interest
Court finds petitioner's arguments untenable.
Manuel R. Dulay Enterprises, Inc. v. Court of Appeals
Contention that Torres never acquired ownership over subject property since the latter
was never in actual possession nor was the property delivered to him is also without
merit.
Par 1, NCC Art 1498: When the sale is made through a public instrument, the execution
thereof shall be equivalent to the delivery of the thing which is the object of the contract,
if from the deed the contrary does not appear or cannot clearly be inferred.
Under the aforementioned article, the mere execution of the deed of sale in a public
document is equivalent to the delivery of the property (formal or symbolic delivery;
authorizes buyer or transferee to use the document as proof of ownership).
Therefore, prior physical delivery or possession is not legally required
Buyer in a foreclosure sale becomes the absolute owner of the property purchased if it is
not redeemed during the period of one year after the registration of the sale. As such, he
is entitled to the possession of the said property and can demand it at any time following
the consolidation of ownership in his name and the issuance to him of new TCT
RE: Possession
Petitioner's contends: Execution of a public document in favor of PNB did not constitute sufficient
delivery to it because the property involved is in the actual and adverse posse ssion of petitioner
and her husband
SC: Must be noted that their possession of the disputed lot is derived from their right as buyers of
the subject parcel of land. As buyers or transferees, petitioner and her husband simply stepped into
the shoes of Lim, who, prior to selling the subject property to them, mortgaged the same to PNB
As Lim's successors-in-interest, their possession could not be said to be adverse to that of Lim.
Thus, they are also bound to recognize and respect the mortgage entered into by the latter
Their possession of the disputed lot could not, therefore, be considered as a legal impediment
which could prevent PNB from acquiring ownership and possession thereof.
RE: Mortgage
Mortgage is an accessory contract intended to secure the performance of the principal obligation.
One of its characteristics is that it is inseparable from the property. It adheres to the property
regardless of who its owner may subsequently be
It subsists notwithstanding a change in ownership; in short, the personality of the owner is
disregarded. Thus, all subsequent purchasers must respect the mortgage whether the transfer to
them be with or without the consent of the mortgagee, for such mortgage until discharged follows
the property
NCC 2126: the mortgage directly and immediately subjects the property upon which it is imposed,
whoever the possessor may be, to the fulfillment of the obligation for whose security it was
constituted.

2B 16-17 SALES AND LEASE Page 194


6. NFF INDUSTRIAL CORP VS G&L ASSOCIATED BROKERAGE

GR 178169 January 12, 2015

FACTS

On July 20, 1999 respondent company (GLAB) ordered 1000 pieces of bulk bags from petitioner (NFF) at Php 380
per piece, for a total of Php 380,000, payable within 30 days from delivery covered by Purchase Order No. 97-002
(PO) dated July 29, 1999. Said PO has an instruction that the goods were for immediate delivery to G & L
Associated Brokerage, Inc., c/o Hi-Cement Corporation, Norzagaray, Bulacan. Shortly thereafter, GLAB ordered
an additional 1000 pieces of bulk bags with the same price, terms of payment and instructions for delivery.
Accordingly, NFF made deliveries to Hi-Cement (HC) of all the ordered bulk bags on several dates.

NFF alleged that the deliveries were duly acknowledged by representatives of GLAB. All deliveries were likewise
covered by sales invoices, stating that the Php 760,000.00 were duly served upon, and received by GLAB's
representative, one Marian Gabay.

On the other hand, GLAB alleges that the PO specifically provides that the bulk bags were to be delivered at Hi-
Cement Corporation to Mr. Raul Ambrosio, GLAB's checker and authorized representative. Subsequently, the
ordered bulk bags were not delivered to GLAB, the same not having been received by the authorized representative
in confirmity with the PO.

Several demand letters were sent but left unheeded, hence a complaint for sum of money was filed.

RTC ruled in favor of plaintiff, but this was reversed by CA.

ISSUE

Whether or not there was a valid delivery on the part of the petitioner in accordance with law, which would give rise
to an obligation to pay on the part of respondent for the value of bulk bags.

HELD

Yes. Ownership does not pass by mere stipulation but only by delivery. Manresa explains, "the delivery of the thing
x x x signifies that title has passed from the seller to the buyer. Moreover, according to Tolentino, the purpose of
delivery is not only for the enjoyment of the thing but also a mode of acquiring dominion and determines the
transmission of ownership, the birth of the real right. The delivery under any of the forms provided by Articles 1497

2B 16-17 SALES AND LEASE Page 195


to 1505 of the Civil Code signifies that the transmission of ownership from vendor to vendee has taken place. Here,
emphasis is placed on Article 1497 of the Civil Code, which contemplates what is known as real o r actual
delivery,when the thing sold is placed in the control and possession of the vendee.

*Testimonial evidence in full case showing the deliveries were made and acknowledged by Mr. Trinidad (indicated
private individual in case name, who is the GM of the company)

Based on the foregoing, it is clear that petitioner has actually delivered the bulk bags to respondent company, albeit
the same was not delivered to the person named in the Purchase Order. In addition, by allowing petitioners
employee to pass through the guard-on-duty, who allowed the entry of delivery into the premises of Hi-Cement,
which is the designated delivery site, respondents had effectively abandoned whatever infirmities may have attended
the delivery of the bulk bags. As a matter of fact, if respondents were wary about the manner of delivery, such issue
should have been brought up immediately after the first delivery was made. Instead, Mr. Trinidad acknowledged
receipt of the first batch of the bulk bags and even followed up the remaining balance of the orders for delivery.

2B 16-17 SALES AND LEASE Page 196


7. SPS. SANTIAGO vs. VILLAMOR et al.,

G.R. No. 168499, November 26, 2012

Brion, J

FACTS

In January 1982, the spouses Domingo Villamor, Sr. and Trinidad Gutierrez Villamor (spouses Villamor, Sr.), the
parents of Mancer Villamor, Carlos Villamor and Domingo Villamor, Jr. (respondents) and the grandparents of
respondent John Villamor, mortgaged their 4.5-hectare coconut land in Sta. Rosa, San Jacinto, Masbate, known as
Lot No. 1814, to the Rural Bank of San Jacinto (Masbate), Inc. (San Jacinto Bank) as security for a P10,000.00 loan.

When the spouses Villamor, Sr. failed to redeem the property within the prescribed period, the San Jacinto Bank
obtained a final deed of sale in its favor sometime in 1991.

Respondents, together with their sister Catalina Villamor Ranchez, to acquire the land from the San Jacinto Bank.
The San Jacinto Bank agreed with the respondents and Catalina to a P65,000.00 sale, payable in installments.

On July 21, 1994, the spouses Villamor, Sr. sold the land to the petitioners for P150,000.00.

Respondents filed a case against the bank to convey the land to the respondents.

The San Jacinto Bank claimed that it already issued a deed of repurchase in favor of the spouses Villamor, Sr.; the
payments made by the respondents and Catalina were credited to the account of Domingo, Sr. since the real buyers
of the land were the spouses Villamor, Sr.

RTC dismissed the specific performance case. It found that the San Jacinto Bank acted in good faith wh en it
executed a deed of "repurchase" in the spouses Villamor, Sr.s names since Domingo, Sr., along with the
respondents and Catalina, was the one who transacted with the San Jacinto Bank to redeem the land.

The CA found that the respondents and Catalina made the installment payments on their own behalf and not as
representatives of the spouses Villamor, Sr.

2B 16-17 SALES AND LEASE Page 197


The petitioners argue that the spouses Villamor, Sr.s execution of the July 21, 1994 deed of sale in the petitioners
favor was equivalent to delivery of the land under Article 1498 and the petitioners are purchasers in good faith since
they had no knowledge of the supposed transaction between the San Jacinto Bank and the respondents and Catalina
so they are the owners

The respondents and respondent John submit that they hold legal title to the land since they perfected the sale with
the San Jacinto Bank as early as November 4, 1991, the first installment payment.

ISSUE

Whether or not there is delivery contemplated by law?

HELD

No.

Execution of the deed of sale only a prima facie presumption of delivery.

Article 1477

Ownership of the thing sold shall be transferred to the vendee upon the actual or constructive delivery thereof.

Article 1497

The thing sold shall be understood as delivered, when it is placed in the control and possession of the vendee.

With respect to incorporeal property, Article 1498 lays down the general rule: the execution of a public instrument
"shall be equivalent to the delivery of the thing which is the object of the cont ract, if from the deed the contrary does
not appear or cannot clearly be inferred."

However, the execution of a public instrument gives rise only to a prima facie presumption of delivery, which is
negated by the failure of the vendee to take actual possession of the land sold.

"A person who does not have actual possession of the thing sold cannot transfer constructive possession by the
execution and delivery of a public instrument.

In this case, no constructive delivery of the land transpired upon the execution of the deed of sale since it was not the
spouses Villamor, Sr. but the respondents who had actual possession of the land. The presumption of constructive

2B 16-17 SALES AND LEASE Page 198


delivery is inapplicable and must yield to the reality that the petitioners were not placed in possession and control of
the land.

2B 16-17 SALES AND LEASE Page 199


8. ESTELITA VILLAMAR vs. BALBINO MANGAOIL

G.R. No. 188661, April 11, 2012

Reyes, J

FACTS

Villamar is the registered owner of a 3.6080 hectares parcel of land in San Francisco, Manuel, Isabela co vered
by a Transfer Certificate of Title (TCT).

On March 30, 1998, she entered into an Agreement with Mangaoil for the purchase and sale of said parcel of
land, under the following terms and conditions:

1. The price of the land is ONE HUNDRED AND EIGHTY THOUSAND (180,000.00) PESOS
per hectare but only the 3.5000 hec. shall be paid and the rest shall be given free, so that the total purchase
or selling price shall be [P]630,000.00 only;

2. ONE HUNDRED EIGHTY FIVE THOUSAND (185,000.00) PESOS of the total price was
already received on March 27, 1998 for payment of the loan secured by the certificate of title covering
the land in favor of the Rural Bank of Cauayan, San Manuel Branch, San Manuel, Isabela, in order that
the certificate of title thereof be withdrawn and released from the said bank, and the rest shall be for the
payment of the mortgages in favor of Romeo Lacaden and Florante Parangan;

3. After the release of the certificate of title covering the land subject -matter of this agreement, the
necessary deed of absolute sale in favor of the PARTY OF THE SECOND PART shall be executed and
the transfer be immediately effected so that the latter can apply for a loan from any lending institution
using the corresponding certificate of title as collateral therefor, and the proceeds of the loan, whatever be
the amount, be given to the PARTY OF THE FIRST PART;

On April 1, 1998, the parties executed a Deed of Absolute Sale whereby Villamar transferred the subject parcel
of land to Mangaoil.

In a letter dated September 18, 1998, Mangaoil informed Villamar that he was backing out from the sale agreed
upon, reasoning that the area is not yet fully cleared by encumbrances as there are tenants who are not willing to
vacate the land without giving them back the amount that they mortgaged the land.

Mangaoil demanded refund of his downpayment, but was unheeded.

Mangaoil filed before the RTC a complaint for rescission of contract against Villamar, alleging that:

- he handed Villamar the sum of P185,000.00 to be applied as follows: P80,000 was for the redemption of the land
which was mortgaged to the Rural Bank of Cauayan, San Manuel Branch, San Manuel, Isabela, to enable the
plaintiff to get hold of the title and register the sale, and P105,000.00 was for the redemption of the said land from
private mortgages to enable plaintiff to possess and cultivate the same;

- That although the defendant had already long redeemed the said land from the said bank and withdrawn the TCT,
she has failed and refused, despite repeated demands, to hand over the said title to the plaintiff and still refuses and
fails to do so;

2B 16-17 SALES AND LEASE Page 200


- That, also, the plaintiff could not physically, actually and materially possess and cultivate the said land because the
private mortgagees and/or present possessors refuse to vacate the same;

Villamar averred that she had complied with her obligations to Mangaoil, having caused the release of the TCT
by the Rural Bank of Cauayan and its delivery to a certain Atty. Antonio (petitioners counsel) who was allegedly
commissioned to facilitate the transfer of the said title in Mangaoils name. She stresses that under Article 1498 of
the NCC, when a sale is made through a public instrument, its execution is equivalent to the delivery of the th ing
which is the contract's object, unless in the deed, the contrary appears or can be inferred.

Mangaoil invokes Articles 1191 and 1458 to argue that when a seller fails to transfer the ownership and
possession of a property sold, the buyer is entitled to rescind the contract of sale. Further, he contends that the
execution of a deed of absolute sale does not necessarily amount to a valid and constructive delivery.

The RTC ordered the rescission of the agreement and the deed of absolute sale executed between the respondent
and the petitioner.

The Court of Appeals affirmed the ruling of the RTC.

ISSUE

Whether or not the failure of the petitioner to deliver to the respondent both the physical possession of the subject
property and the certificate of title amounts to a substantial breach of the petitioners obligations constituting a valid
cause to rescind the agreement and deed of sale entered into by the parties.

HELD

YES.

Article 1498 of the NCC generally considers the execution of a public instrument as constructive delivery by the
seller to the buyer of the property subject of a contract of sale. The case at bar, however, falls among the exceptions
to the foregoing rule since a mere presumptive and not conclusive delivery is created as the respondent failed to take
material possession of the subject property.

It is doctrinally settled that in a contract of sale, the vendor is bound to transfer the ownership of, and to deliver
the thing that is the object of the sale, the way Article 1547 of the Civil Code is worded, viz.:

Art. 1547. In a contract of sale, unless a contrary intention appears, there is:

(1) An implied warranty on the part of the seller that he has a right to sell the thing at the time when the ownership is
to pass, and that the buyer shall from that time have and enjoy the legal and peaceful possession of the thing;

(2) An implied warranty that the thing shall be free from any hidden defaults or defects, or any change or
encumbrance not declared or known to the buyer.

shows that actual, and not mere constructive delivery is warrantied by the seller to the buyer. Peaceful possession of
the thing sold can hardly be enjoyed in a mere constructive delivery.

2B 16-17 SALES AND LEASE Page 201


Although Articles 1458, 1495 and 1498 of the NCC and case law do not generally require the seller to deliver to
the buyer the physical possession of the property subject of a contract of sale and the certificate of title covering the
same, the agreement entered into by the petitioner and the respondent provides otherwise. However, the terms of the
agreement cannot be considered as violative of law, morals, good customs, public order, or public policy, hence,
valid.

Item no. 3 of the agreement executed by the parties expressly states that transfer s hall be immediately effected so
that the latter can apply for a loan from any lending institution using the corresponding certificate of title as
collateral therefore. Item no. 3 is literal enough to mean that there should be physical delivery of the TCT for how
else can the respondent use it as a collateral to obtain a loan if the title remains in the petitioners possession.

Article 1458 of the NCC obliges the seller to transfer the ownership of and to deliver a determinate thing to the
buyer, who shall in turn pay therefor a price certain in money or its equivalent. In addition thereto, Article 1495 of
the NCC binds the seller to warrant the thing which is the object of the sale. On the other hand, Article 1498 of the
same code provides that when the sale is made through a public instrument, the execution thereof shall be equivalent
to the delivery of the thing which is the object of the contract, if from the deed, the contrary does not appear or
cannot clearly be inferred.

2B 16-17 SALES AND LEASE Page 202


PLACE OF DELIVERY

1. ALEJANDRA BUGARIN VDA. DE SARMIENTO vs. JOSEFA R. LESACA

G.R. No. L-15385, June 30, 1960

FACTS:

January 18, 1949 - Plaintiff bought from defendant two parcels of land for P5,000. After the sale, plaintiff tried to
take actual physical possession of the lands but was prevented from doing so by one Martin Deloso who claims to be
the owner thereof. February 1, 1949 - Plaintiff instituted an action before the Tenancy Enforcement Division of the
DOJ to oust said Martin Deloso from the possession of the lands, which action she later abandoned for reasons
known only to her. December 12, 1949, plaintiff wrote defendant asking the latter either to change the lands sold
with another of the same kind and class or to return the purchase price together with t he expenses she had incurred
in the execution of the sale, plus 6% interest; Defendant did not agree with her proposition. TC: Rescinded the sale

ISSUE

Whether the execution of the deed of sale in a public document is equivalent to delivery of possession of the lands
sold?

HELD

Articles 1461 and 1462 of the old Civil Code provide:

ART. 1461. The vendor is bound to deliver and warrant the thing which is the subject -matter of the sale.

ART. 1462. The thing sold shall be deemed delivered when the vendee is placed in the control and possession
thereof.

If the sale should be made by means of a public instrument, the execution thereof shall be equivalent to the delivery
of the thing which is the subject-matter of the contract unless the contrary appears or is clearly to be inferred from
such instrument.

When a contract of sale is executed the vendor is bound to deliver to the vendee the thing sold by placing the vendee
in the control and possession of the subject-matter of the contract. However, if the sale is executed by means of a
public instrument, the mere execution of the instrument is equivalent to delivery unless the contrary appears or is
clearly to be inferred from such instrument.

In the case at bar, there is no stipulation that the vendor did not inten d to deliver outright the possession of the lands
to the vendee. On the contrary, it can be clearly seen therein that the vendor intended to place the vendee in actual
possession of the lands immediately as can be inferred from the stipulation that the ven dee "takes actual possession
thereof ... with full rights to dispose, enjoy and make use thereof in such manner and form as would be most

2B 16-17 SALES AND LEASE Page 203


advantageous to herself." The possession referred to in the contract evidently refers to actual possession and not
merely symbolical inferable from the mere execution of the document.

The vendor did not comply with his commitment. From the execution of the sale up to the present the vendee was
never able to take possession of the lands due to the insistent refusal of Mart in Deloso to surrender them claiming
ownership thereof. And although it is postulated in the same article that the execution of a public document is
equivalent to delivery, this legal fiction only holds true when there is no impediment that may prevent the passing of
the property from the hands of the vendor into those of the vendee.

If, notwithstanding the execution of the instrument, the purchaser cannot have the enjoyment and material tenancy
of the thing and make use of it himself or through another in his name, because such tenancy and enjoyment are
opposed by the interposition of another will, then fiction yields to reality the delivery has not been effected.

Plaintiff can rescind the contract of sale. In a contract of purchase and sale the obligation of the parties is reciprocal,
and, as provided by the law, in case one of the parties fails to comply with what is incumbent upon him to do, the
person prejudiced may either exact the fulfillment of the obligation or rescind the sale. Since plaintiff ch ose the
latter alternative; her action is in accordance with law.

2B 16-17 SALES AND LEASE Page 204


2. NORKIS DISTRIB UTORS, INC., vs. THE COURT OF APPEALS & ALBERTO NEPALES

G.R. No. 91029 February 7, 1991

Grino-Aquino, J

FACTS:

On September 20, 1979 private respondent Alberto Nepales bought a Yamaha Wonderbike Motorcycle
from petitioner Norkis Distributor Inc. (Norkis)-Bacolod Branch for P7,500.00. The price was payable by means of
a Letter of Guaranty from the Development Bank of the Philippines (DBP). As security for th e loan, Nepales would
execute a chattel mortgage on the motorcycle in favour of DBP. Branch Manager Labajo issued Norkis Sales
Invoice No. 0120 showing that the contract of sale of the motorcycle had been perfected. Nepales signed the sales
invoice to signify his conformity with the terms of the sale. In the meantime, however, the motorcycle remained in
Norkis' possession.

On November 6, 1979, the motorcycle was registered in the Land Transportation Commission (LTC) in the
name of Alberto Nepales. The regis tration fees were paid by Nepales, evidenced by an official receipt. On January
22, 1980, the motorcycle was delivered to a certain Julian Nepales who was allegedly the agent of Alberto Nepales
but the latter denies it. The motorcycle met an accident on February 3, 1980 at Binalbagan, Negros Occidental being
driven by a certain Zacarias Payba. The unit was a total wreck. It was returned, and stored inside Norkis' warehouse.
DBP paid Norkis the amount. Nepales demanded delivery of the motorcycle. When Norkis could not deliver, he
filed an action for specific performance with damages against Norkis.

Norkis answered that the motorcycle had already been delivered to private respondent before the accident.
Norkis concedes that while there was no "actual" delivery of the vehicle, there was constructive delivery of the unit
upon: (1) the issuance of the Sales Invoice No. 0120 in the name of the private respondent and the affixing of his
signature thereon (2) the registration of the vehicle with the LTC. and (3) t he issuance of official receipt for
payment of registration fees. Hence, the risk of loss or damage had to be borne by him as owner of the unit.

RTC ruled in favour of Nepales. Court of Appeals affirmed the lower courts decision. Norkis was ordered
to pay Nepales the price of the motorcycle plus interest or deliver a brand new motorcycle of the same kind.

ISSUE

Whether or not Norkis should bear the loss of the motorcycle.

HELD

AFFIRMATIVE

Article 1496 of the Civil Code provides that "in the absence of an express assumption of risk by the buyer,
the things sold remain at seller's risk until the ownership thereof is transferred to the buyer. The issuance of a sales
invoice does not prove transfer of ownership of the thing sold to the buyer. An invoice is n othing more than a
detailed statement of the nature, quantity and cost of the thing sold and has been considered not a bill of sale. In all

2B 16-17 SALES AND LEASE Page 205


forms of delivery, it is necessary that the act of delivery whether constructive or actual, be coupled with the inte ntion
of delivering the thing. The act, without the intention, is insufficient.

When the motorcycle was registered by Norkis in the name of private respondent, Norkis did not intend yet
to transfer the title or ownership to Nepales, but only to facilitate the execution of a chattel mortgage in favor of the
DBP for the release of the buyer's motorcycle loan. The Letter of Guarantee issued by the DBP, reveals that the
execution in its favor of a chattel mortgage over the purchased vehicle is a prerequisite for the approval of the
buyer's loan. If Norkis would not accede to that arrangement, DBP would not approve private respondent's loan
application and, consequently, there would be no sale.

There was neither an actual nor constructive delivery of the thing s old. Hence, the risk of loss should be
borne by the seller, Norkis, which was still the owner and possessor of the motorcycle when it was wrecked. This is
in accordance with the well-known doctrine of res perit domino.

2B 16-17 SALES AND LEASE Page 206


3. CLARA M. BALATBAT vs. COURT OF APPEALS, ET AL.

G.R. No. 109410 August 28, 1996

FACTS

A parcel of land was acquired by plaintiff Aurelio Roque and Maria Mesina during their conjugal union. Upon the
Death of Maria, Aurelio A. Roque filed a complaint for partition doc keted as Civil Case No. 109032 against
Corazon Roque, Alberto de los Santos, Feliciano Roque, Severa Roque and Osmundo Roque before the then Court
of First Instance of Manila, Branch IX. On March 29, 1979, the trial court rendered a decision in favor of plaintiff
Aurelio A. Roque. On June 2, 1979, the decision became final and executory. On October 5, 1979, the Register of
Deeds of Manila issued a Transfer Certificate of Title No. 135671 in the name of the following persons in the
following proportions: 6/10 share for Aurelio A.Roque and 4/10 to the defendant. On April 1, 1980, Aurelio A.
Roque sold his 6/10 share in T.C.T. No. 135671 to spouses Aurora Tuazon -Repuyan and Jose Repuyan as evidenced
by a Deed of Absolute Sale.

On July 21, 1980, Aurora Tuazon Repuyan caused the annotation of her affidavit of adverse claimon the Transfer
Certificate of Title No. 13567. On August 20, 1980, Aurelio A. Roque filed a complaint for Rescission of Contract
docketed as Civil Case No. 134131 against spouses Aurora Tuazon-Repuyan and Jose Repuyan before Branch IV of
the then Court of First Instance of Manila. The complaint is grounded on spouses Repuyans failure to pay the
balance of P45,000.00 of the purchase price. A deed of absolute sale was executed on February 4, 1982 between
Aurelio S. Roque, Corazon Roque, Feliciano Roque, Severa Roque and Osmundo Roque and Clara Balatbat,
married to Alejandro Balatbat. On April 14, 1982, Clara Balatbat filed a motion for the issuance of a writ of
possession which was granted by the trial court on September 14, 1982 subject, however, to valid rights and interest
of third persons over the same portion thereof, other than vendor or any other person or persons privy to or claiming
any rights or interest under it. The corresponding writ of possession was issued on September 20, 1982. On May 20,
1982, petitioner Clara Balatbat filed a motion to intervene in Civil Case No. 134131 (Rescission Case) which was
granted. However, Clara Balatbat failed to file her complaint in intervention. On A pril 15, 1986, the trial court
rendered a decision dismissing the complaint, and declaring the Deed of Absolute Sale (between Aurelio Roque and
Respondents) dated April 1, 1980 as valid and enforceable.

ISSUE

Whether or not there is a transfer of ownership upon the delivery of the thing even if the purchase price is not fully
paid

HELD

Examining the terms and conditions of the Deed of Sale dated April 1, 1980, the P45,000.00 balance is payable only
after the property covered by T.C.T. No. 135671 has been partitioned and subdivided, and title issued in the name of
the BUYER hence, vendor Roque cannot demand payment of the balance unless and until the property has been
subdivided and titled in the name of the private respondents. Devoid of any stipulation that ownership in the thing
shall not pass to the purchaser until he has fully paid the price, ownership in the thing shall pass from the vendor to
the vendee upon actual or constructive delivery of the thing sold even if the purchase price has not yet bee n fully
paid. The failure of the buyer to make good the price does not, in law, cause the ownership to revest to the seller

2B 16-17 SALES AND LEASE Page 207


unless the bilateral contract of sale is first rescinded or resolved pursuant to Article 1191 of the New Civil Code.
Non-payment only creates a right to demand the fulfillment of the obligation or to rescind the contract.

With respect to the non-delivery of the possession of the subject property to the private respondent, suffice it to say
that ownership of the thing sold is acquired only from the time of delivery thereof, either actual or constructive.
Article 1498 of the Civil Code provides that - when the sale is made through a public instrument, the execution
thereof shall be equivalent to the delivery of the thing which is the ob ject of the contract, if from the deed the
contrary does not appear or cannot be inferred. The execution of the public instrument, without actual delivery of the
thing, transfers the ownership from the vendor to the vendee, who may thereafter exercise the rights of an owner
over the same. In the instant case, vendor Roque delivered the owners certificate of title to herein private respondent.
It is not necessary that vendee be physically present at every square inch of the land bought by him, possession of
the public instrument of the land is sufficient to accord him the rights of ownership. Thus, delivery of a parcel of
land may be done by placing the vendee in control and possession of the land (real) or by embodying the sale in a
public instrument (constructive). The provision of Article 1358 on the necessity of a public document is only for
convenience, not for validity or enforceability. It is not a requirement for the validity of a contract of sale of a parcel
of land that this be embodied in a public instrument.

A contract of sale being consensual, it is perfected by the mere consent of the parties. Delivery of the thing brought
or payment of the price is not necessary for the perfection of the contract; and failure of the vendee to pay the price
after the execution of the contract does not make the sale null and void for lack of consideration but results at most
in default on the part of the vendee, for which the vendor may exercise his legal remedies.

2B 16-17 SALES AND LEASE Page 208


4. GONZALES vs. HABERER

G.R. No. L-22604 February 3, 1925

Ostrand, J

FACTS:

This action is brought to recover the sum of P34,260 alleged to be due the plaintiffs from the defendant upon a
written agreement for the sale of a tract of land situated in the Province of Nueva Ecija . The plaintiffs also ask for
damages in the sum of P10,000 for the alleged failure of the defendant to comply with his part of the agreement.

The defendant in his answer admits that of the purchase price stated in the agreement a balance of P31,000 remain s
unpaid, but by way of special defense, cross -complaint and counter-claim alleges that at the time of entering into the
contract the plaintiffs through false representations lead him to believe that they were in possession of the land and
that the title to the greater portion thereof was not in dispute; that on seeking to obtain possession he found that
practically the entire area of the land was occupied by adverse claimants and the title thereto disputed; that he
consequently has been unable to obtain possession of the land; and that the plaintiffs have made no efforts to
prosecute the proceedings for the registration of the land. He therefore asks that the contract be rescinded; that the
plaintiffs be ordered to return to him the P30,000 already paid by him to them and to pay P25,000 as damages for
breach of the contract.

The court below dismissed the plaintiffs' complaint, declared the contract rescinded and void and gave the defendant
judgment upon his counterclaim for the sum of P30,000, with interest from the date upon which the judgment
becomes final. The case is now before this court upon appeal by the plaintiffs from that judgment.

ISSUE

Whether or not there is sufficient ground for recission?

HELD

YES. It is conceded by the plaintiffs that the defendant never obtained actual or physical possession of the land, but
it is argued that under the contract quoted the plaintiffs were under no obligation to place him in possession. This
contention cannot be sustained. The contract gave the defendant the right to take possession of the land immediately
upon the execution of the contract and necessarily created the obligation on the part of the plaintiffs to make good
the right thus granted; it was one of the essential conditions of the agreement and the failure of the plaintiffs to
comply with this condition, without fault on the part of the defendant, is in itself sufficient ground for the rescission,
even in the absence of any misrepresentation on their part.

It is therefore unnecessary to discuss the ques tion whether the defendant was induced to enter into the agreement
through misrepresentation made by the plaintiff Gomez. We may say, however, that the evidence leaves no doubt
that some misrepresentations were made and that but for such misrepresentations the defendant would not have been
likely to enter into the agreement in the form it appeared. As to the contention that the plaintiff Gonzalez cannot be
charged with the misrepresentations of Gomez, it is sufficient to say that the latter in negotiating for the sale of the

2B 16-17 SALES AND LEASE Page 209


land acted as the agent and representative of the other plaintiff, his wife; having accepted the benefit of the
representations of her agent she cannot, of course, escape liability for them.

The contention of the appellants that the symbolic delivery effected by the execution and delivery of the agreement
was a sufficient delivery of the possession of the land, is also without merit. The possession referred to in the
contract is evidently physical; if it were otherwise it would not have b een necessary to mention it in the contract.

2B 16-17 SALES AND LEASE Page 210


5. AGCAOILI VS GSIS

GR NO L 30056 August 30, 1988

FACTS:
Appellee Marcelo Agcaoili applied for purchase of a house and lot in the GSIS Housing Project at Nangka
Marikina, Rizal, subject to the condition that the latter should forthwith occupy the house, which such condition
cannot be fulfilled by Agcaoili as the house was uninhabitable. Agcaoili thereafter refused to pay the first
installment and other fees on the house. GSIS cancelled the award and required Agcaoili to vacate the premises.
Agcaoili then instituted action in the CFI of Manila for specific performance and damages. Pending action, a written
protest was likewise lodged by other awardees in the housing units of the same sub division, regarding the failure of
the System to complete construction of their own houses. Judgment was rendered in favor of Agcaoili. GSIS wanted
to reverse the decision on the ground that perfection of the contract of sale between it and Agcaoili being
conditioned upon the latters immediate occupancy of the house subject thereof, and the latter having failed to
comply with the obligation, no contract ever came in existence between them. Agcaoili's offer to buy from GSIS
was contained in a printed form drawn up by the latter, entitled "Application to Purchase a House and/or Lot."
Agcaoili filled up the form, signed it, and submitted it. The acceptance of the application was also set out in a form
(mimeographed) also prepared by the GSIS. The form sent to Agcaoili, duly filled up, advised him of the approval
of his "application to purchase a house and lot in our GSIS Housing Project at NANGKA, MARIKINA, RIZAL,"
and that "Lot No. 26, Block No.(48) 2, together with the housing unit constructed thereon, has be en allocated to
you."

ISSUE:
Whether or Agcaoili may have the house completed so that it may be put into habitable condition

HELD:
The completion of the unfinished house so that it may be put into habitable condition, as one form of relief to the
plaintiff Agcaoili, no longer appears to be a feasible option in view of the not inconsiderable time that has already
elapsed. That would require an adjustment of the price of the subject of the sale to conform to present prices of
construction materials and labor. It is more in keeping with the realities of the situation, and with equitable norms, to
simply require payment for the land on which the house stands, and for the house itself, in its unfinished state, as of
the time of the contract. In fact, this is an alternative relief proposed by Agcaoili himself, i.e., "that judgment issue . .
(o)rdering the defendant (GSIS) to execute a deed of sale that would embody and provide for a reasonable
amortization of payment on the basis of the present actual unfinished and uncompleted condition, worth and value of
the said house

There was a meeting of the minds upon the purchase by Agcaoili of a determinate house and lot in the GSIS
Housing Project at Nangka Marikina, Rizal at a definite price payable in amortizations a t P31.56 per month, and
from that moment the parties acquired the right to reciprocally demand performance.It was, to be sure, the duty of
the GSIS, as seller, to deliver the thing sold in a condition suitable for its enjoyment by the buyer for the purpos e
contemplated , in other words, to deliver the house subject of the contract in a reasonably livable state. This it failed
to do. It sold a house to Agcaoili, and required him to immediately occupy it under pain of cancellation of the sale.
Under the circumstances there can hardly be any doubt that the house contemplated was one that could be occupied
for purposes of residence in reasonable comfort and convenience. By any objective interpretation of its terms, the
contract can only be understood as imposing on the GSIS an obligation to deliver to Agcaoili a reasonably habitable
dwelling in return for his undertaking to pay the stipulated price.

2B 16-17 SALES AND LEASE Page 211


6. CONSING vs. CA

G.R. No. 78272 August 29, 1989

FACTS

Petitioner is a registered owner of a parcel of land which she caused to be subdivided into 38 lots. Upon approval of
the subdivision, she likewise filed a petition for the segregation of titles for the 38 lots. Certain lots, including the
two lots subject of the controversy, were subject to voluntary right of way. Under the name Mearle Homes, the
spouses Consing engaged in the sale of the 38 lots.
2. Oct 4, 1971, Petitioner and private respondent Santos entered in to an agreement denominated as a Contract of
Sale for a house and lot.

In consideration of which, buyer is to pay seller Php110,000 with interest at 12% per annum. The same is payable as
follows:
- P25,000.00 upon the signing of the contract
- monthly installment of P 1,020.14 payable on or before the fifth day of each month beginning December 1971
without necessity of demand until the amount of the purchase price and interest shall have been fully paid
- after which ownership would be transferred to the buyer.

Starting May 1972, respondent defaulted. Demand letters were sent but were futile.
Respondent manifested her willingness to settle her obligations on the condition that the Consings comply with all
the laws and regulations on subdivisions and after payment to her of damages as a consequence of the use of a
portion of her lot, more or less 168 sq.m., as a subdivision road. Consings filed an ejectment case, decision was in
favor of Consings.
During pendency of ejectment case, respondent filed a complaint for specific performance with damages with CFI.
CFI issued restraining order enjoining Municipal Court from resolving motion for execution.
Consing submitted his subdivision plan to Municipal Council of Marikina for approval. After recommendations to
proper authorities, the same was approved.

ISSUE

Whether reduction of purchase price is proper since part of the lots sold to the private respondents are used as
subdivision roads.

HELD
It is undisputed that the Consings' subdivision plan was approved by the LRC as a simple subdivision which
indicated no streets or roads. However, this does not preclude the need for them within the subdivision. The relative
position of this "right of way" vis -a-vis the lots shows that it is in fact a road without which the subdivision lot
buyers would have no means of access to and from the subdivision. These should not have been the object of sale in
favor of private respondents Santos for they are deprived of its exclusive enjoyment since the same is used by
everyone.
A subdivision seller's duty includes the duty to deliver the thing sold in a condition suitable fo r its enjoyment by the
buyer for the purposes contemplated. Since proper access to a residence is essential to its enjoyment, petitioners are
bound to build roads within the subdivision.

2B 16-17 SALES AND LEASE Page 212


In including as part of Santos' purchase price the value of the subd ivision road, petitioners have shifted to her the
burden of providing for an access to and from the subdivision. The Consings have thus failed in their duty as
subdivision lot sellers and for such failure and consequent unfairness and injustice to Santos, the latter should be
entitled to a proportionate reduction in her purchase price of the two lots.

2B 16-17 SALES AND LEASE Page 213


7. GABILA vs. PEREZ

G.R.No.: L-29541 January 27,1989

FACTS

Siblings Pablo, Ramon and Mercedes Perez executed a Deed of Sale in favor of petitioner Carlos Gabila for a parcel
of land which they inherited from their father. It was stipulated that the siblings would execute an Extra-Judicial
Partition and pay the estate and inheritance taxes in order that the title to the land be transferred in their names, and
afterwards transferred to vendee Gabila. Upon payment of P1,500 (down payment), the possession of the land was
immediately delivered to the vendee.

Despite the complete payment of the monthly installments (P1,000 in total), the siblings failed to comply with their
obligation. Petitioner filed for action praying that the defendants be ordered to execute the Extra-Judicial Partition,
pay the estate and inheritance taxes, and execute transfer of title to him. The trial court dismissed the action on the
ground that the defendants can no longer be compelled to execute the Extra-Judicial Partition for the said land
because it has been sold to the plaintiff; the extrajudicial partition of the property should have been done at the time
of the sale.

ISSUE

Whether or not the execution of an Extra-Judicial Partition is necessary so that the title can be transferred in
petitioners name

HELD

No, there is no need to partition the land among the siblings before the transfer of title to petitioner because the land
in question has already been sold, and the siblings co-ownership over it has been terminated by the sale. The sale of
the property has been consummated by the payment of the price and by the delivery of the peaceful possession of
the land to the plaintiff-vendee. The Supreme Court held that [a]ll that needs to be done now is to register on the
TCT No. 899 of the late Mariano Perez, the deed of sale, which may also be treated as an affidavit of adjudication of
the land, to the vendors in order that their father's title may be cancelled and a new one can be issued to their vendee,
Carlos Gabila.

2B 16-17 SALES AND LEASE Page 214


8. ALFONSO QUIJADA, ET AL. vs. COURT OF APPEALS

G.R. No. 126444, December 4, 1998

Martinez, J

FACTS:

Alfonso, Cresente, Reynelda, Demetrio, Eliuteria, Eulalio, and Warlito, all surnamed Quijada (petitioners), are the
children of the late Trinidad Corvera Vda. de Quijada. Trinidad inherited from Pedro Corvera, the two-hectare
parcel of land subject of the case, situated in the barrio of San Agustin, Talacogon, Agusa n del Sur. Trinidad
Quijada together with her sisters, executed a conditional deed of donation of the Two -hectare parcel of land subject
of the case in favor of the Municipality of Talacogon, with the condition that it shall be used solely and exclusively
as part of the campus of the proposed provincial high school in Talacogon. Apparently, Trinidad remained in
possession of the land despite the donation. On July 29, 1962, Trinidad sold one (1) hectare of the subject parcel of
land to Regalado Mondejar. Subsequently, Trinidad verbally sold the remaining one (1) hectare to Regalado
Mondejar without the benefit of a written deed of sale and evidenced solely by receipts of payment. In 1980, the
heirs of Trinidad, filed a complaint for forcible entry against Reg alado Mondejar, which complaint was, however,
dismissed for failure to prosecute. In 1987, the proposed provincial high school having failed to materialize, the
Sangguniang Bayan of the municipality of Talacogon enacted a resolution reverting the two (2) h ectares of land
donated back to the donors. In the meantime, Regalado Mondejar sold portions of the land to Fernando Bautista,
Rodolfo Goloran, Efren Guden, and Ernesto Goloran (respondents). On July 5, 1988, the Quijadas filed an action
alleging that their deceased mother never sold, conveyed, transferred, or disposed of the property in question to any
person or entity much less to Regalado Mondejar.

RTC RULED IN FAVOR OF THE QUIJADAS. Trinidad Quijada had no legal title or right to sell the land to
defendant Mondejar in 1962, 1966, 1967 and 1968, the same not being hers to dispose of because ownership belongs
to the Municipality of Talacogon

CA REVERSED THE RULING. The sale made by Trinidad Quijada to Mondejar was valid as the former retained
an inchoate interest on the lots by virtue of the automatic reversion clause in the deed of donation.

ISSUE

Whether or not the sale of the property to Mondejar by Trinidad Quejada was valid considering that at the time of
the perfection of contract, the land was already donated to Municipality.

HELD

Yes, the sale is valid. Sale, being a consensual contract, is perfected by mere consent, which is manifested the
moment there is a meeting of the minds as to the offer and acceptance thereof on three (3) elements: subjec t matter,
price and terms of payment of the price. Ownership by the seller on the thing sold at the time of the perfection
of the contract of sale is not an element for its perfection. What the law requires is that the seller has the right
to transfer ownership at the time the thing sold is delivered. Perfection per se does not transfer ownership which
occurs upon the actual or constructive delivery of the thing sold. A perfected contract of sale cannot be challenged
on the ground of non-ownership on the part of the seller at the time of its perfection hence, the sale is still valid.

2B 16-17 SALES AND LEASE Page 215


The consummation, however, of the perfected contract is another matter. It occurs upon the constructive or actual
delivery of the subject matter to the buyer when the seller or her successors-in-interest subsequently acquires
ownership thereof. Such circumstance happened in this case when petitioners who are Trinidad Quijadas heirs
and successors-in-interest became the owners of the subject property upon the reversion of the o wnership of the
land to them. Consequently, ownership is transferred to respondent Mondejar and those who claim their right from
him. Article 1434 of the New Civil Code supports the ruling that the sellers title passes by operation of law to the
buyer. This rule applies not only when the subject matter of the contract of sale is goods, but also to other kinds of
property, including real property.

2B 16-17 SALES AND LEASE Page 216


9. UNION MOTOR CORPORATION, vs. THE COURT OF APPEALS, JARDINE-MANILA FINANCE,
INC., SPOUSES ALBIATO BERNAL and MILAGROS BERNAL

G.R. No. 117187, July 20, 2001

SC cited Addison v. Felix and Tioco The Code imposes upon the vendor the obligation to deliver the thing sold.
The thing is considered to be delivered when it is placed in the hands and possession of the vendee. (Civil Code,
Art. 1462). [Now Art. 1497, Civil Code]

FACTS

On September 14, 1979, the respondent Bernal spouses purchased from petitioner Union Motor Corporation
(UMC) one Cimarron Jeepney for P37,758.60 to be paid in installments. The spouses executed a promissory
note and a deed of chattel mortgage in favor of the petitioner. Meanwhile, the petitioner entered into a contract of
assignment of the promissory note and chattel mortgage with Jardine-Manila Finance, Inc. (JMFI) Through Manuel
Sosmea, an agent of the petitioner, the parties agreed that the respondent spouses would pay the amount of the
promissory note to JMFI the latter being the assignee of the petitioner. To effectuate the sale as well as the
assignment of the promissory note and chattel mortgage, the respondent spouses were required to sign a notice of
assignment, a deed of assignment, a sales invoice, a registration certificate, an affidavit, and a disclosure statement.
The respondent spouses were obliged to sign all these document s for the reason that, according to Sosmea, it was a
requirement of petitioner UMC and JMF, for the application of sale to the spouses be approved. Upon the
respondent spouses tender of the downpayment worth P10,037.00, and the petitioners acceptance of the
same, the latter approved the sale. Although the respondent spouses have not yet physically possessed the
vehicle, Sosmea required them to sign the receipt as a condition for the delivery of the vehicle.

The respondent spouses continued paying the agreed installments even if the subject motor vehicle remained
undelivered, after P7,507.00 worth of installments they discontinued paying on account of non-delivery of the
subject motor vehicle. It was alleged that the non-delivery of the vehicle was because of the fact that Sosmea, the
agent of UMC, took the vehicle in his personal capacity.

JMF filed a complaint for a sum of money against Spouses Bernal, which was later amended to include UMC as
alternative defendant, the reason being that if the respond ent spouses refusal to pay JardineManila Finance, Inc.
was due to petitioners nondelivery of the unit.

The Trial court rendered a Decision ordering

1. JMF to pay spouses the sum of P7,507.15 plus legal interest until fully paid;

2. Union Motor Corporation to pay defendants spouses Bernals the downpayment in the amount of
P10,037.00, plus legal interest, until fully paid;

3. Union Motor Corporation to pay plaintiff P23,268.29, plus legal interest until fully paid, and attorneys
fees equivalent to 20% of the amount due to plaintiff.

Both petitioner and respondent filed an appeal, however the CA affirmed the Decision of the trial court.

Hence this Petition.

2B 16-17 SALES AND LEASE Page 217


The Petitioner UMC maintains that the respondent spouses are not entitled to a return of the down payment for the
reason that there was a delivery of the subject motor vehicle. There was a constructive delivery of the vehicle when
respondent Albiato Bernal signed the registration certificate of the subject vehicle. Inasmuch as there was already
delivery of the subject motor vehicle, ownership has been transferred to the respondent spouses. The Chattel
Mortgage Contract signed by the respondent Bernal spouses in favor of the petitioner likewise proves that ownership
has already been transferred to them for the reason that, under Article 2085 of the New Civil Code, the mortgagor
must be the owner of the property. As owners of the jeepney, the respondent Bernal spouses should bear the loss
thereof in accordance with Article 1504 of the New Civil Code which provides that when the ownership of goods is
transferred to the buyer, the goods are at the buyers risk whether actual delivery has been made or not.

The main allegation of the respondent Bernal spouses , on the other hand, is that they never came into possession
of the subject motor vehicle. Thus, it is but appropriate that they be reimbursed by the petitioner of the initial
payment which they made.

ISSUE

Whether or not there has been a delivery, physical or constructive, of the subject motor vehicle.

HELD

None. We rule in favor of the respondent Bernal spouses.

Undisputed is the fact that the respondent Bernal spouses did not come into possession of the subject Cimarron
jeepney that was supposed to be delivered to them by the petitioner. The registration certificate, receipt and sales
invoice that the respondent Bernal spouses signed were explained during the hearing without any opposition by the
petitioner. According to testimonial evidence adduced by the respondent spouses during the trial of the case, the said
documents were signed as a part of the processing and for the approval of their application to buy the subject motor
vehicle. Without such signed documents, no sale, much less delivery, of the subject jeepney could be made. The
documents were not therefore an acknowledgment by respondent spouses of the physical acquisition of the subject
motor vehicle but merely a requirement of petitioner so that the said subject motor vehicle would be delivered to
them. We have ruled that the issuance of a sales invoice does not prove transfer of ownership of the thing sold to the
buyer; an invoice is nothing more than a detailed statement of the nature, quantity and cost of the thing sold and has
been considered not a bill of sale. The registration certificate signed by the respondent spouses does not conclusively
prove that constructive delivery was made nor that ownership has been transferred to the respondent spouses.

In all forms of delivery, it is necessary that the act of delivery, whether constructive or actual , should be
coupled with the intention of delivering the thing. The act, without the intention, is insufficient.The critical
factor in the different modes of effecting delivery which gives legal effect to the act, is the actual intention of
the vendor to deliver, and its acceptance by the vendee. Without that intention, there is no tradition.

2B 16-17 SALES AND LEASE Page 218


10. SAN LORENZO DEVELOPMENT CORPORATION VS. COURT OF APPEALS, ET AL.

G.R. No. 124242, January 21, 2007

Tinga, J.

FACTS:
Spouses Lu owned two (2) parcels of land which were purportedly sold the two to respondent Pablo Babasanta.
Babasanta made a downpayment of fifty thousand pesos (P50,000.00) as evidenced by a memorandum receipt
issued by Pacita Lu of the same date.
Thereafter, several other payments totaling P200,000.00 were made by Babasanta.
Babasanta wrote a letter to Pacita Lu to demand the execution of a final de ed of sale in his favor so that he could
effect full payment of the purchase price. Babasanta notified the spouses about having received information that the
spouses sold the same property to another
Babasanta demanded that the second sale be cancelled an d that a final deed of sale be issued in his favor.
Babasanta filed a Complaint for Specific Performance and Damages alleging that the lands had been sold to him by
the spouses
Petitioner San Lorenzo Development Corporation (SLDC) filed a Motion for Intervention alleging that it had legal
interest in the subject matter under litigation because the two parcels of had been sold to it by Sps Yu. Babasanta
argued that the latter had no legal interest; two parcels of land involved herein had already been convey ed to him by
the Spouses Lu and hence, the vendors were without legal capacity to transfer or dispose of the two parcels of land

ISSUE:
Who between SLDC and Babasanta has a better right over the two parcels of land subject of the instant case in view
of the successive transactions executed by the Spouses Lu

HELD: (SLDC)

1.) The agreement between Babasanta and the Spouses Lu is a contract to sell and not a contract of sale.

While there is no stipulation that the seller reserves the ownership of the property until full payment of the
price which is a distinguishing feature of a contract to sell, the subsequent acts of the parties convince us that the
Spouses Lu never intended to transfer ownership to Babasanta except upon full payment of the purchase price.

Babasantas letter dated 22 May 1989 was quite telling. He stated therein that despite his repeated requests
for the execution of the final deed of sale in his favor so that he could effect full payment of the price, Pacita Lu
allegedly refused to do so. In effect, Babasanta himself recognized that ownership of the property would not be
transferred to him until such time as he shall have effected full payment of the price. Doubtlessly, the receipt signed
by Pacita Lu should legally be considered as a perfected contract to sell.

The distinction between a contract to sell and a contract of sale is quite germane. In a contract of sale, title
passes to the vendee upon the delivery of the thing sold; whereas in a contract to sell, by agreement the ownership is
reserved in the vendor and is not to pass until the full payment of the price.[22] In a contract of sale, the vendor has
lost and cannot recover ownership until and unless the contract is resolved or rescinded; whereas in a contract to
sell, title is retained by the vendor until the full payment of the price, such payment being a positive suspensive

2B 16-17 SALES AND LEASE Page 219


condition and failure of which is not a breach but an event that prevents the obligation of the vendor to convey title
from becoming effective.

2.) Assuming arguendo there has been a perfected contract of sale, there was still no transfer of ownership
to Babasanta

Sale, being a consensual contract, is perfected by mere consent and from that moment, the parties may
reciprocally demand performance.

The perfection of a contract of sale should not, however, be confused with its consummation. In relation to
the acquisition and transfer of ownership, it should be noted that sale is not a mode, but merely a title. A mode is the
legal means by which dominion or owners hip is created, transferred or destroyed, but title is only the legal basis by
which to affect dominion or ownership.[28] Under Article 712 of the Civil Code, ownership and other real rights
over property are acquired and transmitted by law, by donation, b y testate and intestate succession, and in
consequence of certain contracts, by tradition. Contracts only constitute titles or rights to the transfer or acquisition
of ownership, while delivery or tradition is the mode of accomplishing the same.[29] Therefore, sale by itself does
not transfer or affect ownership; the most that sale does is to create the obligation to transfer ownership. It is
tradition or delivery, as a consequence of sale, that actually transfers ownership.

Explicitly, the law provides that the ownership of the thing sold is acquired by the vendee from the moment
it is delivered to him in any of the ways specified in Article 1497 to 1501.[30] The word delivered should not be
taken restrictively to mean transfer of actual physical possessio n of the property. The law recognizes two principal
modes of delivery, to wit: (1) actual delivery; and (2) legal or constructive delivery.

Actual delivery consists in placing the thing sold in the control and possession of the vendee.[31] Legal or
constructive delivery, on the other hand, may be had through any of the following ways: the execution of a public
instrument evidencing the sale;[32] symbolical tradition such as the delivery of the keys of the place where the
movable sold is being kept;[33] traditio longa manu or by mere consent or agreement if the movable sold cannot yet
be transferred to the possession of the buyer at the time of the sale;[34] traditio brevi manu if the buyer already had
possession of the object even before the sale;[35] and traditio constitutum possessorium, where the seller remains in
possession of the property in a different capacity.[36]

Following the above disquisition, respondent Babasanta did not acquire ownership by the mere execution
of the receipt by Pacita Lu acknowledging receipt of partial payment for the property. For one, the agreement
between Babasanta and the Spouses Lu, though valid, was not embodied in a public instrument. Hence, no
constructive delivery of the lands could have been effected. For another, Bab asanta had not taken possession of the
property at any time after the perfection of the sale in his favor or exercised acts of dominion over it despite his
assertions that he was the rightful owner of the lands. Simply stated, there was no delivery to Baba santa, whether
actual or constructive, which is essential to transfer ownership of the property. Thus, even on the assumption that the
perfected contract between the parties was a sale, ownership could not have passed to Babasanta in the absence of
delivery, since in a contract of sale ownership is transferred to the vendee only upon the delivery of the thing sold

2B 16-17 SALES AND LEASE Page 220


11. FLORES vs. LIM

50 Phil 738

FACTS

Land involved is located in barrio Pinaninding, Laguimanoc, Tayabas owned by Isabel Flores (73 hectares, on which
were 164 coconut bearing trees and 1000 non bearing and about 300 buri trees). Jan 20 1923, Isabel Flores land was
sold to Trinidad Lim at sheriffs sale. Lim refused to render an account of the fruits, profits of the land. Lim took
actual, physical possession of the property even before the 1 yr period of redemption. Flores prays that the defendant
be ordered to render an itemized account, the amount of which should be deducted from the price of the redemption;
that she has the right to redeem and the defendant pay her P1000 as damage and cost. The trial court rendered
judgment in favor of Flores.

ISSUE

Whether or not Lim has become the owner of the property absent the delivery and within period of redemption.

HELD

A purchaser of real property at an ordinary execution sale is not entitled to possession of the land or the accruing
rents and profits until after the period of redemption has expired and the legal title to the land has become vested in
him.

The defendant had no legal right to poss ession of the land in question, and, hence, she was a trespasser from the time
she took possession during the whole period of redemption.

2B 16-17 SALES AND LEASE Page 221


12. TAMBUNTING vs. COURT OF APPEALS

GR No. L-48278 Nov. 9, 1988

Padilla, J.:

FACTS

On Dec. 16, 1959, private respondents obtained a loan of P3,600 from the Spouses Aurora and Antonio Tambunting,
evidenced by a promissory note payable in 4 months, with interest at 12% per annum. As security, a Deed of Real
Estate Mortgaged over a parcel of land owned by Cruz was registered in Rizal. Due to failure to pay upon maturity,
the respondents filed for extrajudicial foreclosure on Mar. 17, 1967. On Aug. 2, 1967, the Cruzes filed for an
annulment of the mortgage and for preliminary injunction over the scheduled sale. Petitioners published on Dec. 20
in the Rizal Chronicle, the sheriff's notice of scheduled sale. Petitioners argue that the CA erred in saying that they
failed to comply with the requirement of publication.

ISSUE

Whether or not petitioner complied with the requirements of publication

HELD

No. Act No. 3135 requires that notices of the sale be published for not less than 20 days in at least 3 conspicuous
places in the city or municipality where the property is located, and if the property exceeds P4000 in value, that the
notice shall be published once a week for at least 3 consecutive weeks. This rule must be strictly complied with.

2B 16-17 SALES AND LEASE Page 222


13. SPS. VICENTE PINGOL, ET AL. vs. COURT OF APPEALS,

G.R. No. 102909, September 6, 1993

Davide Jr, J

FACTS

Petitioner Vicente Pingol is the owner of Lot No. 3223 of the Cadastral Survey of Caloocan. He executed a "DEED
OF ABSOLUTE SALE OF ONE-HALF OF (1/2) [OF] AN UNDIVIDED PORTION OF A PARCEL OF LAND"
in favor of Francisco N. Donasco.

Francisco immediately took possession of the subject lot and constructed a house thereon. In January 1970, he
started paying the monthly installments but was able to pay only up to 1972. On 13 July 1984, Francisco Donasco
died. At the time of his demise, he had paid P8,369.00, plus the P2,000.00 advance payment, leaving a balance of
P10,161.00 on the contract price. Lot No. 3223-A remained in the possession of Donasco's heirs.

On 19 October 1988, the heirs of Francisco Donasco filed an action for "Specific Performance and Damages, with
Prayer for Writ of Preliminary Injunction" against the spouses Vicente and Lourdes Pingol (petitioners herein)
averring that after the death of their father, they offered to pay the balance of P10,161.00 plus the stipulated legal
rate of interest thereon to Vicente Pingol but the latter rebuffed their offer and has "been demanding for a bigger and
unreasonable amount, in complete variance to what is lawfully due and payable." They stated that they had "exerted
earnest efforts to forge or reach an amicable and peaceful settlement with the defendants" for the payment of the
property in question but to no avail. They further alleged that the defendants were committing "acts of forcible entry
and encroachment" upon their land and asked that a writ of preliminary injunction be issued to restrain the
defendants from the acts complained of.

Pingol, on the other hand, contended that (1) xxx (2) the deed of sale embodied a conditional co ntract of sale "as the
consideration is to be paid on installment basis within a period of six years beginning January, 1970"; (3) the
subdivision plan was prepared on the assumption that Francisco Donasco would be able to comply with his
obligation; (4) when Francisco died, he had not fully paid the total consideration agreed upon; and (5) considering
the breach by Francisco of his contractual obligation way back in 1976, the sale was deemed to have been cancelled
and the continuous occupancy of Francisco after 1976 and by his heirs thereafter was by mere tolerance of Vicente
Pingol.

In their Reply and Answer to Counterclaim, the plaintiffs pointed out that there is no provision in the deed of sale
for its cancellation in case of default in the payment of the monthly installments and invoked Article 1592 of the
New Civil Code.

ISSUE

Whether the contract is a contract of sale (if so, property belongs to respondent) or a contract to sell (if so, property
still belongs to petitioner)

HELD

Contract of sale.

2B 16-17 SALES AND LEASE Page 223


A perusal of Exhibit "A" leads to no other conclusion than that it embodies a contract of sale. The plain and clear
tenor of the "DEED OF ABSOLUTE SALE OF ONE-HALF (1/2) [OF] AN UNDIVIDED PORTION OF A
PARCEL OF LAND" is that "the VENDOR hereby . . . SELL, CONVEY AND CONVEY by way Absolute Sale
the one-half (1/2) portion . . . to the VENDEE . . . his heirs, assigns and successors -in-interest." That the vendor,
petitioner Vicente Pingol, had that clear intention was further evidenced by his failure to reserv e his title thereto
until the full payment of the price.

In Dignos vs. Court of Appeals, we held that a deed of sale is absolute in nature although denominated as a "Deed of
Conditional Sale" where there is no stipulation in the deed that title to the prop erty sold is reserved in the seller until
the full payment of the price, nor is there a stipulation giving the vendor the right to unilaterally resolve the contract
the moment the buyer fails to pay within a fixed period. Exhibit "A" contains neither stipu lation. What is merely
stated therein is that "the VENDEE agrees that in case of default in the payment of the installments due the same
shall earn a legal rate of interest, and to which the VENDOR likewise agrees."

Furthermore, as found by the Court of Appeals, the acts of the parties, contemporaneous and subsequent to the
contract, clearly show that an absolute deed of sale was intended, by the parties and not a contract to sell:

[P]ursuant to the deed, the vendor delivered actual and constructive possess ion of the property to the vendee, who
occupied and took such possession, constructed a building thereon, had the property surveyed and subdivided and a
plan of the property was prepared and submitted to the Land Registration Commission which approved it
preparatory to segregating the same and obtaining the corresponding TCT in his name. Since the sale, appellee
continuously possessed and occupied the property as owner up to his death on July 13, 1984 and his heirs, after his
death, continued the occupancy and possession of the property up to the present.

The delivery of the object of the contract divested the vendor of the ownership over the same and he cannot recover
the title unless the contract is resolved or rescinded pursuant to Article 1592 of the Ne w Civil Code. No notarial or
judicial rescission of the contract had been made.

2B 16-17 SALES AND LEASE Page 224


ON SALE ON TRIAL/SATIS FACTION

1. VICTORIA R. VALLARTA vs. COURT OF APPEALS,

G.R. No. L-40195, May 29, 1987

FACTS:

Rosalinda Cruz, the private offended party, and accused Victoria Vallarta are long time friends and business
acquaintances. On November 20, 1968, Cruz entrusted to Victoria Vallarta seven pieces of jewelry. In December of
the same year, Vallarta decided to buy some items, exchanged one item with another, and issued a post-dated check
in the amount of P5,000 dated January 30, 1969. Upon presentment of said check to the Security Bank, Cruz found
out it was dishonored and she was informed that Vallarta's account had been closed. Cruz apprised Vallarta of th e
dishonor and the latter promised to give another check. Later, Vallarta pleaded for more time. Still later, she started
avoiding Cruz. A criminal action for Estafa was instituted. Thereafter, the trial court found Vallarta guilty. The
decision was affirmed by the Court of Appeals. Petitioner seeks a reversal of the CA decision affirming the lower
courts judgment convicting her of estafa. In seeking acquittal, Vallarta stresses that the transaction between her and
Cruz was a "sale or return," perfected and consummated on November 20, 1968 when the seven pieces of jewelry
were delivered. The check issued in December 1968 was therefore in payment of a pre -existing obligation. Thus,
even if it was dishonored, petitioner claims that she can only be held civilly liable. Vallarta thus assigns as errors the
finding of that Court a quo that the jewelries were entrusted on November 20, 1968, but the sale was perfected in
December 1968, and the finding that there was deceit in the issuance of the postdated check.

ISSUE

Whether or not the transaction between Vallarta and Cruz was a sale or return

HELD

The transaction entered into by Cruz and Vallarta was a SALE ON APPROVAL (also called SALE ON
ACCEPTANCE, SALE ON TRIAL or SALE ON SATISFACTION under Art. 1502 (2), NCC)

The Supreme Court ruled that in order to arrive at the proper characterization of said transaction, it is necessary to
determine the intention of the parties.

Excerpts from Direct and Cross -Examinations of Cruz would indicate that:

o 1) Only in December 1968 that Vallarta finally agreed to buy two sets and changed the ruby ring with another
ring, where she gave Cruz a postdated check;

o 2) Although the pieces of jewelry were taken by Mrs. Vallarta at one instance, Cruz testified that sh e entrusted
them to her so she can select what (jewelry) she wants.

2B 16-17 SALES AND LEASE Page 225


o 3) Cruz agreed to reduce the cost to 5,800 php when she went to her house to finalize what jewelries Vallarta
wanted

The Court noted that Vallarta changed the ruby ring because it was not acceptable to her, and chose another ring.
Likewise, the price to be paid for the jewelry was finally agreed upon only in December 1968. Thus, there was a
meeting of the minds between the parties as to the object of the contract and the consideration therefore only
in December 1968, the same time that the check was issued. The delivery made on November 20, 1968 was only
for the purpose of enabling Vallarta to select what jewelry she wanted.

There was no meeting of the minds on November 20, 1968, then, as of that date, there was yet no contract of sale
which could be the basis of delivery or tradition.

The delivery made on November 20, 1968 was not a delivery for purposes of transferring ownership the
prestation incumbent on the vendor. If ownership ov er the jewelry was not transmitted on that date, then it could
have been transmitted only in December 1968, the date when the check was issued.

IT WAS A "SALE ON APPROVAL" SINCE OWNERSHIP PASSED TO THE BUYER, VALLARTA,


ONLY WHEN SHE SIGNIFIED HER APPROVAL OR ACCEPTANCE TO THE SELLER, CRUZ, AND
THE PRICE WAS AGREED UPON.

Finding no error in the assailed decision of the CA, the same is affirmed by the SC. The check which later bounced
was not issued in payment of a pre-existing obligation as petitioner claimed to exonerate herself from criminal
liabilities. Instead, the issuance of the check was simultaneous with the transfer of ownership over the jewelry.

** note: Sale or Return, the ownership passes to the buyer on delivery (CIVIL CODE, art. 1502). (The
subsequent return of the goods reverts ownership in the seller [CIVIL CODE, art. 1502]). Delivery, or tradition, as a
mode of acquiring ownership must be in consequence of a contract (CIVIL CODE, art. 712), e.g. sale. **

2B 16-17 SALES AND LEASE Page 226


ON LOSS OF THE THING SOLD

1. ROMAN vs. GRIMALT

6 Phil. 96

FACTS
On July 2, 1904, counsel for Pedro Roman filed a complaint in the Court of First Instance of this city against Andres
Grimalt praying that the latter pay for the price of Santa Marina 1.5k and for other remedies available. He alleges
that defendant agreed to buy his schooner in three installments to which he agreed and said that from that date, the
vessel was at defendantss disposal and offered to deliver such at his disposal.The ship sunk in the harbor of Manila
as a result of a storm. The same month demand was made upon defendant for the payment of the ship in the manner
stipulated by them.
Defendant claims that they did not have a contract because when he found out that the title of the vessel was in the
name of one Paulina Giron and not the Petitioner, defendant required petitioner to perfect his title before the former
would agree.

ISSUES

(1)Whether or not there is a perfected contract.


(2)Whether or not defendant should bear the loss of the sunken vessel.

HELD

(1)A sale shall be considered perfected and binding as between vendor and vendee when they have agreed as to the
thing which is the object of the contract and as to the price, even though neither has been actually delivered.

The sale of the schooner was not perfected and the purchaser did not consent to the execution of the deed of transfer
for the reason that the title of the vessel was not in the name of Pedro Roman, the alleged owner. Roman promised,
however, to perfect his title to the vessel, but he failed to do so. The papers presented by him did not show that he
was the owner of the vessel.

The vessel was sunk in the bay on the afternoon of the 25th of June, 1904, during a severe storm and before the
owner had complied with the condition exacted by the proposed purchaser.

(2).If no contract of sale was actually executed by the parties the loss of the vessel must be borne by its owner and
not by a party who only intended to purchase it and who was unable to do so on account of failure on the part of the
owner to show proper title to the vessel and thus enable them to draw up the contract of sale.

The defendant was under no obligation to pay the price of the vessel, the purchase of which had not been concluded.
The conversations had between the parties and the letter written by defendant to plaintiff did not establish a contract
sufficient in itself to create reciprocal rights between the parties.

2B 16-17 SALES AND LEASE Page 227


2. JOSE DE LEON, ET AL. vs. ASUNCION SORIANO

G.R. No. L-2724, August 24, 1950

Tuason, J.

FACTS:

Felix de Leon died in the City of Manila on November 23, 1940, leaving properties in Manila, Bulacan and Nueva
Ecija. He was survived by his wife Asuncion Soriano and by his three acknowledged natural children Jose P. de
Leon, Cecilio P. de Leon and Albina P. de Leon. On March 23, 1943, an amicable agreement was executed between
Asuncion Soriano on the one hand and the acknowledged natural children on the other, in which it is stipulated,
among other things, that the three acknowledged natural children would deliver to Asuncion Soriano certain
amounts of palay annually beginning the year 1943 and continuing during the lifetime of Asuncion Soriano. The
agreement also provides that said obligation constitutes a first lien upon all the rice lands of the estate of Felix de
Leon in San Miguel, Bulacan.

Defendant failed to delivwer the exact cavans of rice to plaintiff due to "the Huk troubles in Central Luzon which
rendered impossible full compliance with the terms of the agreement;" and it was contended that "inasmuch as the
obligations of the defendants to deliver the full amount of the palay is depending upon the produce as this is in the
nature of an annuity, . . . the obligations of the defendants have been fully fulfilled by delivering in good faith all
that could be possible under the circumstances."

ISSUE:

Whether or not plaintiiffs obligation was extinguish

HELD:

Article 1182 of the Civil Code which was in force at the time agreement in question was entered into, provide that
"Any obligation which consists in the delivery of a determinate thing shall be extinguished if such thing should be
lost or destroyed without fault on the part of the debtor and before he is in default. Inversely, the obligation is not
extinguished if the thing that perishes is indeterminate.

Except as to quality and quantity, the first of which is itself generic, the contract sets no bounds or limits to the palay
to be paid, nor was there even any stipulation that the cereal was to be the produce of any particular land. Any palay
of the quality stipulated regardless of origin on however acquired (lawfully ) would be obligatory on the part of the
obligee to receive and would discharge the obligation. It seems therefore plain that the alleged failure of crops
through alleged fortuitous cause did not excuse performance.

2B 16-17 SALES AND LEASE Page 228


3. LAWYERS COOPERATIVE PUBLISHING COMPANY vs.PERFECTO A. TABORA

G.R. No. L-21263 April 30, 1965

Bautista Angelo, J.

FACTS:

Appellant bought from appellee one set of American Jurisprudence, payable on installment plan. It was provided in
the contract that "title to and ownership of the books shall remain with the seller until the purchase price shall have
been fully paid. Loss or damage to the books after delivery to the buyer shall be borne by the buyer." In the midnight
of the same date, however, a big fire broke out in that locality which destroyed and burned all the buildings standing
on one whole block including at the law office and library of Tabora where he placed the books he bought from the
company.

The company, as a token of goodwill sent to Tabora free of charge 4 volumes of the Philippine Reports. Tabora,
however failed to pay the monthly installments agreed upon on the balance which prompted the company to file the
action for the recovery of the balance due. Plaintiff also prayed that defendant be ordered to pay 25% of the amount
due as liquidated damages, and the cost of action.

Appellant now contends that since it was agreed that the title to and the ownership of the books shall remain with the
seller until the purchase price shall have been fully paid, and the books were burned or destroyed immediately after
the transaction, appellee should be the one to bear the loss for, as a result, the loss is always borne by the owner.
Moreover, even assuming that the ownership of the books were transferred to the buyer after the perfection of the
contract the latter should not answer for the loss since the same occurred through force majeure.

ISSUE:

Whether or not the seller should bear the loss.

RULING:

NO.

While as a rule the loss of the object of the contract of sale is borne by the owner or in case of force majeure the one
under obligation to deliver the object is exempt from liability, the application of that rule does not here obtain
because the law on the contract entered into on the matter argues against it. It is true that in the contract entered into
between the parties the seller agreed that the ownership of the books shall remain with it until the purchase price
shall have been fully paid, but such stipulation cannot make the seller liable in case of loss not only because such
was agreed merely to secure the performance by the buyer of his obligation but in the very contract it was expressly
agreed that the "loss or damage to the books after delivery to the buyer shall be borne by the buye r." Any such
stipulation is sanctioned by Article 1504 of our Civil Code, which in part provides:

(1) Where delivery of the goods has been made to the buyer or to a bailee for the buyer, in pursuance of the contract
and the ownership in the goods has been retained by the seller merely to secure performance by the buyer of his
obligations under the contract, the goods are at the buyer's risk from the time of such delivery.

2B 16-17 SALES AND LEASE Page 229


Neither can appellant find comfort in the claim that since the books were destroyed by fire without any fault on his
part he should be relieved from the resultant obligation under the rule that an obligor should be held exempt from
liability when the loss occurs thru a fortuitous event. This is because this rule only holds true when the obligation
consists in the delivery of a determinate thing and there is no stipulation holding him liable even in case of fortuitous
event. Here these qualifications are not present. The obligation does not refer to a determinate thing, but is pecuniary
in nature, and the obligor bound himself to assume the loss after the delivery of the goods to him. In other words, the
obligor agreed to assume any risk concerning the goods from the time of their delivery, which is an exception to the
rule provided for in Article 1262 of our Civil Code.

2B 16-17 SALES AND LEASE Page 230


4. NORKIS DISTRIB UTORS INC. vs. COURT OF APPEALS, ET AL.

G.R. No. 91029 February 7, 1991

FACTS:

On September 20, 1979 private respondent Alberto Nepales bought a Yamaha Wonderbike Motorcycle from
petitioner Norkis Distributor Inc. (Norkis)-Bacolod Branch for P7,500.00.

The price was payable by means of a Letter of Guaranty from the Development Bank of the Philippines (DBP). As
security for the loan, Nepales would execute a chattel mortgage on the motorcyc le in favour of DBP.

Branch Manager Labajo issued Norkis Sales Invoice No. 0120 showing that the contract of sale of the motorcycle
had been perfected. Nepales signed the sales invoice to signify his conformity with the terms of the sale. In the
meantime, however, the motorcycle remained in Norkis' possession.

On November 6, 1979, the motorcycle was registered in the Land Transportation Commission (LTC) in the name of
Alberto Nepales. The registration fees were paid by Nepales, evidenced by an official rece ipt.

On January 22, 1980, the motorcycle was delivered to a certain Julian Nepales who was allegedly the agent of
Alberto Nepales but the latter denies it.

The motorcycle met an accident on February 3, 1980 at Binalbagan, Negros Occidental being driven by a certain
Zacarias Payba. The unit was a total wreck. It was returned, and stored inside Norkis' warehouse.

DBP paid Norkis the amount. Nepales demanded delivery of the motorcycle. When Norkis could not deliver, he
filed an action for specific performance with damages against Norkis.

Norkis answered that the motorcycle had already been delivered to private respondent before the accident. Norkis
concedes that while there was no "actual" delivery of the vehicle, there was constructive delivery of the unit upon:
(1) the issuance of the Sales Invoice No. 0120 in the name of the private respondent and the affixing of his signature
thereon (2) the registration of the vehicle with the LTC. and (3) the issuance of official receipt for payment of
registration fees. Hence, the risk of loss or damage had to be borne by him as owner of the unit.

RTC ruled in favour of Nepales. Court of Appeals affirmed the lower courts decision. Norkis was ordered to pay
Nepales the price of the motorcycle plus interest or deliver a brand new motorcycle of the same kind.

ISSUE

Whether or not Norkis should bear the loss of the motorcycle.

HELD

Yes, Norkis should bear the loss of the motorcycle!

Article 1496 of the Civil Code provides that "in the absence of an express assumption of risk by the buyer,
the things sold remain at seller's risk until the ownership thereof is transferred to the buyer.

The issuance of a sales invoice does not prove transfer of ownership of the thing sold to the buyer. An
invoice is nothing more than a detailed statement of the nature, quantity and cost of the thing sold and has been
considered not a bill of sale.

2B 16-17 SALES AND LEASE Page 231


In all forms of delivery, it is necessary that the act of delivery whether constructive or actual, be coupled
with the intention of delivering the thing. The act, without the intention, is insufficient.

When the motorcycle was registered by Norkis in the name of private respondent, Norkis did not intend yet
to transfer the title or ownership to Nepales, but only to facilitate the execution of a ch attel mortgage in favor of the
DBP for the release of the buyer's motorcycle loan. The Letter of Guarantee issued by the DBP, reveals that the
execution in its favor of a chattel mortgage over the purchased vehicle is a prerequisite for the approval of the
buyer's loan. If Norkis would not accede to that arrangement, DBP would not approve private respondent's loan
application and, consequently, there would be no sale.

There was neither an actual nor constructive delivery of the thing sold. Hence, the risk o f loss should be
borne by the seller, Norkis, which was still the owner and possessor of the motorcycle when it was wrecked. This is
in accordance with the well-known doctrine of res perit domino.

2B 16-17 SALES AND LEASE Page 232


5. YU TEK CO vs. GONZALES

29 Phil. 384

FACTS

Mr. Basilio Gonzalez received the sum of P3,000 Philippine currency from Messrs. Yu Tek and Co., and that in
consideration of said sum he obligates himself to deliver to the said Yu Tek and Co., 600 piculs of sugar of the first
and second grade, according to the result of the polarization, within the period of three months, beginning on the 1st
day of January, 1912, and ending on the 31st day of March of the same year, 1912.

That the said Mr. Basilio Gonzales obligates himself to deliver to the said Messrs. Yu Tek and Co., of this city the
said 600 piculs of sugar at any place within the said municipality of Santa Rosa which the said Messrs. Yu Tek and
Co., or a representative of the same may designate.

That in case the said Mr. Basilio Gonzales does not deliver to Messrs. Yu Tek and Co. the 600 piculs of sugar within
the period of three months, referred to in the second paragraph of this document, this contract will be rescinded and
the said Mr. Basilio Gonzales will then be obligated to return to Mes srs. Yu Tek and Co. the P3,000 received and
also the sum of P1,200 by way of indemnity for loss and damages.

Plaintiff proved that no sugar had been delivered to it under this contract nor had it been able to recover the P3,000.

Defendant assumes that the contract was limited to the sugar he might raise upon his own plantation; that the
contract represented a perfected sale; and that by failure of his crop(due to a storm) he was relieved from complying
with his undertaking by loss of the thing due. (Arts. 1452, 1096, and 1182, Civil Code.)

ISSUE

Whether there was loss of the thing due

HELD

This argument is faulty in assuming that there was a perfected sale. Article 1450 defines a perfected sale as follows:

The sale shall be perfected between vendor and vendee and shall be binding on both of them, if they have agreed
upon the thing which is the object of the contract and upon the price, even when neither has been delivered.

Article 1452 reads: "The injury to or the profit of the thing sold shall, after th e contract has been perfected, be
governed by the provisions of articles 1096 and 1182."

2B 16-17 SALES AND LEASE Page 233


This court has consistently held that there is a perfected sale with regard to the "thing" whenever the article of sale
has been physically segregated from all other articles. Thus, a particular tobacco factory with its contents was held
sold under a contract which did not provide for either delivery of the price or of the thing until a future time.

In the case at bar the undertaking of the defendant was to sell to th e plaintiff 600 piculs of sugar of the first and
second classes. Was this an agreement upon the "thing" which was the object of the contract within the meaning of
article 1450? Sugar is one of the staple commodities of this country. For the purpose of sale its bulk is weighed, the
customary unit of weight being denominated a "picul." There was no delivery under the contract. Now, if called
upon to designate the article sold, it is clear that the defendant could only say that it was "sugar." He could only us e
this generic name for the thing sold. There was no "appropriation" of any particular lot of sugar. Neither party could
point to any specific quantity of sugar and say: "This is the article which was the subject of our contract.

We conclude that the contract in the case at bar was merely an executory agreement; a promise of sale and not a
sale. At there was no perfected sale, it is clear that articles 1452, 1096, and 1182 are not applicable. The defendant
having defaulted in his engagement, the plaintiff is entitled to recover the P3,000 which it advanced to the
defendant. (since the subject matter is not determinate or specific, sugar being generic, there was no risk of it being
lost)

2B 16-17 SALES AND LEASE Page 234


6. BUNGE CORPORATION and UNIVERSAL COMMERCIAL AGENCIES, Vs ELENA CAMENFORTE
and COMPANY,

G.R. No. L-4440 August 29, 1952

Bautista Angelo, J

FACTS

Bunge Corp. (represented by the Universal Commercial Agencies) entered into a contract with Visayan
Products Company (VPC) on Oct. 22, 1947, whereby the latter sold to the former 500 long tons of merchantable
Philippine copra in bulk at the prices of $188.80, U.S. currency, per ton, less 1 per cent brokerage per short ton of
2,000 pounds, C & F Pacific Coast, U.S.A..

According to the terms and conditions of the contract, the vendor should ship the stipulated copra during the
month of November or December 1947, to San Francisco, California, U.S.A. for delivery to the vendee.

Notwithstanding repeated demands made by the vendee, the vendor failed to ship and deliver the copra during
the period agreed upon.

Bunge Corp.,believing in good faith that the vendor would ship and deliver the copra on time, sold to El
Dorado Oil Works the quantity of copra it had purchased at the same price agreed upon.

However, due to the failure of the vendor to fulfill its contract to ship and deliver the quantity of copra agreed
upon within the period stipulated, the vendee has suffered damages in the amount of P180,00.

VPC contented that NO CONTRACT OF SALE (COS) was perfected.If any, it was that signed by Vicente
Kho,the manager and controlling stickholder in VPC Tacloban. Vicente Kho who signed for and in behalf of the
company never had any authority to act for that company either expressly or impliedly, inasmuch as the only ones
who had the authority to do so are Elena Camenforte, the general manager, Tan Se Chong, the manager, and Tiu
Kee, the assistant manager.

Kho admitted that he signed the contracts and tried his best to deliver the said copra but due to force majeure,
he failed to do so. Lower court ordered for payment of damages by VPC to Bunge Corp.

Upon appeal, VPC now admitted that a COS was entered into, alleging as a defense that the copra they had
gathered and stored for delivery to the appellees in Samar was destroyed by force majeure which under the law has
the effect of exempting them from liability for damages. Thus, they contend that the lower court erred in
condemning them for damages despite the fact that their failure to fulfill the contract is due to force majeure.

ISSUES

1. Whether or not there was a perfected COS?


2. Whether or not VPC is liable for failure to deliver copra even if lost by force majeure

HELD

2B 16-17 SALES AND LEASE Page 235


Yes. VPC is liable for failure to deliver the copra, a generic thing notwithstanding the fact that it was lost by force
majeure.

A perusal of this contract shows that the subject matter is Philippine copra. The sale is to be made by weight, 500
long tons. It does not refer to any particular or specific lot of copra, nor does it mention the place where the copra is
to be acquired. No portion of the copra has been earmarked or segregated. The vendor was at liberty to acquire the
copra from any part of the Philippines. The sale simply refers to 500 long tons of the Philippine copra. The subject-
matter is, therefore, generic, not specific.

It appearing that the obligation of appellant is to deliver copra in a generic sense, the obligation cannot be deemed
extinguised by the destruction or disappearance of the copra stored in San Ramon, Samar. Their obligation subsists
as long as that commodity is available. A generic obligation is not extinguished by the loss of a thing belonging to a
particular genus. Genus nunquan perit.

2B 16-17 SALES AND LEASE Page 236


7. PHILIPPINE VIRGINIA TOBACCO ADMINISTRATION vs. DE LOS ANGELES

G.R. No. L-33079 December 11, 1978

Fernando, J

FACTS

On July 24, 1963, a fire occurred in the redrying plant of petitioner Philippine Virginia Tobacco Administration
(PVTA for brevity) at Agoo, La Union, as a result of which private respondents suffered losses arising from the sale
and delivery of tobacco to Central Cooperative Exce, Inc., (CCE for brevity), the authorized agent of petitioner. The
tobacco shipments were authorized to be unloaded and awaiting inspection, grading, and weighing when they were
burned on July 24, 1963.

In its answer the defendant PVTA, now the petitioner, alleged that there was no inspection or acceptance of tobacco
shipments by it and the CCE.

Plaintiffs in the lower court, now respondents, made demands for the payment of their shipments but these demands
were ignored.

ISSUE

Who should bear the loss of tobacco due to the fire accident? Specifically, the question that arises is whether the
PVTA is liable to pay therefor and bear the loss considering that fact that the said tobacco shipments were still to be
inspected, graded, and weighed to determine the class and compensation therefor. In other words, were the tobaccos
legally delivered to and accepted by the PVTA?

HELD

In upholding the judgment holding PVTA liable, the Supreme Court provided reasons as follows:

1. It bears repeating that the trial court was satisfied as to the fact of delivery of the tobacco in question at the
redrying plant of petitioner agent, the CCE. It was also found by it that the PVTA directed supervised and controlled
the CCE in receiving shipments of tobacco and in the performance of its activities, and that the tobacco, once
received from the trading entities, were under its control, not subject to withdrawal without its authority. The
procedure was so carefully designed that the supervision by it could be rendered most effective. Thus any attempt to
exculpate itself thereafter on alleged deficiencies could succeed only if the evidence offered by petitioner were of
such a nature as to justify evasion of what is required by law no less than by morality. Clearly, proof of such
character was lacking in this case.

2. In this petition for review, the PVTA would assail the judgment reached on the allegation that the contract of sale
was not perfected. Such an assertion, on the face of the facts as found, would appear to be clearly untenable.
Nonetheless, it was sought to lend it plausibility in the eight-page brief of petitioner by the argument that the
shipments of the tobacco in question "were still to be inspected, graded and weighed." Such a contention certainly
cannot suffice to overturn the decision. For one thing, it raised an issue of fact, the ruling on which, as could be
expected, was adverse to petitioners. For its own fieldmen had the responsibility of such tobacco being graded,

2B 16-17 SALES AND LEASE Page 237


weighed, baled and loaded on trucks duly sealed for transportation to its redrying plant. That responsibility was
fulfilled as found by the trial court. The grading was done according to the standards on samples provided by
petitioner. The shipping documents were in order. The weight and grades of such tobacco were certified by such
fieldmen and thereafter processed by its provincial tobacco agent. It was only then that clearance was given, the
PVTA requirements having been met. The futility of the effort to deny the perfection of the contract of sale is thus
rather apparent. So it has been from Irureta v. Tambunting, a 1902 decision. All that was required was t hat there be
an agreement on the thing which is the subject of the contract and upon the price. So it was provided by Article 1450
of the Civil Code of Spain of 1889 then in force. There is difference in phraseology but not in meaning under the
present-Civil Code: "The contract of sale is perfected at the moment there is a meeting of minds upon the thing
which is the object of the contract and upon the price."

3. Smith Bell and Company v. Jimenez, 19 decided in 1963, comes to mind. In that case, there was a delivery by
petitioner of a typewriter upon requisition of the Municipality of Paniqui, Tarlac, but ten days thereafter, the
municipal building was totally razed by a fire. Notwithstanding the fact that the Municipal Treasurer, as well as the
Provincial Treasurer of Tarlac recommended payment, respondent Auditor disapproved the claim on the ground that
the article in question was never presented for inspection and verification, Justice Barrera, speaking for the Court,
after noting that there was indeed such delivery, stated that even on the assumption then that not all the terms of the
contract as to inspection were fully complied with, "yet upon the facts obtaining in this case, we believe that
injustice would be done the petitioner if we apply said principle to the present claim ." He stressed both "the law and
equity of the case [in holding that] the municipality of Paniqui is legally bound to pay for the price of the typewriter
involved herein and, therefore, the decision of the Auditor General is hereby reversed ." In La Fuerza, Inc. v. Court
of Appeals, this Court, through the then Chief Justice Concepcion, stressed the doctrine that the decisive factor is the
delivery of the thing sold. So that it is placed in the control and possession of the vendee. This was what happened in
this case. The liberality with which this Court views the stage of perfection in a contract of sale is likewise manifest
in Republic v. Lichauco, where this Court held that there could be a valid and binding agreement providing fo r sale
of property yet to be adjudicated by the court. Only thus may the law be infused with the highest concept of equity
and fair dealing. As it was in those cases, so it should be now.

Justice Teehankee and Justice Makasiar concurred in the dissent of Justice Aquino, to wit:

The judgment (of the trial court) is erroneous. At the time the tobacco was burned, the ownership thereof had not yet
passed to the PVTA. The tobacco was still owned by the sixteen plaintiffs or sellers. The CCE was merely an agent
of the PVTA. Even as agent, it had not yet accepted delivery of the tobacco before it was lost during the fire. There
was no acceptance of delivery because the tobacco, at the time it was lost, had not yet been properly inspected,
graded and weighed.

Inasmuch as the PVTA did not become the owner of the lost tobacco and as the sixteen trading entities were still the
owners thereof, the loss should be borne by them, not by the PVTA. Res perit domino. Hence, the PVTA was not
obligated to pay for the tobacco (Roman vs. Grimalt, 6 Phil. 96; Yu Tek & Co. vs. Gonzalez, 29 Phil. 384)

2B 16-17 SALES AND LEASE Page 238


8. ALLIANCE TOBACCO, INV. vs. PHILIPPINE VIRGINIA TOBACCO ADMINISTRATION,

179 SCRA 336

FACTS

The Philippine Virginia Tobacco Administration (PVTA) entered into a contract of proc uring, redrying and
servicing with the Farmers Virginia Tobacco Redrying Company, Inc. (FVTR) for the 1963 tobacco trading
operation. In June of that year, the PVTA also entered into a merchandising loan agreement with the petitioner
whereby the PVTA agreed to lend P25,500 to the petitioner for the purchase of flue-cured Virginia tobacco from
bona fide Virginia tobacco farmer-producers. The following month, petitioner shipped to the FVTR 96 bales of
tobacco covered by Guia No. 1 and and 167 bales covered by Guia No. 2.

Upon the arrival of the tobacco shipments in the Redrying Plant, defendant FVTR, at Bauang, La Union (Bauang for
short), they were listed in the Log Book, after which the tobacco were brought inside the Redrying Compound. After
several days, the grading of the plaintiffs tobacco took place but only 89 bales from Guia No. 2 were graded,
weighed and accepted. The operations of defendant FVTR in Bauang, stopped in October 1963. The plaintiff asked
that its ungraded and unweighted tobacco be with drawn from the Redrying Plant. The defendants PVTA and FVTR
refused to allow the plaintiffs request because according to them the tobacco sought to be withdrawn were subject
of a merchandising loan and owned by defendant, PVTA.

Unfortunately, the remaining ungraded and unweighted 174 bales with a total value of P28,382 were lost while they
were in the possession of the FVTR. Having learned of such loss in 1965, petitioner demanded for its value and the
application of the same to its merchandising loan with PVTA but both the latter and the FVTR refused to heed said
demands. Consequently, petitioner filed in the then Court of First Instance of La Union a complaint against PVTA
and FVTR praying that the two defendants be ordered to pay the value of the 89 bales which were weighed, graded
and accepted by the defendants and the value of the lost bales of tobacco and/or that the said amount be applied to
its loan with PVTA.

In its decision, the lower court, ruled that the PVTA should not be held responsible for the lost bales of tobacco
because they were not yet properly graded and weighed and that petitioner failed to present the weighers tally
sheets and warehouse receipts or quedans. Petitioner appealed to the then Intermediate Appellate Court which, in its
decision of March 20, 1984,11 affirmed in toto the lower courts decision.

ISSUE

Whether or not petitioners delivery of 174 bales of tobacco to the FVTR, a contractee of the PVTA, perfected the
contract of sale between petitioner and the (PVTA) so as to h old the latter liable for the loss of said bales while in
the possession of the FVTR.

HELD

YES. The Civil Code provides that ownership of the thing sold shall be transferred to the vendee upon the actual or
constructive delivery thereof. There is delivery when the thing sold is placed in the control and possession of the
vendee. Indeed, in tobacco trading, actual delivery plays a pivotal role. The peculiar procedure undergone in trading
reveals that delivery seals the contract of sale because the trader los es not only possession but also control over the
shipment. While under an ideal situation, there would have been merit in respondent PVTAs contention that the
contract of sale could not have been perfected pursuant to Article 147522 of the Civil Code beca use to determine the
price of the tobacco traded, the shipment should first be inspected, graded and weighed, the said contention is
misplaced herein. A strict interpretation of the provision of Article 1475 may result in adverse effects to small

2B 16-17 SALES AND LEASE Page 239


planters who would not be paid for the lost products of their toil. Since PVTA had virtual control over the lost
tobacco bales, delivery thereof to the FVTR should also be considered effective delivery to the PVTA.

Notes.Possession is acquired by the material occupation of a thing or the exercise of a right or by the fact it is
subject to the action of our will, or by the proper acts and legal formalities established for acquiring such right.

2B 16-17 SALES AND LEASE Page 240


ARTICLES 1505 AND 1506 IN RELATION TO ARTICLE 55 9

1. JOSE R. CRUZ vs. REYNALDO PAHATI, ET AL.

G.R. No. L-8257, April 13, 1956

Bautista Angelo, J

FACTS

Factual Finding of the CFI:

(NB: These facts show that the letter was falsified by Belizo to enable him to sell the car to Bulahan for a valuable
consideration)

Nothern Motors, Inc. originally owned the automobile in question which it later sold to Chinaman Lu Dag.
This Chinaman sold it afterwards to Jesusito Belizo (who was then a dealer in second hand cars) and the latter in
turn sold it to Cruz.

One year thereafter, Belizo offered to Cruz to sell the same automobile as Belizo claimed that he has a
buyer for it. Cruz agreed. At that time, Cruz certificate of registration was missing so he wrote a letter addressed to
the Motor Section of the Bureau of Public Works for the issuance of a new registration certificate alleging as reason
the loss of the one previously issued to him and stating that he was intending to sell his car.

This letter was delivered to Belizo on March 3, 1952. He also turned over Belizo the automobile on the
pretext that Belizo will show it to a prospective buyer. On March 7, 1952, the letter was falsified and converted into
an authorized deed of sale in favor of Belizo by erasing a portion thereof and adding in its place the words " sold the
above car to Mr. Jesusito Belizo of 25 Valencia, San Francisco del Monte, for Five Thousand Pesos (P5,000)."

Armed with this deed of sale, Belizo succeeded in obtaining a certificate of registration in his name on the
same date, March 7, 1952, and also on the same date, Belizo sold the car to Felixberto Bulahan who in turn sold it to
Reynaldo Pahati, a second hand car dealer.

Pahati admitted having bought the automobile from Bulahan which he paid in check. When the Manila
Police Department impounded the automobile, he cancelled the sale and stopped the payment of the check and as a
result he returned the automobile to Bulahan who in turned surrendered the check for cancellation

Bulahan on his part claims that:

1. He acquired the automobile from Jesusito Belizo for value and without having any knowledge of any
defect in the title of the latter

2. Cruz had previously acquired title to said automobile by purchase from Belizo as evidenced by a deed
of sale executed to that effect;

3. Later Cruz delivered the possession of the automobile to Belizo for resale and to facilitate it he gave
the latter a letter of authority to secure a new certificate of registration in his name (plaintiff's) and that by

2B 16-17 SALES AND LEASE Page 241


having clothed Belizo with an apparent ownership or authority to sell the automobile, plaintiff is now
estopped to deny such ownership or authority.

Therefore, he claimed that between the two innocent parties, he who gave occasion, through his conduct, to the
falsification committed by Belizo, should be the one to suffer the loss and this one is the plaintiff.

Decision of the CFI: Both were found by the lower court to be innocent and to have acted in good faith. They were
found to be the victims of Belizo who falsified the letter given him by plaintiff to enable him to sell the car of
Bulahan for profit.

The court declared Bulahan entitled to the automobile and consequently ordered Cruz to return it to Baluhan and,
upon his failure to do so, to pay him the sum with legal interest from the date of the decision. The claim for damages
and attorney's fees of Bulahan was denied. Defendant Belizo was however ordered to indemnify the plaintiff in the
amount of P4,900 and pay the sum of P5,000 as moral damages. The counterclaim of defendant Pahati was denied
for lack of evidence.

Hence, this action before the Court.

Contention of the Defendant: The common law principle that "Where one of two innocent parties must suffer by a
fraud perpetrated by another, the law imposes the loss upon the party who, by his misplaced confidence, has enabled
the fraud to be committed" must be relied on and contends that, as between plaintiff and Bulahan, the former should
bear the loss because of the confidence he reposed in Belizo which enabled the latter to commit the falsification.

ISSUE

Who has a better right of the two (Cruz or Baluhan) over the car?

HELD

Cruz has a better right.

The law applicable to the case is Article 559 of the new Civil Code which provides:

ART. 559. The possession of movable property acquired in good faith is equivalent to a title.
Nevertheless, one who has lost any movable or has been unlawfully deprived thereof, may recover
it from the person in possession of the same.

2B 16-17 SALES AND LEASE Page 242


If the possessor of a movable lost or of which the owner has been unlawfull y deprived, has
acquired it in good faith at a public sale, the owner cannot obtain its return without reimbursing
the price paid therefor.

It appears that "one who has lost any movable or has been unlawfully deprived thereof, may recover it from the
person in possession of the same" and the only defense the latter may have is if he "has acquired it in good faith at a
public sale" in which case "the owner cannot obtain its return without reimbursing the price paid therefor."

Article 1505, of the same Code provides that:

"where goods are sold by a person who is not the owner thereof, and who does not sell them under
authority or with the consent of the owner, the buyer acquires no better title to the goods than the
seller had, unless the owner of the goods is by his conduct precluded from denying the seller's
authority to sell.

Applying the above legal provisions to the facts of this case, one is inevitably led to the conclusion that Cruz has a
better right to the car in question than Bulahan for it cannot be disputed that Cruz had been illegally deprived
thereof because of the ingenious scheme utilized by Belizo to enable him to dispose of it as if he were the owner
thereof.

Cruz therefore can still recover the possession of the car even if defendant Bu lahan had acted in good faith in
purchasing it from Belizo. Nor can it be pretended that the conduct of plaintiff in giving Belizo a letter to secure the
issuance of a new certificate of registration constitutes a sufficient defense that would preclude rec overy because of
the undisputed fact that that letter was falsified and this fact can be clearly seen by a cursory examination of the
document. If Bulahan had been more diligent he could have seen that the pertinent portion of the letter had been
erased which would have placed him on guard to make an inquiry as regards the authority of Belizo to sell the car.
This he failed to do.

On the defendants contention, the Court held that the principle invoked cannot be applied to this case which is
covered by an express provision of our new Civil Code. Between a common law principle and a statutory provision,
the latter must undoubtedly prevail in this jurisdiction. Moreover we entertain serious doubt if, under the
circumstances obtaining, Bulahan may be considered more innocent than the plaintiff in dealing with the car in
question. We prefer not to elaborate on this matter it being necessary considering the conclusion we have reached.

Disposition: Wherefore, the decision appealed from is reversed. The Court declares plaintiff to be entitled to
recover the car in question, and orders defendant Jesusito Belizo to pay him the sum of P5,000 as moral damages,
plus P2,000 as attorney's fees. The Court absolves defendant Bulahan and Pahati from the complaint as regard s the
claim for damages, reserving to Bulahan whatever action he may deem proper to take against Jesusito Belizo. No
costs.

2B 16-17 SALES AND LEASE Page 243


2. CHUA HAI vs. HON. RUPERTO KAPUNAN,

G.R. No. L-11108, June 30, 1958

FACTS
Ong Shu, owner of Youngstown Hardware, sold 700 corrugated galvanized iron sheets to Roberto Soto.
Soto failed to make payment on the price of the sheets. Soto sold 100 of said sheets to Chua Hai (petitioner).
Ong Shu charged Soto with estafa. With the case was pending, the 700 sheets were deposited with Ma nila Police
District. Ong Shu filed a motion under Art. 105 of RPC (restitution) for the release of the sheets to him. Chua Hai
opposed the motion, but the trial court granted Shus motion.
Hai contends that he was deprived of the due process of law since restitution can only happen upon final judgment,
which in this case was wanting because the case was still pending.

ISSUE
Whether or not the trial court was correct in granting the motion of Shu for the release of the sheets.

HELD
No. Mere filling of a charge of estafa does not mean that estafa was indeed committed. Civil liability of the offender
to make restitution does not arise until his criminal liability has been declared.
It cannot be assumed at this stage of the proceedings that respondent Ong Sh u is still the owner of the
property to do so it take for granted that the estafa was in fact committed, when so far, the trial on the merits has not
even started, and the presumption of innocence holds full sway.
The facts do not justify a finding that the owner Ong Shu was illegally deprived of the iron sheets, at least
in so far as appellant was concerned. It is not denied that Ong Shu delivered the sheets to Soto upon a perfected
contract of sale, and such delivery transferred title or ownership to th e purchaser. Art1496. The ownership of the
thing sold is acquired by the vendee from the moment it is delivered to him in any of the ways specified in articles
1497 to 1501, or in any other manner signifying an agreement that the possession is transferre d from the vendor to
the vendee.
The failure of the buyer to make good the price does not, in law, cause the ownership to revest in the seller
until and unless the bilateral contract of sale is first rescinded or resolved pursuant to Article 1191 of the new Civil
Code.
And, assuming that the consent of Ong Shu to the sale in favor of Sotto was obtained by the latter through
fraud or deceit, the contract was not thereby rendered void ab initio, but only voidable by reason of the fraud.
ART. 1390. The following contracts are voidable or annullable, even though there may have been no damage to the
contracting parties:
(1) Those where one of the parties is incapable of giving consent to a contract
(2) Those where the consent is vitiated by mistake, violence, intimidation, undue influence or fraud.
These contracts are binding, unless they are annulled by a proper action in court. They are susceptible of ratification.
(C.C.)
Agreeably to this provision, Article 1506 prescribes:
ART. 1506. Where the seller of goods has a voidable title thereto, but his title has not been avoided at the time of the
sale, the buyer acquires a good title to the goods, provided he buys them in good faith, for value, and without notice
of the seller's defect of title. (C.C.)
Hence, until the contract of Ong Shu with Sotto is set aside by a competent court (assuming that the fraud is
established to its satisfaction), the validity of appellant's claim to the property in question cannot be disputed, and his
right to the possession thereof should be respected.

2B 16-17 SALES AND LEASE Page 244


3. VICENTE TAN ET AL. v. COURT OF APPEALS

G. R. No. 90365 March 18, 1991

Sarmiento, J.

FACTS:
June 15, 1974, Tan was arrested by the military authorities pursuant to an Arrest, Search and Seizure Order (ASSO)
on the basis of investigation Service for alleged irregular transactions at Continental Bank. The bank's chairman of
the board, Cornelio Balmaceda, and its President, Jose Moran, were not arrested, and in fact continued to run the
operations of the bank.

Because of a possible bank run as a result of the arrests, the officers of Continental Bank requested an emergency
loan to meet pending withdrawals of depositors. The Monetary Board approved the request. On June 24, 1974, the
Director of petitioner's department of Commercial and Savings Banks, after conducting said verification, reported
that Continental Bank's assets cannot meet its liabilities. On the basis of the report, petitioner ordered the closure of
Continental Bank and designated the Director of its Department of Co mmercial and Savings Banks as receiver with
instructions to take charge of the bank's assets pursuant to Sec. 29 of R.A. No. 265. The same report indicated,
however, that the bank may be allowed to reorganize under an entirely new management subject to cer tain
conditions foremost of which was the infusion of fresh funds into the bank.

While still under detention by the military, respondent Tan executed certain agreements on February 2, 1977, May
12, 1977 and July 5, 1977 transferring and assigning 359,615 shares of stock in Continental Bank, as well as other
properties belonging to him and his affiliate firms in consideration of the assumption by these assignees of the
liabilities and obligations of respondents Tan and his companies.

The assignees of respondents Tan and his companies rehabilitated Continental Bank, Tan wrote the petitioner that
they have no objection to the reopening and rehabilitation of Continental Bank under its new name, International
Corporate Bank or Interbank.

Interbank reopened in 1977 and since then operated as a banking institution with controlling ownership thereof
changing hands during the past decade.

On January 13, 1987, after the lapse of more than twelve (12) years, private respondents filed the present case of
reconveyance of shares of stock with damages and restraining order before the respondent court but was dismissed
on the grounds that the action is barred by the statute of limitations or prescription and that plaintiffs therein (private
respondents herein) have no cause of action against the defendant (herein petitioner), as well as laches on the part of
plaintiffs. On April 1, 1987, private respondents filed their Opposition to the Motion to Dismiss to which petitioner
filed its Reply dated April 10, 1987.

ISSUE: Whether or not Tan has cause of action against the respondent for the reconveyance of his shareholdings in
the former Continental Bank.

HELD: NO

On the issue of cause of action, the Court of Appeals is of the opinion that the complaint states no cause of a ction,
since the Central Bank is not one of the assignees of the shares the petitioners are seeking to recover, and hence, no

2B 16-17 SALES AND LEASE Page 245


reconveyance is possible against it.

The Court notes that as the complaint itself avers, the petitioners' shares in the Continent al Bank were assigned to
the firms which Herminio Disini allegedly controlled, and not to the Central Bank. It is therefore fairly obvious that
if any claim for reconveyance may be prosecuted, it should be prosecuted against the Disini companies.

It is true that the Central Bank is alleged to be the "indirect owner," arising from certain loans supposedly facilitated
by the Bank that enabled yet two other companies, the National Development Company and the American Express
Bank, to acquire about ninety-nine percent of International Corporate Bank, subject to the conditionality that any
transfer of shares shall be approved by the Central Bank. Clearly, however, if the Central Bank were "owner"
which as we shall see, it is notit is "owner" only because it is preserving its money exposure to the National
Development Corporation and the American Express Bank. It is not "owner" for reconveyance purposes.

2B 16-17 SALES AND LEASE Page 246


4. JOSE B. AZNAR vs. RAFAEL YAPDIANGCO

G.R. No. L-18536 March 31, 1965

FACTS:

Teodoro Santos advertised the sale of his FORD FAIRLANE 500. L. De Dios went to the Santos residence who told
Irineo Santos, Teodoro Santos son that his uncle, Vicente Marella, was interested to buy the advertised car. Marella
agreed to buy the car for P14,700.00 on the understanding that the price would be paid only after the car had been
registered in his name.

The deed of the sale for the car was then executed in Marella's favor. Thereafter, the registration of the car in
Marella's name was effected. Up to this stage of the transaction, the purchased price had not been paid.

Vicente Marella who said that the amount he had on hand then was short by some P2,000.00 and begged off to be
allowed to secure the shortage from a sister. He ordered L. De Dios to go to the said sister and suggested that Irineo
Santos go with him. He requested the registration papers and the deed of sale from Irineo Santos on the pretext that
he would like to show them to his lawyer. Irineo handed over the same to the latter and proceeded to the alleged
house of Marella's sister with L. De Dios and another unidentified person.

At the supposed sisters house, Irineo Santos and L. De Dios alighted from the car and entered the house while their
unidentified companion remained in the car. L. De Dios asked Irineo Santos to wait at the sala while he went inside
a room. After waiting in vain for De Dios, Irineo discovered that neither the car nor their unidentified companion
was there anymore. Irineo Santos rushed to see Marella. He found the house closed and Marella gone. His father
promptly advised the police authorities.

That very same day Vicente Marella was able to sell the car to Jose B. Aznar, for P15,000.00 who acquired the car
from Vicente Marella in good faith, for a valuable consideration an d without notice of the defect appertaining to the
vendor's title.

Agents of the Philippine Constabulary seized and confiscated the car from Jose B. Aznar.

Jose B. Aznar filed a complaint for replevin against Captain Rafael Yapdiangco, the head of the Philippine
Constabulary unit which seized the car in question Claiming ownership of the vehicle.

The lower court rendered a decision awarding the car to Teodoro Santos and ruled that Teodoro Santos had been
unlawfully deprived of his personal property by Vicente Marella.

From this decision, Jose B. Aznar appeals.

ISSUE

Whether or not Teodoro Santos has a better right over Jose B. Aznar to the possession of the automobile. YES

HELD

The plaintiff-appellant accepts that the car in question originally belonged to a nd was owned by the intervenor-
appellee, Teodoro Santos, and that the latter was unlawfully deprived of the same by Vicente Marella. However, the

2B 16-17 SALES AND LEASE Page 247


appellant contends that upon the facts of this case, the applicable provision of the Civil Code is Article 1506 and not
Article 559 as was held by the decision under review.

The contention is clearly unmeritorious. Under Art. 1506, it is essential that the seller should have a voidable title at
least. It is very clearly inapplicable where, as in this case, the seller had no title at all.

Vicente Marella did not have any title to the property under litigation because the same was never delivered to him.
He sought ownership or acquisition of it by virtue of the contract. Vicente Marella could have acquired ownership or
title to the subject matter thereof only by the delivery or tradition of the car to him.

Under Article 712 of the Civil Code, "ownership and other real rights over property are acquired and transmitted by
law, by donation, by testate and intestate succession, and in consequence of certain contracts, by tradition." As
interpreted by this Court in a host of cases, by this provision, ownership is not transferred by contract merely but by
tradition or delivery. Contracts only constitute titles or rights to t he transfer or acquisition of ownership, while
delivery or tradition is the mode of accomplishing the same.

In the case on hand, the car in question was never delivered to the vendee by the vendor as to complete or
consummate the transfer of ownership by virtue of the contract. It should be recalled that while there was indeed a
contract of sale between Vicente Marella and Teodoro Santos, the former, as vendee, took possession of the subject
matter thereof by stealing the same while it was in the custody of the latter's son.

There is no adequate evidence on record as to whether Irineo Santos voluntarily delivered the key to the car to the
unidentified person who went with him and L. De Dios to the place on Azcarraga where a sister of Marella allegedly
lived. But even if Irineo Santos did, it was not the delivery contemplated by Article 712 of the Civil Code. For then,
it would be indisputable that he turned it over to the unidentified companion only so that he may drive Irineo Santos
and De Dios to the said place on Azcarraga and not to vest the title to the said vehicle to him as agent of Vicente
Marella. Article 712 above contemplates that the act be coupled with the intent of delivering the thing.

2B 16-17 SALES AND LEASE Page 248


ON ARTICLES 1539, 1541 AND 1542

1. RUDOLF LIETZ VS CA

G.R. No. 122463 December 19, 2005

FACTS:

Respondent Agapito Buriol owned a parcel of unregistered land situated at Capsalay Island, Port Barton, San
Vicente, Palawan. Buriol leased 1 hectare of said land to Italians Turatello and Sani. Later, he so ld the same parcel
of land to Petitioner Rudolf Lietz. The Deed of Absolute Sale states that it is a parcel of land, consisting of Five (5)
hectares, situated in San Vicente, Palawan with the following boundaries, NORTH, EAST. It was sold for
P30,000. Lietz discovered that Buriol owned only 4 hectares and 1 hectare was even subject of lease, therefore only
3 hectares were delivered. Lietz filed for recovery of possession against lessees Turatello. He also prayed for the
restoration of the amount paid by him in excess of the value of the property sold to him. Both RTC & CA ruled that
it is a sale of land in lump sum and Lietz is not entitled to a proportionate reduction of the contract price.

ISSUE:

Is it a sale of immovable by unit under Art. 1539 or a s ale of immovable by lump sum under 1542?

HELD

It is a sale by lump sum. Where both the area and the boundaries of the immovable are declared, the area covered
within the boundaries of the immovable prevail over the stated area. In cases of conflict betwee n areas and
boundaries, boundaries should prevail. What really defines a piece of ground is not the area, calculated with more or
less certainty, mentioned in its description, but the boundaries therein laid down, as enclosing the land and
indicating its limits.

The Sale between Lietz and Buriol is one of lump sum. The deed of Sale shows that the parties agreed on the
purchase price on a predetermined area of five hectares within the specified boundaries and not based on a particular
rate per area. In accordance with Art. 1542, there shall be no reduction in the purchase price even if the area
delivered to petitioner is less than that stated in the contract. Also, the leased land is not part of the land sold.

2B 16-17 SALES AND LEASE Page 249


2. CEBU WINLAND DEVELOPMENT CORP VS ONG SIAO HUA

G.R. No. 173215, May 21, 2009

FACTS:

Cebu Winland Development Corporation sold two (2) Condominium Units and 4 Parking Spaces of Cebu Winland
Tower Condominium to Ong Siao Hua. The area per condo unit as indicated in petitioners price list 155 sq.m and
price per sq.m P22,378.95 . Hua paid P2,298,658.08 as down and issued 24 post dated checks in the amount of
P223,430. 70 per check to cover balance total of P5,363,385.19. On October 10,1996 , possession of the
Condominium units were transferred to Hua. After full payment of the purchase price, on Jan. 31, 1997 the Condo
Certificates of Title were issued. Hua discovered that the unit was only 110 sq.m contrary to the 155 sq.m indicated.
Hua demanded for a proportionate reduction of the purchase price eq uivalent ot the difference of 90 sq.m x
P22,378.95 = P2,014, 105.50 under Art. 1539 on Sales of immovable per unit price. The Housing and Land Use
Regulatory Board (HLURB) ruled that his action under Art. 1539 or 1542 is prescribed based on Art. 1543 which
states that actions based on 1539 and 1542 shall prescribe in six months, counted from the day of delivery.

ISSUES

1. Is Huas action prescribed based on Art. 1543?

2. May he demand a proportionate reduction of the price?

HELD

1. No, delivery under Art. 1543 requires both transfer of ownership and possession. Therefore, prescriptive period
shall be counted from the delivery of the Certificate of Ownership of Condominium Unit, upon the full payment of
the purchase price.

2. Yes, it is a unit price contract under Art. 1539. In a unit price contract, the statement of area of an immovable is
not conclusive and the price may be reduced or increased depending on the area actually delivered. If the vendor
delivers less than the area agreed upon, the vendee may oblige delivery of all that is stated in the contract or demand
a proportionate reduction of the purchase price if delivery is no longer possible.

2B 16-17 SALES AND LEASE Page 250


DOUBLE SALES

1. CARBONELL VS. CA

69 SCRA 99

FACTS:

Jose Poncio, a native of Batanes, sold a parcel of land in Agan St. San Juan, Rizal to his neighbor Rosario Carbonell
on the condition that Carbonell assumes the mortgage payment of the land to the Republic Savings Bank. Carbonell
then paid arrears (back payments/ balance) of the Mortgage to the Republic Savings bank in the amount of P247.
They executed a document in Batanes dialect for one-half lot bought from Poncio. Wherein Poncio can continue to
live for a period of one year on the land sold, afterwards he shall pay rent. Carbonell prepared a deed of sale, but
Poncio refused to proceed, arguing he has already sold the land to respondents Infante who made a better offer.
Carbonell immediately filed an adverse claim on the land title. Infante 4 days later, registered the Sale in her favor
with the adverse claim still annotated. Both Carbonell and Infantes claim title to the property.

ISSUE:

Who owns the land?

HELD:

Carbonell owns the land. Infante may have registered first, but according to Art. 1544 (2), registration must be in
good faith, otherwise it will not operate as an inscription. The adverse claim was already filed 4 days before Infante
registered the sale in her favor. Infante is already aware that there was a prior sale made to Carbonell. Both the
registration, and possession of Infante is in Bad Faith. The Older Title of Carbonell in good faith shall prevail.

2B 16-17 SALES AND LEASE Page 251


2. NOEL and JULIE ABRIGO VS. ROMANA DE VERA

G.R. No. 154409, June 21, 2004

FACTS:

Gloria Villafania sold a house and lot located at Banaoang, Mangaldan, Pangasinan covered b y Tax declaration to
Rosenda Tigno-Salazar and Rosita Cave-Go. Tigno-Salazar and Cave-Go declared the lot in their name. Unknown
to Tigno-Salazar and Cave- Go, Gloria Villafania obtained a free-patent on the land. Tigno-Salazar and Cave Go
sold the house and lot to Spouses Abrigo. Villafania sold the same land to Romana De Vera, who registered the sale,
and had a TCT issued in her name. The CA ruled that De Vera relied is a purchaser in good faith and for value and
must be protected since she relied on the Torrens title of Villafania. Sps. Abrigo appeal.

ISSUE:

Who has better right to the property?

HELD:

De Vera owns the land. The predecessor of Sps. Abrigo registered the sale with Act. 3344, not being aware the lot is
under the Torrens System. The predecessor of De Vera has an OCT, thereby allowing her to issue a TCT in her
name. Registration must be done in the proper registry in order to bind the land. Since the property in dispute in the
present case was already registered under the Torrens System, Sp s. Abrigos registration of the sale under Act. 3344
was not effective for purposes of Art. 1544 (Double Sales) of the Code. Registration under Act. 3344 does not apply
as constructive notice to 2nd buyer of land if it is already registered under the Torrens System.

2B 16-17 SALES AND LEASE Page 252


3. DAGUPAN TRADING VS. MACAM

L-18497, May 31, 1965, 4 SCRA 179.

FACTS:

Sammy Maron and his seven brothers and sisters were pro -indiviso owners of a parcel of unregistered land located
in barrio Parayao, Pangasinan.(8 co-owners). While their application for registration of the said land was pending,
they executed two deed of sale conveying the property to Rustico Macam, who immediately took possession thereof.
One month after the sale, Original Certificate Title of the land was issued in the names of the Marons free from all
liens and encumbrances. Several months later, when Sammy Maron lost a case against Manila Trading and Supply
Company, an order was issued for the sale of Sammys interest in the property at a public auction. After t he Manila
Trading and Supply Company acquired the property in the auction, it sold all of its right and title to the property to
Dagupan Trading Co. Dagupan Trading Co. then filed a case against Macam praying it be named the owner of the
one-eighth of the property(Sammys interest) and for Macam to leave said property.

ISSUE

Whether or not Dagupan Trading Co. has the better right to the one -eighth share of Sammy Maron in the property
since it acquired said interest after the registration of the property as against Macam who acquired the same before
the registration.

HELD

No, it was Rustico Macam who has the better right to the interest. What was the interest and claim of Sammy
Maron on the one-eighth portion of the property inherited by him and his co -heirs, at the time of the levy? The
answer must necessarily be that he had none, because for a considerable time prior to the levy, his interest had
already been conveyed to Macam, fully and irretrievablyas the Court of Appeals held. Consequently,
subsequent levy made on the property for the purpose of satisfying the judgment rendered against Sammy Maron in
favor of the Manila Trading Company was void and of no effect. Needless to say, the unregistered sale and the
consequent conveyance of title and ownership in favor of Macam could not have been cancelled and rendered of no
effect upon the subsequent issuance of the Torrens title over the entire parcel of land.

NOTE: RULE ON DOUBLE SALE - If the property covered by the conflicting sales were unregistered land,
Macam would undoubtedly have the better right in view of the fact that his claim is based on a prior sale coupled
with public, exclusive and continuous possession thereof as owner. On the other hand, were the land involved in the
conflicting transactions duly registered land, We would be inclined to hold that Dagupan Trading Co. has the better
right because the registration of the deed of sale is the operative act that gives validity to the transfer. This would be
fatal to Macams claim, the deeds of sale executed in his favor by the Marons not having been registered, while the
levy in execution and the provisional certificate of sale as well as the final deed of sale in favor of appellant were
registered. Consequently, this registered conveyance must prevail although posterior to the one executed in favor of
appellee, and appellant must be deemed to have acquired such right, title and interest as appeared on the certificate
of title issued in favor of Sammy Maron, subject to no lien, encumbrance or burde n not noted thereon. This case
does not fall within either situation.

2B 16-17 SALES AND LEASE Page 253


4. RICARDO CHENG vs. RAMON B. GENATO and ERNESTO R. DA JOSE AND SOCORRO DA JOSE,
G.R. No. 129760, December 29, 1998

FACTS:

On September 6, 1989, respondent Genato entered into an agreement with respondent- spouses Ernesto R. Da Jose
and Socorro B. Da Jose over two parcels of land located at Paradise Farms, San Jose del Monte, Bulacan

The agreement culminated in the execution of a contract to sell for which the purchase price was P80.00 per square
meter. The contract was in a public instrument and was duly annotated at the back of the two certificates of title on
the same day.

On October 4, 1989, the Da Jose spouses, not having finished verifying the titles mentioned in Clause 3 as afore-
quoted, asked for and was granted by respondent Genato an extension of another 30 days or until November 5,
1989. However, according to Genato, the extension was granted on condition that a new set of documents is made
seven (7) days from October 4, 1989. This was denied by the Da Jose spouses.

Pending the effectivity of the aforesaid extension period, and without due notice to the Da Jose spouses, Genato
executed an Affidavit to Annul the Contract to Sell, on October 13, 1989. Moreover, no annotation of the said
affidavit at the back of his titles was made right away.

(Cheng) went to Genatos residence and expressed interest in buying the subject properties. On that occasion,
Genato showed to Ricardo Cheng copies of his transfer certificates of title and the annotations at the back thereof of
his contract to sell with the Da Jose spouses. Genato also showed him the aforementioned Affidavit to Annul the
Contract to Sell which has not been annotated at the back of the titles.Despite these, Cheng went ahead and issued
a check for P50,000.00 upon the assurance by Genato that the previous contract with the Da Jose spouses will be
annulled for which Genato issued a handwritten receipt.

After Genato deposited Chengs receipt, the latter called the former to remind him to annul the contract to sell
between him and the Da Jose sps. Upon acting on this request the next day, Genato saw the sps in the Registry of
Deeds and when the sps found out they told Genato that the 30day extenson is still in effect and they are ready to
comply with the payment, because of this Genato decided to honor their contract, and return the check of Cheng.

Cheng refused to accept the check and asserts that there is already a perfected contract between him an d Genato.
Cheng executed an affidavit of adverse claim and had it annotated on the subject TCTs.On the same day, Genato
and the sps continued with their contract to sell.

Trial Court ruled in favor of Cheng that the receipt issued by Genato to Cheng unerringly meant a sale and not just
a priority or an option to buy. Also that there was valid rescisiion of the contract to sell with the sps by virtue of the
affidavit to annul under art.1191 of the civil code.

The CA reversed such judgment and ruled that the prior contract to sell in favor of the Da Jose spouses was not
validly rescinded; that the subsequent contract to sell between Genato and Cheng, embodied in the handwritten
receipt, was without force and effect due to the failure to rescind the prior cont ract; and that Cheng should pay
damages to the respondents herein being found to be in bad faith.

ISSUE

2B 16-17 SALES AND LEASE Page 254


Whether or not there was double sales. Who has the better right

HELD

None. The Jose spouses have the right over the subject property.

The trial court and the appellate court correctly held that the agreement between Genato and Cheng is a contract to
sell, which was, in fact, petitioners contention in his pleadings before the said courts. Consequently, both
agreements, between Genato and the Da Jose spouses, on the one hand, and that of Genato and Cheng, on the other
hand, actually involve two contracts to sell. The provisions of Art. 1544 of the Civil Code on double sales come to
mind, a meticulous reading of the provision shows that said law is not apro pos to the instant case. This provision
connotes that the following circumstances must concur:

(a) The two (or more) sales transactions in issue must pertain to exactly the same subject matter, and must
be valid sales transactions;
(b) The two (or more) buyers at odds over the rightful ownership of the subject matter must each represent
conflicting interests; and
(c) The two (or more) buyers at odds over the rightful ownership of the subject matter must each have
bought from the very same seller.
These situations obviously are lacking in a contract to sell for neither a transfer of ownership nor a sales transaction
has been consummated. The contract to be binding upon the obligee or the vendor depends upon the fulfillment or
non-fulfillment of an event.

Notwithstanding this contrary finding with the appellate court, we are of the view that the governing principle of
Article 1544, Civil Code, should apply in this situation. Jurisprudence teaches us that the governing principle is
PRIMUS TEMPORE, PORTIOR JURE (first in time, stronger in right). For not only was the contract between
herein respondents first in time it was also registered long before petitioners intrusion as a second buyer. This
principle only applies when the special rules provided in the afo recited article of the Civil Code do not apply or fit
the specific circumstances mandated under said law or by jurisprudence interpreting the article.

The rule exacted by Article 1544 of the Civil Code for the second buyer to be able to displace the first buyer are:

. (1) that the second buyer must show that he acted in good faith (i.e. in ignorance of the first sale and of
the first buyers rights) from the time of acquisition until title is transferred to him by registration or failing
registration, by delivery of possession;

. (2) the second buyer must show continuing good faith and innocence or lack of knowledge of the first
sale until his contract ripens into full ownership through prior registration as provided by law.

Thus, in the case at bar, the knowledge gained by the Da Jose spouses, as first buyers, of the new agreement between
Cheng and Genato will not defeat their rights as first buyers except where Cheng, as second buyer, registers or
annotates his transaction or agreement on the title of the subject properties in good faith ahead of the Da Jose
spouses. Moreover, although the Da Jose spouses, as first buyers, knew of the second transaction it will not bar them
from availing of their rights granted by law, among them, to register first t heir agreement as against the second
buyer.

In contrast, knowledge gained by Cheng of the first transaction between the Da Jose spouses and Genato defeats his
rights even if he is first to register the second transaction, since such knowledge taints his prior registration with bad
faith.

2B 16-17 SALES AND LEASE Page 255


5. CORONEL VS. COURT OF APPEALS

G.R. No. 103577 October 7, 1996

FACTS:

On January 19, 1985Coronel executed a document entitled "Receipt of Down Payment" in favor of plaintiff
Alcaraz with the ff conditions appurtenant to the sale:
o Ramona will make a down payment of P50,000.00 upon execution of the document aforestated;
o The Coronels will cause the transfer in their names of the title of the property registered in the
name of their deceased father upon receipt of the P50,000.00 Pesos down payment;
o Upon the transfer in their names of the subject property, the Coronels will execute the deed of
absolute sale in favor of Ramona and the latter will pay the former the whole balance of
P1,190,000.00.
February 18, 1985Coronels sold the property to Mabanag; Coronels cancelled and rescinded the contract with
Alcaraz by depositing the down payment paid by Concepcion in the bank in trust for Ramona Patricia Alcaraz.
Concepcion, et al., thereafter filed a complaint for specific performance against the Coronels . The Coronels on the
other hand executed a Deed of Absolute Sale over the subject property in favor of Catalina

ISSUE

Whether or not there is a double sale (to whom should the land be granted Alcaraz; Mabanag was not a registrant
in good faith although was considered a buyer in good faith)

HELD

Yes.

(Necessity of distinguishing contract to sell v. conditional contract of sale )

It is essential to distinguish between a contract to sell and a conditional contract o f sale specially in
cases where the subject property is sold by the owner not to the party the seller contracted with, but to a third person,
as in the case at bench. In a contract to sell, there being no previous sale of the property, a third person buyin g such
property despite the fulfillment of the suspensive condition such as the full payment of the purchase price, for
instance, cannot be deemed a buyer in bad faith and the prospective buyer cannot seek the relief of reconveyance of
the property. There is no double sale in such case. Title to the property will transfer to the buyer after registration
because there is no defect in the owner-seller's title per se, but the latter, of course, may be used for damages by the
intending buyer.

In a conditional contract of sale, however, upon the fulfillment of the suspensive condition, the sale
becomes absolute and this will definitely affect the seller's title thereto. In fact, if there had been previous delivery of
the subject property, the seller's ownership or title to the property is automatically transferred to the buyer such that,
the seller will no longer have any title to transfer to any third person. Applying Article 1544 of the Civil Code, such
second buyer of the property who may have had actual or con structive knowledge of such defect in the seller's title,
or at least was charged with the obligation to discover such defect, cannot be a registrant in good faith. Such second
buyer cannot defeat the first buyer's title. In case a title is issued to the s econd buyer, the first buyer may seek
reconveyance of the property subject of the sale.

2B 16-17 SALES AND LEASE Page 256


(The contract between Coronel and Alcaraz is a conditional contract of sale. The happening of the suspensive
condition, i.e. the transfer of the certificate of title from the name of petitioners' father, Constancio P. Coronel, to
their names rendered the contract absolute)

With the foregoing conclusions, the sale to the other petitioner, Catalina B. Mabanag, gave rise to a case
of double sale where Article 1544 of the Civil Code will apply.

In a case of double sale, what finds relevance and materiality is not whether or not the second buyer was
a buyer in good faith but whether or not said second buyer registers such second sale in good faith, that is, without
knowledge of any defect in the title of the property sold.

As clearly borne out by the evidence in this case, p etitioner Mabanag could not have in good faith,
registered the sale entered into on February 18, 1985 because as early as February 22, 1985, a notice of lis pendens
had been annotated on the transfer certificate of title in the names of petitioners, wherea s petitioner Mabanag
registered the said sale sometime in April, 1985. At the time of registration, therefore, petitioner Mabanag knew that
the same property had already been previously sold to private respondents, or, at least, she was charged with
knowledge that a previous buyer is claiming title to the same property. Petitioner Mabanag cannot close her eyes to
the defect in petitioners' title to the property at the time of the registration of the property.

2B 16-17 SALES AND LEASE Page 257


6. LORETO ADALIN, ET AL. vs. THE HON. COURT OF APPEALS, ET AL.

G.R. No. 120191 October 10, 1997

FACTS

Elena Kado and her siblings owned a lot with a 5 door commercial building fronting Imperial Hotel. The units were
leased. Elena contracted the services of Bautista, who brought Yu and Lim to her for the purpose of buying the
premises. During the meeting, it was agreed that the Yu and Lim would buy the said units except for the 5th which
is to be bought by Adalin. They entered into a Conditional Sale where Elena was obligated to evict the tenants
before the full payment of the purchase price. Elena offered the same for sale to the lessees but they refused
claiming that they could not afford; thus, she filed a case for ejectment against them. Thereafter, the lessees decided
to exercise their right to buy the units Kalaw ruled that since the sale to Yu and Lim was conditional, the
subsequent sale to the lessees must be preferred.

ISSUE

Who has a better title, Yu and Lim or the lessees?

HELD

YU AND LIM. While it is true that the Deed was for Conditional Sale, examination of the contents thereof
would show that it was one for the actual sale. During the meeting, the property was already sold; the only
conditions were that Elena would evict the lessees before the full payment of the price. The choice of to
whom to sell the property had already been decided. That being the case, since the sale in favor of Yu and
Lim was the prior sale, it must be preferred.Besides, Elena was guilty of double-dealing, which cannot be sanctioned
in law. It was, after all, her obligation to evict the lessees. The lessees were in bad faith as well for having
knowledge of the supposed sale in favor of Yu and Lim. Their subsequent registration of the sale cannot shield them
in their fraud.

2B 16-17 SALES AND LEASE Page 258


7. DOLORITA C. BEATINGO vs. LILIA BUGASIS

G.R. No. 179641, February 9, 2011

FACTS

Petitioner Dolorita Beatingo filed a Complaint for Annulment and Cancellation of Sale, Reconveyance, Delivery of
Title and Damages against respondent Lilia Bu Gasis before the RTC Iloilo.

Petitioner alleged that, on May 19, 1998, she bought a piece of land, denominated as Lot No. 7219 (hereafter
referred to as the subject property), from Flora G. Gasis (Flora). The subject property was registered in the name of
Floras predecessor-in-interest. The sale was evidenced by a notarized Deed of Absolute Sale. On October 18, 1999,
petitioner went to the Register of Deeds to have the sale registered. She, however, failed to obtain registration as she
could not produce the owners duplicate certificate of title. She, thus, filed a petition for the issuance of the owners
duplicate certificate of title but was opposed by respondent, claiming that she was in possession of the Original
Certificate of Title (OCT) as she purchased the subject pro perty from Flora on January 27, 1999, as evidenced by a
Deed of Sale. This prompted petitioner to file the Complaint, insisting that she is the rightful owner of the subject
property. She also maintained that respondent had been keeping the OCT despite kno wledge that petitioner is the
rightful owner. She further accused respondent of inducing Flora to violate the contract with her, which caused her
damage, prejudice, mental anguish, and serious anxiety

On the other hand, respondent claimed that she purchased the subject property from Flora without knowledge of the
prior sale of the same subject property to petitioner, which makes her an innocent purchaser for value. Respondent
denied having induced Flora to violate her contract with petitioner as she never knew the existence of the alleged
first contract. Lastly, respondent declared that, upon payment of the purchase price, she immediately occupied the
subject property and enjoyed its produce.

ISSUE

Who has a better right?

HELD: Bugasis.

Admittedly, the two sales were not registered with the Registry of Property. Since there was no inscription, the next
question is who, between petitioner and respondent, first took possession of the subject property in good faith. As
aptly held by the trial court, it was res pondent who took possession of the subject property and, therefore, has a
better right.

Petitioner insists that, upon the execution of the public instrument (the notarized deed of sale), she already acquired
possession thereof, and thus, considering that the execution thereof took place ahead of the actual possession by
respondent of the subject property, she has a better right.

Indeed, the execution of a public instrument shall be equivalent to the delivery of the thing that is the object of the
contract. However, the Court has held that the execution of a public instrument gives rise only to a prima facie
presumption of delivery. It is deemed negated by the failure of the vendee to take actual possession of the land sold.

2B 16-17 SALES AND LEASE Page 259


In this case, though the sale was evidenced by a notarized deed of sale, petitioner admitted that she refused to make
full payment on the subject property and take actual possession thereof because of the presence of tenants on the
subject property. Clearly, petitioner had not taken posses sion of the subject property or exercised acts of dominion
over it despite her assertion that she was the lawful owner thereof.

Respondent, on the other hand, showed that she purchased the subject property without knowledge that it had been
earlier sold by Flora to petitioner. She had reason to believe that there was no defect in her title since the owners
duplicate copy of the OCT was delivered to her by the seller upon full payment of the purchase price. She then took
possession of the subject property and exercised acts of ownership by collecting rentals from the tenants who were
occupying it.

2B 16-17 SALES AND LEASE Page 260


8. CARMELITA FUDOT vs. CATTLEYA LAND, INC., VELASCO

G.R. No. 171008, September 13, 2007

FACTS

Respondent Cattleya Land, Inc. asked someone to check the titles of nine (9) lots, the subject land included which it
intended to buy from spouses Troadio and Asuncion Tecson. Finding no defect respondent purchased said 9 lots
through Deed of Conditional Sale. Subsequently, a DOAS was executed over same properties. Said deeds were
registered with Register of Deeds.
The Register of Deeds refused to actually annotate because of an existing notice of attachment in connection with a
ciil case pending before RTC Bohol. The attachment was eventually cance lled by virtue of a compromise agreement
between spouses and their creditor. Titles to 6 of 9 lots were issued, RoD refused to issue titles to the remaining 3
since the same were still unaccounted for.
Petitioner presented for registration before the Regis ter of Deeds the owners copy of the title of the subject
property, together with the deed of sale purportedly executed by the Tecsons in favor of petitioner. On the following
day, respondent sent a letter of protest/opposition to petitioners application, and it was when respondent found out
that Register of Deeds had already registered the deed of sale in favor of petitioner Fudot and issued a new title
Fudot's name.
Respondent then filed its complaint before the RTC. Asuncion responded claiming that she never signed any deed of
sale covering any part of their conjugal property in favor of petitioner. She averred that her signature in petitioners
deed of sale was forged thus, said deed should be declared null, and it was only then when she found out that there
was an amorous relationship between petitioner Fudot and her husband.
However, Petitioner alleged that spouses Tecson had sold to her the subject property for P20,000.00 and delivered to
her the owners copy of the title and she claimed that she sub sequently presented the said title to the Register of
Deeds but the latter refused to register the same because the property was still under attachment.
RTC pointed out that respondent had recorded in good faith the deed of sale in its favor ahead of petitioner.
Moreover, based on Asuncions convincing and unrebutted testimony, the trial court concluded that the purported
signature of Asuncion in the deed of sale in favor of petitioner was forged, thereby rendering the sale void.
Petitioner sought recours e to the Court of Appeals, arguing in the main that the rule on double sale was applicable to
the case. The appellate court, however, dismissed her appeal. MR was filed but was also denied by CA for lack of
merit. Therefore, this petition.

ISSUE

Whether or not defense of petitioner that there was double sale was valid.

HELD

No, her defense of double sale is invalid. There is no double sale to speak of. Art. 1544 of the Civil Code,24 which
provides the rule on double sale, applies only to a situation where the same property is validly sold to different
vendees. In this case, there is only one sale to advert to, that between the spouses Tecson and respondent.

2B 16-17 SALES AND LEASE Page 261


The trial court declared that the sale between the spouses Tecson and petitioner is invalid, as it bears the forged
signature of Asuncion. Since for there to be a valid sale, signatures of both spouses who designated their property
regime to be conjugal must be present.
Even assuming that there was double sale in this case, petitioner would still not prevail. Knowledge gained by the
first buyer of the second sale cannot defeat the first buyers rights, except where the second buyer registers in good
faith the second sale ahead of the first as provided by the aforequoted provision of the Civil Code. Suc h knowledge
of the first buyer does not bar him from availing of his rights under the law, among them to register first his
purchase as against the second buyer. However, knowledge gained by the second buyer of the first sale defeats his
rights even if he is first to register the second sale, since such knowledge taints his prior registration with bad faith.
It is thus essential, to merit the protection of Art. 1544, second paragraph, that the second realty buyer must act in
good faith in registering his deed of sale.

2B 16-17 SALES AND LEASE Page 262


9. SPOUSES TEOFILO, et al. vs. DONATO REYES, et al.

G.R. No. 150913, February 20, 2003

Bellosillo, J

FACTS:

AT STAKE IN THIS PETITION FOR REVIEW is the ownership of three (3) parcels of unregistered land with an
area of approximately 130,947 square meters situated in Brgy. Sapa, Burgos, Pangasinan.

The three (3) parcels were formerly owned by the spouses Francisco and Asuncion Tazal who on 1 September 1957
sold them for P724.00 to respondents predecessor-in-interest, one Mamerto Reyes, with right to repurchase
within two (2) years from date thereof by paying to the vendee the purchase price and all expenses incident to their
reconveyance. After the sale the vendee a retro took physical possession of the properties and paid the taxes thereon.
The otherwise inconsequential sale became controversial when two (2) of the three (3) parcels were again sold on
24 December 1958 by Francisco Tazal for P420.00 in favor of petitioners predecessor-in-interest Blas Rayos
without first availing of his right to repurchase the properties.

In the meantime, on 1 September 1959 the conventional right of redemption in favor of spouses Francisco and
Asuncion Tazal expired without the right being exercised by either the Tazal spouses or the vend ee Blas Rayos.

After the expiration of the redemption period, Francisco Tazal attempted to repurchase the properties from Mamerto
Reyes by asserting that the 1 September 1957 deed of sale with right of repurchase was actually an equitable
mortgage and offering the amount of P724.00 to pay for the alleged debt. But Mamerto Reyes refused the tender of
payment and vigorously claimed that their agreement was not an equitable mortgage.

On May 1960, Francisco Tazal instituted an action against Mamerto Reyes, docketed as Civil Case No. A-245, for
the declaration of its transaction with Reyes as equitable mortgage.

On 22 June 1961 Francisco Tazal again sold the third parcel of land previously purchased by Mamerto Reyes to
petitioner-spouses Teofilo and Simeona Rayos for P400.00. On 1 July 1961 petitioner-spouses bought from Blas
Rayos for P400.00 the two (2) lots that Tazal had sold at the first instance to Mamerto Reyes and thereafter to Blas
Rayos, while Civil Case No. A-245 was pending before the trial court.

The trial court in Civil Case No. A-245 rejected the contention of Francisco Tazal that the deed of sale executed on
1 September 1957 was an equitable mortgage but held that Tazal could nonetheless redeem the three (3) parcels of
land within thirty (30) days from finality of judgment by paying to Mamerto Reyes the purchase price of P724.00
and all expenses to execute the reconveyance.

Mamerto Reyes appeal was elevated to the SC because only questions of law were involved. When Mamerto Reyes
died in 1986, petitioner-spouses Teofilo and Simeona Rayos wrested physical possession of the disputed properties
from Reyess heirs.

On 20 June 1990 the Court rendered the judgment in Civil Case No. A -245 final and executory for failure of the
parties therein to prosecute the case. Subsequent to the finality of said judgment, petitioner-spouses did nothing to
repurchase the three (3) parcels of land within the thirty (30) - day grace period from finality of judgment
since, according to them, they believed that the consignat ion of P724.00 in the civil case had perfected the
repurchase of the disputed properties.

2B 16-17 SALES AND LEASE Page 263


On 6 July 1992 respondents as heirs of Mamerto Reyes executed an affidavit adjudicating to themselves the
ownership of the parcels of land and declared the properties in their names for assessment and collection of real
estate taxes. On 19 January 1993 respondents registered the 1 September 1957 deed of sale with right of
repurchase with the register of deeds.

On 8 July 1993 respondents filed a complaint for damages and recovery of ownership and possession of the three (3)
parcels of land in dispute against herein petitioner-spouses Teofilo and Simeona Rayos and petitioner George Rayos
as administrator thereof. It was respondents theory that neither petitioners nor their predecessors-in-interest
Francisco Tazal and Blas Rayos repurchased the properties before buying them in 1958 and 1961 or when the
judgment in Civil Case No. A-245 became final and executory in 1990, hence the sale of the three (3) parcels of land
to petitioner-spouses did not transfer ownership thereof to them.

ISSUES

(1) Whether or not the petitioners can be considered as buyers in good faith

(2) Whether or not the application of Art. 1544 is warranted in the instant case

HELD

(1) NO. As held in David v. Bandin, the issue of good faith or bad faith of the buyer is relevant only where the
subject of the sale is registered land and the purchaser is buying the same from the registered owner
whose title to the land is clean x x x in such case the purchaser who relies on the clean title of the
registered owner is protected if he is a purchaser in good faith for value. Since the properties in question
are unregistered lands, petitioners as subsequent buyers thereof did so at their peril. Their claim of
having bought the land in good faith, i.e., without notice that some other person has a right to or interest in
the property, would not protect them if it turns out, as it actually did in this case, that their seller did not
own the property at the time of the sale.

At any rate, petitioners failed to discharge their burden of proof that they were purchasers of the three (3)
parcels of land in good faith. For, as the Court ruled in Embrado v. Court of Appeals, the burden of proving
the status of a purchaser in good faith and for value lies upon him who asserts that status, which is not
discharged by simply invoking the ordinary presumption of good faith, i.e., that everyone is presumed to
act in good faith, since the good faith that is here essential is integral with the very status which must be
established.

In the proceedings a quo, what is evident is the admitted fact of payment made by Mamerto Reyes as
respondents predecessor-in-interest of the taxes on the properties prior to and at the time when the contracts
of sale in favor of petitioner-spouses were perfected, which undoubtedly confirms the precedence of
respondents possession of the parcels of land in question. This situation should have compelled petitioners
to investigate the right of respondents over the p roperties before buying them, and in the absence of such
inquiry, the rule is settled that a buyer in the same circumstances herein involved cannot claim to be a
purchaser in good faith.

(2) NO. The absence of good faith on the part of petitioner-spouses Teofilo and Simeona Rayos in purchasing
the three (3) parcels of unregistered land precludes the application of the rule on double sales enunciated in
Art. 1544 of the Civil Code. In any event, even if we apply Art. 1544, the facts would nonetheless show

2B 16-17 SALES AND LEASE Page 264


that respondents and their predecessor-in-interest registered first the source of their ownership and
possession, i.e., the 1 September 1957 deed of sale with right to repurchase, held the oldest title, and
possessed the real properties at the earliest time. Applying the doctrine of priority in time, priority in rights
or prius tempore, potior jure, respondents are entitled to the ownership and possession of the parcels of
land in dispute.

2B 16-17 SALES AND LEASE Page 265


10. SPOUSES SABITSANA VS. MUERTEGUI

G.R. No. 181359, August 5, 2013

FACTS
Alberto Garcia executed an unnotarized deed of absolute sale in favor of Juanito Muertegui over an unregistered
land in Leyte. Juanitos father Domingo Muertegui Sr. and brother Domingo Jr. took actual possession of the land
and also payed taxes.After 10 years, Garcia sold the land already sold to Muertegui to the latters family lawyer
Sabitsana, herein petitioner. The said deed of sale was notarized and after a year, registered to the Register of Deeds.
Taxes were also paid by petitioner from the time of sale.
When Domingo Sr. passed away, his heirs applied for land registration under the Public land act or CA 141 but
petitioner appealed to DENR claiming that he is the legal owner of the disputed land.
The heirs of Juanito thereafter through Domingo Jr. as an attorney-in-fact applied for the quieting of title and to have
the deed of sale between Garcia and Sabitsana be declared null and void for it was done in bad faith.
Atty. Sabitsana countered that the sale between Garcia and Juanito was void due to the absence of marital consent of
Garcias wife. RTC had no jurisdiction over the case the land being under 20,000 peeos.
The RTC ruled in favor of Juanito, applying 1544 of the NCC,Sabitsanas registration not in good faith, therefore,
because Juanito was the first possessor of the lot in good faith, the sale to him is preferable. The sale to petitioner
must be declared null and void as it casts cloud over Juanitos title.
CA affirmed in toto RTC ruling. In this appeal, petitioner contends that 1544 is not applicable but PD 1529
considering that the lot is unregistered.

ISSUE
Whether or not Art.1544 of the NCC applies in the case given that the land in dispute is unregistered

HELD No

Both the trial court and the CA are, however, wrong in applying Article 1544 of the Civil Code. Both courts seem
to have forgotten that the provision does not apply to sales involving unregistered land. Suffice it to state that the
issue of the buyers good or bad faith is relevant only where the su bject of the sale is registered land, and the
purchaser is buying the same from the registered owner whose title to the land is clean. In such case, the purchaser
who relies on the clean title of the registered owner is protected if he is a purchaser in go od faith for value.

Act no. 3344 applies in this case. 3344 expressly declares that any registration made shall be without prejudice to a
third party with a better right. The question to be resolved therefore is: who between petitioners and respondent ha s
a better right to the disputed lot? Respondent has a better right to the lot.The sale to respondent Juanito was executed
on September 2, 1981 via an unnotarized deed of sale, while the sale to petitioners was made via a notarized
document only on October 17, 1991, or ten years thereafter. Thus, Juanito who was the first buyer has a better right
to the lot, while the subsequent sale to petitioners is null and void, because when it was made, the seller Garcia was
no longer the owner of the lot. Nemo dat quod non habet.

The fact that the sale to Juanito was not notarized does not alter anything, since the sale between him and Garcia
remains valid nonetheless. Notarization, or the requirement of a public document under the Civil Code,is only for
convenience, and not for validity or enforceability.

Nor can petitioners registration of their purchase have any effect on Juanitos rights. The mere registration of a sale

2B 16-17 SALES AND LEASE Page 266


in ones favor does not give him any right over the land if the vendor was no longer the owner of the land, having
previously sold the same to another even if the earlier sale was unrecorded.Neither could it validate the purchase
thereof by petitioners, which is null and void. Registration does not vest title; it is merely the evidence of such
title.Our land registration laws do not give the holder any better title than what he actually has.

2B 16-17 SALES AND LEASE Page 267


11. ROSAROSO VS. LUCILA LABORTE SORIA

G.R. No. 194846, June 19, 2013

Mendoza, J.

FACTS:

SPOUSES Louis Rosaroso and Honorat Duazo acquired several real properties through the years. When Honorata
died, Loius married Lourdes Pastor Rosaroso.

Sometimes later, a complaint was filed by some of Louis children against their sibling Luicilla., Lourdes and
Meridian Realty Corporation among others. It was alleged by the petitioner that Louis with the full knowledge and
consent of his second wife, executed a Deed of Absolute Sale of some the properties in their favor.

Petitioner also alleged that a second sale took place when the respondent through unscrupulous means, made Louis
sign a Deed of Absolute Sale conveying to Meridian three parcels of residen tial land.

They further averred that Meridian was in bad faith when it did not make any inquiry as to who were the occupants
and owners of said lots; and that had Meridian only investigated , it would have been informed as to the true status
of the subject properties and would have desisted in pursuing their acquisition.

ISSUE:

Whether or not Meridian is a buyer in bad faith

HELD:

YES. Meridian is a buyer in bad faith. The fact that Meridian had them first registered will not help its cause.
In case of double sale, Article 1544 of the Civil Code provides:
ART. 1544. If the same thing should have been sold to different vendees, the ownership shall be transferred to the
person who may have first possession thereof in good faith, if it should be movable property.
Should it be immovable property, the ownership shall belong to the person acquiring it who in good faith first
recorded it in the Registry of Property.
Should there be no inscription, the ownership shall pertain to the person who in good faith was first in possession;
and, in the absence thereof; to the person who presents the oldest title, provided there is good faith.

Otherwise stated, ownership of an immovable property which is the subject of a double sale shall be transferred: (1)
to the person acquiring it who in good faith first recorded it in the Registry of Property; (2) in default thereof, to the
person who in good faith was first in possession; and (3) in default thereof, to the person who presents the oldest
title, provided there is good faith. The requirement of the law then is two-fold: acquisition in good faith and
registration in good faith. Good faith must concur with the registration. If it would be shown that a buyer was in
bad faith, the alleged registration they have made amounted to no registration at all.

2B 16-17 SALES AND LEASE Page 268


When a piece of land is in the actual possession of persons other than the seller, the buyer must be wary and should
investigate the rights of those in possession. Without making such inquiry, one cannot claim that he is a buyer in
good faith. When a man proposes to buy or deal with realty, his duty is to read the public manuscript, that is, to look
and see who is there upon it and what his rights are. A want of caution and diligence, which an honest man of
ordinary prudence is accustomed to exercise in making purchases, is in contemplation of law, a want of good faith.
The buyer who has failed to know or discover that the land sold to him is in adverse possession of another is a buyer
in bad faith.

2B 16-17 SALES AND LEASE Page 269


WARRANTY AGAINST EVICTION

1. BINALBAGAN TECH VS CA

G.R. No. 100594. March 10, 1993.

FACTS:

A property owned by deceased Luis Puentevella, thru Angelina P. Echaus, in her capacity as Judicial Administratrix
of the intestate estate of the former, was sold to one Raul Javellana with the cond ition that the vendee-promisee
would not transfer his rights to said lots without the express consent of Puentevella and that in case of the
cancellation of the contract by reason of the violation of any of the terms thereof. Javellana having failed to pay the
installments, a case was filed by defendant Puentevella against Javellana and the Southern Negros Colleges which
was impleaded as a party defendant it being in actual possession thereof, for the rescission of their contract to sell
and the recovery of possession of the lots and buildings with damages. Judgment was rendered in favor of
Puentevilla.. A case for injunction and damages was filed and trial court issued an ex-parte writ of preliminary
injunction which was dissolved by the CA pending the reso lution of the civil case. Defendant Puentevella was
restored to the possession of the lots and buildings subject of this case. Consequently, the property was restored to
the petitioner in the Civil Case. Notably, petitioner was not in possession of the pro perty from 1974-1982.

Echaus then demanded payment from petitioner for the subdivision lots, enclosing in the letter of demand a
statement of account as of September 1982 showing a total amount due of P367,509.93, representing the price of the
land and accrued interest as of that date. Binalbagan failed to pay which prompted Echaus to file a case for recovery
of title plus damages. The CA ruled in favor of Echaus. Hence, this petition.

ISSUE:

Whether or not private respondents may demand payment from 1974-1982.

HELD

No. A party to a contract cannot demand performance of the other party's obligations unless he is in a position to
comply with his own obligations. Similarly, the right to rescind a contract can be demanded only if a party thereto is
ready, willing and able to comply with his own obligations thereunder (Art. 1191, Civil Code; Seva vs. Berwin, 48
Phil. 581 [1926]; Paras, Civil Code of the Philippines, 12th ed. Vol. IV, p. 200). In a contract of sale, the vendor is
bound to transfer the ownership of and deliver, as well as warrant, the thing which is the object of the sale (Art.
1495, Civil Code); he warrants that the buyer shall, from the time ownership is passed, have and enjoy the legal and
peaceful possession of the thing

2B 16-17 SALES AND LEASE Page 270


ARTICLE 1547. In a contract of sale, unless a contrary intention appears, there is:

(1) An implied warranty on the part of the seller that he has a right to sell the thing at the time when the
ownership is to pass, and that the buyer shall from that time have and enjoy th e legal and peaceful
possession of the thing.

xxx xxx xxx

As afore-stated, petitioner was evicted from the subject subdivision lots in 1974 by virtue of a court order in Civil
Case No. 293 and reinstated to the possession thereof only in 1982. During the period, therefore, from 1974 to 1982,
seller private respondent Angelina Echaus' warranty against eviction given to buyer petitioner was breached though,
admittedly, through no fault of her own. It follows that during that period, 1974 to 1982, private res pondent Echaus
was not in a legal position to demand compliance of the prestation of petitioner to pay the price of said subdivision
lots. In short, her right to demand payment was suspended during that period, 1974-1982.

2B 16-17 SALES AND LEASE Page 271


2. HEIRS OF SOFIA QUIRONG VS DBP

G.R. No. 173441, December 3, 2009

FACTS:

The late Emillo Daloppe left a parcel of land to his wife Felisa and nine children. To enable one of the children
(Rosa Dalope-Funcion) to get a loan from the Development Bank of the Philippines (DBP), Felisa sold the parcel of
land to Funcions. The Funcions failed to pay the loan. DBP subsequently foreclosed the mortgage and made a
conditional sale of the land to Sofia Quirong for PhP78,000. In their contract of sale, Sofia Quirong waived any
warranty against eviction. The contract provided that the DBP did not guarantee possession of the property and
that it would not be liable for any lien or encumbrance on the same. Quirong gave a down payment of P14,000.00.

Two months after the conditional sale to Quirong, Felisa and her eight other children subsequently filed an action for
partition and declaration of nullity of documents with damages against DBP and the Funcions before the Regional
Trial Court (RTC) of Dagupan City. Despite the suit, the DBP executed a deed of absolute sale of the subject lot in
Sofia Quirongs favor. The deed of sale carried substantially the same waiver of warranty against eviction and of
any adverse lien or encumbrance.

Sofia Quirong having since died, her heirs filed an answer in intervention in which they asked the RTC to award the
lot to them and, should it instead be given to the Dalopes, to allow the Quirong heirs to recover the lots value from
the DBP. Because the heirs failed to file a formal offer of evidence, the trial court did not rule on the merits of their
claim to the lot and, alternatively, to relief from DBP.

The RTC rendered a decision, declaring DBPs sale to Sofia Quirong valid only with respect to the shares of
Felisa and Rosa Funcion in the property. It declared Felisas sale to the Funcions, the latters mortgage to the
DBP, and the latters sale to Sofia Quirong void insofar as they prejudiced the shares of the eight other
children of Emilio and Felisa who were each entitled to a tenth share in the subject lot.

The Quirong heirs then filed an action against DBP before the RTC of Dagupan City for rescission of the contract of
sale between Sofia Quirong, their predecessor, and the DBP and praying for the reimbursement of the price of
P78,000.00 that she paid the bank plus damages. The heirs alleged that they were entitled to the rescission of the sale
because the decision in Civil Case D-7159 stripped them of nearly the whole of the lot that Sofia Quirong, their
predecessor, bought from DBP. DBP filed a motion to dismiss the action on ground of prescription and res judicata
but the RTC denied their motion.

After hearing the case, the RTC rendered a decision, rescinding the sale between Sofia Quirong and DBP and
ordering the latter to return to the Quirong heirs the PhP78,000.00 Sofia Quirong paid the bank. On appeal by DBP,
Court of Appeals (CA) reversed the RTC decision and dismissed the heirs action on the ground of prescription. The
CA concluded that, reckoned from the finality of the December 16, 1992 decision in Civil Case D-7159, the
complaint filed on June 10, 1998 was already barred by the 4-year prescriptive period under Article 1389 of the
Civil Code. The Quirong heirs filed a motion for reconsideration of the decision but the CA court denied it .

According to DBP, the prescriptive period should be 4 years as provided under Article 1389 of the Civil Code,
which provides that the action to claim rescission must be commenced within four years. On the other hand, the

2B 16-17 SALES AND LEASE Page 272


Quirong heirs argue that it should be 10 years as provided under Article 1144 which states that actions upon a
written contract must be brought within 10 years from the date the right of action accrues.

ISSUE

If the heirs of a lot buyer were evicted from the lot because of a final judgment based on a right prior to the sale (i.e.,
the seller did not validly acquire the lot from the person who sold the lot to the seller), should the evicted heirs file
an action for rescission under article 1381 or an action for rescission/resolution under Article 1191?

HELD

The Supreme Court agreed with DBP that the prescriptive period was 4 years because the action involved was
one for rescission under Article 1381.

The Court distinguished between a rescission under Article 1381 and a rescission u nder Article 1191:
"Rescission" is a subsidiary action based on injury to the plaintiffs economic interests as described in
Articles 1380 and 1381.
"Resolution," the action referred to in Article 1191, on the other hand, is based on the defendants
breach of faith, a violation of the reciprocity between the parties.
As an action based on the binding force of a written contract, therefore, rescission (resolution) under
Article 1191 prescribes in 10 years. Ten years is the period of prescription of actions based on a
written contract under Article 1144.
Article 1191 gives the injured party an option to choose between, first, fulfillment of the contract and, second, its
rescission. An action to enforce a written contract (fulfillment) is definitely an actio n upon a written contract,
which prescribes in 10 years.
The action filed by the Quirong heirs was an action for rescission (not resolution). Here, the Quirong heirs alleged in
their complaint that they were entitled to the rescission of the contract of s ale of the lot between the DBP and Sofia
Quirong because the decision in Civil Case D-7159 deprived her heirs of nearly the whole of that lot.
The status of that contract at the time of the filing of the action for rescission, apparently, was a contract of sale that
had already been fully performed when Sofia Quirong paid the full price for the lot and when, in exchange, the DBP
executed the deed of absolute sale in her favor.
There was a turnover of control of the property from DBP to Sofia Quirong since she assumed under their contract,
the ejectment of squatters and/or occupants on the lot, at her own expense.
With the loss of 80% of the subject lot to the Dalopes by reason of the judgment of the RTC in Civil Case D-7159,
the Quirong heirs had the right to file an action for rescission against the DBP pursuant to the provision of Article
1556.
However, the Court concluded that the action for rescission, which is based on a subsequent economic loss suffered
by the buyer, was barred by prescription as it was filed beyond the 4-year prescriptive period under Article 1389.

2B 16-17 SALES AND LEASE Page 273


3. HEIRS OF NICOLAS OROSA VS MIGRINO

G.R. Nos. 99338-40 February 1, 1993

FACTS

In Maria Mayug Vda. de Cailles v. Dominador Mayuga, et. al., the Court decided confirming ownership ove r a
fifty-three (53) hectare parcel of land located in Las Pias, Rizal in favor of one Dominador Mayuga. The Court also
extended the benefit of such confirmation to the latter's successor-in-interest, the late Nicolas Orosa.

The heirs of Nicolas Orosa (petitioners herein) moved for execution of judgment. This motion was granted by the
lower court in its Order dated 25 October 1989, directing the Land Registration Authority ("LRA") to submit the
property's amended technical description for approval.

However, the LRA did not comply with said order because, among others, its records indicated that the property had
previously been decreed in favor of one Jose T. Velasquez, to whom was issued Original Certificate of Title No.
6122.

On 10 September 1990, Goldenrod, Inc. ("Goldenrod") filed a motion for leave to intervene in the execution
proceeding, alleging an interest in the property which is the subject matter of LRC No. 2839.

Goldenrod claims that in 1977, during the pendency of this case before the Court, Delta Motors Corporation (Delta)
acquired for value the contingent rights of Nicolas Orosa over the property, as well as the conflicting claims thereto
of one Jose Velasquez. In 1980, the land registration court trying Jose Velasquez' claims in LRC No. N-5416
excluded therefrom the land referred to as Lot 9 Psu -11411 Amd-2 in G.R. No. L-30859. Meanwhile, Delta
somehow managed to obtain transfer certificates of titles over the land and sold this acquisition to Goldenrod in
1987. The latter then succeeded in obtaining issuance in its favor of Transfer Certificates of Title Nos. 4893 and
4901, whose technical descriptions overlapped "big portions" of the land referred to as Lot 9 Psu -11411 Amd-2 in
G.R. No. L-30859. In February 1989, Goldenrod sold the land covered by said transfer certificates of title to a
consortium composed of Fil Estate Management Inc., Arturo Y. Dy, Megatop Realty Development Inc., Peaksun
Enterprises and Export Corporation, and Elena D. Jao ("Consortium"). The contract of sale contained an undertaking
on Goldenrod's part to "defend the title of the VENDEES to the property against claims of any third person
whatsoever." It is on the basis of this stipulation that Goldenrod seeks to intervene in the execution Proceedings of
LRC No. 2839.

ISSUE

Whether Goldenrod has shown in its pleadings in intervention a sufficient legal interest in the land which is the
subject matter of LRC No. 2839

HELD

Taking Goldenrod's own admissions at their face value, it is quite apparent that whatever direct and act ual legal
interest it may have had over the land had been disposed of by it for value in favor of the consortium in 1989 and
that whatever residual legal interest in the property can be premised on Goldenrod's contractual undertaking,
actually an express warranty against eviction, is expectant or contingent in nature. Presently, Goldenrod has no legal
interest in the property and its warranty can only be enforced by the consortium if the latter is dispossessed of the

2B 16-17 SALES AND LEASE Page 274


land by virtue of a proper action instituted by the Orosa heirs as registered owners thereof. (SEE ART. 1557 &
1558)

But, the legal interest which entitles a person to intervene in a suit must be actual and material, direct and
immediate. A party seeking to intervene in a pending case must show that he will either gain or lose by the direct
legal operation and effect of a judgment.

In the present case, Goldenrod has failed to meet this criteria and the lower court gravely abused its discretion in
permitting intervention after having overlooked this matter.

2B 16-17 SALES AND LEASE Page 275


4. BIGNAY EX-IM PHILIPPINES INC VS UNION BANK

G.R. No. 171590, February 12, 2014

FACTS

In 1984, Alfonso de Leon (Alfonso) mortgaged in favor of Union Bank of the Philippines (Union Bank) real
property which was registered in his and his wife Rosarios name and covered by TCT 286130. The property was
foreclosed and sold at auction to Union Bank. After the redemption period expired, the bank consolidated its
ownership and issued in its name in 1987. In 1988, Rosario filed against Alfonso and Union Bank for annulment of
the 1984 mortgage, claiming that Alfonso mortgaged the property without her consent, and for reconveyance.

In a September 6, 1989 LetterProposal,Bignay ExIm Philippines, Inc. (Bignay), through its President, Milagros
Ong Siy (Siy), offered to purchase the property.

One of the terms of the deed of sale is quoted below:


Section 1. The VENDEE hereby recognizes that the Parcel/s of Land with improvements thereon is
acquired through foreclosure proceedings and agrees to buy the Parcel/s of Land with improvement[s]
thereon in its present state and condition. The VENDOR therefore does not make any x x x representations
or warranty with respect to the Parcel/s of Land but that it will defend its title to the Pa rcel/s of Land with
improvement[s] thereon against the claims of any person whomsoever.

On December 27, 1989, Bignay mortgaged the property to Union Bank, presumably to secure a loan obtained from
the latter. On December 12, 1991, the Court declared NULL and VOID the following documents:)((1)said Mortgage
Contract dated April 11, 1984 executed by and between defendants Alfonso de Leon, Jr. alone and Union Bank of
the Philippines(2)Sheriffs Sale dated June 12, 1985(3)T.C.T. issued in the name of defendan t Union Bank on June
10, 1987;(4) Sale and mortgage by and between Union Bank and Bignay ExIm Phil. Inc. on December 1989.

The order dated February 2, 1988 granting a writ of possession in favor of Union Bank is hereby SET ASIDE and
QUASHED. Meanwhile, as a result of the 1991 Decision , Bignay was evicted from the property; by then, it had
demolished the existing structure on the lot and begun construction of a new building. Bignay filed a case against
Union Bank. The RTC on the matter declared that Union Bank, through Robles, acted in bad faith in selling the
subject property to Bignay; for this reason, the stipulation in the December 20, 1989 deed of sale limiting Union
Banks liability in case of eviction cannot apply, because under Article 1553 of the Civil Code, [a]ny stipulation
exempting the vendor from the obligation to answer for eviction shall be void, if he acted in bad faith. Moreover, it
held that the bank was guilty of gross negligence amounting to bad faith, which thus contravened its undertaking
in the deed of sale to defend its title to the Parcel/s of Land with improvement thereon against the claims of any
person whatsoever. The CA affirmed the RTC decision upon appeal. CA added that the declaration contained in the
letterproposalcannot bind Bignay because the proposal was supposedly prepared and signed by Siy in her
personal capacity.

ISSUE

2B 16-17 SALES AND LEASE Page 276


Whether or not the application of Arts. 1548 and 1549 of the Civil Code was patently erroneous.

HELD

No. The Court finds for Bignay.

Indeed, this Court is convinced that Bignay purchased the property without knowledge of the pending Civil Case.
Union Bank is therefore answerable for its express undertaking under the December 20, 1989 deed of sale to
defend its title to the Parcel/s of Land with improvement thereon against the claims of any person whatsoever. By
this warranty, Union Bank represented to Bignay that it had title to the property, and by assuming the obligation to
defend such title, it promised to do so at least in good faith and wit h sufficient prudence, if not to the best of its
abilities.

Eviction shall take place whenever by a final judgment based on a right prior to the sale or an act imputable to the
vendor, the vendee is deprived of the whole or of a part of the thing purchased (Art. 1548). In case eviction occurs,
the vendee shall have the right to demand of the vendor, among others,(a) the return of the value which the thing
sold had at the time of the eviction, be it greater or less than the price of the sale; (b) the expense s of the contract, if
the vendee has paid them; and (c) the damages and interests, and ornamental expenses, if the sale was made in bad
faith( Art. 1555).

There appears to be no dispute as to the value of the building constructed on the property by Bignay; the only issue
raised by Union Bank in these Petitions is the propriety of the award of damages, and the amount thereof is not in
issue. The award in favor of Bignay of P4 million, or the consideration or cost of the property, and P20 million the
value of the building it erected thereon is no longer in issue and is thus in order.

2B 16-17 SALES AND LEASE Page 277


ON WARRANTY AGAINST HIDDEN DEFECTS/ENCUMBRANCES (Articles 1561 -1581)

1. ALBERT BRYAN VS THOMAS HANDKINS AND J BIALOGLOWS KI

G.R. No. L-18999 November 24, 1922

Johns, J

FACTS

In Manila, the defendants made a bill of sale to the plaintiff of the motor schooner Sultan for which the plaintiff paid
a down payment and executed to the defendants two promissory notes and the possession of the schooner was
delivered to the plaintiff for use in his own personal business.

It was at once taken to Iloilo where it was for the first time inspected by the inspector of hulls and boilers of Iloilo.
As a result of the inspection, it was found to be unseaworthy, and was o rdered to be placed on the slipways for
immediately repairs.

Plaintiff commenced this action in which he alleged the execution of the contract and the promissory notes and "that
said defects in the vessel existed at the time of the delivery to plaintiff; t hat said defects rendered said vessel unfit
for the use of plaintiff; that if plaintiff had had knowledge of such defects he would not have bought said vessel; that
said defects were hidden and plaintiff could not perceive them; that defendants warranted s aid vessel to be in
seaworthy condition that said vessel was not in such condition when delivered to plaintiff.

As a counterclaim, the defendants seek to recover upon the promissory notes only and contend that the plaintiff
agreed to purchase it without any inspection.

The defects in the lumber and material used in the construction of the vessel were hidden and concealed and
unknown to the plaintiff until the official inspection was made in Iloilo.

ISSUE

Whether or not the defendants should be held liable for the hidden defects.

HELD

YES.

The instant case comes under the provisions of article 1484 of the Civil Code, which provides:

The vendor is liable for any hidden defects which the thing sold may have should they render it unfit for the use for
which it was intended, or if they should diminish its adaptability to such use to such an extent that had the vendee
had knowledge thereof he would not have bought it or would have given a lower price for it; but such vendor shall
not be liable for patent or vis ible defects, or for those which are not visible, if the vendee should be an expert who by
reason of his trade or profession should easily perceive them.

2B 16-17 SALES AND LEASE Page 278


And article 1485, which provides:

The vendor is liable to the vendee for any latent faults or defects in the thing sold, even if they were unknown to
him.

This provision shall not apply if the contrary has been stipulated and the vendor was not aware of such latent faults
or defects.

The defects of its construction were hidden and concealed and were unknown to the plaintiff until the official
inspection was made, when he promptly brought this action. The proof is conclusive that such hidden defects
rendered the vessel unfit for the use for which it was intended, and that the plaintiff did not have any knowledge of
such defects, and that no sane man would ever have purchased it with such knowledge.

Article 1485 expressly provides that the vendor is liable to the vendee for any latent faults or defects of the thing
sold, even if they were unknown to him.

The plaintiff was entitled to recover the down payment of which he had paid on the contract, and that he should be
charged with the use of the rental value of the vessel during the time that he used it and had it in his possession.

2B 16-17 SALES AND LEASE Page 279


2. GOCHANGO VS DEAN

G.R. No. L-23109 March 20, 1925

Romualdez, J

FACTS:

The plaintiffs had purchased a land of the Pasay Estate by installments. The defendant was the owner of two parcels
of land situated in Masbate. The plaintiffs and defendant agreed to exchange their respective p roperties, but before
the final execution of the contract of exchange, the plaintiff Gochangco went to Masbate to make an examination of
the parcels of land offered for exchange by the defendant. The contract of exchange was later executed.

In the deed, the defendant stated, among other things, the following:

It is also declared that the said described property is sold will all coconut trees growing on it, and I
declared that I believe there are more than 6,000 coconut trees so growing, together with any and all
improvements of any kind whatsoever existing on the said land including all movable goods, chattel, etc.,
found thereof.

The plaintiffs claim that there was no such 6,000 number of trees and such existence of said number was the
primary consideration of the contract of exchange, without which the plaintiffs would not have accepted the carrying
out of the transaction between them.

The Court of First Instance of Manila, rendered judgment absolving the defendant from the complaint.

ISSUE:

Whether or not the CFI erred in absolving the defendant?

HELD

No.

The plaintiffs allege that defendant made them false and fraudulent representations as to the existence of 6,000
coconut trees on his lands in Masbate offered for exchange. This was not proven. It do es not appear in the record
that the defendant deliberately violated the truth in stating his belief that there were such a number of coconut trees
on said lands. Furthermore, it was shown that the plaintiff viewed the lands and himself estimated that there
were there more than six thousand coconut trees.

The facts herein proven, considered in the light of the provisions contained in article 1484 of the Civil Code, made
applicable to this case by article 1541 of said Code, prevent us from holding the action brought by the plaintiffs to
be of any merit. They have not established their alleged right to the judgment prayed for in their complaint.

2B 16-17 SALES AND LEASE Page 280


3. E. C. MCCULLOUGH VS R. AENILLE & CO.

G.R. No. 1300 February 3, 1904


Willard, J.

FACTS
August 27, 1901, the parties made a written contract which contained the following clause:

For the purpose of carrying into effect the contract of sale, said Francisco Gonzalez y de la Fuente and Don
Antonio la Puente y Arce, in the name and on behalf of R. Aenlle & Co., solemnly declare that they sell, absolutely
and in fee simple, to E. C. McCullough, the tobacco and cigarette factory known as "La Maria Cristina,said sale
including the trade-mark "La Maria Cristina," the stock of tobacco in leaf and manufactu re, machinery, labels,
wrappers, furniture, fixtures, and everything else belonging to the said factory, all of the same for the following
sums: xxx xxx

This sum is subject to modification, in accordance with the result shown by the inventory to be dr awn up. xxx
xxx

The inventory mentioned in this contract was afterwards made by the defendant and delivered to the plaintiff, xxx ,
and on September 26 the parties executed a second instrument, which contained the following clauses:

That the parties hereto have completed the beforementioned inventory xxx representing a total and effective value
of 131,000 pesos, after deducting the discount agreed upon for each article, xxx and that E. C. McCullough, the
purchaser, remained in possession of the above-mentioned tobacco, and of all its appurtenances and the stock on
hand

That by virtue of the conditions set forth, the parties hereto fix selling price xxx at the said sum of $131,000, on
account of which the vendee, Mr. McCullough, authorizes the vendors, Don Francisco Gonzalez y de la Fuente and
Don Antonio la Puente y Arce, to collect and receive the 20,000 pesos deposited in the Spanish -Philippine Bank for
that purpose and binds himself to pay the said vendees the $111,000 remaining for the complete and t otal payment
of the said purchase price by the 30th day of September, instant, on which date said sum must be paid, and in case
payment shall not be made by Mr. McCullough on said date, the said contract of sale of the said factory will be
rescinded, the said sum of 20,000 pesos before mentioned accruing to the benefit of the representatives of R. Aenlle
& Co

On September 30 they executed a third contract, in which the defendant acknowledged the receipt at that time of the
full purchase price of the sale.

Among other items of leaf tobacco in the inventory were the two following:

1. Y. P. I. 4. S.a Angadanan 99 221 bales, net weight qqs. 571.35 at 40. $22,854.

2. Isabela, 99 loose leaves. 1.a 2.a 3.a 76 bales re-baled, net weight, qqs. 130.32 at 42. $5,473.44.

2B 16-17 SALES AND LEASE Page 281


In December, 1901, the plaintiff, with others, organized a company, to which the plaintiff sold all the tobacco
bought by him from the defendant. The purchaser, the new company, on examining these two lots, rejected them
because the tobacco was not of the quality indicated in the inventory.

Thereupon the plaintiff, claiming that the tobacco in these two lots was worthless, brought this action against the
defendant to recover what he paid therefor, namely, the two sums of $22,854 and $5,473.44.

At the time in question the plaintiff was the owner of a printing establishment and he testified that he desired to
move it to the building in which the defendant had its cigar factory; that it was impossible for him to get the building
without buying the tobacco factory, and for that reason he bought it, intending to sell it as soon as he could without
loss. The said contract of August 27 contained provisions for the leasing and ultimate purchase of the building by
the plaintiff.

The court below found that the first lot was worth at the time of the sale only 8 pesos a quintal instead of 40, the
price paid by the plaintiff; that the second lot was worth 11 pesos instead of 42, and ordered judgment against the
defendant for the difference, which amounted to 24,109.24 pesos. The defendant excepted to the judgment moved
for a new trial on the ground that the evidence was insufficient to support the judgment, and excepted to the order
denying this motion.

ISSUE

Whether or not plaintiff Mccullough can recover purchase price because of the inferior quality of the items sold?
No!

HELD

The document of August 27 was a completed contract of sale. (Art. 1450, Civil Code.) xxx By its terms the appellee
was bound to take all the leaf tobacco then belonging to the facto ry and to pay therefor the prices named in the
invoices. This obligation was absolute and did not depend at all upon the quality of the tobacco or its value. The
appellee did not, in this contract, reserve the right to reject the tobacco if it were not of a specific crop. He did not
buy tobacco of a particular kind, class, or quality. He bought all the tobacco which the appellant owned and agreed
to pay for it what the defendant had paid for it.

There is nothing in this contract to show that he bought 221 bales of fourth-class superior Angadanan of the crop of
1899. xxx If it correctly gave the number of bales and the price paid therefor by the appellant, according to the
invoices, it was a sufficient compliance with the contract. The fact that the tobacco was described as of one class
instead of another would be unimportant. The appellee did not purchase by class or quality, but by quantity

2B 16-17 SALES AND LEASE Page 282


There is no evidence to show that any representations as to the quality of the tobacco were made to the plaintiff by
the defendant prior to the contract of August 27, nor that there was any agreement prior to that time as to an
exhibition of samples nor that the plaintiff prior to that time made any examination or inquiry as to the quality of the
tobacco. The fact is that the plaintiff in order to get the building had to buy the factory and everything that went with
it. He saw himself obliged to take all the tobacco which the defendant had, no matter what its quality was. The
defendant was not willing to sell him the building and the good tobacco which it had on hand, retaining itself of
poorer quality. He had to take it all or not get the building. He probably thought that he was safe in agreeing to pay
no more than the defendant had paid. But, however this may be and wha tever may have been his reasons therefor, it
is certain that the plaintiff bound himself by the contract of August 27 to take all the tobacco which the defendant
then had any pay therefor the prices that the company had paid. He could relieve himself from this obligation only
by showing either that the tobacco in the inventory was not owned by the defendant on August 27 or that the prices
stated therein were not the prices which the defendant paid for it. He undertook to do neither of these things, and his
action must fail. The right to rescind a contract for lesion when the value is less than half of the purchase price,
given by Law 56, title 5, partida 5, has been expressly taken away by article 1293 of the Civil Code. Article 1474 of
the Civil Code has no application in this case. The fact that an article is of one grade or quality instead of another
does not constitute a hidden defect within the meaning of that article.

The judgment below is reversed. We find the facts to be as hereinbefore stated and up on such facts we hold as a
conclusion of law that the plaintiff can not recover. Judgment will be entered that the plaintiff take nothing by the
action and that the defendant recover the costs of both instances, and after the expiration of twenty days the cause
shall be returned to the lower court for execution.

2B 16-17 SALES AND LEASE Page 283


4. CHANG YONG TEK VS GENEROSA SANTOS

G.R. No. 4386. February 24, 1909

Johnson, J

FACTS:

Plaintiff sold to the defendant a certain quantity of tobacco, amounting in all to the sum of P894. The defendant does
not deny the purchase of the tobacco in question nor the amount which was originally promised to be paid for the
same. The plaintiff alleges that P442 of the said amount was to be paid at the end of January, 1904, and that P452 of
said amount was to be paid at the end of the month of September, 1905. No part of the said amount of P894 having
been paid, the plaintiff commenced an action on the 24th of January, 1907, for its recovery.

ISSUE

Whether or not defendant should be held liable for the payment of the amount agreed upon?

HELD

YES, the only defense presented by the defendant was that the tobacco delivered by the plaintiff was not of good
quality. The defendant does not allege, however, that it was not of the quality pu rchased nor that the plaintiff
delivered a different kind or quality of tobacco than that purchased. Neither does the defendant allege that she
intended to purchase or did purchase a particular quality of tobacco and that quality was not delivered. Neither does
the record show that the defendant had made any complaint to the plaintiff concerning the quality of the tobacco or
that it was not the kind of tobacco which she had purchased, until after the present action had been commenced. The
defendant admits that she had sold the tobacco in question.

Nearly three years had elapsed from the time the defendant purchased the tobacco in question until the time
the present action was commenced. The record does not disclose when the defendant sold the tobacco in qu estion.
The defendant does not allege nor attempt to prove that she did not have an opportunity to examine the tobacco
delivered, for the purpose of determining its quality. Neither does the record disclose that the plaintiff made any
false representations with reference to the quality or kind of tobacco sold. Neither does the defendant allege or
attempt to prove that the tobacco in question contained any hidden defects which might not have been disclosed
upon the slightest investigation. There is no attempt to show that the plaintiff undertook to warrant the quality of the
tobacco. In the absence of an express warranty, a vendor or merchandise only warrants; first, the legal and peaceable
possession of the thing sold; and second, that there are no hidden faults or defects therein. (Art. 1474, Civil Code.)

2B 16-17 SALES AND LEASE Page 284


Moreover, it appears from the record that the defendant did examine the tobacco in question at the time of
the sale by opening many of the bundles and examining the contests thereof. It not being proven t hat the plaintiff
made any warranty or any misrepresentations with reference to the quality of the tobacco in question, and it having
been proven that the defendant had an opportunity to and did examine the tobacco in question at the time of
purchase and not having made any objection whatever until after a lapse of more than three years and not then until
after an action had been brought, and making no objection whatever as to the price agreed upon, nor as to the
quantity of the tobacco delivered, in our opinion she should be held liable for the payment of the amount agreed
upon.

2B 16-17 SALES AND LEASE Page 285


5. MARTINEZ VS CA

G.R. No. L31271 April 29, 1974

Esguerra, J

FACTS:

The spouses Romeo Martinez and Leonor Suarez are the registered owners of two (2) parcels of land
located in Lubao, Pampanga. Both parcels of land are fishponds. The property involved in the instant case is the
second parcel mentioned in the above-named TCT.

Potenciano Garcia, from whom the spouses bought the property, was prevented by the then municipal
president of Lubao, Pedro Beltran, from restoring the dikes constructed on the contested property, the former filed a
case with the Court of First Instance against the said Pedro Beltran to restrain the latter in his official capacity from
molesting him in the possession of said second parcel, and on even date, applied for a writ of preliminary injunction,
which was issued against said municipal president.

Potenciano Garcia applied for the registration of both parcels of land in his name. Pu rsuant to the Court's
decision, OCT was issued to the spouses Potenciano Garcia and Lorenza Sioson. These parcels of land were
subsequently bought by Emilio Cruz de Dios. Thereafter, the ownership of these properties changed hands until
eventually they were acquired by the herein appellee spouses.

To avoid any untoward incident, the disputants agreed to refer the matter to the Committee on Rivers and
Streams, it ruled that the disputed properties were not a public river but a private fishpond owned by the h erein
spouses.

Some four years later, Honorable Florencio Moreno, then Secretary of Public Works and Communications,
ordered another investigation of the said parcel of land, directing the appellees herein to remove the dikes they had
constructed, on the strength of the authority vested in him by Republic Act No. 2056. The said order which gave rise
to the instant proceedings, embodied a threat that the dikes would be demolished should the herein appellees fail to
comply therewith within thirty (30) days.

The lower court ruled in favour of the spouses. The Court of Appeals reversed the decision and declared the
land to be a public river. Hence, ordering the return of the property to the government.

ISSUE

Whether or not the decision is a collateral attack to the indefeasibility of the Torrens title issued to the spouses?

HELD

NO! The conclusiveness and incontestability of a Torrens certificate of title do not apply here. The Land
Registration Court has no jurisdiction over nonregisterable properties, such as public navigable rivers which are
parts of the public domain, and cannot validly adjudge the registration of title in favor of a private applicant.

2B 16-17 SALES AND LEASE Page 286


The evidence submitted before the trial court which was passed upon by the respondent Court of Appeals
shows is a river of the public domain.

There is no weight in the appellants' argument that, being a purchaser for value and in good faith of, the
nullification of its registration would be contrary to the law and to the applicable decisions of the Supreme Cou rt as
it would destroy the stability of the title which is the core of the system of registration. Appellants cannot be deemed
purchasers for value and in good faith.

Before purchasing a parcel of land, it cannot be contended that the appellants who were t he vendees did not
know exactly the condition of the land that they were buying and the obstacles or restrictions thereon that may be
put up by the government in connection with their project of converting it into a fishpond.

Nevertheless, they willfully and voluntarily assumed the risks attendant to the sale of said lot. One who
buys something with knowledge of defect or lack of title in his vendor cannot claim that he acquired it in good faith.

The ruling that a purchaser of a registered property cannot g o beyond the record to make inquiries as to the legality
of the title of the registered owner, but may rely on the registry to determine if there is no lien or encumbrances over
the same, cannot be availed of as against the law and the accepted principle t hat rivers are parts of the public domain
for public use and not capable of private appropriation or acquisition by prescription.

2B 16-17 SALES AND LEASE Page 287


6. JERRY MOLES VS IAC

G.R. No. 73913, January 31, 1989

FACTS

Diolosa Publishing House, petitioner together with Rogelio Yusay, a letter press machine operator, decided to buy
the linotype machine, Model 14. The transaction was basically verbal in nature but to facilitate the loan application
with the DBP, a pro forma invoice, dated April 23, 1977 and reflectin g the amount of P50,000.00 as the
consideration of the sale, was signed by petitioner with an addendum that payment had not yet been made but that he
promised to pay the full amount upon the release of his loan from the aforementioned bank on or before the end of
the month. Although the agreed selling price was only P40,000.00, the amount on the invoice was increased by
P10,000.00, said increase being intended for the purchase of new matrices for said machine. Under date of August
29, 1977, private respondent issued a certification wherein he warranted that the machine sold was in A -1 condition,
together with other express warranties. On November 29, 1977, petitioner wrote private respondent that the machine
was not functioning properly as it needed a new distributor bar. Private respondent made no reply to said letter, so
petitioner engaged the services of other technicians. Later, after several telephone calls regarding the defects in the
machine, private respondent sent two technicians to make the necessary repairs but they failed to put the machine in
running condition. In fact, since then petitioner was never able to use the machine. An expert witness for the
petitioner, one Gil Legaspina, declared that he inspected the linotype machine involved in this case at the instance of
petitioner. In his inspection thereof, he found the following defects. It turned out that the said linotype machine was
the same machine that witness Legaspina had previously inspected for Sy Brothers, a firm which also wanted to bu y
a linotype machine for their printing establishment. Having found defects in said machine, the witness informed Sy
Brother about his findings, hence the purchase was aborted. In his opinion, major repairs were needed to put the
machine back in good running condition.

ISSUE

Whether or not there is an Implied Warranty, as such if whether or nor rescission s warranted

HELD

Now, when an article is sold as a secondhand item, a question arises as to whether there is an implied warranty of its
quality or fitness. It is generally held that in the sale of a designated and specific article sold as secondhand, there is
no implied warranty as to its quality or fitness for the purpose intended, at least where it is subject to inspection at
the time of the sale. On the other hand, there is also authority to the effect that in a sale of a secondhand articles
there may be, under some circumstances, an implied warranty of fitness for the ordinary purpose of the article sold
or for the particular purpose of the buyer.

Thus, in finding for private respondent, the respondent court cited the ruling in Sison vs. Ago, et al. to the effect that
unless goods are sold as to raise an implied warranty, as a general rule there is no implied warranty in the sale of
secondhand articles.

2B 16-17 SALES AND LEASE Page 288


Said general rule, however, is not without exceptions. Article 1562 of our Civil Code, which was taken from the
Uniform Sales Act, provides:

Art. 1562. In a sale of goods, there is an implied warranty or condition as to the quality or fitness of the goods, as
follows:

(1) Where the buyer, expressly or by implication, makes known to the seller the particular purpose for which the
goods are acquired, and it appears that the buyer relies on the seller's skill or judgment (whether he be the grower or
manufacturer or not), there is an implied warranty that the goods shall be reasonably fit for such purpose;

It must be remembered that the certification was a condition sine qua non for the release of petitioner's loan which
was to be used as payment for the purchase price of the machine. Private respondent failed to refute this material
fact. Neither does he explain why he made that express warranty on the condition of the machine if he had not
intended to be bound by it. In fact, the respondent court, in de claring that petitioner should have availed of the
remedy of requiring repairs as provided for in said certification, thereby considered the same as part and parcel of
the verbal contract between the parties.

On the basis of the foregoing circumstances, the inescapable conclusion is that private respondent is indeed bound
by the express warranty he executed in favor of herein petitioner.

We disagree with respondent court that private respondents express warranty as to the A -1 condition of the machine
was merely dealer's talk. Private respondent was not a dealer of printing or linotype machines to whom could be
ascribed the supposed resort to the usual exaggerations of trade in said items. His certification as to the condition of
the machine was not made to induce petitioner to purchase it but to confirm in writing for purposes of the financing
aspect of the transaction his representations thereon. Ordinarily, what does not appear on the face of the written
instrument should be regarded as dealer's or trader's talk; conversely, what is specifically represented as true in said
document, as in the instant case, cannot be considered as mere dealer's talk.

On the question as to whether the hidden defects in the machine is sufficient to warrant a rescission of th e contract
between the parties, we have to consider the rule on redhibitory defects contemplated in Article 1561 of the Civil
Code. A redhibitory defect must be an imperfection or defect of such nature as to engender a certain degree of
importance. An imperfection or defect of little consequence does not come within the category of being redhibitory.

As already narrated, an expert witness for the petitioner categorically established that the machine required major
repairs before it could be used. This, plus the fact that petitioner never made appropriate use of the machine from the
time of purchase until an action was filed, attest to the major defects in said machine, by reason of which the
rescission of the contract of sale is sought. The factual finding, therefore, of the trial court that the machine is not
reasonably fit for the particular purpose for which it was intended must be upheld, there being ample evidence to
sustain the same.

2B 16-17 SALES AND LEASE Page 289


At a belated stage of this appeal, private respondent came up for th e first time with the contention that the action for
rescission is barred by prescription. While it is true that Article 1571 of the Civil Code provides for a prescriptive
period of six months for a redhibitory action a cursory reading of the ten preceding articles to which it refers will
reveal that said rule may be applied only in case of implied warranties. The present case involves one with and
express warranty. Consequently, the general rule on rescission of contract, which is four years 27 shall apply.
Considering that the original case for rescission was filed only one year after the delivery of the subject machine, the
same is well within the prescriptive period. This is aside from the doctrinal rule that the defense of prescription is
waived and cannot be considered on appeal if not raised in the trial court, 28 and this case does not have the features
for an exception to said rule.

2B 16-17 SALES AND LEASE Page 290


7. ENGINEERING & MACHINERY CORPORATION vs. COURT OF APPEALS and PONCIANO L.
ALMEDA

G.R. No. 52267. January 24, 1996

Panganiban, J

FACTS:

Pursuant to the contract dated September 10, 1962 between petitioner and private respondent, the former undertook
to fabricate, furnish and install the air-conditioning system in the latters building along Buendia Av enue, Makati in
consideration of P210,000.00. Petitioner was to furnish the materials, labor, tools and all services required in order
to so fabricate and install said system. The system was completed in 1963 and accepted by private respondent, who
paid in full the contract price.

On September 2, 1965, private respondent sold the building to the National Investment and Development
Corporation (NIDC). Howevwer, due to NIDCs noncompliance with the terms and conditions of the deed of sale,
private respondent was able to secure judicial rescission thereof. It was then that he learned from some NIDC
employees of the defects of the air-conditioning system of the building.

Acting on this information, private respondent commissioned Engineer David R. Sapico to rend er a technical
evaluation of the system in relation to the contract with petitioner. In his report, Sapico enumerated the defects of the
system and concluded that it was not capable of maintaining the desired room temperature of 76F - 2F. On the basis
of this report, private respondent filed an action for damages against petitioner, complaint alleged that the air-
conditioning system installed by petitioner did not comply with the agreed plans and specifications. Petitioner
moved to dismiss the complaint, alleging that the prescriptive period of six months had set in pursuant to Articles
1566 and 1567, in relation to Article 1571 of the Civil Code, regarding the responsibility of a vendor for any hidden
faults or defects in the thing sold.

Private respondent countered that the contract dated September 10, 1962 was not a contract of sale but a contract for
a piece of work under Article 1713 of the Civil Code. Thus, in accordance with Article 1144 (1) of the same Code,
the complaint was timely brought within the ten-year prescriptive period.

In due course, the trial court rendered a decision finding that petitioner failed to install certain parts and accessories
called for by the contract, and deviated from the plans of the system, thus reducing its operational e ffectiveness to
the extent that 35 window-type units had to be installed in the building to achieve a fairly desirable room
temperature. On the question of prescription, the trial court ruled that the complaint was filed within the ten -year
prescriptive period although the contract was one for a piece of work, because it involved the installation of an air-
conditioning system which the defendant itself manufactured, fabricated, designed and installed.

Petitioner appealed to the Court of Appeals, which affirmed the decision of the trial court. Hence, it instituted the
instant petition.

ISSUE

Whether or not the action for breach of warranty against hidden defects has prescribed?

2B 16-17 SALES AND LEASE Page 291


HELD

No. Where there is an express warranty in the contract, as in the case at bench, the prescriptive period is the one
specified in the express warranty, and in the absence of such period, the general rule on rescission of contract, which
is four years shall apply. However, a close scrutiny of the complaint filed in the trial court reveals that the original
action is not really for enforcement of the warranties against hidden defects, but one for breach of the contract itself.

From all of the foregoing, the Court is persuaded to believe the plaintiff that not only had the defendant failed to
install items and parts provided for in the specifications of the air-conditioning system be installed, like face and by-
pass dampers and modulating thermostat and many others, but also that there are items, parts and accessories which
were used and installed on the air-conditioning system which were not in full accord with contract specifications.
These omissions to install the equipments, parts and accessories called for in the specifications of the contract, as
well as the deviations made in putting into the air-conditioning system equipments, parts and accessories not in full
accord with the contract specification naturally resulted to adversely affect the operational effectiveness of the air -
conditioning system which necessitated the installation of thirty-five window type of air-conditioning units
distributed among the different floor levels in order to be able to obtain a fairly desirable room temperature for the
tenants and actual occupants of the building. The Court opines and so holds tha t the failure of the defendant to
follow the contract specifications and said omissions and deviations having resulted in the operational
ineffectiveness of the system installed makes the defendant liable to the plaintiff in the amount necessary to rectify
to put the air conditioning system in its proper operational condition to make it serve the purpose for which the
plaintiff entered into the contract with the defendant.

Having concluded that the original complaint is one for damages arising from breach o f a written contract - and not
a suit to enforce warranties against hidden defects - we herewith declare that the governing law is Article 1715.
However, inasmuch as this provision does not contain a specific prescriptive period, the general law on prescription,
which is Article 1144 of the Civil Code, will apply. Said provision states, inter alia, that actions upon a written
contract prescribe in ten (10) years. Since the governing contract was executed on September 10, 1962 and the
complaint was filed on May 8, 1971, it is clear that the action has not prescribed.

2B 16-17 SALES AND LEASE Page 292


8. CARLOS DE GUZMAN VS TOYOTA CUBAO

G.R. No. 141480 November 29, 2006

FACTS:

On November 27, 1997, petitioner purchased from respondent a brand new white Toyota Hi-Lux. The vehicle was
delivered to petitioner two days later. On October 18, 1998, Mr. de Guzman demanded for the replacement of the
engine of the vehicle because it showed a crack during a trip when the vehicle passed by Marcos Highway during
heavy rain. Mr. De Guzman demanded for the Toyota dealer to replace the engine with a new one based on an
implied warranty.

On the other hand, Toyota Cubao maintains that Mr De Guzman's claim for replacement was already barred by the
statute of limitations amd had therefore prescribed under Article 1571 of the Civil Code for claiming cause of action
for more than six months from the time the vehicle was purchased and/or delivered. Respondent reiterates that
Article 169 of Republic Act No. 7394 does not apply.

The RTC granted the respondents motion and dismissed the complaint, agreeing with the respondents that the
subject car was a consumer product because it is used for personal, family, a gricultural purposes, contrary to
defendant counsels claim that it is not because it is a non -consumable item. Further, the RTC ruled that Since no
warranty card or agreement was attached to the complaint, the contract of sale of the subject pick-up carried an
implied warranty that it was free from any hidden faults or defects, or any charge or encumb rance not declared or
known to the buyer. The prescriptive period thereof is six (6) months under the Civil Code.

ISSUE

Whether or not petitioner can validly demand for the replacement of the car

HELD:

No.

Under Article 1599 of the Civil Code, once an express warranty is breached, the buyer can accept or keep the goods
and maintain an action against the seller for damages. In the absence of an existing express warranty on the part of
the respondent, as in this case, the allegations in petitioners comp laint for damages were clearly anchored on the
enforcement of an implied warranty against hidden defects, in which in this case, that the engine of the vehicle
which Toyota had sold to Mr. de Guzman was not defective.

By filing this case, Mr. de Guzman wants to hold Toyota responsible for breach of implied warranty for having sold
a vehicle with defective engine. Such being the case, petitioner should have exercised this right within six months
from the delivery of the thing sold. Since petitioner filed th e complaint on April 20, 1999, or more than nineteen

2B 16-17 SALES AND LEASE Page 293


months counted from November 29, 1997 (the date of the delivery of the motor vehicle), his cause of action had
prescribed.

2B 16-17 SALES AND LEASE Page 294


9. JAIME ANG VS CA

GR No. 177874; September 29, 2008

Carpio-Morales, J.

FACTS

Respondent Soledad sold his Mitsubishi GSR sedan 1982 model to petitioner Ang by Deed of Absolute Sale. The
latter conveyed to the former his Mitsubishi Lancer model 1988 and an additional P55,000 also by Deed of Absolute
Sale. Ang later on sold the GSR through Far Eastern Motors to a certain Bugash on August 14, 1992. Before the sale
was registered, the vehicle was seized by a Writ of Replevin pursuant to a case between BA Finance Corporation
and Panes (vehicle owner prior to Soledad). The latter failed to pay an indebtedness in favor of the bank, this the
mortgage was foreclosed.

Ang paid BA Finance to secure the release of the vehicle. Soledad refused to pay him back. Ang charged him for
estafa which was dismissed for lack of evidence. Ang filed a case for damages but was dismissed for failure to
submit to baranggay conciliation. Further cases were filed but were again dismissed. Finally, RTC rendered a
decision in favor of Ang pursuant to justice and equity and the principle of unjust enrichment. Upon appeal, CA
reversed the decision on the ground of prescription.

ISSUES

(1) Whether the action has prescribed.


(2) Whether there has been a breach of warranty under the contract of sale.

HELD

A warranty is a statement of representation made by the seller of the goods, contemporaneously and as part of the
contract of sale, having reference to the character, quality or title of the goods, and by which he promises or
undertakes that certain facts are or shall be as he represents them. It may b e express as provided for under Article
1546 or implied as when it is derived by law by application or inference from the nature and transaction or the
relative situation or circumstances of the parties, irrespective of any intention of the seller to creat e it.

The prescriptive period for instituting actions for breach of an express warranty is that specified in the contract, in
the absence of such, 4 years. As for implied warranties, they prescribe 6 months from the date of the delivery of the
thing sold.

Soledad actually gave an implied warranty of title in the contract since he declared that he owned and had clean title
to the vehicle at the time the Deed of Absolute Sale was executed. He also gave a warranty against eviction for
pledging that he will defend the title to it. Given the nature of Angs business as a car dealer he could not have relied

2B 16-17 SALES AND LEASE Page 295


solely on the warranties and representations of Soledad. Since what was given is a implied warranty, the prescriptive
period is 6 months from the date of delivery of the thing sold. The action was filed beyond the prescriptive period.

The action for damages for breach of contract will likewise not prosper even if the same had been invoked timely
since the requisites have not been met. The following are the requisites: (a) the purchaser has been deprived of the
ole or a part of the thing sold, (b) the eviction is by virtue of a final judgement, (c)basis of the final judgment is a
right existing prior to the sale, and (d) the vendor has been summoned in the suit for eviction at the instance of the
vender. There was no judgment which deprived Ang of the vehicle. Neither was there a suit where Soledad was
impleaded.

Finally, Ang settled the mortgage debt in his own volition. Thus, he cannot claim any refund from Soledad. Ang
paid in order to not be obliged to return the payment made by Bugash. Moreover, Soledad was not in any way
benefitted by the payment of Ang.

2B 16-17 SALES AND LEASE Page 296


10. COCA-COLA BOTTLERS PHILS INC VS CA

G.R. No. 110295 October 18, 1993

Davide Jr, J

FACTS

Private respondent Lydia Geronimo owned and operated Kindergarten Wonderland Canteen in Dagupan City which
catered to the students of Kindergarten Wonderland and the general public. Upon receiving complaints from parents
of the students that they found fiber-like matter and some other foreign particles in the Coke and Sprite bottles she
had sold, the private respondent examined those in stock and brought the same to the Regional Health Office of La
Union for examination. The results yielded that the substances were adulterated and due to such, private
responded suffered decrease in sales for the soft drinks (P200-300 loss/day) which ultimately led to her losing the
shop.

Private respondent filed a case for damages, and petitioner thereafter moved to dismiss the c omplaint on the grounds
of failure to exhaust administrative remedies and prescription. Petitioner contended that the action should have been
brought within 6 months as it is an action based on a breach of implied warranty under Article 1561 of the Civil
Code because there was no tortious or wrongful conduct alleged in the complaint.

On the other hand, private respondent argued that her claim for damages was based on injury to her rights and was
timely brought under Article 1146 (4 years). Additionally, s he posited that Article 1567 provides for rescission or
proportionate reduction, with damages in either case; she only prayed for damages arising from quasi-delict.

The trial court ruled in favor of petitioner and noted that there was a pre -existing contractual relations between the
parties and as such, the action for damages was based on breach for implied warranty and the same should have been
brought within 6 months.

ISSUE

Is the action for damages one for breach of implied warranty for hidden defects , which should be filed within 6
months, or for quasi-delict, which should be filed within 4 years?

HELD

The Supreme Court held that the action for damages was based on quasi-delict which should be filed within 4 years.
Contrary to petitioners contention, the complaint alleged reckless and negligent manufacture of adulterated food
items intended to be sold for public consumption. Additionally, the existence of a contract between parties does not
bar the commission of a tort and the recovery of damages therefrom; an action for breach of warranty does not bar
an action for torts.

2B 16-17 SALES AND LEASE Page 297


ON THE OBLIGATIONS OF THE VENDEE

1. LAFONT VS PASCASIO

G.R. No. 1556, December 7, 1905

Willard, J

FACTS:

Joaquin Lafont, was the owner of an undivided half of the building known as the Philippine Theater, in
Manila, and the appellee, Maria Yia Pascasio, was the owner of the other half. On the 13th day of November, 1899,
the two of the, entered into a contract of sale wherein Lafont will transfer his ownership to Pascasio, with the
reservation for his right to repurchase the same. Thereafter, Lafont manifested his intention to repurchase the
property to which Pascasio answered stating that upon receipt of the purchase price together with the payment for
the expenses incurred, she will execute the necessary deed. Thereafter, Lafont deposited P1,000.00 in the CFI as the
sum required by him and thereafter filed a complaint asking the court to declare, among others, that his right to
repurchase still existed.

ISSUE:

Whether or not the obligations of paying the money and executing the deed are simultaneous in a pacto de
retro sale.

HELD:

YES. In a contract of pacto de retro the return of the money and the execution of the deed of resale are simultaneous
acts. In a contract of purchase and sale the seller is not required to deliver the thing sold until the price is paid, in the
absence of an agreement to the contrary (art. 1466). Neither is the purchaser bound to pay the price before the article
is delivered to him (art 1500), and we hold in this case that the payment of the price and the execution of the deed of
resale were simultaneous acts, and that the plaintiff was not required to deliver the money in advance of the
execution of the deed.

2B 16-17 SALES AND LEASE Page 298


2. ENGEL VS VELASCO & COM 47 PHIL

GR Nos. 2165121653. December 29, 1924

Street, J

FACTS

The plaintiffs Engel, Upmann & Co., were export brokers, or jobbers, of textile merchandise in the City of New
York, while the defendant (Mariano Velasco & Co.,) of a large store in Manila where general merchandise is sold
both at wholesale and retail. In connection with this business the defendant from time to time has occasion to import
textile fabrics on a large scale. In the beginning of the year 1920 commercial relations were established between the
plaintiffs and the defendant, and in the succeeding three months the defendant, sent to the plaintiffs numerous orders
for merchandise. The defendant would first obtain from the plaintiffs by cable information as to the prices of the
goods desired, and would thereupon send a cablegram to the plaintiffs, instructing them to buy and hold specified
qualities of goods in the amount and at the prices stated. Contemporaneously with the sending of the cablegram the
defendant would dispatch by mail more extended instructions, confirming the cablegram and giving such other
advice as was desirable.

as many as 37 orders were given by the defendant to the plaintiffs beginning in the month of January, 1920. A
number of these orders were duly honored by the defendant upon the receipt of the goods and the price paid in due
course.

16 or 17 orders appear to have been promptly placed with the manufacturers by the plaintiffs, but delay occurred in
the matter of shipment; and when delivery was finally tendered in Manila, acceptance was refused. These goods
were thereupon sold by the plaintiffs in Manila and claim made upon the defendant for the difference between the
amount realized and the contract price.

The other goods were sold in New York City after it was found that said goods would not be accepted in Manila.

Plaintiff claim breach of contract on the part of Defendant in its failure to accept and to pay for the goods covered by
the orders referred to.

Defendant assail the admissibility of the correspondence by cable, the delay in shipment and even the different yard
length than what was ordered, money scarcity and drop in prices as defense to avoid its liability.

Trial court ruled in favor of plaintiff to claim from defendant.

Defendant appealed.

ISSUE

Whether or not Defendant is liable

HELD

Yes. The relationship between plaintiff and defendant is one of buyer and seller.

2B 16-17 SALES AND LEASE Page 299


From the correspondence it is plain that the defendant is not in a position successfully to invoke delay in the making
of shipments as ground for its release from the obligation to pay for the merchandise. When the defendant found
itself caught with these large orders in a paralyzed market, the only hope that presented itself to the defendant's
officers was that the shipments might be. delayed for a few months until business should improve, as was expected
would be the case in the autumn. The requests for delay contained in the cables and letters of July 7 and September
14 were accordingly dispatched, and the plaintiffs were kept well informed as to the situation in which the defendant
was placed. The requests for delay, as well as the proven inability of the defendant to comply with its promise to
supply the credit necessary to move the goods, completely enervate the delay in the shipments of the goods,
considered as a ground for the dissolution of the defendant's obligations. It should be noted that the cable and the
letter of September 14 were dispatched at a date subsequent to the times originally stipulated for shipments of most
of the goods, indicating that the defendant waived the delay in delivery. Upon referring to the plaintiffs' acceptances
it will be noted that only one small shipment of moderate amount was to be shipped in December, while only three
others were scheduled for dates as late as September. All the other shipments had been intended for dates then
already past, namely, in May, June, July and August.

We may observe in passing that it is not necessary to hold that the original contracts were abrogate d and a new
agreement substituted by mutual agreement of the parties as a consequence of the communications above
mentioned; It is enough to say that delay in the shipment of the goods was favorable to the defendant and was in
effect requested by it.

Other contentions advanced for the defendant as justifications of the nonacceptance of the goods are based upon
minor deviations in the consignments from the literal terms of the defendant's orders. In this connection we note: (1)
That the length of some of the pieces of merchandise shipped by the plaintiffs were not exactly fifty yards as
specified in the written order, but deviated therefrom in some instances to the extent of several yards; (2) that some
of the orders of the defendant were delivered in single shipments whereas partial shipments had been specified in the
orders, and vice versa; (3) that the plaintiffs had shipped some of the goods in question after they had received the
cablegram of November 9 instructing them to cease shipments; (4) and, finally, that the dates of contemplated
delivery specified in the sales notes did not conform in all respects to the exact terms of the written orders.

As to the deviation in the lengths of pieces, it appears that manufacturers of fabrics of the kind here in que stion do
not put up the goods in pieces of exactly fifty yards length, the practice being to make the pieces of about the
specified lengths. In conformity with this usage, the private code of the plaintiffs, which was being used by the
defendant, was so constructed that the code words referring to particular qualities of goods indicated pieces of a
length of about fifty yards; and the defendant's cablegrams therefore called for pieces of this length. In preparing the
written orders, however, corresponding to some of these cablegrams, the individual in charge of defendant's
correspondence department by mistake indicated lengths of fifty yards without qualification, instead of about fifty
yards, in some of the orders. The plaintiffs' sales notes corresponded with the cabled order, with the result that some
deviation from the stated length was discovered in these lots, the pieces being longer than had been called for. As
the goods were to be paid for by the yard, the resulting price was somewhat greater than it otherwise would have
been.

With reference to the irregular deliveries, namely, delivery by partial shipments where delivery of the whole was
called for, or of the whole where partial shipments were called for, we note that in one or more instances the
defendant's written orders specified the manner of delivery, and that these instructions were not in all cases observed
by the plaintiffs. The explanation given of this is that in view of the delays that had occurred the plaintiffs
considered that the advice to ship part of an order on one boat and part on the following boat had lapsed.
Furthermore, in view of the confessed inability of the defendant to finance the shipments, the plaintiffs say that they
shipped as much as they could on their own credit but were not able to ship all; and, therefore, in the exercise of a

2B 16-17 SALES AND LEASE Page 300


fair discretion, and with a view to relieving the defendant as much as possible, they thought best to distribute their
shipments over several classes of goods rather than to ship one or more larg er orders entire.

In regard to the plaintiffs' shipment of goods after the receipt of the cablegram of November 9 from the defendant
instructing the plaintiffs to cease shipment, the plaintiffs suggest that said cablegram was not interpreted as a flat
repudiation of the contract but was taken rather as a temporary cry of distress, owing to the inability of the defendant
to meet its financial engagements; and it was believed that the plaintiffs' representative, Mr. Kummer, could adjust
the matter by making arrangements f or long extensions in Manila. Some of the goods were therefore forwarded that
might have been stopped; but others had already been embarked and the shipment could not be recalled though the
sailing of the vessel occurred after the cablegram was received.

None of the contentions above referred to, nor others of less moment, constitutes in our opinion any sufficient
ground for absolving the defendant from liability.

The question about the undue delay in the shipment of the goods was never raised by the defendant until the drafts
covering the goods shipped in the autumn began to arrive in Manila and were presented by the bank to the
defendant. Then, finding itself without funds and unable to confront the situation, the defendant put forth the claim
that the shipment of the goods had been out of time.

It is true that considerable time elapsed between default and the dates of the sales; but it does not appear that it
would have been practicable to have made the sales sooner, and the proofs shows that the market price of textiles at
the time the sales were effected were at least equal to the market price at the date of defendant's default, while the
two lots which were sold latest sold for a higher price. The defendant was therefore not prejudiced by t his delay.

2B 16-17 SALES AND LEASE Page 301


3. CASTLE BROTHERS ET AL VS GUTIERREZ HERMANOS

G.R. No. L-4232 April 1, 1908

Willard, J

FACTS

Plaintiffs closed a contract with the defendants, by which the latter agreed to sell the plaintiffs 500 bales or hemp at
P24 picul. The contract was a verbal one. Plaintiff testified that he bought and respondent agreed to sell him 500
bales of good current Manila. Meanwhile, respondent denied that the phrase good current was ever mentioned
in the conversation but a number of evidence corroborative of the claim of the plaintiff was found by the court

ISSUE

Whether or not plaintiff had the right to inspect goods bought (Yes)

HELD

Whether the hemp is sold by the marks or by the grades above mentioned, the buyer always has the right to inspect it
for the purpose of seeing if the quality of the hemp corresponds to that indicated by the mark, when the sale is made
with reference to the mark, or if it is sold with reference to one of the other grades, such as "fair current," to see if it
is equal to that grade.

The court below found upon the facts in favor of the plaintiffs and we think that the evidence sustains such finding.

On the 500 bales, delivered, the plaintiffs accepted and paid for 210 bales and no question as to these bales on mad e
in the case. They refused to accept 299 bales on the ground that it was not good current Manila hemp, called upon
the defendants to furnish 299 bales of that quality and notified them that, on failure to do so, they would buy the
same in the market and charge the increased cost of the defendants. The defendants refused to substitute other bales
and the plaintiffs bought 299 bales of good current Manila hemp at P28.50 a picul, P4.50 more per picul than the
price at which the defendants had agreed to furnis h them that quality of hemp. The loss to the plaintiffs was
therefore P2,691 and to recover that sum this action was brought.

2B 16-17 SALES AND LEASE Page 302


4. DELA CRUZ VS LEGASPI

G.R. No. L-8024 November 29, 1955

Bengzon, J

FACTS

Eusebio de la Cruz sued Apolonio Legaspi and his wife to compel delivery of the parcel of land they had sold to
him. The complaint alleged the execution of the contract, the terms thereof, the refusal of defendants to accept
payment of the purchase price of P450 which he had tendered, and undue retent ion of the realty.

The defendants, in their answer, admitted the sale and the price; but they alleged that before the document (of sale)
"was made, the plaintiff agreed to pay the defendants the amount of P450 right after the document is executed that
very day December 5, 1949, but after the document was signed and ratified by the Notary Public and after the
plaintiff has taken the original of the said document, the sad plaintiff refused to pay the sum of P450. They asserted
that for lack of consideration and for deceit, the document of sale should be annulled.

The judge, rendered judgment (a) ordering plaintiff to pay the price of P450 to defendants: (b) ordering the latter to
receive such price and immediately after such receipt, to deliver possession of t he property to plaintiff.

The defendants appealed. They contended that the contract should be rescinded because it lacks consideration.

ISSUE

Whether not not the contract lacks consideration and rescission is proper.

HELD

The defendants err err in the as sertion that as plaintiff failed to pay the price after the execution of the document of
sale as agreed previously, the contract became null and void for lack of consideration. It cannot be denied that when
the document was signed the cause or consideration existed: P450. The document specifically said so; and such was
undoubtedly the agreement. Subsequent non-payment of the price at the time agreed upon did not convert the
contract into one without cause or consideration: a nudum pactum.

The situation was rather one in which there is failure to pay the consideration.

In addition, rescission is not proper because defendants had not made a previous demand on him, by suit or notarial
act. Art. 1504 states that, he vendee may pay even after the expiration of t he period, at any time before demand has
been made upon him either by suit or by notarial act. After such demand has been made the judge cannot grant him
further time.

2B 16-17 SALES AND LEASE Page 303


5. RODOLFO ALFONSO VS CA

G.R. No. L-63745, June 8, 1990

Narvasa, J.:

FACTS

Private respondents owned a 239 sq. meter lot in Sta. Cruz, Laguna. They entered into a verbal agreemen t with the
petitioners, to which the petitioners paid P2,000 in advance. They executed a private agreement in which the
Chancos accept the P2000 as downpayment for the lot. Petitioners claim that they would pay the remaining P4000
upon obtaining a loan with the PNB, and that they offered payment on Oct. 6, 1973, but were refused, and that
respondents sold the same to the Namit Spouses. Respondents claim, on the other hand, that they were in dire need
of money, so they only gave the Alfonso Spouses one week to pay the remaining balance. They claim that when the
Alfonsos failed to pay, the respondents told them that they were cancelling the sale, and that they tried to return the
P2000, but that the Alfonsos refused. The petitioners brought suit to cancel the sale to the Namits, and to convey the
land to them. The Alfonsos argue that the Chancos made no demand by judicial or notarial notice. The lower court
ruled in favor of petitioners, while the CA reversed the ruling.

ISSUE

Whether or not the lot may conveyed to the petitioner

HELD

No. The contract here is an oral contract to sell. Article 1592 requiring judicial or notarial acts, on which the
petitioners rely, cannot be made to apply. The absence of a formal deed conveyance indicated that the respondents
only wanted to transfer the property upon full payment of the price.

2B 16-17 SALES AND LEASE Page 304


6. ABELARDO VALARAO, GLORIOSA VALARAO and CARLOS VALARAO vs. COURT OF
APPEALS and MEDEN A. ARELLANO

G.R. No. 130347 March 3, 1999

Panganiban, J

FACTS:

7, spouses Abelardo and Gloriosa Valarao, thru their son Carlos Valarao as their attorney -in-fact, sold to Private
Respondent Meden Arellano under a Deed of Conditional Sale a parcel of land situated in the District of Diliman, Q.
C., covered by TCT No. 152879 with an area of 1,504 square meters, for the sum of P3,225,000.00 payable under a
schedule of payment stated therein. Respondent obligated herself to encumber her separate piece of property for
security. It was further stipulated that a failure of 3 consecutive monthly payments anyone year-end lump sum
payment within the period stipulated, the sale shall be considered automatically rescinded without the necessity of
judicial action and all payments made by the vendee shall be forfeited in favor of the vendors by way of rental.

Private respondent appellant alleged that as of September, 1990, she had alre ady paid the amount of P2,028,000.00
pesos, although she admitted having failed to pay the installments due in October and November, 1990. Petitioner,
however, had tried to pay the installments due in the said months, including the amount due in the month of
December, 1990 on December 30 and 31, 1990, but was turned down by the vendors -petitioners thru their maid,
Mary Gonzales, who refused to accept the payment offered upon orders from her employer.

When all her efforts to make payment were unsuccessful, private respondent sought judicial action. by filing this
petition for consignation on January 4, 1991. On the other hand, vendors -petitioners, sent private respondent a letter
notifying her that they were enforcing the provision on automatic rescission.

ISSUES

(1) Whether the stipulation declaring the contract rescinded upon failure to pay is tantamount to a judicial demand
and notice of rescission under Art. 1592 of the Civil Code.

(2) Whether the automatic forfeiture clause is valid and binding between the parties.

HELD

1. The issue of whether the requirement of a judicial demand or a notarial act has been fulfilled is immaterial to the
resolution of the present case. Article 1592 of the Civil Code. states:

Art. 1592. In the sale of immovable property, even though it may have been stipulated that upon failure to pay the
price at the time agreed upon the rescission of the contract shall of right take place, the vendee may pay, even after
the expiration of the period, as long as no demand for rescissio n of the contract has been made upon him either
judicially or by notarial act. After the demand, the court may not grant him a new term.

2B 16-17 SALES AND LEASE Page 305


It is well-settled that the above-quoted provision applies only to a contract of sale, and not to a sale on installment
or a contract to sell. In the present case, the Deed of Conditional Sale is of the same nature as a sale on installment
or a contract to sell, which is not covered by Article 1592.

2. Yes. We concede the validity of the automatic forfeiture clause, which deems any previous payments forfeited
and the contract automatically rescinded upon the failure of the vendee to pay three successive monthly installments
or any one yearend lump sum payment. However, petitioners failed to prove the conditions that would warrant the
implementation of this clause.

it is clear that petitioners were not justified in refusing to accept the tender of payment made by private respondent
on December 30 and 31, 1990. Had they accepted it on either of said dates, she would have paid all three monthly
installments due. In other words, there was no deliberate failure on her part to meet her responsibility to pay. The
Court takes note of her willingness and persistence to do so, and, petitioners cannot now say otherwise. The fact is:
they refused to accept her payment and thus have no reason to demand the enforcement of the automatic forfeiture
clause.

2B 16-17 SALES AND LEASE Page 306


7. PILAR OCAMPO VS CA

G.R. No. 97442 , June 30, 1994

Bellosillo, J

FACTS

The property in question is registered under OCT No. 0-7743 in the name of seller Severino Tolosa. Tolosa
mortgaged the land to the Philippine Veterans Bank and had the encumbrance annotated.

Tolosa and Pilar T. Ocampo, the latter being then represented by Teresa T. Borres, entered into an
"Agreement to Sell Real Property" whereby Tolosa "sells, cedes and transfers" the land to Ocampo in consideration
of P25,000.00, P12,500.00 of which was paid upon signing of the deed and the balance to be due within six (6)
months thereafter. Paragraph 4 of the contract provides that "immediately upon complete payment of the purchase
price . . . by the VENDEE, the VENDOR . . . agrees to execute and deliver unto the VENDEE whatever pertinent
document or documents necessary to implement this sale and to transfer title to the VENDEE." Ocampo failed to
complete her payment. Nonetheless, Tolosa still accepted her late payments. Upon learning of the mortgage lien,
Ocampo caused her adverse claim to be annotated on Tolosas certificate.

Meanwhile, Tolosa and Magdalena S. Villaruz executed a "Contract to Sell". The contract stipulated the
immediate conveyance of the physical possession of the land to Villaruz, although no deed of definite sale would be
delivered to her unless the price was fully paid. Tolosa wrote Ocamp o offering to reimburse her what she paid
provided she would sign a document canceling her adverse claim. Failing to convince Ocampo, Tolosa filed a
petition in the Court of First Instance of Iloilo to cancel the adverse claim of Ocampo. The same was denie d.
However, during the pendency of the civil case, Tolosa succeeded in securing from another branch of the court the
cancellation of the adverse claims of Ocampo without notice to her. Consequently, the contract of sale of Villaruz
was registered and TCT was subsequently issued in her name.

The RTC rendered a decision dismissing the complaint of Tolosa as well as the complaint in intervention of
Villaruz, declaring the contract to sell executed between plaintiff Severino Tolosa and third -party defendant
Magdalena Villaruz as null and void as well as the Transfer of Certificate of Title issued in connection therewith, if
any On appeal, the CA reversed and set aside the trial courts decision. It upheld the sale in favor of Villaruz on the
theory that the agreement of Tolosa and Ocampo was merely a contract to sell. It claimed that in the absence of a
deed of absolute sale in favor of Ocampo, in relation to par. 4 of the contract, Tolosa retained ownership over the
land and validly conveyed the same to Villaruz. On the other hand, the agreement between Tolosa and Ocampo
although titled "Agreement to Sell Real Property" was a perfected contract of absolute sale wherein Tolosa forthwith
sold, ceded and transferred the land to Ocampo.

ISSUE

Whether or not par. 4 of the contract between Tolosa and Ocampo was merely a contract to sell.

2B 16-17 SALES AND LEASE Page 307


HELD

NO. The Court ruled that Paragraph 4 pertains to the undertaking of the seller to execute and deliver to the buyer
any document deemed necessary by law to implement the sale and transfer title since the parties were unsure of what
documents were pertinent. If the intent was for the seller to retain ownership and possession of the land through non -
delivery of certain documents unless the price be fully paid, par. 4 alone should be inutile; it should have been
complemented with a proviso that the sale would not be implemented nor the title considered transferred unless
another document specifically for said purpose be first executed and delivered to the buyer. In this regard, no right to
retain ownership and possession of the land pending full payment of the price can be inferred from the fact that no
delivery was made to Ocampo. The failure of the buyer to pay the price in full within a fixed period does not,
by itself, bar the transfer of the ownership or possession, much less dissolve the contract of sale. Under Art.
1592 of the Civil Code, the failure of Ocampo to complete her payment of the purchase price within the stipulated
period merely accorded Tolosa the option to rescind the contract of sale upon judicial or notarial demand. In any
case, however, the breach on the part of Ocampo was only slight if not outweighed by the bad faith of Tolosa in
reneging in his own prestations, hence, judicial rescission of the contract can not be justified. And while the contract
dated 3 June 19 in favor of Villaruz is also a contract of sale, that of Ocampo dated 21 April 1975 should prevail
pursuant to Art. 1544 of the Civil code on double sales. While Villaruz may have registered his cont ract or came
into possession ahead of Ocampo, Villaruz was never in good faith as Ocampo had her adverse claim annotated on
Tolosas OCT before Villaruz and Tolosas transaction.

2B 16-17 SALES AND LEASE Page 308


REMEDIES OF THE AGGRIEVED PARTY

SUSPENSION OF PAYMENT

1. CORINTHIAN REALTY VS CA

G.R. No. 150240, December 26, 2002

Carpio Morales, J.:

FACTS:

Private respondents Emilio Martin, Matilde Martin, Teofilo Guinto, Prudencio Guinto, Margarita Guinto and
Delfin Guinto, along with the heirs of spouses Tomas de Leon an d Francisca Medina, are the co-owners of a 47,737
sq. m. parcel of land situated in Pamplona, Las Pinas, Metro Manila covered by OCT No. 0-18 of the Register of
Deeds of Rizal (the property).

On June 13, 1983, private respondents and petitioner entered int o a Deed of Conditional Sale (the deed)
covering the property. While the names of co-owner respondent Delfin Guinto and the heirs of spouses Tomas de
Leon and Francisca Medina appeared in the deed as among the vendors, they did not affix their respective s ignatures
thereon.
Under the deed, the following terms and conditions were stipulated:

1. The selling price of the above described realty is P 10.00 per square meter; the total consideration of
this Contract therefore is P477,370.00
2. Upon the signing of this Instrument, the VENDEE will pay unto the VENDORS a sum equivalent to
P3.00 per square meter of the above-described property, or a total amount of P142,211.00

3. The remaining balance in the amount of P335,159.00 will be paid by the VENDEE to the VENDORS
within a period of NINETY (90) DAYS from the execution of this Instrument ;

4. If for no justifiable reason, the VENDEE fails and/or refuses to comply with this obligation, the
VENDORS, without prior notice to the VENDEE, shall forfeit-the earnest money; but as soon as the
VENDEE complied (sic) with his obligation under this Contract, then the VENDORS shall
immediately execute the absolute deed conveying the above described property unto the VENDEE, its
executors and administrators; and, in the event the VENDORS for no justifiable reason refuse and/or
fail to execute the necessary absolute deed of sale in favor of the VENDEE, then the VENDEE shall
have every right to institute the necessary action for the protection of its rights and interests .

On separate occasions before the execution of the deed, petitioner paid the private respondents -signatories to
the deed specific amounts of money totalling P82,956.42. Thus Emilio Martin and Matilde Martin received
P47,403.67 on September 22, 1982, Teofilo Guinto and Margarita Guinto received P11,850.92 each on April
14,1983, and Prudencio Guinto received P11,850.91 on June 2, 1983

Petitioner did not, contrary to the stipulation in the deed, pay the balance of the purchase price within ninety
(90) day.

ISSUE:
Whether or not petitioner is justified in suspending payment

2B 16-17 SALES AND LEASE Page 309


HELD:
Petitioners invocation of Article 1590 of the Civil Code which reads:

Art. 1590. Should the vendee be disturbed in the possession or ownership of the thing acquired, or should he have
reasonable grounds to fear such disturbance, by a vindicatory action or a foreclosure of mortgage, he may suspend
the payment of the price until the vendor has caused the disturbance or danger to cease, unless the latter gives
security for the return of the price in a proper case, or it has been stipulated that, notwithstanding any such
contingency, the vendee shall be bound to make the payment. A mere act of trespass shall not authorize the
suspension of the payment of the price.

to justify its suspension of the payment of the balance of the purchase price on the basis of, so it claims, a well-
grounded fear that its possession or ownership of the property would be disturbed by a vindicatory action which
private respondent Delfin Guinto may institute against it fails. And so does its invocation of Article 1191 of the
Civil Code which provides:

ART. 1191. The power to rescind obligations is implied in reciprocal ones, in case one of the obligors should not
comply with what is incumbent upon him.

The injured party may choose between the fulfillment and the rescission of the obligation, with the payment of
damages in either case. He may also seek rescission, even after he has chosen fulfillment, if the latter should become
impossible.

The court shall decree the rescission claimed, unless there be just cause authorizing the fixing of a period.

This is understood to be without prejudice to the rights of third persons who have acquired the thing, in accordance
with Articles 1385 and 1388 and the Mortgage Law.

For under Article 493 of the Civil Code which provides:

Article 493 Each co-owner shall have the full ownership of his part and of the fruits and benefits pertaining thereto,
and he may therefore alienate, assign or mortgage it, and even substitute another person in its enjoyment, except
when personal rights are involved. But the effect of the alienation or the mortgage, with respect to the co-
owners, shall be limited to the portion which may be allotted to him in the division upon the termination of the co-
ownership,

a co-owner has the right to sell his undivided share. If he sells the entire property without obtaining the consent of
the other co-owners, the sale is not null and void. Only the rights of the co -owner-seller are transferred, thereby
making the buyer a co-owner of the property.
The transferee gets only what his transferor would have been entitled to after partition.

2B 16-17 SALES AND LEASE Page 310


2. VICENTE BARENG VS CA

G.R. No. L-12973, April 25, 1960

Reyes, JBL, J.

FACTS:

On November 29, 1951, petitioner Bareng purchased from respondent Alegria the cinematographic
equipment installed at the Pioneer (now Rosamil) Theater in Laoag, Ilocos Norte, for the sum of P15,000, P10,000
of which was paid, and for the balance, Bareng signed four promissory notes falling due on the following dates:
P1,000 on December 15, 1951; P1,500 on February 15, 1952; P1,500 on March 15, 1952; and P1,000 on April,
1952.

The first promissory note was duly paid by petitioner. On February 12, 1952, shortly before the second note
fell due, the other respondent Agustin Ruiz informed petitioner that he was a co-owner of the equipment in question,
and several days thereafter, Ruiz sent petitioner a telegram instructing him to suspend payments to Alegria of the
balance of the price as he was not agreeable to the sale. On the same day, Alegria sought to collect upon the second
note, but petitioner refused to pay on account of Ruiz's claims. Only P400 was paid on the second note and
thereafter, petitioner refused to make any more payments to Alegria until the latter had settled his dispute with Ruiz.

On March 31, 1952, Ruiz filed suit against Alegria and petitioner Bareng (Civil Case No. 1527) for his
share in the price of the cinema equipment in question. On May 21, 1952, Alegria and Ruiz reached a compromise
in the case, wherein the former recognized the latter as co-owner of the equipment sold to petitioner, and promised
to pay him 2/3 of whatever amount he could recover from the latter. Whereupon, on May 28, 1952. Alegria sued
Bareng for the amount of P13,500 allegedly representing the unpaid balance of the price of said equipment. Bareng
answered the complaint, alleging that only P3,600 had not been paid on the price of the equipment in question,
prayed for the rescission of the sale for supposed violation by Alegria of certain express warranties as to the quality
of the equipment, and asked for payment of damages for alleged violation of Alegria's warranty of title. Petitioner
Bareng claims he is not liable to pay interests to Alegria because he was justified in suspending payment of the
balance of the price of the equipment in question from the time he learned of Ruiz' adverse claims over said
equipment.

ISSUE:

Whether or not suspension of payment was justified.

HELD

NO.

There is no question that, as found by the Court of Appeals, petitioner Bareng ha d the right to suspend payment of
the balance of the price of the cinema equipment in question to his vendor, respondent Alegria, from the time he was
informed by Ruiz of the latter's claims of co-ownership thereof, especially upon his receipt of Ruiz' telegram

2B 16-17 SALES AND LEASE Page 311


wherein the latter asserted that he was not agreeable to the sale. Nevertheless, said right of Bareng ended as soon as
"the vendor has caused the disturbance or danger to cease". In this case, respondent Alegria had caused the
disturbance or danger to petitioner's ownership or possession to cease when he (Alegria) reached a compromise with
Ruiz in Civil Case No. 1527 whereby Ruiz expressed his conformity to the sale to Bareng, subject to the payment of
his share in the price by Alegria. Petitioner Bareng cannot claim that he was not aware of this compromise
agreement between the two owners, because he was a party -defendant in Civil Case No. 1527. From the time
Alegria and Ruiz reached this settlement, there was no longer any danger of threat to Bareng's ownership and full
enjoyment of the equipment he bought from Alegria. And it was by virtue of this settlement that Alegria, two days
later, sued petitioner for the unpaid balance of the price of said equipment.

2B 16-17 SALES AND LEASE Page 312


3. ADELFA PROPERTIES, INC. vs. COURT OF APPEALS, ROSARIO JIMENEZ-CASTAEDA
and SALUD JIMENEZ.

G.R. No. 111238 January 25, 1995

Regalado, J

FACTS

Herein private respondents and their brothers, Jose and Dominador Jimenez, were the registered co -owners
of a parcel of land consisting of 17,710 square meters, covered by a TCT 309773, situated in Las Pias, Metro
Manila. On July 28, 1988, Jose and Dominador sold their share consisting of one-half of said parcel of land,
specifically the eastern portion thereof, to herein petitioner pursuant to a "Kasulatan sa Bilihan ng Lupa."
Subsequently, a "Confirmatory Extrajudicial Partition Agreement" was executed by the Jimenezes, wherein the
eastern portion of the subject lot, with an area of 8,855 square meters was adjudicated to Jose and Dominador, while
the western portion was allocated to herein private respondents.

Thereafter, herein petitioner expressed interest in buying the western portion of the property from private
respondents. Accordingly, on November 25, 1989, an "Exclusive Option to Purchase" was executed between
petitioner and private respondents, under the terms and conditions that: the selling price of said 8,655 square meters
of the subject property is P2,856,150.00; the sum of P50,000.00 received from ADELFA PROPERTIES, INC. as an
option money shall be credited as partial payment upon the consummation of the sale and the balance in the sum of
P2,806,150.00 to be paid on or before November 30, 1989; that in case of default on the part of ADELFA
PROPERTIES, INC. to pay said balance, this option shall be cancelled and 50% of the option money to be forfeited
in favor of respondents and the remaining 50% of said money will be refunded upon the sale of said property to a
third party; Eventually, a new owner's copy of the certificate of title was issued but it remained in the possession of
Atty. Bernardo until he turned it over to petitioner Adelfa Properties, Inc.

Before petitioner could make payment, it received summons together with a copy of a complaint filed by
the nephews and nieces of private respondents against the latter, Jose and Dominador, and herein petitioner for
annulment of the deed of sale in favor of Household Corporation and recovery of ownership of the property covered
by TCT 309773. As a consequence petitioner suspended payment of the full purchase price and suggested that
private respondents settle the case. Respondent Salud Jimenez refused to heed the suggestion of petitioner.

On December 7, 1989, petitioner caused to be annotated on the title of the lot its option contract with
private respondents, and its contract of sale with Jose and Dominador Jimenez, as Entry No. 1437-4 and entry No.
1438-4, respectively. On December 14, 1989, private respondents informed Atty. Bernardo, in his capacity as
petitioner's counsel, that they were cancelling the transaction. In turn, Atty. Bernardo offered to pay the purchase
price provided that P500,000.00 be deducted therefrom for the settlement of the civil case. This was rejected by
private respondents. On February 23, 1990, the RTC dismissed Civil Case No. 89-5541. Thus, on February 28,
1990, petitioner caused to be annotated anew on TCT No. 309773 the exclusive option to purchase. On the same
day, February 28, 1990, private respondents executed a Deed of Conditional Sale 10 in favor of Emylene Chua over
the same parcel of land for P3,029,250, of which P1,500,000.00 was paid to private respondents on said date, with

2B 16-17 SALES AND LEASE Page 313


the balance to be paid upon the transfer of title to the specified one -half portion. On April 16, 1990, Atty. Bernardo
wrote private respondents that in view of the dismissal of the case against them, petitioner was willing to pay the
purchase price, and he requested that the corresponding deed of absolute sale be executed. This was ignored by
private respondents.

On July 27, 1990, private respondents' counsel sent a letter to petitioner enclosing therein a check for
P25,000.00 representing the refund of fifty percent of the option money paid under the exclusive option to purchase,
requesting petitioner to return the owner's duplicate copy of the certificate of title of respondent Salud Jimenez.

Petitioner failed to surrender the certificate of title, hence private respondents filed Civil Case for
annulment of contract with damages, praying that the exclus ive option to purchase be declared null and void; that
defendant, herein petitioner, be ordered to return the owner's duplicate certificate of title; and that the annotation of
the option contract on TCT No. 309773 be cancelled.

RTC: ruled that the agreement entered into by the parties was merely an option contract, and declared that the
suspension of payment by herein petitioner constituted a counter-offer which was tantamount to a rejection of the
option; that herein petitioner could not validly suspend payment in favor of private respondents on the ground that
the vindicatory action filed by the latter's kin did not involve the western portion of the land covered by the contract
between petitioner and private respondents, but the eastern portion thereof which was the subject of the sale between
petitioner and the brothers Jose and Dominador Jimenez; hence cancelled the exclusive option to purchase, declared
the sale to intervenor Emylene Chua as valid and binding, and ordered petitioner to pay damages and attorney's fees
to private respondents, with costs.

CA: affirmed

ISSUES

(1) Whether or nto "Exclusive Option to Purchase" executed between petitioner Adelfa Properties, Inc. and private
respondents Rosario Jimenez-Castaeda and Salud Jimenez is an option contract; and

(2) Whether or not there was a valid suspension of payment of the purchase price by said petitioner, and the legal
effects thereof on the contractual relations of the parties.

HELD

(1) NO
The distinction between an "option" and a contract o f sale is that an option is an unaccepted offer. It states
the terms and conditions on which the owner is willing to sell the land, if the holder elects to accept them within the
time limited. If the holder does so elect, he must give notice to the other p arty, and the accepted offer thereupon
becomes a valid and binding contract. If an acceptance is not made within the time fixed, the owner is no longer
bound by his offer, and the option is at an end. A contract of sale, on the other hand, fixes definitely the relative

2B 16-17 SALES AND LEASE Page 314


rights and obligations of both parties at the time of its execution. The offer and the acceptance are concurrent, since
the minds of the contracting parties meet in the terms of the agreement.

The test in determining whether a contract is a "contract of sale or purchase" or a mere "option" is whether
or not the agreement could be specifically enforced. There is no doubt that the obligation of petitioner to pay the
purchase price is specific, definite and certain, and consequently binding and enforceable. Had private respondents
chosen to enforce the contract, they could have specifically compelled petitioner to pay the balance of
P2,806,150.00. This is distinctly made manifest in the contract itself as an integral stipulation, compliance with
which could legally and definitely be demanded from petitioner as a consequence. An agreement is only an "option"
when no obligation rests on the party to make any payment except such as may be agreed on between the parties as
consideration to support the option until he has made up his mind within the time specified.

An option, and not a contract to purchase, is effected by an agreement to sell real estate for payments to be
made within specified time and providing forfeiture of money paid upon failure to make payment, where the
purchaser does not agree to purchase, to make payment, or to bind himself in any way other than the forfeiture of the
payments made. As hereinbefore discussed, this is not the situation obtaining in the case at bar. While there is
jurisprudence to the effect that a contract which provides that the initial payment shall be totally forfeited in case of
default in payment is to be considered as an option contract, still we are not inclined to conform with the findings of
respondent court and the court a quo that the contract executed between the parties is an option contract, for the
reason that the parties were already contemplating the payment of the balance of the purchase price, and were not
merely quoting an agreed value for the property. The term "balance," connotes a remainder or something remaining
from the original total sum already agreed upon. In other words, the alleged option money of P50,000.00 was
actually earnest money which was intended to form part of the purchase price . The amount of P50,000.00 was not
distinct from the cause or consideration for the sale of the property, but was itself a part thereof. It is a statutory rule
that whenever earnest money is given in a contract of sale, it shall be considered as part of th e price and as proof of
the perfection of the contract. It constitutes an advance payment and must, therefore, be deducted from the total
price. Also, earnest money is given by the buyer to the seller to bind the bargain.

(2) YES but petitioners are now estopped


At a glance, it is easily discernible that, although the complaint prayed for the annulment only of the
contract of sale executed between petitioner and the Jimenez brothers, the same likewise prayed for the recovery of
therein plaintiffs' share in that parcel of land specifically covered by TCT No. 309773. In other words, the plaintiffs
therein were claiming to be co-owners of the entire parcel of land described in TCT No. 309773, and not only of a
portion thereof nor, as incorrectly interpreted by the lower courts, did their claim pertain exclusively to the eastern
half adjudicated to the Jimenez brothers. Such being the case, petitioner was justified in suspending payment of the
balance of the purchase price by reason of the aforesaid vindicatory act ion filed against it. The assurance made by
private respondents that petitioner did not have to worry about the case because it was pure and simple harassment is
not the kind of guaranty contemplated under the exceptive clause in Article 1590 wherein the v endor is bound to
make payment even with the existence of a vindicatory action if the vendee should give a security for the return of
the price.

Be that as it may, and the validity of the suspension of payment notwithstanding, we find and hold that
private respondents may no longer be compelled to sell and deliver the subject property to petitioner for two
reasons: 1) petitioners failure to duly effect the consignation of the purchase price after the disturbance had ceased
and 2) the fact that the contract to sell had been validly rescinded by private respondents. The records of this case
reveal that as early as February 28, 1990 when petitioner caused its exclusive option to be annotated anew on the
certificate of title, it already knew of the dismissal of civil Case No. 89-5541. However, it was only on April 16,
1990 that petitioner, through its counsel, wrote private respondents expressing its willingness to pay the balance of

2B 16-17 SALES AND LEASE Page 315


the purchase price upon the execution of the corresponding deed of absolute sa le. At most, that was merely a notice
to pay. There was no proper tender of payment nor consignation in this case as required by law.

The mere sending of a letter by the vendee expressing the intention to pay, without the accompanying
payment, is not considered a valid tender of payment. Besides, a mere tender of payment is not sufficient to compel
private respondents to deliver the property and execute the deed of absolute sale. It is consignation which is essential
in order to extinguish petitioner's obligation to pay the balance of the purchase price.

THE RULE IS DIFFERENT IN CASE OF AN OPTION CONTRACT OR IN LEGAL REDEMPTION


OR IN A SALE WITH RIGHT TO REPURCHASE, wherein consignation is not necessary because these cases
involve an exercise of a right or privilege (to buy, redeem or repurchase) rather than the discharge of an obligation,
hence tender of payment would be sufficient to preserve the right or privilege . This is because the provisions on
consignation are not applicable when there is no obligatio n to pay. A contract to sell, as in the case before us,
involves the performance of an obligation, not merely the exercise of a privilege of a right. consequently,
performance or payment may be effected not by tender of payment alone but by both tender and consignation.

Furthermore, petitioner no longer had the right to suspend payment after the disturbance ceased with the
dismissal of the civil case filed against it. Necessarily, therefore, its obligation to pay the balance again arose and
resumed after it received notice of such dismissal. Unfortunately, petitioner failed to seasonably make payment, as
in fact it has deposit the money with the trial court when this case was originally filed therein. By reason of
petitioner's failure to comply with its obligation, private respondents elected to resort to and did announce the
rescission of the contract through its letter to petitioner dated July 27, 1990. That written notice of rescission is
deemed sufficient under the circumstances. Article 1592 of the Civil Code which requires rescission either by
judicial action or notarial act is not applicable to a contract to sell. Furthermore, judicial action for rescission of a
contract is not necessary where the contract provides for automatic rescission in case of breach, as in the contract
involved in the present controversy.

In the case at bar, it has been shown that although petitioner was duly furnished and did receive a written
notice of rescission which specified the grounds therefore, it failed to reply theret o or protest against it. Its silence
thereon suggests an admission of the veracity and validity of private respondents' claim. If private respondents had
not taken the initiative of filing Civil Case No. 7532, evidently petitioner had no intention to take any legal action to
compel specific performance from the former. By such cavalier disregard, it has been effectively estopped from
seeking the affirmative relief it now desires but which it had theretofore disdained.

2B 16-17 SALES AND LEASE Page 316


4. ARRA REALTY VS GUARANTEE DEVELOPMENT CORP AND INSURANCE AGENCY

G.R. No. 142310, September 20, 2004

Callejo Sr, J

FACTS

Arra Realty Corporation was the owner of a parcel of land, located in Makati City. Through its president, Arguelles,
the ARC decided to construct a five-story building on its property and engaged the services of Engineer Erlinda
Penaloza as project and structural engineer. Penaloza and the ARC, agreed that Penaloza would share the purchase
price of one floor of the building for the price of P3,105,838. Sometime in May 1983, Penaloza took possession of
the one-half portion of the second floor where she put up her office and a school. Unknown to her, ARC had
executed a real estate mortgage over the lot and the entire building in favor of th e China Banking Corporation.
Penaloza was able to pay P1,175,124.59 for the portion of the second floor of the building she had purchased from
the ARC. Then she learned that the property had been mortgaged to the China Banking Corporation. When the ARC
failed to pay its loan to China Banking Corporation, the subject property was foreclosed extrajudicially, and,
thereafter, sold at public auction to China Banking Corporation. Penaloza filed a complaint for "specific
performance or damages" with a prayer for a writ of preliminary injunction against the petitioners.

Plaintiff wrote the defendants on August 31, 1984 proposing to defendants the execution of a deed of sale with
assumption of mortgage in her favor of the portion of the loan corresponding to the s econd floor of the said edifice
and informing them of her resolve to hold further payments on the purchase price of the second floor until her rights
and interest over the same shall have been adequately and properly secured.

The petitioners posit that no contract of sale over the subject property was perfected between the petitioner ARC, on
the one hand, and respondent Pealoza, on the other, because the latter failed to pay the balance of the total purchase
price of a portion of the second floor of the building as provided in their November 18, 1982 agreement. They aver
that respondent Pealoza bound and obliged herself to pay the downpayment of P901,738 on or before January
1983, and the balance in twenty (20) equal quarterly payments of P110,205. Howeve r, the petitioners aver,
respondent Pealoza was able to complete the downpayment only on March 4, 1983 and managed to pay only three
quarterly installments, and part of the fourth quarterly installment.

ISSUE

Whether Penaloza had the right to suspend payment

HELD

In a contract of sale, until and unless the contract is resolved or rescinded in accordance with law, the vendor cannot
recover the thing sold even if the vendee failed to pay in full the initial payment for the property. The failure of the

2B 16-17 SALES AND LEASE Page 317


buyer to pay the purchase price within the stipulated period does not by itself bar the transfer of ownership or
possession of the property sold, nor ipso facto rescind the contract.

Admittedly, respondent Pealoza failed to pay the downpayment on time. But then, the petitioner ARC accepted,
without any objections, the delayed payments of the respondent; hence, as provided in Article 1235 of the New Civil
Code, the obligation of the respondent is deemed complied with.

The respondent cannot be blamed for suspending further remittances of payment to the petitioner ARC because
when she pushed for the issuance of her title to the property after taking possession thereof, the ARC failed to
comply. She was aghast when she discovered that in July 1984, even before s he took possession of the property, the
petitioner ARC had already mortgaged the lot and the building to the China Banking Corporation; when she offered
to pay the balance of the purchase price of the property to enable her to secure her title thereon, the petitioner ARC
ignored her offer. Under Article 1590 of the New Civil Code, a vendee may suspend the payment of the price of the
property sold.

Respondent Pealoza was impelled to cause the annotation of an adverse claim. In this case, respondent Pealoza
suspended the payment of the balance of the purchase price of the property because she had the right to do so. While
she failed to pay the purchase price on time, the petitioner ARC nevertheless accepted such delayed payments. The
respondent even proposed to assume the loan account of the petitioner ARC with the China Banking Corporation in
an amount equivalent to the balance of the purchase price of the subject property, which the petitioner ARC rejected.
In fine, respondent Pealoza acted in accord with law and in utmost good faith.

2B 16-17 SALES AND LEASE Page 318


RECTO LAW

1. PAMECA WOOD TREATMENT PLANT, INC., HERMINIO G. TEVES, VICTORIA V. TEVES and
HIRAM DIDAY R. PULIDO vs. HON. COURT OF APPEALS and DEVELOPMENT BANK OF THE
PHILIPPINES

G.R. No. 106435. July 14, 1999

Gonzaga-Reyes, J

FACTS

On April 17, 1980, petitioner PAMECA Wood Treatment Plant, Inc. (PAMECA) obtained a loan of
US$267,881.67, or the equivalent of P2,000,000.00 from respondent Bank. By virtue of this loan, petitioner
PAMECA, through its President, petitioner Herminio C. Teves, executed a promissory note for the said amount,
promising to pay the loan by installment. As security for the said loan, a chattel mortgage was also executed over
PAMECAs properties in Dumaguete City, consisting of inventories, furniture and equipment, to cover the whole
value of the loan.

On January 18, 1984, and upon petitioner PAMECAs failure to pay, respondent bank extrajudicially
foreclosed the chattel mortgage, and, as sole bidder in the public auction, purchased the foreclosed properties for a
sum of P322,350.00, which is only 1/6 of the market value of the property.

On June 29, 1984, respondent bank filed a complaint for the collection of the balance of P4,366,332.46
with Regional Trial Court of Makati City against petitioner PAMECA and private petitioners herein, as solidary
debtors with PAMECA under the promissory note.

RTC Makati rendered a judgement ordering the defendants to pay jointly and severally plaintiff the (1) sum
of P4,366,332.46 representing the deficiency claim of the latter and (2) the costs of the suit.

The Court of Appeals affirmed the RTC decision. Hence, this Petition.

ISSUES

(1) Whether or not the public auction sale of PAMECAs chattels were tainted with fraud, hence unconscionable and
inequitable, and therefore null and void

(2) Whether or not CA gravely erred in not applying by analogy Article 1484 and Article 2115 of the Civil Code

HELD

2B 16-17 SALES AND LEASE Page 319


(1) No. The respondent court declared that it is not at all unlikely for the chattels to have sufficiently
deteriorated as to have fetched such a low price at the time of the auction sale. Petitioners never assailed the
validity of the sale in the RTC, and only in the Court of Appeals did they attempt to prove inadequacy of
price through the documents, Basic is the rule that parties may not bring on appeal issues that were not
raised on trial.

Neither did respondent court find anything irregular or fraudulent in the circumstance that respondent bank was
the sole bidder in the sale, as all the legal procedures for the conduct of a foreclosure sale have been complied
with, thus giving rise to the presumption of regularity in the performance of public duties. Fraud is a serious
allegation that requires full and convincing evidence, and may not be inferred from the lo ne circumstance that it
was only respondent bank that bid in the sale of the foreclosed properties.

(2) No. The Court ruled that the application by analogy of Article 1484 of the Civil Code to the instant case is
untenable. As correctly pointed out by the trial court, the said article applies clearly and solely to the sale of
personal property the price of which is payable in installments. Although Article 1484, paragraph (3)
expressly bars any further action against the purchaser to recover an unpaid balan ce of the price, where the
vendor opts to foreclose the chattel mortgage on the thing sold, should the vendees failure to pay cover two
or more installments, this provision is specifically applicable to a sale on installments.

To accommodate petitioners prayer even on the basis of equity would be to expand the application of the
provisions of Article 1484 to situations beyond its specific purview, and ignore the language and intent of
the Chattel Mortgage Law. Equity, which has been aptly described as just ice outside legality, is applied
only in the absence of, and never against, statutory law or judicial rules of procedure.

2B 16-17 SALES AND LEASE Page 320


2. SUPERLINES TRANSPORTATION COMPANY, INC., and MANOLET LAVIDES, vs. ICC
LEASING & FINANCING CORPORATION

G.R. No. 150673, February 28, 2003

Callejo Sr, J

Facts:

In 1995, Superlines Transportation Co., Inc. (Superlines, for brevity) decided to acquire five new buses from the
Diamond Motors Corporation for the price of P10,873,582.00. Because of lacked financial resourc es for the
purpose, Superlines authorized its President and General Manager, Manolet Lavides, to look for and negotiate with a
financing corporation for a loan for the purchase of said buses.

Lavides negotiated with ICC Leasing & Financing Corporation (ICC, for brevity) for a financial scheme for the
planned purchase. ICC agreed to finance the purchase of the new buses via a loan and proposed a three -year term for
the payment thereof at a fixed interest rate of 22% per annum. The new buses to be purchased were to be used by
Superlines as security for the loan. On October 19, 1995, Diamond Motors Corporation sold to Superlines five new
buses.

Superlines, through Lavides, executed two documents, namely: a deed of chattel mortgage over the said buses as
security for the purchase price of the buses in the amount of P13,114,287.00 loaned by ICC to Superlines, which
deed was annotated on the face of said certificates of registration, and a promissory note in favor of ICC binding and
obliging itself to pay to the latter the amount of P10,873,582.00 in monthly installments of P415,290.00, the first
instalment to start on December 23, 1995, with interest thereon at the rate of 22% per annum until full payment of
said amount.

After paying only seven monthly amortizations, Superlines defaulted in the payment of its obligation to ICC. On
April 2, 1997, ICC wrote Superlines demanding full payment of its outstanding obligation, which as of March 31,
1997 amounted to P12,606,020.55. However, Superlines failed to heed said demand.

ICC filed a complaint for collection of sum of money with prayer for a writ of replevin on April 21, 1997 against
Superlines and Lavides. On May 29, 1997, the sheriff took possession of the five buses in compliance with the writ
of seizure issued by the trial court. Thereafter, ICC instituted extra-judicial foreclosure proceedings over the subject
buses. An auction sale was held on July 2, 1997. ICC offered a bid of P7,200,000.00 for the motor vehicles and was
declared the winning bidder, resulting in a deficiency of P5,406,029.55. In addition, ICC incurred necessary
expenses in the amount of P920,524.62. Superlines thus still owed ICC the amount of P6,326,556.17.

Superlines and Lavides asserted that the real agreement of the parties was one of financin g a sale of personal
property, the prices for which shall be payable on installments, with ICC as the vendor, and Superlines as the
vendee, of said buses. Relying on Article 1484(3) of the Civil Code, Superlines and Lavides claimed that since the
chattel mortgage on subject buses was already foreclosed by ICC, the latter had no further action against Superlines
and Lavides for the unpaid balance of the price.

Leonardo Serrano, Jr., the Executive Vice-President and Chief Operations Officer of ICC, however, testified that the
transaction forged by ICC and Superlines was an amortized commercial loan and not a consumer loan.

ISSUE

2B 16-17 SALES AND LEASE Page 321


Whether or not ICC is entitled to the deficiency claim or unpaid balance after the extra -judicial foreclosure had been
made.

HELD

ICC and Superlines entered into an amortized commercial loan agreement with ICC as creditor-mortgagee and
Superlines as debtor-mortgagor. Superlines was ordered to pay to ICC the sum of P5,956,351.18 as deficiency.

It was Diamond Motors Corporation and not ICC which sold the subject buses to Superlines. It held that no evidence
had been presented by Superlines to show that ICC bought the said buses from Diamond Motors Corporation under
a special arrangement and that ICC sold the buses to Superlines.

It should be noted that Article 1484(3) is applicable only where there is vendor-vendee relationship between the
parties and since ICC did not sell the buses to Superlines, the latter cannot invoke said law. Diamond is the seller of
the five units of buses and not the plaintiff. The evidence shows that the transaction between the parties was an
amortized commercial loan to be paid in installments.

The evidence on record shows that under the Promissory Note, Chattel Mortgage and Continuing Guaranty,
respondent was the creditor-mortgagee of petitioner Superlines and not the vendor of the new buses. Hence,
petitioners cannot find refuge in Article 1484(3) of the New Civil Code. What should apply was the Chattel
Mortgage executed by petitioner Superlines and respondent in relation to the Chattel Mortgage Law. This Court had
consistently ruled that if in an extra-judicial foreclosure of a chattel mortgage a deficiency exists, an independent
civil action may be instituted for the recovery of said deficiency. To deny the mortgagee the right to maintain an
action to recover the deficiency after foreclosure of the chattel mortgage would be to overlook the fact that the
chattel mortgage is only given as security and not as payment for the debt in case of failure of payment. Bo th the
Chattel Mortgage Law and Act 3135 governing extra-judicial foreclosure of real estate mortgage, do not contain any
provision, expressly or impliedly, precluding the mortgagee from recovering deficiency of the principal obligation.

2B 16-17 SALES AND LEASE Page 322


3. PHILIPPINE NATIONAL BANK vs. COURT OF APPEALS, and SPOUSES EDILBERTO and
ELENA NATIVIDAD

G.R. No. 121739, June 14, 1999

Mendoza, J

FACTS

On December 29, 1975, respondent spouses were granted by the petitioner bank a one-year Time Loan Commercial
(TLC) of P34,000.00.To secure the payment of said loan, respondent spouses executed in favor of petitioner a real
estate mortgage over 9 parcels of individually titled lands located in Sta. Maria, Pangasinan with an aggregate area
of 2,292 square meters. The properties after due inspection and evaluation was appraised by petitioner for loan
purposes in the total amount of P49, 000.00 thereby justifying the grant of the loan applied for.

Due to financial constraints, after paying P15, 000.00, respondent spouses failed to pay the balance despite repeated
demands by petitioner.

An auction sale was conducted in Pangasinan with the petitioner bank as the highest bidder for P7, 000.00. The
amount however was short of P64, 624.31 representing the balance on the principal obligation, accrued interest,
penalties, attorneys fees, and expenses of litigation. For failure to redeem the properties within the statutory period
allowed by law, petitioner filed the instant deficiency claim.

Respondent spouses on the other hand interposed as justification of their inability to redeem the properties their
continuing financial hardships. More than this, they professed that they should not be made to pay the deficit among
other grounds as the aforesaid public auction sale was tainted with fraud and similar irregularities .

ISSUE

Whether or petitioner is estopped from pursuing its deficiency claim arising from the extrajudicial foreclosure
against respondent spouses properties.

HELD

NO.

To begin with, it is settled that if the proceeds of the sale are insufficient to cover the debt in an extrajudicial
foreclosure of the mortgage, the mortgagee is entitled to claim the deficiency from the debtor. For when the
legislature intends to deny the right of a creditor to sue for any deficiency resulting from foreclosure of security
given to guarantee an obligation it expressly provides as in the case of pledges (Civil Code, Art. 2115) and in chattel
mortgages of a thing sold on installment basis [Civil Code, Art. 1484(3)]. Act No. 3135, which governs the
extrajudicial foreclosure of mortgages, while silent as to the mortgagees right to recover, does not, on the other
hand, prohibit recovery of deficiency. Accordingly, it has been held that a deficiency claim arising from the
extrajudicial foreclosure is allowed.

Both the trial court and the Court of Appeals held petitioner to be estopped from pursuing its deficiency claim on the
ground that the deficiency arose from petitioner banks reappraisal of the properties in question for purposes of fixing
the bid price thereof. Both courts held that petitioner had lowered its appraisal of the properties for the purpose of
acquiring the properties and still collecting from respondent spouses a deficiency claim. In their view, respondent

2B 16-17 SALES AND LEASE Page 323


spouses relied in good faith on petitioners initial appraisal of their properties as worth P49,000.00 in mortgaging
their properties on the theory that in case of their failure to pay their loan, their properties can answe r for their
obligation.

There are, however, several factors militating against this view. First, based on the evidence presented, it does
appear that the reappraisal of the properties was their fair value. Second, it appears that rather than being passive
bystanders in the original appraisal of their properties, respondent spouses actually played an active part in the
valuation. Lastly, respondent spouses were benefited rather than harmed by the substantially lower reappraised value
of their properties.

Indeed, as pointed out by petitioner bank, respondents had several options. They could have participated in the
public bidding or exercised their right of redemption or sold such right to redeem or simply settled their debt.
However, they did none of these things despite due notice to them. Respondents are thus to blame for their
predicament. Their claim of financial distress is not an excuse to evade their clear obligation to the bank.

2B 16-17 SALES AND LEASE Page 324


4. VISAYAN SAWMILL COMPANY, INC., and ANG TAY vs. THE HONORABLE COURT OF APPEALS
and RJH TRADING, represented by RAMON J. HIBIONADA, proprietor

G.R. No. 83851. March 3, 1993

Davide Jr, J

FACTS

On May 1, 1983, herein plaintiff-appellee and defendants -appellants entered into a sale involving scrap iron located
at the stockyard of defendant-appellant corporation at Cawitan, Sta. Catalina, Negros Oriental, subject to the
condition that plaintiff-appellee will open a letter of credit in favor of defendant-appellant corporation on or before
May 15, 1983. This is evidenced by a contract entitled `Purchase and Sale of Scrap Iron' duly signed by both parties.
On May 17, 1983, plaintiff-appellee through his man, started to dig and gather and (sic) scrap iron at the defendant -
appellant's (sic) premises, proceeding with such endeavor until May 30 when defendants -appellants allegedly
directed plaintiff-appellee's men to desist from pursuing the work in view of an alleged case filed against plaintiff-
appellee by a certain Alberto Pursuelo. This, however, is denied by defendants -appellants who allege that on May
23, 1983, they sent a telegram to plaintiff-appellee cancelling the contract of sale because of failure of the latter to
comply with the conditions thereof.

On May 24, 1983, plaintiff-appellee informed defendants -appellants by telegram that the letter of credit
was opened May 12, 1983 at the Bank of the Philippine Islands main office in Ayala, but then (sic) the transmittal
was delayed. On May 26, 1983, defendants -appellants received a letter advice from the Dumaguete City Branch of
the Bank of the Philippine Islands dated May 26, 1983. On July 19, 1983, plaintiff-appellee sent a series of
telegrams stating that the case filed against him by Pursuelo had been dismissed and demanding that defendants -
appellants comply with the deed of sale, otherwise a case will be filed against them. On July 20, 1983 informed
plaintiff-appellee's lawyer that defendant-appellant corporation is unwilling to continue with the sale due to plaintiff-
appellee's failure to comply with essential pre-conditions of the contract. On July 29, 1983, plaintiff-appellee filed
the complaint below with a petition for preliminary attachment. The writ of attachment was returned unserved
because the defendant-appellant corporation was no longer in operation and also because the scrap iron as well as
other pieces of machinery can no longer be found on the premises of the corporation.

In his complaint, private respondent prayed for judgment ordering the petitioner corporation to comply with
the contract by delivering to him the scrap iron subject thereof; he further sought an award of actual, moral and
exemplary damages, attorney's fees and the costs of the suit. In their Answer with Counterclaim, petitioners insisted
that the cancellation of the contract was justified because of private respondent's non -compliance with essential pre-
conditions, among which is the opening of an irrevocable and unconditional letter of credit not later than 15 May
1983.

On 29 November 1985, the Regional Trial Court (RTC) of Iloilo rendered its judgment in favor of plaintiff
and against the defendants ordering the latter to pay jointly and severally plaintiff. Petitioners appealed from said
decision to the Court of Appeals, which affirmed with modification, in respect to the moral damages, the decision of
the trial court.

2B 16-17 SALES AND LEASE Page 325


Hence, this petition for review under Rule 45 of the Rules of Court, where petitioners urge theCourt to set
aside the decision of public respondent Court of Appeals.

ISSUE

Whether or not rescission or cancellation of the contract is justified.

HELD

YES. There is merit in the instant petition. Both the trial court and the public respondent erred in the appreciation of
the nature of the transaction between the petitioner corporation and the private respondent. To this Court's mind,
what obtains in the case at bar is a mere contract to sell or promise to sell, and not a contract of sale.

The trial court assumed that the transaction is a contract of sale and, influenced by its view that there was
an "implied delivery" of the object of the agreement, concluded that Article 1593 of the Civil Code was
inapplicable; citing Guevarra vs. Pascual 10 and Escueta vs. Pando, it ruled that rescission under Article 1191 of the
Civil Code could only be done judicially. The trial court further classified the breach committed by the private
respondent as slight or casual, foreclosing, thereby, petitioners' right to rescind the agreement.

In this case, the petitioner corporation's obligation to sell is unequivocally subject to a positive suspensive
condition, i.e., the private respondent's opening, making or indorsing of an irrevocable and unconditional letter of
credit. The former agreed to deliver the scrap iron only upon payment of the purchase price by means of an
irrevocable and unconditional letter of credit. Otherwise stated, the contract is not one of sale where the buyer
acquired ownership over the property subject to the resolutory condition that the purchase price would be paid after
delivery. Thus, there was to be no actual sale until the opening, making or indorsing of the irrevocable and
unconditional letter of credit. Since what obtains in the case at bar is a mere promise to sell, the failure of the private
respondent to comply with the positive suspensive condition cannot even be considered a breach casual or serious
but simply an event that prevented the obligation of petitioner corporation to convey title from acquiring binding
force.

Consequently, the obligation of the petitioner corporation to sell did not arise; it therefore cannot be
compelled by specific performance to comply with its prestation. In short, Article 1191 of the Civil Code does not
apply; on the contrary, pursuant to Article 1597 of the Civil Code, the petitioner corporation may totally rescind, as
it did in this case, the contract. Said Article provides:

"ART. 1597. Where the goods have not been delivered to the buyer, and the buyer has
repudiated the contract of sale, or has manifested his inability to perform his obligations,
thereunder, or has committed a breach thereof, the seller may totally rescind the contract of sale
by giving notice of his election so to do to the buyer."

2B 16-17 SALES AND LEASE Page 326


Respecting the allegations of the contending parties, while it is true that Article 1593 of the New Civil
Code provides that with respect to movable property, the rescis sion of the sale shall of right take place in the interest
of the vendor, if the vendee fails to tender the price at the time or period fixed or agreed, however, automatic
rescission is not allowed if the object sold has been delivered to the buyer. There being already an implied delivery
of the items, subject matter of the contract between the parties in this case, the defendant having surrendered the
premises where the scraps were found for plaintiff's men to dig and gather, as in fact they had dug and ga thered, this
Court finds the mere notice of resolution by the defendants untenable and not conclusive on the rights of the
plaintiff.

In the case at bar, the trial court ruled that rescission is improper because the breach was very slight and the
delay in opening the letter of credit was only 11 days. Where time is not of the essence of the agreement, a slight
delay by one party in the performance of his obligation is not a sufficient ground for rescission of the agreement.
Equity and justice mandates that the vendor be given additional period to complete payment of the purchase price.

2B 16-17 SALES AND LEASE Page 327


5. FILINVEST CREDIT CORPORATION VS. PHILIPPINE ACETYLENE, CO., INC.,

G.R. No. L-50449 January 30, 1982

De Castro, J

FACTS

Philippine Acetylene Co. (PAC) bought a Chevrolet 1969 model vehicle from Alexander Lim for 55,000 to be paid
through installment. To secure the payment, Filinvest Credit executed a chattel mortgage over the said vehicle in
favor of Lim, who later assigned all his rights, title and interest to the Plaintiff by virtue of Deed of Assignment The
defendant failed to pay 9 consecutive installments that pushed the plaintiff to issue a demand letter. The latter
replied, stating their desire to return the mortgaged property for their debts sat isfaction. So the vehicle was returned
to the plaintiff together with the document Voluntary Surrender with Special Power of Attorney to sell. The
plaintiff had a hard time selling the thing due to the unpaid taxes imposed on the vehicle. So the latter instituted an
action for collection of a sum of money with damages, at the same time, they offered to deliver back the motor
vehicle to the appellant but the latter refused to accept it with the following defense: The delivery of the motor
vehicle to Filinvest extinguished its money obligation as it amounted to a dation in payment.

ISSUE

Whether or not the delivery of PAC is considered a valid payment of obligation through dation en pago thus
extinguishing their obligation.

HELD

No. Dacion en pago, according to Manresa, is the transmission of the ownership of a thing by the debtor to the
creditor as an accepted equivalent of the performance of an obligation. In dacion en pago, as a special mode of
payment, the debtor offers another thing to the creditor wh o accepts it as equivalent of payment of an outstanding
debt. The undertaking really partakes in one sense of the nature of sale, that is, the creditor is really buying the thing
or property of the debtor, payment for which is to be charged against the deb tors debt. As such, the essential
elements of a contract of sale, namely, consent, object certain, and cause or consideration must be present. In its
modern concept, what actually takes place in dacion en pago is an objective novation of the obligation wh ere the
thing offered as an accepted equivalent of the performance of an obligation is considered as the object of the
contract of sale, while the debt is considered as the purchase price.

Mere delivery of mortgaged motor vehicle by mortgagor does not mean transfer of ownership to mortgagee without
his consent under the principle of dacion en pago; What is transferred is merely possession of the property.

2B 16-17 SALES AND LEASE Page 328


6. NONATO VS. INTERMEDIATE APPELLATE COURT

G.R. No. L-74049, November 29, 1988

FACTS:

On 1976, Spouses Restituto Nonato and Ester Nonato purchased a Volkswagen from the Peoples Car Inc
(PCI) on installment basis. Nonato executed a promissory note and a chattel mortgage in favor of PCI to secure their
complete payment then PCI subsequently, assigned its rights and interest over the note and mortagge in favor of
Investors Finance Corp (IFC).

Spouses Nonato failed to pay two or more installments, despite demands. Hence, the car was repossessed
by IFC. Despite repossession, IFC still demanded from Spouses Nonato that they pay the balance of the price of the
car. IFC, then, filed a complaint for the payment of the price of the car with damages

In their defense, the spouses argued that when the company repossessed the car, IFC had, by that act,
effectively cancelled the sale of the vehicle. As such, it was barred from exacting the recovery of the unpaid balance
of the purchase price as mandated by Art 1484 of the New Civil Code.

The trial court rendered in favor of IFC and ordered the Spouses Nonato p ay the balance of the purchase
price of the car with interest. CA affirmed the same.

ISSUE:

Whether or not IFC may still demand payment of the balance of the purchase price.

HELD: NO.

Art 1484 of the New Civil Code provides that should the vendee or the purchaser of a personal property
default in the payment of two or more of the agreed installments, the vendor or the seller has the option to avail any
of these 3 remedies, either to exact fulfillment by the purchaser of the obligation, or to cancel the s ale, or to
foreclose the mortgage on the purchased personal property, if one was constituted. These remedies have been
recognized as an alternative, not cumulative, that the exercise of one should bar the exercise of the others.

In the case at bar, IFC had taken possession of the car purchased by Spouses Nonato after they defaulted in
their payments. The defense of IFC that it the repossession of the vehicle was only for the purpose of appraising its
value and for storage and safekeeping pending full payment of the spouses is untenable. The receipt issued by IFC to
the spouses when it took possession of the vehicle that the vehicle could be redeemed within 15 days. This could
only mean that should the spouses fail to redeem the car within the period provide d, IFC would retain permanent
possession of the vehicle. IFC even notified the Spouses Nonato that the value of the car was not sufficient to cover
the balance of the purchase price and there was no attempt at all on the part of the company to return the c ar.

The acts performed by IFC are consistent with the conclusion that it had opted to cancel the sale of the
vehicle. Therefore, it is barred from exacting payment from the petitioners of the balance of the price of the vehicle
which it had already reposs essed

2B 16-17 SALES AND LEASE Page 329


7. BORBON II VS. SERVICEWIDE SPECIALIS TS, INC

258 SCRA 634

FACTS:

On December 7, 1984, defendants Daniel L. Borbon and Francisco Borbon signed a promissory note in
favor of Pangasinan Auto Mart for the purchase of certain chattels.

The rights of Pangasinan Auto Mart, Inc. was later assigned to Filinvest Credit Corporation which also
assigned all its rights, interest and title over the Promissory Note and the chattel mortgage to the plaintiff. The
promissory note stipulates that the installment of P10,238.00 monthly should be paid on the 7th day of each month
starting January 1985, but the defendants failed to comply with their obligation.

Because the defendants did not pay their monthly installments, Filinvest demanded from the defendants the
payment of their installments due on January 29, 1985. After the accounts were assigned to the plaintiff, the plaintiff
attempted to collect by sending a demand letter to the defendants for them to pay their entire obligation which, as of
March 12, 1985, totaled P185,257.80.

The defendants claim that what they intended to buy from Pangasinan Auto Mart was a jeepney type Isuzu
K. C. Cab. The vehicle that they bought was not delivered. Instead, through misrepresentation and machination, the
Pangasinan Motor, Inc. delivered an Isuzu crew cab

The appellate court ruled that petitioners could not avoid liability under the promissory note and the chattel
mortgage that secured it since private respondent took the note for value and in good faith.

In their appeal to this Court, petitioners merely seek a modification of the decision of the appellate court
insofar as it has upheld the court a quo in the award of liquidated damages and attorney's fees in favor of private
respondent. Petitioners invoke the provisions of Article 1484.

ISSUE:

Whether or not the award of liquidated damages and attorneys fees in addition to the proceeds of the auction sale
were proper.

HELD:

No. ART. 1484. In a contract of sale of personal property the price of which is payable in installments, the vendor
may exercise any of the following remedies:

"(1) Exact fulfillment of the obligation, should the vendee fail to pay;

"(2) Cancel the sale, should the vendee's failure to pay cover two or more installments;

"(3) Foreclose the chattel mortgage or the thing sold, if one has been constituted, should the vendee's failure to pay
cover two or more installments. In this case, he shall have no further action against the purchaser to recover any
unpaid balance of the price. Any agreement to the contrary shall be void."

2B 16-17 SALES AND LEASE Page 330


The remedies under Article 1484 of the Civil Code are not cumulative but alternative and exclusive, which
means that should the vendee or purchaser of a personal property default in the payment of two or more of the
agreed installments, the vendor or seller has the option to avail of any of these three remedies either to exact
fulfillment by the purchaser of the obligation, or to cancel the sale, or to foreclose the mortgage on the purchased
personal property, if one was constituted. These remedies have been recognized as alternative, not cumulative, that
the exercise of one would bar the exercise of the others.

When the seller assigns his credit to another person, the latter is likewise bound by the same law.
Accordingly, when the assignee forecloses on the mortgage, there can be no further recovery of the deficiency, and
the seller-mortgagee is deemed to have renounced any right thereto. [6] A contrario, in the event the seller-mortgagee
first seeks, instead, the enforcement of the additional mortgages, guarantees or other security arrangements, he must
then be held to have lost by waiver or non-choice his lien on the chattel mortgage of the personal propert y sold by
any mortgaged back to him, although, similar to an action for specific performance, he may still levy on it.

Given the circumstances, we must strike down the award for liquidated damages made by the court a quo
but we uphold the grant of attorney's fees which we, like the appellate court, find to be reasonable. Parenthetically,
while the promissory note may appear to have been a negotiable instrument, private respondent, however, clearly
cannot claim unawareness of its accompanying documents so as to thereby gain a right greater than that of the
assignor.

2B 16-17 SALES AND LEASE Page 331


8. LUIS RIDAD and LOURDES RIDAD VS. FILIPINAS INVESTMENT and FINANCE
CORPORATION, JOSE D. SEBASTIAN and JOSE SAN AGUSTIN

G.R. No. L-39806 January 27, 1983

De Castro, J

FACTS

On April 14, 1964, plaintiffs purchased from the Supreme Sales arid Development Corporation two (2) brand new
Ford Consul Sedans payable in 24 monthly installments. To secure payment thereof, plaintiffs executed on the same
date a promissory note covering the purchase price and a deed of chattel mortgage on the two vehicles purchased
and also on another car (Chevrolet) and plaintiffs' franchise or certificate of public convenience the operation of a
taxi fleet.

With the conformity of the plaintiffs, the vendor assigned its rights, title and interest to the above-mentioned
promissory note and chattel mortgage to defendant Filipinas Investment and Finance Corporation (Filipinas
Investment).

Plaintiffs failed to pay their monthly installments, thus the defendan t corporation foreclosed the chattel mortgage
extra-judicially. Without notice to the plaintiffs, a public auction sales was held for the sale of the two Ford Consul
cars. The defendant corporation was the highest bidder and purchaser.

Another auction sale was held on November 16, 1965, involving the remaining properties subject of the deed of
chattel mortgage since plaintiffs' obligation was not fully satisfied by the first auction sale of the aforesaid vehicles.

On the second public auction sale, the franchise of plaintiffs to operate five units of taxicab service was sold to
defendant Filipinas Investment, which subsequently sold and conveyed the same to defendant Jose D. Sebastian,
who then filed with the Public Service Commission an application for a pproval of said sale in his favor.

Plaintiffs filed an action for annulment of contractwith Filipinas Investment and Finance Corporation, Jose D.
Sebastian and Sheriff Jose San Agustin, as party-defendants at the Court of First Instance of Rizal Branch I.

The lower court declared the chattel mortgage in the taxicab franchise and Chevrolet and the second auction sale
null and void and the assignment made by Filipinas Investment in favor of Jose Sebastian as void.

The defendants appealed to the Court of Appeals arguing that the lower court erred in declaring the second auction
sale and other transactions relating thereto as void and of no legal effect.

ISSUE

Whether or not the chattel mortgage on the taxicabs and subsequent sale thereof is valid.

HELD

The resolution of said issue is unquestionably governed by the provisions of Article 1484 of the Civil Code which
states:

Art. 1484. In a contract of sale of personal property the price of which is payable in installments, the vendor may

2B 16-17 SALES AND LEASE Page 332


exercise y of the following remedies:

(1) Exact fulfillment of the obligation, should the vendee fail to pay;

(2) Cancel the sale, should the vendee's failure to pay cover two or more installments;

(3) Foreclose the chattel mortgage on the thing sold, if one has been const ituted, should the vendee's failure to
pay cover two or more installments. In this case, he shall have no further action against the purchaser to recover any
unpaid balance of the price. Any agreement to the contrary shall be void.

The remedies provided in the foregoing article are alternative, not cumulative. Furthermore, if the vendor avails
himself of the right to foreclose his mortgage, the law prohibits him from further bringing an action against the
vendee for the purpose of recovering whatever balance of the debt secured not satisfied by the foreclosure sale.

In the instant case, defendant corporation elected to foreclose its mortgage upon default by the plaintiffs in the
payment of the agreed installments. Having chosen to foreclose the chattel mortgage, and bought the purchased
vehicles at the public auction as the highest bidder, it is deemed to have renounced any and all rights which it might
otherwise have under the promissory note and the chattel mortgage as well as the payment of the unpaid ba lance.

2B 16-17 SALES AND LEASE Page 333


9. NORTHERN MOTORS VS. SAPINOSO

G.R. No. L-28074 May 29, 1970

Villamor, J

FACTS

Sapinoso purchased a car from Northern Motors, at a price of P12,171. Sapinoso paid the downpayment, while the
balance will be paid on installments. A chattel mortgage was executed on the car in order to secure the payment. In
case of default, the mortgagee is entitled to the following remedies: sell the car, cancel the contract, extrajudicial
foreclosure, judicial foreclosure, exact fulfillment. In each elected remedy, the mortgagor waives his right to retrieve
his payment. Sapinoso failed to pay the installments several times, and most of the payments made by him were only
to apply to the interest. In consequence thereof, a complaint against him was filed by Northern Motors. Norther
Motors prayed in the complaint that it elects the remedy of extrajudicial foreclosure with a writ of replevin upon
filing a bond. In case of refusal to deliver the car, Sapinoso shall be liable for the amount due plus 25% attorneys
fees. After the commencement of the action but before the filing of an answer, Sapinoso paid P1,250. The sheriff
then seized the car and delivered the same to Northern Motors. Sapinoso thereafter made an answer, stating that he
already paid a good portion of the value of the car, and that the reason why he was not paying the installments is
because the car was defective and its value is only P5,000. The trial court held that Northern Motors may possess the
car, but it must return the P1250 paid by Sapinoso. Northern Motors, however, appealed the part of the decision
which orders them to return the P1250 paid to Sapinoso. According to Northern Motors, under Art 1484 of the Civil
Code, it is the exercise and not the election of remedy of foreclo sure that bars the creditor from recovering the
unpaid balance of the debt. Sapinoso, in paying Northern before he files his answer, and by not filing a counter -
claim, in effect renounced any right he has to recover.

ISSUE

Whether or not Northern Motors has an obligation to return P1,250

HELD

No. It is the fact of foreclosure and actual sale that bars the mortgagee from recovering any unpaid debts from the
mortgagor. Article 1484 of the New Civil Code prohibits further action against the purchaser to re cover any unpaid
balance of the price in which the Court construed the word action as any judicial or extrajudicial proceeding by
virtue of which the vendor may lawfully be enabled to exact recovery of the unpaid balance. Since there has not yet
been any act of foreclosure, the strict provisions of Art 1484 has no application yet. The payment of the P1,250 was
a voluntary act on the part of Sapinoso and does not operate as a further action of the part of Northern on its election
for extrajudicial foreclosure.

2B 16-17 SALES AND LEASE Page 334


10. AGUSTIN VS. COURT OF APPEALS

GR no: 107846 Date: April 18, 1997

Francisco, J

FACTS

The dispute stemmed from an unpaid promissory note dated October 28, 1970, executed by petitioner
Leovillo C. Agustin in favor of ERM Commercial for the amount of P43,480.80. The note was payable in monthly
installments and secured by a chattel mortgage over an Isuzu diesel truck, both of which were subsequently assigned
to private respondent Filinvest Finance Corporation. When petitioner defaulted in paying the installments, private
respondent demanded from him the payment of the entire balance or, in lieu thereof, the possession of the
mortgaged vehicle. Neither payment nor surrender was made. Aggrieved, private respondent filed a complaint with
the Regional Trial Court of Manila, Branch 26 (RTC Branch 26) against petitioner praying for the issuance of a writ
of replevin or, in the alternative, for the payment of P32,723.97 plus interest at the rate of 14% per annum from due
date until fully paid. Trial ensued and, thereafter, a writ of replevin was issued by RTC Branch 26. By virtue thereof,
private respondent acquired possession of the vehicle. Upon repossession, the latter discovered that the vehicle was
no longer in running condition and that several parts were missing which private respondent replaced. The vehicle
was then foreclosed and sold at public auction.

Private respondent subsequently filed a supplemental complaint claiming additional reimbursement


worth P8,852.76 as value of replacement parts and for expenses incurred in transporting the mortgaged vehicle from
Cagayan to Manila. In response, petitioner moved to dismiss the supplemental complaint arguing that RTC Branch
26 had already lost jurisdiction over the case because of the earlier extra-judicial foreclosure of the mortgage. The
lower court granted the motion and the case was dismissed. This was reversed by CA which remanded the case to
RTC of Manila, Branch 40 for determination of the amount due from petitioner. The trial court dete rmined said
amount which was appealed to CA by both parties. CA affirmed the decision of the trial court lowering the amount.
Petitioner filed a motion for reconsideration, but to no avail. Hence, this petition for review on certiorari.

ISSUE

Whether or not the award of repossession expenses to private respondent as mortgagee is "contrary to the letter,
intent and spirit of Article 1484 of the Civil Code"

HELD

The SC held that the appellate court had already settled the propriety of awarding repossessio n expenses in
favor of private respondent. The remand of the case to RTC Branch 40 was for the sole purpose of threshing out the
correct amount of expenses and not for re-litigating the accuracy of the award. Thus, the findings of RTC Branch 40,
as affirmed by the appellate court in CA-G.R. No. 24684, was confined to the appreciation of evidence relative to
the repossession expenses for the query or issue passed upon by the respondent court in CA -G.R. No. 56718-R
(propriety of the award for repossession expenses) has become the law of the case.

At any rate, even if we were to brush aside the law of the case doctrine we find the award for repossession
expenses still proper. In Filipinas Investment & Finance Corporation v. Ridad , the Court recognized an exception to
the rule stated under Article 1484(3) upon which petitioner relies. Thus:

2B 16-17 SALES AND LEASE Page 335


x x x Where the mortgagor plainly refuses to deliver the chattel subject of the mortgage upon his failure to pay two
or more installments, or if he conceals the chattel to place it beyond the reach of the mortgagee, what then is the
mortgagee expected to do? x x x It logically follows as a matter of common sense, that the necessary expenses
incurred in the prosecution by the mortgagee of the action for replevin so that he can regain possession of the
chattel, should be borne by the mortgagor. Recoverable expenses would, in our view, include expenses properly
incurred in effecting seizure of the chattel and reasonable attorneys fees in prosecuting the action for replevin

2B 16-17 SALES AND LEASE Page 336


11. MAGNA FINANCIAL SERVICES GROUP, INC. vs. ELIAS COLARINA

G.R. No. 158635 December 9, 2005

Chico-Nazario, J.

FACTS

On 11 June 1997, Elias Colarina bought on installment from Magna Financial Services Group, Inc., one unit of
Suzuki Multicab. After making a down payment, Colarina executed a promissory note for the balance of
P229,284.00 payable in thirty-six (36) equal monthly installments. To secure payment, Colarina executed an
integrated promissory note and deed of chattel mortgage ov er the motor vehicle. He failed to pay the monthly
amortization beginning January 1999, accumulating an unpaid balance of P131,607.00. Despite repeated demands,
he failed to make the necessary payment.

Magna filed a Complaint for Foreclosure of Chattel Mortgage with Replevin. A Writ of Replevin was issued by the
MTCC. Upon service of the writ, Colarina voluntarily surrendered physical possession of the vehicle to the Sheriff,
and the same was turned over to Magna. MTCC rendered judgement in favor of Magna. Legaspi City RTC affirmed
in toto the decision of the MTCC.

The CA found merit in Colarinas contention that the MTC and the RTC erred in ordering the defendant to pay the
unpaid balance of the purchase price of the subject vehicle. The fact that Magna had unconscionably sought
payment of the unpaid balance regardless of its complaint for the foreclosure of said mortgage is glaring proof that it
intentionally devised the same to deprive the defendant of his rights. A judgment in its favor will in effect allow it to
retain the possession and ownership of the subject vehicle and at the same time claim against the defendant for the
unpaid balance of its purchase price. In such a case, the respondent would luckily have its cake and eat it too.
Decision was reversed and case was remanded to the lower court. The foreclosure of the chattel mortgage was
ordered, without any right to seek the payment of the unpaid balance of the purchase price

ISSUE

What is the true nature of a foreclosure of chattel mortgage, extrajudicial or judicial, as an exercise of the 3rd option
under NCC article 1484, paragraph 3?

HELD

A contract of chattel mortgage, which is the transaction involved in the present case, is in the nature of a conditional
sale of personal property given as a security for the payment of a debt, or the performance of some other obligation
specified therein, the condition being that the sale shall be void upon the seller paying to the purchaser a sum of
money or doing some other act named. If the condition is performed according to its terms, the mortgage and sale
immediately become void, and the mortgagee is thereby divested of his title. On the other hand, in case of
nonpayment, foreclosure is one of the remedies available to a mortgagee by which he subjects the mortgaged
property to the satisfaction of the obligation to secure that for which the mortgage was given.

2B 16-17 SALES AND LEASE Page 337


ISSUE

Was the CA correct in holding that Magna Financial sought to avail the inconsistent remedies; namely, BOTH
foreclosure and payment of the balance of the purchase price?

HELD

YES.

NCC Article 1484 explicitly provides: In a contract of sale of personal property the price of which is payable
in installments, the vendor may exercise any of the following remedies:

(1) Exact fulfillment of the obligation, should the vendee fail to pay;

(2) Cancel the sale, should the vendees failure to pay cover two or more installments;

(3) Foreclose the chattel mortgage or the thing sold, if one has been constituted, should the vendees failure to pay
cover two or more installments. In this case, he shall have no further action against the purchaser to recover any
unpaid balance of the price. Any agreement to the contrary shall be void.

Art 1484 par (3) prevents mortgagees from seizing the mortgaged property, buying it at foreclosure sale for a low
price and then bringing the suit against the mortgagor for a deficiency judgment. The almost invariable result of this
procedure was that the mortgagor found himself minus the property and still owing practically the full amount of his
original indebtedness.

2B 16-17 SALES AND LEASE Page 338


ON ARTICLE 1485

1. ELISCO TOOL MANUFACTURING CORPORATION VS.COURT OF APPEALS, ROLANDO


LANTAN, and RINA LANTAN

G.R. No. 109966 May 31, 1999.

Mendoza, J

FACTS

Private respondent Rolando Lantan entered into an agreement with his employer, Elisco tool Manufacturing Corp,
leasing unto the former a Colt lancer for a period of 5 years. The contract also provided that at the end of the 5 year
period, Lantan may exercise the option to purchase price of the car and he should just pay the remaining balance.
Said option is limited to the employee. It also provided that upon Lantans failure to pay 3 accumulated monthly
rentals, the petitioner will have the option to lease said vehicle to another. Lantan also has to return the car in case he
resigns or is dismissed. He was laid off after petitioner ceased operations in 1981. At that time, he has paid P61,
070.94. Petitioner then filed a replevin case against Lantan and his wife, alleging that they have failed to pay the
monthly rentals despite repeated demands. The trial court rendered judgement in favor of the defendants declaring
defendants as the lawful owners of the Colt Lancer. CA affirmed in toto the judgement.

ISSUE

Whether or not spouses Lantan are the lawful owners of the Colt Lancer.

HELD

Although the agreement provides for the payment by Lantan of monthly rentals, the 5 th paragraph thereof gives
them the option to purchase the motor vehicle at the end of the 5th year or upon payment of the 60 th monthly rental
when all monthly rentals shall be applied to the payment of the full purchase price of the car. Clearly, the
transaction is a lease in name only. The so-called monthly rentals are in truth monthly amortizations on the cars
price. Being a contract of sale on installment, Article 1484 &1485 apply. As such, the case should be considered as
one for specific performance pursuant to A rticle 1484 (1). The prayer for a writ of replevin is only for the purpose
of ensuring specific performance by private respondents. However, the private respondents could no longer be held
liable for the payment of interest on unpaid monthly rentals since it was entered into in pursuance of a car plan
adopted by petitioner for the benefit of its deserving employees. Further, private respondents default in payment
was due to the cessation of operations of petitioners sister company. Elizalde Steel Company. That petitioner
accepted payments from Lantan more than 2 years after the latters employment have been terminated constitutes a
waiver of petitioners right to collect interest upon delayed payment. What private respondents paid should be
considered the payment in full.

2B 16-17 SALES AND LEASE Page 339


ON ARTICLES 1191 AND 1381

1. SPOUSES JAIME BENOS v. SPOUSES GREGORIO LAWILAO

G.R. No. 172259, December 5, 2006

Ynares-Santiago, J.

FACTS

On February 11,1999, petitioner-spouses Benos and respondent Lawilao executed a Pacto de Retro Sale where
Benos sold their lot and the building erected thereon for P300,000, one-half of which to be paid in cash to the Benos
and the other half to be paid to the bank to pay off the loans of the Benos which was secured by the same lot and
building. Under the contract, Benos could redeem the property within 18 months from the date of execution by
returning the contract price, otherwise, the sale would become irrevocable. After paying the P150,000, Lawilao took
possession of the property, restructured it twicw, eventually the loan become due and demandable. On August 14,
2000, a son of Benos and Lawilao paid the bankl but the bank refused. Lawilao filed for consignation against the
bank and deposited the amount of P159,000.00. RTC declared Lawilao of the ownership of the subject property,
which was affirmed by the Court of Appeals.

ISSUE

Whether or not the contract of Pacto de Retro Sale be rescinded by the petitioner

HELD

In the instant case, records show that Lawilao filed the petition for consignation against the bank in Civil Case
without notifying the Benos. Hence, Lawilao failed to prove their offer to pay the balance, even before the filing of
the consignation case. Lawilao never notified the Benos. Thus, as far as the Benos are concerned, there was no full
and complete payment of the contract price which gives them the right to rescind.

Petition is granted. Court of Appeals decision is reversed and set aside, that the Pacto de Retro Sale is rescinded and
petitioner are ordered to return the amount of P150,000 to respondents.

2B 16-17 SALES AND LEASE Page 340


2. UFC VS CA

G.R. No. L-29155 May 13, 1970

Castro, J.:

FACTS

That as far back as 1938, plaintiff Magdalo V. Francisco, Sr. discovered or invented a formula for the manufacture
of a food seasoning (sauce) derived from banana fruits popularly known as MAFRAN sauce; that the manufacture of
this product was used in commercial scale in 1942, and in the same year plaintiff registered his trademark in his
name as owner and inventor with the Bureau of Patents; that due to lack of sufficient capital to finance the
expansion of the business, in 1960, said plaintiff secured the financial assistance of Tirso T. Reyes who, after a
series of negotiations, formed with others defendant Universal Food Corporation eventually leading to the execution
on May 11, 1960 of the aforequoted "Bill of Assignment"

Conformably with the terms and conditions of Exh. A, plaintiff Magdalo V. Francisco, Sr. was appointed Chief
Chemist with a salary of P300.00 a month, and plaintiff Victoriano V. Francisco was appointed auditor and
superintendent with a salary of P250.00 a month.

However, due to the alleged scarcity and high prices of raw materials, on November 28, 1960, Secretary -Treasurer
Ciriaco L. de Guzman of defendant issued a Memorandum (Exhibit B), duly approved by the President and General
Manager Tirso T. Reyes that only Supervisor Ricardo Francisco should be retained in the factory and that the salary
of plaintiff Magdalo V. Francisco, Sr., should be stopped for the time being until the corporation should resume its
operation.

Some five (5) days later, that is, on December 3, 1960, President and General Manager Tirso T. Reyes, issued a
memorandom to Victoriano Francisco ordering him to report to the factory and produce "Mafran Sauce" at the rate
of not less than 100 cases a day so as to cope with the orders of the corporation's various distributors and dealers,
and with instructions to take only the necessary daily employees without employing permanent employees.
However, they failed to do so.

On February 14, 1961 Magdalo V. Francisco, Sr. and Victoriano V. Francisco filed with the Court of First Instance
of Manila, against, the Universal Food Corporation, an action for rescission of a contract entitled "Bill of
Assignment."

ISSUEWhether or not rescission will prosper?

HELD

The general rule is that rescission of a contract will not be permitted for a slight or casual breach, but only for such
substantial and fundamental breach as would defeat the very object of the parties in making the agreement. The
question of whether a breach of a contract is substantial depends upon the attendant circumstan ces. The petitioner
contends that rescission of the Bill of Assignment should be denied, because under article 1383, rescission is a

2B 16-17 SALES AND LEASE Page 341


subsidiary remedy which cannot be instituted except when the party suffering damage has no other legal means to
obtain reparation for the same. However, in this case the dismissal of the respondent patentee Magdalo V. Francisco,
Sr. as the permanent chief chemist of the corporation is a fundamental and substantial breach of the Bill of
Assignment. He was dismissed without any fault or negligence on his part. Thus, apart from the legal principle that
the option to demand performance or ask for rescission of a contract belongs to the injured party, the fact
remains that the respondents -appellees had no alternative but to file the present action for rescission and damages. It
is to be emphasized that the respondent patentee would not have agreed to the other terms of the Bill of Assignment
were it not for the basic commitment of the petitioner corporation to appoint him as it s Second Vice-President and
Chief Chemist on a permanent basis; that in the manufacture of Mafran sauce and other food products he would
have "absolute control and supervision over the laboratory assistants and personnel and in the purchase and
safeguarding of said products;" and that only by all these measures could the respondent patentee preserve
effectively the secrecy of the formula, prevent its proliferation, enjoy its monopoly, and, in the process afford and
secure for himself a lifetime job and steady income.

The salient provisions of the Bill of Assignment, namely, the transfer to the corporation of only the use of the
formula; the appointment of the respondent patentee as Second Vice -President and chief chemist on a permanent
status; the obligation of the said respondent patentee to continue research on the patent to improve the quality of the
products of the corporation; the need of absolute control and supervision over the laboratory assistants and personnel
and in the purchase and safekeeping of the chemicals and other mixtures used in the preparation of said product
all these provisions of the Bill of Assignment are so interdependent that violation of one would result in virtual
nullification of the rest.

2B 16-17 SALES AND LEASE Page 342


3. GUZMAN, BOCALING & CO. VS RAOUL S. V. BONNEVIE

G.R. No. 86150 March 2, 1992

Cruz, J.

FACTS

A parcel of land with two buildings constructed thereon, belonging to the Intestate Estate of Jose L. Reynoso, was
leased to Raoul S. Bonnevie and Christopher Bonnevie by the admin istratrix, Africa Valdez de Reynoso, for a
period of one year beginning August 8, 1976 with payment of a monthly rental.

The Contract of lease contained the following stipulation:

20. In case the LESSOR desire or decides to sell the lease property, the LESSEES shall be given a first priority to
purchase the same, all things and considerations being equal.

On November 3, 1976, according to Reynoso, she notified the private respondents by registered mail that she was
selling the leased premises and was giving them 30 days from receipt of the letter within which to exercise their
right of first priority to purchase the subject property, and that in the event that they did not exercise the said right,
she would expect them to vacate the property not later than March 1977.

On January 20, 1977, Reynoso sent another letter to private respondents advising them that in view of their failure to
exercise their right of first priority, she had already sold the property.

The private respondents wrote Reynoso informing h er that neither of them had received her first letter and that they
had already advised her agent to inform them officially should she decide to sell the property so negotiations could
be initiated.

The leased premises were formally sold to petitioner Guzman, Bocaling & Co.

The selling price quoted to the Bonnevies was P600,000.00, to be fully paid in cash less only the mortgage lien of
P100,000.00. On the other hand, the selling price offered to and accepted by the petitioner was only P400,000.00
and only P137,500.00 was paid in cash while the balance of P272,500.00 was to be paid "when the property (was)
cleared of tenants or occupants.

Reynoso wrote a letter to the private respondents demanding that they vacate the premises within 15 days for their
failure to pay the rentals for four months. When they refuse, Reynoso filed a complaint for ejectment against them.

While the ejectment case was pending in the City Court, the private respondents filed an action for annulment of the
sale between Reynoso and herein petitioner Guzman, Bocaling & Co. and cancellation of the transfer certificate of
title in the name of the latter.

ISSUE

Whether or not the contract of sale is rescissible.

2B 16-17 SALES AND LEASE Page 343


HELD

YES.

Reynoso would be guilty of violating Paragraph 20 of the Contract o f Lease which specifically stated that the private
respondents could exercise the right of first priority, "all things and conditions being equal." The Court reads this to
mean that there should be identity of the terms and conditions to be offered to the Bonnevies and all other
prospective buyers, with the Bonnevies to enjoy the right of first priority. Even if the Bonnevies could not buy it at
the price quoted, Reynoso could not sell it to another for a lower price and under more favorable terms and
conditions. Only if the Bonnevies failed to exercise their right of first priority could Reynoso lawfully sell the
subject property to others, and at that only under the same terms and conditions offered to the Bonnevies.

The private respondents still had a right to rescind the Contract of Sale because of the failure of Reynoso to comply
with her duty to give them the first opportunity to purchase the subject property.

The Contract of Sale was not voidable, but rescissible. Under Article 1380 to 1381 (3) of the Civil Code, a contract
otherwise valid may nonetheless be subsequently rescinded by reason of injury to third persons, like creditors. The
status of creditors could be validly accorded the Bonnevies for they had substantial interests that were prejudiced b y
the sale of the subject property to the petitioner without recognizing their right of first priority under the Contract of
Lease.

Rescission is a remedy granted by law to the contracting parties and even to third persons, to secure reparation for
damages caused to them by a contract, even if this should be valid, by means of the restoration of things to their
condition at the moment prior to the celebration of said contract. It is a relief allowed for the protection of one of the
contracting parties and even third persons from all injury and damage the contract may cause, or to protect some
incompatible and preferment right created by the contract. Recission implies a contract which, even if initially valid,
produces a lesion or pecuniary damage to someone that justifies its invalidation for reasons of equity.

It is true that the acquisition by a third person of the property subject of the contract is an obstacle to the action for
its rescission where it is shown that such third person is in lawful possess ion of the subject of the contract and that
he did not act in bad faith. However, this rule is not applicable in the case before us because the petitioner is not
considered a third party in relation to the Contract of Sale nor may its possession of the sub ject property be regarded
as acquired lawfully and in good faith.

A purchaser in good faith and for value is one who buys the property of another without notice that some other
person has a right to or interest in such property and pays a full and fair price for the same at the time of such
purchase or before he has notice of the claim or interest of some other person in the property. Good faith connotes an
honest intention to abstain from taking unconscientious advantage of another. Tested by these princip les, the
petitioner cannot tenably claim to be a buyer in good faith as it had notice of the lease of the property by the
Bonnevies and such knowledge should have cautioned it to look deeper into the agreement to determine if it
involved stipulations that would prejudice its own interests.

2B 16-17 SALES AND LEASE Page 344


4. EQUATORIAL REALTY DEVELOPMENT, INC. & CARMELO & BAUERMANN, INC., VS

MAYFAIR THEATER, INC..

G.R. No. 106063 November 21, 1996

Hermosisima, J

FACTS:

Carmelo owned a parcel of land, together with two 2-storey buildings constructed thereon located at Claro M
Recto Avenue, Manila, and covered by TCT No. 18529 issued in its name.

On June 1, 1967 Carmelo entered into a contract of lease with Mayfair for the latter's lease of a portion of
Carmelo's property particularly, a portion of the second floor of a two storey bldg (1,640 sqm floor area) and the
second floor and the mezzanine (150 sqm) for use by Mayfair as a motion picture theater and for a term of twenty
(20) years. Mayfair thereafter constructed on the leased property a movie house known as "Maxim Theatre."

Two years later, Mayfair entered into a second contract of lease with Carmelo for the lease of another portion
of Carmelo's property, to wit:

A PORTION OF THE SECOND FLOOR of the two-storey building, with a floor area of 1,064 square meters.

THE TWO (2) STORE SPACES AT THE GROUND FLOOR and MEZZANINE of the two -storey building with a
floor area of 300 square meters.

for similar use as a movie theater and for a similar term of twenty (20) years. Mayfair put up another movie house
known as "Miramar Theatre"

Both contracts of lease provides (sic) identically worded paragraph 8, which reads:

That if the LESSOR should desire to sell the leased premises, the LESSEE shall be given 30-days exclusive option
to purchase the same.

In the event, however, that the leased premises is sold to someone other than the LESSEE, the LESSOR is bound
and obligated, as it hereby binds and obligates itself, to stipulate in the Deed of Sale hereof that the purchaser shall
recognize this lease and be bound by all the terms and conditions thereof.

1974 - Mr. Henry Pascal of Carmelo informed Mr. Henry Yang, President of Mayfair, that Carmelo was
desirous of selling the entire Claro M. Recto property. Mr. Pascal told Mr. Yang that a certain Jose Araneta was
offering to buy the whole property for US Dollars 1,200,000, and Mr. Pascal asked Mr. Yang if the latter was
willing to buy the property for Six to Seven Million Pesos.

Sept 1974 - Mayfair sent a letter to Carmelo purporting to express interest in acquiring not only the leased
premises but "the entire building and other improvements if the price is reasonable. However, both Carmelo and
Equatorial questioned the authenticity of the second letter.

Four years later, Carmelo sold its entire C.M. Recto Avenue land and building, which included the leased
premises housing the "Maxim" and "Miramar" theatres, to Equatorial by virtue of a Deed of Absolute Sale, for the
total sum of P11,300,000.00.

2B 16-17 SALES AND LEASE Page 345


In September 1978, Mayfair instituted the action a quo for specific performance and annulment of the sale of
the leased premises to Equatorial.

TC ruled in favour of Carmelo and Equitorial, holding that the option clause cannot be deemed to be bind ing
on Carmelo because of lack of distinct consideration therefor.

CA reversed, holding that since the two lease contracts does not state a fixed price for the purchase of the
leased premises, which is an essential element for a contract of sale to be perfected, what paragraph 8 is, must be a
right of first refusal and not an option contract. It explicated:

Although the provision giving Mayfair "30-days exclusive option to purchase" cannot be legally categorized as an
option, it is, nevertheless, a valid and binding stipulation. What the trial court failed to appreciate was the intention
of the parties behind the questioned proviso.

ISSUE

Whether or not CA ERRED IN RULING THAT THE OPTION CLAUSE IN THE CONTRACTS OF LEASE IS
ACTUALLY A RIGHT OF FIRST REFUSAL PROVISO

HELD:

We agree with the respondent Court of Appeals that the aforecited contractual stipulation provides for a right
of first refusal in favor of Mayfair. It is not an option clause or an option contract. It is a contract of a right of first
refusal.

An option contract as one necessarily involving the choice granted to another for a distinct and separate
consideration as to whether or not to purchase a determinate thing at a predetermined fixed price.

The rule so early established in this jurisdiction is that the deed of option or the option clause in a contract, in
order to be valid and enforceable, must, among other things, indicate the definite price at which the person granting
the option, is willing to sell.

The option is not the contract of sale itself. The optionee has the right, but not the obligation, to buy. Once the
option is exercised timely, i.e., the offer is accepted before a breach of the option, a bilateral promise to sell and to
buy ensues and both parties are then reciprocally bound to comply with their respective undertakings.

Theres no option to purchase in contemplation of the second paragraph of Article 1479 of the Civil Code, has
been granted to Mayfair under the said lease contracts.

An option is a contract granting a privilege to buy or sell within an agreed time and at a determined price. It is a
separate and distinct contract from that which the parties may enter into upon the consummation of the option. It
must be supported by consideration. 22 In the instant case, the right of first refusal is an integral part of the contracts
of lease. The consideration is built into the reciprocal obligations of the parties.

The Court of Appeals is correct in stating that Paragraph 8 was incorporated into the contracts of lease for the
benefit of Mayfair which wanted to be assured that it shall be given the first crack or the first option to buy the
property at the price which Carmelo is willing to accept. It is not also correct to say that there is no consideration in
an agreement of right of first refusal. The stipulation is part and parcel of the entire contract of lease. The
consideration for the lease includes the consideration for the right of first refusal. Thus, Mayfa ir is in effect stating
that it consents to lease the premises and to pay the price agreed upon provided the lessor also consents that, should

2B 16-17 SALES AND LEASE Page 346


it sell the leased property, then, Mayfair shall be given the right to match the offered purchase price and to bu y the
property at that price.

2B 16-17 SALES AND LEASE Page 347


ON RESCISSION OF IMMOVABLE PROPERTY (Articles 1191, 1591 and 1592, Republic Act No. 6552
(the Maceda Law)

1. LUZON BROKERAGE VS MARITIME BUILDING CO

G.R. No. 25885 Date: January 31, 1972

Reyes, JBL J

FACTS

This is an action for interpleading.


April 30, 1949, the defendant Myers Building Co., Inc., entered into a contract of "Deed of Conditional Sale" with
Bary Building Co., Inc., later known as Maritime Building Co., Inc., whereby the former sold to the latter 3 parcels
of land with the improvements thereon for P1M. P50K of this price was paid upon the execution of the contract and
the balance of P950K was to be paid in monthly installments at the rate of P10K with interest of 5% per annum.

They agreed that in case of failure to pay any of the installments, the contract shall be annulled and all payments
made shall be forfeited and the vendor shall have right to re-enter the property and take possession thereof. The
monthly installment of P10K was decreased to P5K per month. Vendee failed to pay for the month of March 1961,
for which the Vice-President, George Schedler, of the Maritime Building Co., Inc., wrote a letter to the President of
Myers, Mr. C. Parsons, requesting for a moratorium on the monthly payment of the installments until the end of the
year 1961. However, Mr Parsons answered that the Board of Directors of the Myers Co., Inc. refused to grant the
request for moratorium for suspension of payment.

The vendee failed to pay the installments for March, April and May, 1961. On May 16, 1961, the vendor made a
demand upon the vendee, which letter, however, was returned unclaimed. On June 5, 1961, the vendor wrote the
vendee another letter advising it of the cancellation of the Deed of Conditional Sale and demanding the return of the
possession of the properties and holding it liable for use and occupation of the said properties at P10K monthly.

In the meantime, the vendor demanded upon the Luzon Brokerage Co., Inc. to whom the Maritime Building Co.,
Inc. leased the properties, the payment of monthly rentals of P10K and the surrender of the same to it. As a
consequence, the Luzon Brokerage Co., Inc. found itself in a payment to the wrong party, filed this action for
interpleader against the Maritime Building Co., Inc.

After the filing of this action, the Myers Building Co., Inc. in its answer filed a cross -claim against the Maritime
Building Co., Inc. praying for the confirmation of its right to cancel the said contract. In the meantime, the contract
between the Maritime Building Co., Inc. and the Luzon Brokerage Co., Inc. was extended by mutual agreement for a
period of four (4) more years, from April, 1964 to March 31, 1968.

2B 16-17 SALES AND LEASE Page 348


The Maritime Building Co., Inc. now contends (1) that the Myers Bu ilding Co., Inc. cannot cancel the contract
entered into by them for the conditional sale of the properties in question extrajudicially and (2) that it had not failed
to pay the monthly installments due under the contract and, therefore, is not guilty of h aving violated the same.

It should be further elucidated that the suspension by the appellant Maritime Building Co., Inc. of the payment of
installments arose from an award of backwages made by the Court of Industrial Relations in favor of members of
Luzon Labor Union who served the Fil-American forces in Bataan in early 1942 at the instance of the employer
Luzon Brokerage Co. and for which F. H. Myers, former majority stockholder of the Luzon Brokerage Co., had
allegedly promised to indemnify E. M. Schedler (who controlled Maritime) when the latter purchased Myers' stock
in the Brokerage Company. Schedler contended that he was being sued for the backpay award of some P325,000,
when it was a liability of Myers, or of the latter's estate upon his death.

The trial court found the position of Schedler indefensible, and that Maritime, by its failure to pay, committed a
breach of the sale contract; that Myers Company, from and after the breach, became entitled to terminate the
contract, to forfeit the installments paid, as well as to repossess, and collect the rentals of, the building from its
lessee, Luzon Brokerage Co., in view of the terms of the conditional contract of sale

ISSUE

Whether or not there has been a breach of contract which warrants the rescissio n of the same? Yes!

HELD
It is difficult to understand how appellant Maritime can seriously contend that its failure or refusal to pay did not
constitute a breach of contract with Myers, when said agreement expressly stipulated.

Contrary to appellant Maritime's averments, the default was not made in good faith. The non -payment of the
installments was the result of a deliberate course of action on the part of appellant, designed to coerce the appellee
Myers Corporation into answering for an alleged promise o f the late F. H. MYERS to indemnify E. W. Schedler, for
any payments to be made to the members of the Luzon Labor Union.

Under the circumstances, the action of Maritime in suspending payments to Myers Corporation was a breach of
contract tainted with fraud or malice (dolo), as distinguished from mere negligence (culpa), "dolo" being succinctly
defined as a "conscious and intentional design to evade the normal fulfillment of existing obligations" and therefore
incompatible with good faith.

Well settled is, however, the rule that a judicial action for the rescission of a contract is not necessary where the
contract provides that it may be revoked and cancelled for violation of any of its terms and conditions

Resort to judicial action for rescission is obviously not contemplated.... The validity of the stipulation can not be
seriously disputed. It is in the nature of a facultative resolutory condition which in many cases has been upheld by
this Court.

2B 16-17 SALES AND LEASE Page 349


The obvious remedy of the party opposing the rescission for any reason being to file the corresponding action to
question the rescission and enforce the agreement.

~University of the Philippines vs. Walfrido de los Angeles

In other words, the party who deems the contract violated may consider it resolved or res cinded, and act
accordingly, without previous court action, but it proceeds at its own risk. For it is only the final judgment of the
corresponding court that will conclusively and finally settle whether the action taken was or was not correct in law.
But the law definitely does not require that the contracting party who believes itself injured must first file suit and
wait for a judgment before taking extrajudicial steps to protect its interest. Otherwise, the party injured by the
other's breach will have to passively sit and watch its damages accumulate during the pendency of the suit until the
final judgment of rescission is rendered when the law itself requires that he should exercise due diligence to
minimize its own damages

2B 16-17 SALES AND LEASE Page 350


2. ROQUE VS LAPUZ

96 SCRA 741

FACTS

In 1954 parties entered into an agreement of sale covering Lots 1,2 and 9, Block 1 in Rockville Subdivision in
Balintawak, Quezon City payable in 120 equal monthly installments at the rate of P16, P15 per square meters
respectively. Defendant payed P150 as deposit and P740.5 for four monthly installments

The plan covering plaintiffs property was merely tentative. After the approval of the Subdivision plan by the
Bureau, defendant requested plaintiff that he be allowed to abandon and substitute Lots 1, 2, and 9 with Lots 4 and
12 ,Block 2 of the said plan to which plaintiff graciously acceded. The purchase price changed to P17 per square
meter with 8% annual interest on the unpaid balance. Defendant occupied the land, built a house, enclosed it with
barbed wire and adobe walls.

Defendant failed to pay monthly installments apart from the first four months. Three years later (1957) plaintiff
asked defendant through a formal letter to vacate the Lots but the latter did not answer. On 1960, plaintiff instituted
an action for rescission of contract. Defendant argued that in their contract he was given anytime within ten years to
pay, therefore he was not in default, that there was no prior demand from plaintiff and that being recipr ocal
obligation plaintiff must first comply with his obligation which was to construct the facilities on the subdivision
allegedly promised.

RTC and CA ruled in favor of plaintiff rescinding the contract however upon MR, CA gave defendant 90 day
period to fulfill his obligation as given by 3rd paragraph of 1191 of the Civil code. Upon appeal, plaintiff argued
among others that 1191 should not have been applied because 1592(provides that no extension must be given in case
demand for rescission already made) must be applied instead of the former in case of rescission of sale of
immovable properties.

ISSUES:

(1) Whether or not 1592 not 1191 of the Civil code is applicable in the case at bar.
(2) Whether or not defendant is entitled to an extension of the period in which to pay the balance long due applying
3rd paragraph of 1191.

HELD

(1) No, 1191 instead if 1592 is should be applied.

It is imperative for the Court to identify the contract as one of sale or to sell because 1592 does not apply to
contracts to sell. There is no writing or document evidencing the agreement originally entered into between
petitioner and private respondent except the receipt showing the initial deposit of P150.00 as shown in Exh. A and
the payment of the 4months installment made by respondent corresponding to July, 1954 to October, 1954 in the
sum of P740.56 as shown in Exh. B. Neither is there any writing or document evidencing the modified agreement
when the 3 lots were changed to Lots 4 and 12 with a reduced area of 725 sq. meters, which are corner lots. This
absence of a formal deed of conveyance is a very strong indication that the parties did not intend immediate
transfer of ownership and title, but only a transfer after full payment of the price.

2B 16-17 SALES AND LEASE Page 351


The overwhelming weight of authority culminating in the Luzon Brokerage vs. Maritime cases laid down
the rule that Article 1592 of the New Civil Code does not apply to a contract to sell where title remains with the
vendor until full payment of the price as in the case at bar. The decision also stressed that there can be no
rescission or resolution of an obligation as yet nonexistent, because the suspensive condition did not happen.
The contract is a contract to sell NOT a contract of sale. Thus 1592 is no t applicable.

(2)No.

His refusal to pay further installments on the purchase price, his insistence that he had the option to pay the
purchase price any time in ten years inspite of the clearness and certainty of his agreement with the petitioner as
evidenced further by the receipt, Exh. B, his dilatory tactic of refusing to sign the necessary contract of sale on the
pretext that he will sign later when he shall have upstated his monthly payments in arrears but which he never
attempted to update, and his failure to deposit or make available any amount since the execution of Exh. B on
June 28, 1954 up to the present or a period of 26 years, are all unreasonable and unjustified which altogether
manifest clear bad faith and malice on the part of respondent Lapuz, making inapplicable and unwarranted the
benefits of paragraph 3, Art. 1191, N.C.C. To allow and grant respondent an additional period for him to pay the
balance of the purchase price, which balance is about 92% of the agreed price, wou ld be tantamount to his bad faith
and sanctioning the deliberate infringement of a contractual obligation that is repugnant and contrary to the stability,
security and obligatory force of contracts. Moreover, respondents failure to pay the succeeding 116 monthly
installments after paying only 4 monthly installments is a substantial and material breach on his part, not merely
casual, which takes the case out of the application of the benefits of paragraph 3, Art. 1191, N.C.C.

One final point, on the supposed substantial improvements erected on the land, respondents house. To grant
the period to the respondent because of the substantial value of his house is to make the land an accessory to the
house. This is unjust and unconscionable since it is a rule in Our Law that buildings and constructions are regarded
as mere accessories to the land which is the principal, following the Roman maxim omne quod solo inadeficatur
solo cedit (Everything that is built on the soil yields to the soil).

Note:
CAs grant of extension period is hinged on the reasoning that defendant already expended on much improvements
on the land insomuch that allowing outright rescission would unduly prejudice him of his substantial rights.

2B 16-17 SALES AND LEASE Page 352


3. CARIDAD ESTATES VS PABLO SANTERO

G.R. No. L 47231; December 19, 1940

Laurel, J.:

FACTS:

On November 28, 1934, Caridad Estates, Inc., through its manager, Hammon H. Buck, leased to Pablo Santero
cadastral lots Nos. 1080 B-1, 1080 B-2 and 1116 in the Municipality of Cavite, Cavite, for one year for P2,200. Said
lands were used for fish pond and salt bed purposes. Three months prior to the expiration of the contract, or on
August 24, 1935, the lessor sold the same lots to the lessees for P30,000, payable as follows: P1,500 on the
execution of the agreement; P4,000 on or before December, 1935; P4,500 on or before March, 1936; and the
remaining balance of P20,000 in ten years. In said contract, the parties stipulated that should the vendee fail to make
the payments agreed upon within sixty days of the date they fall due, the total balance shall become due and payable
and recoverable by an action at law, or the vendor may recover possession of the property and consider any and all
sums paid by the vendee applied as rent. By March, 1936 the vendee Santero was delinquent for P2,445. This led
Caridad Estates to choose rescission and demand the return of the property. Santero argues that the stipulation
forfeiting the payments as rent amounts to Pactum Comissorium or the automatic acquisit ion a debtors property
by the creditor for the payment of a debt. Thus, the stipulation is illegal.
ISSUE: Is the stipulation illegal and amounting to Pactum Comissorium ?
HELD: No, the stipulation that the vendor shall apply the installments to rent, should the vendee fail to pay the
installments on time is valid. Pactum Comissorium applies only when the debtor is already the owner of the
property. In this contract of sale, the transfer of ownership was reserved by the vendor until the payment of th e last
installment, the vendee never acquired ownership of the thing.
The forfeiture of installment payments is not contrary to law, morals, or public policy.

THE CARIDAD ESTATES, INC., plaintiff-appellee, vs. PABLO SANTERO, defendant-appellant

G.R. No. L-47231. December 19, 1940. LAUREL, J.:

FACTS: On November 28, 1934, plaintiff-appellee, through its manager, Hammon H. Buck, leased to defendant -
appellant cadastral lots in the municipality of Cavite, Cavite, for one year for P2,200. Said lands were u sed for
fishpond and salt bed purposes. About three months prior to the expiration of the contract of lease, or on August 24,
1935, the lessor sold the same lots to the leases for P30,000, payable as follows: P1,500 on the execution of the
agreement; P4,000 on or before December, 1935; P4,500 on or before March, 1936; and the remaining balance of
P20,000 in ten years, each annual installment to be paid on or before the month of August of each year beginning
1937. In said contract, the parties stipulated that should the vendee fail to make the payments agreed upon within
sixty days of the date they fall due, the total balance shall become due and payable and recoverable by an action at
law, or the vendor may recover possession of the property and consider any and all sums paid by the vendee
forfeited.

2B 16-17 SALES AND LEASE Page 353


On account of the purchase price of P30,000, the vendee, defendant -appellant, made several payments. As
things thus stood, the amount outstanding in vendee's account as of March, 1936, was P2,445.20. The defend ant-
appellant claims that he offered to pay this amount on September 21, 1936, but the plaintiff refused to accept
payment on the ground that the contract of sale had been definitely cancelled since September 15, 1936, when the
same lands were conveyed by sale to Triston Sison. On the other hand, it is alleged by the plaintiff-appellee that on
August 31, 1936, its general manager by formal communication, advised the defendant of the revocation of the
contract of sale and asked the latter to vacate the premises immediately thereafter.

As the defendants-appellant would not surrender possession of the lands in question, plaintiff-appellee,
filed a complaint for illegal detainer and recovery of rentals against defendant -appellant in the justice of the peace of
court of Cavite. The court rendered its decision, ordering the defendant to surrender the property in question, and to
pay the plaintiff, for its use and occupation, a monthly rental of P200, beginning September 16, 1936, until actual
delivery, with legal interest from the commencement of the suit.

Defendant-appellant brought the case on appeal to the Court of First Instance which affirmed the ruling of
the justice of the peace court.His exception and motion for new trial having been denied, the defendan t-appellant
moved to declare the provincial sheriff in contempt of court for the reasons stated in his petition, and presented
another motion praying for the dissolution of the order of execution issued by the justice of the peace court of
Cavite. The Court of First Instance disallowed the two motions of the defendant.

From this judgment, the defendant appealed.

ISSUE

Whether or not provisions of the contract of sale, more specifically paragraphs 3 and 4 thereof, violate those legal
principles which condemn pacto commissorio?

HELD

No. As provided in the contract of sale, paragraph 4 gives the vendor, if the vendee fails to make the specified
payments, the option of (1) considering the total remaining purchase price due and payable and recoverable by an
action at law or (2) recovering the possession of the property in which case any and all sums paid by the vendee
shall be regarded as rental for the use and occupancy of the property. On the other hand, paragraph 3 obligates the
vendee to deliver the possession of the property and the improvements thereon in good condition and repair in the
event that the vendor should demand the return of the same on account of noncompliance with the terms and
conditions of payment.

It is quite plain, therefore, that the course followed by the vendor in cancelling the contract and demanding
the repossession of the property was well supported by the employed in consonance with, the covenants embodied in

2B 16-17 SALES AND LEASE Page 354


their agreement. As the stipulations in question do not violate the prohibitive provisions of the land or defeat morals
and public order they constitute the law between the parties, binding and effectual upon them.

Appellant, however, gives full reliance on article 1504 of the Civil Code, and vigorously argues that
whatever be the provision of the contract, resolution may not be declared in the absence of a demand upon the
vendee "either judicially or by a notarial act." A cursory reading of the provision would be the best refutation of the
appellant's argument, as it leaves no doubt as to its inapplicability in the present instance. The contract is a sale in
installment, in which the parties have laid down the procedure to be followed in the event the vendee failed to fulfill
his obligation. There is, consequently, no occasion for the application of the requirements of article 1504.

Taking up the argument that the stipulations outlined in paragraphs 3 and 4 of the contract have resulted in
a pactum commissorium, the court is of the opinion that the objection is without legal basis. Historically and in point
of strict law, pactum commissorium, presupposes the existence of mortgage or pledge or that of antichresis. Upon
this account, it becomes hardly conceivable, although the argument has been employed here rather extrava gantly,
that the idea of pactum commissorium should occur in the present contract of sale, considering that, it is admitted,
the person to whom the property is forfeited is the real and equitable owner of the same because title would not pass
until equitable owner of the same because title would not pass until the payment of the last installment.

At most, the provisions in point, as the parties themselves have indicated in the contract, is a penal clause
which carries the express waiver of the vendee to any and all sums he paid when the vendor, upon his inability to
comply with his duty, seeks to recover possession of the property, a conclusive recognition of the right of the vendor
to said sums, and avoid unnecessary litigation designed to enforce fulfillment of the terms and conditions agreed
upon. Said provisions are not unjust or inequitable and does not, as appellant contends, make the vendor unduly rich
at his cost and expense. The charge that the amount forfeited greatly exceeded that which should be paid had the
contract been one of lease loses its weight when we consider that during the years 1935 and 1936, when the
agreement was full force and effect, the price of salt rose high to bring big profits and returns.

2B 16-17 SALES AND LEASE Page 355


4. EUSEB IO MANUEL vs.EULOGIO RODRIGUEZ, SR., ET AL.,

G.R. No. L13435 July 27, 1960

Reyes, JBL, J

FACTS:

Januaria Rodriguez was the original registered owner of a big tract of land (part of which is the land in
question). In 1924, Januara Rodriguez ceded and transferred said land to the Payatas Subdivision Inc., to be
administered by said firm, subdivided, sold, leased or otherwise disposed of.

Eulogio Rodriguez was then the Secretary Treasurer of said Payatas Subdivision Inc. Manuel offered to
buy the lot in question in instalments. It was agreed that a downpayment of P1,300 is to be made and the balance be
paid within 9 to 10 months without interest.

After making the initial payment of P1,300, plaintiff-appellant was placed in the possession of the lot. It
also appears that plaintiff-appellant did not make any payments within the 9 to 10month period. Payatas Subdivision
Inc. sent him a letter urging immediate payment of his unpaid account with the Company, which, including interest,
amounted to P819.23. Thereafter, plaintiff-appellant made another payment of P300. This appears to be the last
payment made.

Payatas Subdivision Inc. then considered his contract cancelled and extinguished, and the amounts already
paid forfeited to the Company, the transaction being merely a contract to sell or promise to sell.

Sometime in 1939, the Payatas Subdivision Inc., having sold all its properties (except some

properties it was administering for Januaria Rodriguez), was extrajudically dissolved, all unsold properties
belonging to Januaria Rodriguez were returned to her.

Januaria Rodriguez, who was the aunt of Eulogio Rodriguez, sold several properties to the latter, including
the lot in question. Eulogio Rodriguez, Sr., then Mayor of Manila, urged Manuel to pay his unsettled account with
the Payatas Subdivision, Inc. Still, there was no payment.

Eulogio Rodriguez, Sr. sold the lot to John Landahl. The sale was duly registered.

A little less than 23 years after the alleged sale to him of Lot 51 in 1926, Manuel brought the instant case,
as aforesaid, to compel the execution of a formal deed of conveyance in his favour covering the purported sale in
1926 to compel receipt of the unpaid balance of the price which plaintiffappellant consigned in court and to annul
the subsequent sales to Eulogio Rodriguez and to John Landahl, and the corresponding transfer certificates to title
issued to them.

ISSUE:

Whether or not Payatas had the right to rescind the contract?

HELD:

2B 16-17 SALES AND LEASE Page 356


YES! The contention of Manuel that Payatas Subdivision Inc. had no right to c ancel the contract as there was only a
"casual breach" is likewise untenable. In contracts to sell, where ownership is retained by the seller and is not to pass
until the full payment of the price, such payment, as we said, is a positive suspensive condition, the failure of which
is not a breach, casual or serious, but simply an event that prevented the obligation of the vendor to convey title from
acquiring binding force, in accordance with Article 1117 of the Old Civil Code. To argue that there was only a
casual breach is to proceed from the assumption that the contract is one of absolute sale, where non -payment is a
resolutory condition, which is not the case.

The contention that Payatas Subdivision had no right to cancel the contract, as there was no demand by suit
or notarial act, as provided by Article 1504 of the Old Code (now Art. 1592, N. C. C.) was untenable. Article 1504
requiring demand by suit or notarial act in case the vendor of realty wants to rescind, does not apply to a contract to
sell or promise to sell, where title remains with the vendor until fulfillment to a positive suspensive condition, such
as full payment of the price.

2B 16-17 SALES AND LEASE Page 357


5. UNIVERSITY OF THE PHILIPPINES VS WALFRIDO DE LOS ANGELES, in his capacity as JUDGE of
the COURT OF FIRST INSTANCE IN QUEZON CITY, et al

G.R. No. L-28602 September 29, 1970

Reyes, JBL J

FACTS:

On November 2, 1960, UP and ALUMCO entered into a logging agreement whereby the latter was granted
exclusive authority to cut, collect and remove timber from the Land Grant for a period starting from the date of
agreement to December 31, 1965, extendible for a period of 5 years by mutual agreement.

On December 8, 1964, ALUMCO incurred an unpaid account of P219,362.94. Despite repeated demands,
ALUMCO still failed to pay, so UP sent a notice to rescind the logging agreement. On the other hand, ALUMCO
executed an instrument entitled Acknowledgment of Debt and Proposed Manner of Payments. It was approved by
the president of UP, which stipulated the following: 3. In the event that the payments called for are not sufficient to
liquidate the foregoing indebtedness, the balance outstanding after the said payments have been applied shall be paid
by the debtor in full no later than June 30, 1965; 5. In the event th at the debtor fails to comply with any of its
promises, the Debtor agrees without reservation that Creditor shall have the right to consider the Logging
Agreement rescinded, without the necessity of any judicial suit

ALUMCO continued its logging operations, but again incurred an unpaid account. On July 19,1965, UP informed
ALUMCO that it had, as of that date, considered rescinded and of no further legal effect the logging agreement, and
that UP had already taken steps to have

another concessionaire take over the logging operation. ALUMCO filed a petition to enjoin UP from conducting the
bidding. The lower court ruled in favor of ALUMCO, hence, this appeal.

ISSUE

Whether or not petitioner UP treat its contract with ALUMCO rescinded, and may disregard the s ame before any
judicial pronouncement to that effect?

HELD

Yes. In the first place, UP and ALUMCO had expressly stipulated that upon default by the debtor, UP has the right
and the power to consider the Logging Agreement of December 2, 1960 as rescinded without the necessity of any
judicial suit. As to such special stipulation and in connection with Article 1191 of the Civil Code, the Supreme
Court, stated in Froilan vs. Pan Oriental Shipping Co:

2B 16-17 SALES AND LEASE Page 358


There is nothing in the law that prohibits the parties fro m entering into agreement that violation of the terms of the
contract would cause cancellation thereof, even without court intervention. In other words, it is not always necessary
for the injured party to resort to court for rescission of the contract.

Of course, it must be understood that the act of party in treating a contract as cancelled or resolved on account of
infractions by the other contracting party must be made known to the other and is always provisional, being ever
subject to scrutiny and review by the proper court. If the other party denies that rescission is justified, it is free to
resort to judicial action in its own behalf, and bring the matter to court. Then, should the court, after due hearing,
decide that the resolution of the contract was not warranted, the responsible party will be sentenced to damages; in
the contrary case, the resolution will be affirmed, and the consequent indemnity awarded to the party prejudiced.

In other words, the party who deems the contract violated may consider it resolved or rescinded, and act
accordingly, without previous court action, but it proceeds at its own risk . For it is only the final judgment of the
corresponding court that will conclusively and finally settle whether the action taken was or was no t correct in law.
But the law definitely does not require that the contracting party who believes itself injured must first file suit and
wait for a judgment before taking extrajudicial steps to protect its interest. Otherwise, the party injured by the oth er's
breach will have to passively sit and watch its damages accumulate during the pendency of the suit until the final
judgment of rescission is rendered when the law itself requires that he should exercise due diligence to minimize its
own damages (Civil Code, Article 2203).

2B 16-17 SALES AND LEASE Page 359


6. ADELFA PROPERTIES, INC., vs. COURT OF APPEALS, ROSARIO JIMENEZ-CASTAEDA and
SALUD JIMENEZ

G.R. No. 111238 January 25, 1995

Regalado, J

FACTS

Herein private respondents and their brothers, Jose and Dominador Jimen ez, were the registered co-owners of a
parcel of land. On July 28, 1988, Jose and Dominador Jimenez sold their share consisting of one -half of said parcel
of land, specifically the eastern portion thereof Adelfa Properties Inc. pursuant to a " Kasulatan sa Bilihan ng Lupa."
Subsequently, a "Confirmatory Extrajudicial Partition Agreement" was executed by the Jimenezes. Thereafter,
Adelfa Properties Inc. expressed interest in buying the western portion of the property from private respondents.
Accordingly, on November 25, 1989, an "Exclusive Option to Purchase" 5 was executed between petitioner and
private wherein the sum of P50,000.00 received from ADELFA PROPERTIES, INC. is an option money shall be
credited as partial payment upon the consummation of the sale and the balance in the sum of TWO MILLION
EIGHT HUNDRED SIX THOUSAND ONE HUNDRED FIFTY PESOS to be paid on or before November 30,
1989; and in case of default on the part of ADELFA PROPERTIES, INC. to pay option shall be cancelled and 50%
of the option money to be forfeited in our favor and we will refund the remaining 50% of said money upon the sale
of said property to a third party.

Considering, however, that the owner's copy of the certificate of title issued to respondent Salud Jimenez had been
lost and a new owner's copy of the certificate of title was issued but it remained in the possession of Atty. Bernardo
until he turned it over to petitioner Adelfa Properties, Inc. Before petitioner could make payment, it received
summons on November 29, 1989, together with a copy of a complaint filed by the nephews and nieces of private
respondents against the latter, Jose and Dominador Jimenez, and herein petitioner in the Regional Trial Court of
Makati for annulment of the deed of sale in favor of Household Co rporation and recovery of ownership of the
property.

Adelfa Properties Inc. informed private respondents that it would hold payment of the full purchase price and
suggested that private respondents settle the case with their nephews and nieces. Respondent Salud Jimenez refused
to heed the suggestion of petitioner and attributed the suspension of payment of the purchase price to "lack of word
of honor." Adelfa Properties Inc. On December 7, 1989, petitioner caused to be annotated on the title of the lot its
option contract with private respondents, and its contract of sale with Jose and Dominador Jimenez. However,
private respondents sent Francisca Jimenez to see Atty. Bernardo, in his capacity as petitioner's counsel, and to
inform the latter that they were cancelling the transaction. In turn, Atty. Bernardo offered to pay the purchase price
provided that P500,000.00 be deducted therefrom for the settlement of the civil case. This was rejected by private
respondents. On December 22, 1989, Atty. Bernardo wrote private respondents on the same matter but this time
reducing the amount from P500,000.00 to P300,000.00, and this was also rejected by the latter.

Atty. Bernardo wrote private respondents informing the latter that in view of the dismissal of the case ag ainst them,
petitioner was willing to pay the purchase price, and he requested that the corresponding deed of absolute sale be
executed. This was ignored by private respondents. Subsequently, private respondents' counsel sent a letter to
petitioner enclosing therein a check for P25,000.00 representing the refund of fifty percent of the option money paid
under the exclusive option to purchase. Private respondents then requested petitioner to return the owner's duplicate

2B 16-17 SALES AND LEASE Page 360


copy of the certificate of title of respondent Salud Jimenez. Petitioner failed to surrender the certificate of title,
respondents filed a case. RTC ruled in favor of respondents. CAffirmed.

ISSUE:

Whether or not the contract with Adelfa Properties Inc was validly rescinded?

HELD:

YES. Private respondents may no longer be compelled to sell and deliver the subject property to petitioner for two
reasons, that is, petitioner's failure to duly effect the consignation of the purchase price after the disturbance had
ceased; and, secondarily, the fact that the contract to sell had been validly rescinded by private respondents.

In the case in bar, there was no proper tender of payment nor consignation in this case as required by law.The mere
sending of a letter by the vendee expressing the intention t o pay, without the accompanying payment, is not
considered a valid tender of payment.Furthermore, petitioner no longer had the right to suspend payment after the
disturbance ceased with the dismissal of the civil case filed against it. Necessarily, therefo re, its obligation to pay the
balance again arose and resumed after it received notice of such dismissal. Unfortunately, petitioner failed to
seasonably make payment, as in fact it has deposit the money with the trial court when this case was originally filed
therein.

By reason of petitioner's failure to comply with its obligation, private respondents elected to resort to and did
announce the rescission of the contract through its letter to petitioner dated July 27, 1990. That written notice of
rescission is deemed sufficient under the circumstances. Article 1592 of the Civil Code which requires rescission
either by judicial action or notarial act is not applicable to a contract to sell. Furthermore, judicial action for
rescission of a contract is not neces sary where the contract provides for automatic rescission in case of breach, as in
the contract involved in the present controversy.

In the case at bar, it has been shown that although petitioner was duly furnished and did receive a written notice of
rescission which specified the grounds therefore, it failed to reply thereto or protest against it. Its silence thereon
suggests an admission of the veracity and validity of private respondents' claim. Furthermore, the initiative of
instituting suit was transferred from the rescinder to the defaulter by virtue of the automatic rescission clause in the
contract. But then, the records bear out the fact that aside from the lackadaisical manner with which petitioner
treated private respondents' latter of cancellation, it utterly failed to seriously seek redress from the court for the
enforcement of its alleged rights under the contract. If private respondents had not taken the initiative of filing Civil
Case No. 7532, evidently petitioner had no intention to take any legal action to compel specific performance from
the former. By such cavalier disregard, it has been effectively estopped from seeking the affirmative relief it now
desires but which it had theretofore disdained.

2B 16-17 SALES AND LEASE Page 361


7. PNB vs. CA

G.R. No. 119580 September 26, 1996

FACTS

The subject matter of the case is a parcel of land situated at the corner of Carlos Palanca and Helios Streets, Sta.
Cruz, Manila, covered by a Transfer Certificate of Title owned and registered under the name of PNB. July 14,
1983, Lapaz made a formal offer to purchase the parcel of land. PNB advised Lapaz of its approval of the offer to
purchase the subject property subject to some conditions. Lapaz signified her conformity to the same. One of the
conditions indicated therein was to clear the subject property of its occupants, so Lapaz undertook the ejectment of
the squatters therein at her expense. Later, Lapaz would ask for adjustment of payment proposals. On February 28,
1984, PNB wrote Lapaz reminding her of her failure to remit the amount of P978,860.00 as embodied in its letter
dated December 6, 1983 and of her refusal to send her letter of conformity to the letter-agreement. Lapaz was
likewise advised to remit her cash payment of the full price amounting to P5,378,902.50; otherwise, the subject
property shall be sold to other interested party/ies and her deposit forfeited. Lapaz's request for adjustment of
payments was likewise denied. Lapaz, due to a significant reduction in the land area being purchased, requested for
the reduction of the selling price from P5,394,300.00 to P5,135,599.17 on cash basis or a total of P6,066,706.49 on
installment. This was favorably acted upon by Lapaz. However, when no further payment was received by PNB
from Lapaz, the former notified the latter by telegram that it was giving her a last chance to pay the balance of the
required downpayment of P563,341.29; failure of which shall cause the cancellation of the sale in her favor and the
forfeiture of her P100,000.00 deposit.

The sale in favor of Lapaz never materialized because of her failure to remit the required amount agreed upon;
hence, the proposed sale was cancelled and the [private respondent's] deposit of P100,000.00 was forfeited by the
defendant [petitioner]. PNB then leased the property to a certain Morse Rivera. On October 3, 1984 Lapaz requested
for a refund of her deposit in the total amount of P660,000.00 (P550,000.00) with a further request that since the
Bank was willing to refund to her her deposit provided that the P100,000.00 is forfeit ed in favor of the Bank, the
amount of P100,000.00 be reduced to P30,000.00 because her deposit of P660,000.00 (P550,000.00) had, after all,
already accumulated to a sizable amount of interest and, besides there was a delay in the approval of the contract or
proposal. Lapaz further intimated that her request for refund shall be subject to the release of the fund within one (1)
week from receipt thereof; otherwise, she would insist on purchasing the property subject to mutually agreed grace
period. On October 16, 1984, PNB released in favor of Lapaz the amount of P550,000.00 representing the refund of
deposit made on the offer to purchase the subject property. On August 30, 1985, [Lapaz] wrote a letter to the former
President of the Philippines, Ferdinand E. Marcos, requesting for the lifting of the directive suspending the sale of
the subject property, which letter was transmitted to the then President of the PNB for comment and/or action.

In its letter dated May 14, 1986, PNB advised Lapaz of the approval of her request for revival of the previously
approved offer to purchase the subject property subject to some further terms and conditions. The conditions were
adhered to except the sixth one. It was asked to be deleted because there has already been defrayal of such expenses
for ejectment beforehand.This was however not accepted by the bank. In a letter dated January 14, 1987, Lapaz
through counsel informed PNB that she was willing to pay and remit the amount of P827,119.83 representing the
balance of the 20% downpayment of the approved purchase price as soon as the subject property was cleared of its
present tenants/occupants. However, the bank in its letter dated January 30, 1987 informed Lapaz that it could no
longer grant her any extension to pay the abovestated amount, and cancelled on January 30, 1987 the approved sale
in plaintiff's [private respondent's] favor for being stale and unimplemented and forfeited her deposit of
P200,000.00. Lapaz through counsel sent the letter dated February 6, 1987 asking for a reconsideration of bank's

2B 16-17 SALES AND LEASE Page 362


position on the matter by honoring the approved sale in plaintiff's [private respondent's] favor as well as her deposit.
In reply, the Bank denied any further extension in favor of the plaintiff [private respondent] and likewise informed
her that it had already decided to sell the property for not less than P7,082,972.00 through negotiated or sealed
bidding. As a consequence of the cancellation of the approved offer to purchase in her favor, Lapaz filed [an] action
for Specific Performance and Damages with Prayer for a Writ of Preliminary Injunction and Temporary Restraining
Order.

ISSUE

Whether or not Mrs. Ngos refusal to pay the downpayment is not a valid basis for PNBs cancellation of the
approved sale

HELD

YES. Also, the subject contracts are contracts to sell, contrary to the court a quo finding.

When the first letter-agreement was cancelled by petitioner, and private respondent agreed to that cancellation upon
receiving P550,000.00 as refund of her aggregate deposit, all the effects of that agreement were terminated. Upon
mutual assent to that cancellation, the agreement so cancelled thereafter no longer existed. Thus, compliance by
private respondent with the terms and conditions of that first agreement served the pu rposes of that agreement and
cannot be made to serve the purposes of the second letter-agreement. Respondent court fallaciously tacked the two
agreements with each other and commingled their effects; it incorrectly considered petitioner's successful ejectment
of the subject property's 1983 occupants under the first letter-agreement to be sufficient compliance with the
condition under the second letter-agreement that the subject property be cleared of its 1986 occupants.

The records attest to the fact that private respondent refused to accept condition No. 6 of the second letter-
agreement, dated May 14, 1986. Private respondent offered, for the second time, after the first letter-agreement was
cancelled, to buy the subject property from petitioner who accepted such offer but subject to specified terms and
conditions. Thus, petitioner's acceptance of private respondent's offer was a qualified acceptance, which in effect, is
a counter-offer necessitating private respondent's acceptance in return. Refusing to bin d herself to bear the expenses
for a second ejectment suit involving the subject property, private respondent in effect rejected petitioner's counter
offer or at the least, accepted the same subject to the deletion of condition No. 6. This, it has to be no ted, is another
counter-offer necessitating acceptance this time by petitioner. Petitioner was unwilling to accept the same and
demanded remittance of the remainder of the downpayment, the failure of which payment, petitioner warned private
respondent, would result in the forfeiture of the initial deposit of P200,000.00 and the ipso facto cancellation of the
second letter-agreement enabling petitioner to sell the subject property through sealed bidding.

From the foregoing, it is clear that private respondent and petitioner were negotiating for terms mutually acceptable
to them. Unfortunately, a mutually acceptable set of terms was not reached between them, and petitioner exercised
its right under the second letter-agreement to cancel the same. This process of negotiation undertaken in 1986 by
herein private parties is undeniably distinct from and entirely independent of the events that transpired in 1983 in the
context of the first letter agreement. Precisely another negotiation was necessary because this 1986 transaction is
different and separate from that undertaken by the said parties in 1983.

NOTE

2B 16-17 SALES AND LEASE Page 363


'1. That the selling price shall be P5,135,599.17 (P200,000.00) already deposited x x x

2. a. That upon your failure to pay the additional deposit of P827,119.83 upon receipt of advice of approval, your
P200,000.00 deposit shall be forfeited and for this purpose, the Bank can sell the property to other interested parties;

xxx

3. That your previous deposit of P100,000.00 which was forfeited by the Bank due to your failure to consummate
the previously-approved sale, shall not be considered as part of the purchase price;

4. That the Bank sells only whatever rights, interests and participation it may have in the property and you are
charged with full knowledge of the nature and extent of said rights, interests and participation and waives [sic] your
right to warranty against eviction;

xxx

6. That the property shall be cleared of its present tenants/occupants but all expenses to be incurred in connection
with the ejectment proceedings shall be for your account;

7. That the sale shall be subject to all terms and conditions covering sale of similar acquired real estate properties;

8. That the sale shall also be subject to all terms and conditions that the Legal Department may impose to protect the
interest of the Bank.'

2B 16-17 SALES AND LEASE Page 364


8. ROMERO VS CA

G.R. No. 107207 November 23, 1995

Vitug, J

FACT

Petitioner Engr. Virgilio Romero was engaged in the business of permalite insulation and processed perlite ore, and
he planned to put up a central warehouse on a land approximately 2,000sq.m. in Metro Manila. Private respondent
Enriqueta De Ongsiong owned a land in San Dionisio, Paranaque, which although inhabited by illegal settlers, was a
suitable place for the central warehouse. Parties entered into a Deed of Conditional Sale which stated that upon
payment of P50,000 by Romero, Ongsiong would commence an ejectment proceeding and would sell the property
for P800/sq.m. It was stipulated that upon failure of Ongsiong to evict the squatters within 60days after execution of
contract, he is bound to return the P50,000, and that Romero is only obliged to pay the remaining purchase price
upon the eviction.

Romero paid the amount and the ejectment suit was filed. Despite winning the ejectment suit, Ongsiong was not
able to oust the illegal settlers from the property. She later went to Romero and returned the amount, informing him
that their contract has been rendered null and void by reason of her failure to comply with the condition of evicting
the squatters. Due to Romeros refusal to accept the amount, Ongsiong filed a case for rescission of the conditional
sale and consigned the amount.

Romero contended that the remedy of rescission cannot be had because such remed y is only available to the
aggrieved party, not to the party who had caused the injury.

ISSUE

Whether or not a vendor may demand rescission of a contract of sale for a parcel of land for a cause traceable to his
own failure to have the squatters evicted within the stipulated period?

HELD

No, only the injured party may claim the remedy of rescission. Under Article 1191, the right of resolution of a party
to an obligation is predicated on a breach of faith by the other party that violates the reciprocity b etween them.
Romero did not breach the agreement and even offered to undertake the eviction himself, which is considered an act
of waiving the condition upon which his obligation to pay the remaining purchase price was hinged on.

2B 16-17 SALES AND LEASE Page 365


9. SALAZAR VS CA

G.R. No. 118203, July 5, 1996

Davide Jr, J

FACTS:

Dr. Salazar offered to sell his properties to Jonette Borres for One Million Pesos (P1,000,000.00) with the
payment to be made within three (3) months. On May 28, 1989, Jonette Borres together with a certain Emilio T.
Salazar went to see Dr. Salazar at the latters residence in Bataan bearing a copy of a Deed of Absolute Sale and
Deed of Warranty but Dr. Salazar refused to sign because Jonette Borres did not have the money ready then. On
June 2, 1989, Jonette Borres met again Dr. Salazar who was about to leave for the USA. With here is the Deed of
Absolute Sale to which she asked Dr. Salazar to sign, but the latter was reluctantly agreed to sign the document
provided that Borres pays of the price in case on June 15, and the other half on June 30. Dr. Salazar constituted
Teresa Dizon as custodian at the Deed of Absolute Sale together with the Titles of the Land in question with the
instruction to Teresa Dizon not to surrender said documents to Jonette Borres until upon payment of the full price in
cash. On June 16, 1992, Dr. Salazar made an overseas call to Dizon to inquire if Jonette Borres had already paid
the down payment of P500,000.00 and Teresa Dizon replied to Dr. Salazar that Jonette Borres h ad not paid the down
payment. Dr. Salazar then ordered Dizon to stop the sale.

ISSUE:

Whether or not the contract entered into by Salazar and Borres is a perfected contract of sale.

HELD: NO, the contract was a contract to sell. If we are to consider only the Deed of Absolute Sale, we can
easily say that the contract between Salazar and Borres is one of sale. However, the Deed of Warranty and the oral
testimony on the circumstances surrounding the execution of the Deed of Absolute Sale, as well a s the other pieces
of evidence submitted by Borres, sustain the finding and conclusion of the trial court that the true agreement
between the parties was a contract to sell in that the true intent of Salazar was to transfer ownership of the property
to Borres only after the latter pays the full consideration. Undoubtedly, Salazar and Borres mutually agreed that
despite the Deed of Absolute Sale title to the two lots in question was not to pass to the latter until full payment of
the consideration of P1 million. The form of the instrument cannot prevail over the true intent of the parties as
established by the evidence.

2B 16-17 SALES AND LEASE Page 366


10. MANILA RACING CLUB VS THE MANILA JOCKEY CLUB

No. 46533. October 28, 1939

Avancena, J

FACTS

Rafael J. Campos purchased a land from Defendant Manila Jockey Club with the amount of P 1,200,000 payable as
follows P50,000 upon the signing of the contract; P50,000 on or before September 28, 1936; P300,000 on or before
December 24, 1936; P200,000 on or before March 24, 1937; and P600,000 on or before September 24, 1937. It was
agreed that should the purchaser fail to pay the amount corresponding to each installment in due time, the vendor
may rescind the contract and keep the amounts paid for itself. Also the purchaser may form a corporation called
Manila Racing Club Inc., to whom he may transfer all his rights and obligations under the contract.

The purchaser made the down payment and the second installment.

On 22 October 1936, the Manila Racing Club Inc. was organized and Campos transferred to it all his rights and
obligation under his contract with the Manila Jockey Club.

The 3rd installment of P300, 000 became due on 24 December 1936 and the purchaser could not pay it, the vendor
on 11 January 1937 declared the contract cancelled and kept the amount of P100, 000 already paid corresponding to
the first 2 installments. The purchaser was however granted an extension until 22 January 1937, to revive the
contract by paying the P300,000 but having failed to do this, the partners of the vendor rat ified on 23 January 1937
the cancellation of the contract agreed upon by its board of directors and the forfeiture of the P100,000 paid by the
purchaser.

The Plaintiff Manila Racing Club filed an action to recover the forfeited amount of P100, 000 and for the payment
of P50,000 as damages. The appealed judgment absolves the defendants.

ISSUE

Whether or not defendant is liable for damages and should return the forfeited amount.

HELD

No. The clause of the contract referring to the forfeiture of the P100,000 already paid, should the purchaser Campos
fail to pay the subsequent installments, is valid, the case does not present any difficulty because the contract is clear
on this point.

This clause regarding the forfeiture of what has been partially paid is valid. It is in the nature of a penal clause,
which may be legally established by the parties (articles 1152 and 1255 of the Civil Code). In its double purpose of
insuring compliance with the contract and of otherwise measuring beforehand the damages, which may result from
noncompliance, it is not contrary to law, morals or public order because the parties voluntarily and knowingly
agreed it upon. Viewing concretely the true effects thereof in the present case, the amount forfeited constitutes only
eight per cent of the stipulated price, which is not excessive if considered as the profit that would have been obtained
had the contract been complied with. There is, moreover, evidence that the defendants, because of this contract with
Campos, had to reject other propositions to buy the same property. At any rate, the penal clause does away with the
duty to prove the existence and measure of the damages caused by the breach.

2B 16-17 SALES AND LEASE Page 367


11. ACTIVE REALTY & DEVELOPMENT CORP VS DAROYA

G.R. No. 141205 May 9, 2002

Puno, J

FACTS:

Petitioner Active Realty & Development Corporation is the owner and developer of Town and Country
Hills Executive Village in Antipolo. It entered into a contract to sell with Daroya whereby the latter agreed to buy a
515 sq.m. lot for P224,025.00n petitioners subdivision.

The contract to sell stipulated that the respondent shall pay the initial amount of P53,766.00 upon execution
of the contract and the balance of P170,259.00 in sixty (60) monthly installments of P4,893.35. Adding the down
payment and installment payments, it would appear that the total amount is P346,367.00, a figure higher than that
stated as the contract price.

On May 5, 1989, petitioner accepted respondents amortization in the amount of P40,000.00. By August 8,
1989, respondent was in default of P15,282.85 representing three (3) monthly amortizations. Petitioner sent
respondent a notice of cancellation of their contract to sell, to take effect thirty (30) days from receipt of the letter. It
does not appear from the records, however, when respondent received the letter. Nonetheless, when respondent
offered to pay for the balance of the contract price, petitioner refused as it has allegedly sold the lot to another buyer.

Respondent filed an action for specific performance against petition er before the Housing and Land Use
Regulatory Board. The HLURB Arbiter ruled that the cancellation of the contract to sell was void as petitioner failed
to pay the cash surrender value to respondent as mandated by law. However, as the subject lot was alrea dy sold to a
third party and the respondent had agreed to a full refund of her installment payments, petitioner was ordered to
refund to respondent all her payments in the amount of P314,816.70, with 12% interest per annum from August 26,
1991 (the date of the filing of the complaint).

On appeal, the HLURB Board of Commissioners set aside the above decision The Board refused to apply
the remedies provided under the Maceda Law and instead deemed it fit to formulate an equitable solution to the
case. It ruled that, as both parties were at fault, i.e., respondent incurred in delay in her installment payments and
respondent failed to send a notarized notice of cancellation, petitioner was ordered to refund to the respondent one
half of the total amount she has paid or P157,408.35, which was allegedly akin to the remedy provided under the
Maceda Law.Respondent appealed to the Office of the President reversed the HLURBs decision.

ISSUE:

Whether or not the seller can be compelled to refund to the buyer the value of the lot or deliver a substitute lot at
buyers option

2B 16-17 SALES AND LEASE Page 368


HELD:

YES

Section 3 of R.A. No. 6552 provided for the rights of the buyer in case of default in the payment of
succeeding installments, where he has already paid at least two (2) years of installments, thus:

(a) To pay, without additional interest, the unpaid installments due within the total grace period earned by him,
which is hereby fixed at the rate of one month grace period for every one year of installment payments made; x
xx

(b) If the contract is cancelled, the seller shall refund to the buyer the cash surrender value of the payments on
the property equivalent to fifty per cent of the total payments made; provided, that the actual cancellation of
the contract shall take place after thirty days from receipt by the buyer of the notice of cancellation or the
demand for rescission of the contract by a notarial act and upon full payment of the cash surrender value to the
buyer.

In this case, respondent has already paid in four (4) years a total of P314,860.76 or P90,835.76 more than
the contract price of P224,035.00. In April 1989, petitioner decided to cancel the contract when the respondent
incurred in delay in the payment of P15,282.85, representing three (3) monthly amortizations. Petitioner refused to
accept respondents subsequent tender of payment of the outstanding balance alleging that it has already cancelled
the contract and sold the subject lot to another buyer. However, the records clearly show that the petitioner failed to
comply with the mandatory twin requirements for a valid and effective cancellation under the law, [19] i.e., he failed
to send a notarized notice of cancellation and refund the cash surrender value. At no time, from the date it gave a
notice of cancellation up to the time immediately before the respondent filed the case against petitioner, did the latter
exert effort to pay the cash surrender value. In fact, the records disclo se that it was only during the preliminary
hearing of the case before the HLURB arbiter when petitioner offered to pay the cash surrender value.Petitioner
justifies its inaction on the ground that the respondent was always out of the country. Even then, th e records are
bereft of evidence to show that petitioner attempted to pay the cash surrender value to respondent through her last
known address. The omission is surprising considering that even during the times respondent was out of the country,
petitioner has been sending her written notices to remind her to pay her installment arrears through her last known
address. Clearly, had respondent not filed a case demanding a final deed of sale in her favor, petitioner would not
have lifted a finger to give respondent what was due her actual payment of the cash surrender value, among others.
In disregard of basic equitable principles, petitioners stance would enable it to resell the property, keep respondents
installment payments, not to mention the cash surrender value which it was obligated to return. The Layug [20] case
cited by petitioner is inapropos. In Layug, the lot buyer did not pay for the outstanding balance of his account and
the Court found that notarial rescission or cancellation was no longer necessary as the seller has already filed in
court a case for rescission of the contract to sell. In the case at bar, respondent offered to pay for her outstanding
balance of the contract price but respondent refused to accept it.Neither did petitioner adduce proof that the
respondents offer to pay was made after the effectivity date stated in its notice of cancellation. Moreover, there was
no formal notice of cancellation or court action to rescind the contract. Given the circumstances, we find it illegal
and iniquitous that petitioner, without complying with the mandatory legal requirements for canceling the contract,
forfeited both respondents land and hard-earned money after she has paid for, not just the contract price, but more
than the consideration stated in the contract to sell.

2B 16-17 SALES AND LEASE Page 369


Thus, for failure to cancel the contract in accordance with the procedure provided by law, we hold that the
contract to sell between the parties remains valid and subsisting.Following Section 3(a) of R.A. No. 6552,
respondent has the right to offer to pay for the balance of the purchase price, without interest, which she did in this
case. Ordinarily, petitioner would have had no other recourse but to accept payment. However, respondent can no
longer exercise this right as the subject lot was already sold by the petitioner to another buyer which lot, as admitted
by the petitioner, was valued at P1,700.00 per square meter. As respondent lost he r chance to pay for the balance of
the P875,000.00 lot, it is only just and equitable that the petitioner be ordered to refund to respondent the actual
value of the lot resold, i.e., P875,000.00, with 12% interest per annum computed from August 26, 1991 un til fully
paid or to deliver a substitute lot at the option of the respondent

2B 16-17 SALES AND LEASE Page 370


12. LEANO VS CA

G.R. No. 129018. November 15, 2001

Pardo, J

FACTS

Hermogenes Fernando, as vendor and Carmelita Leao, as vendee executed a contract to sell involving a piece of
land, Lot No. 876-B, with an area of 431 square meters, located at Sto. Cristo, Baliuag, Bulacan. Carmelita Leao
bound herself to pay Hermogenes Fernando the sum of P107,750.00 as the total purchase price of the lot. The
manner of paying the total purchase price was as follows:

The sum of P10,775.00 shall be paid at the signing of this contract as DOWN PAYMENT, the balance of
P96,975.00shall be paid within a period of TEN (10) years at a monthly amortization of P1,747.30 to begin from
December 7, 1985 with interest at eighteen per cent (18%) per annum based on balances.

The contract provided for a one month grace period. Should the month of grace expire without the installments for
both months having been satisfied, interest will be charged.

Should a period of ninety (90) days elapse from the expiration of the grace period without the overdue and unpaid
installments having been paid with the corresponding interests up to that date, respondent Fernando, as vendor, was
authorized to declare the contract cancelled and to dispose of the parcel of land, as if the contract had not been
entered into. The payments made, together with all the improvements made on the premises, shall be considered as
rents paid for the use and occupation of the premises and as liquidated damages.

Carmelita Leao made several payments in lump sum, then she constructed a house on the lot valued at
P800,000.00.The last payment that she made was on April 1, 1989.

he trial court rendered a decision in an ejectment case ordering petitioner Leao to vacate the premises.

Petitioner Leao filed with the Regional Trial Court of Malolos, Bulacan a complaint for specific performance with
preliminary injunction. Petitioner assailed the validity of the judgment of the municipal trial court. The court
rendered judgement Ordering the plaintiff to pay to the defendant the sum of P103,090.70 corresponding to her
outstanding obligations under the contract to sell, among other things.

Fernando filed a motion for reconsideration thereto and the court increased the amount to P183,687.00.

petitioner Leao appealed the decision to the Court of Appeals. Court of Appeals promulgated a decision affirming
that of the Regional Trial Court in toto.

ISSUE

Whether there was a proper cancellation of the contract to sell.

HELD

2B 16-17 SALES AND LEASE Page 371


R. A. No. 6552 recognizes in conditional sales of all kinds of real estate (industrial, commercial, residential) the
right of the seller to cancel the contract upon non-payment of an installment by the buyer, which is simply an event
that prevents the obligation of the vendor to convey title from acquiring binding force.

Sec. 3 (b) of the law provides that, the actual cancellation of the contract shall take place after thirty days from
receipt by the buyer of the notice of cancellation or the demand for rescission of the contract by a notarial act and
upon full payment of the cash surrender value to the buyer.

The decision in the ejectment case operated as the notice of cancellation required by Sec. 3(b). As petitioner Leao
was not given the cash surrender value of the payments that she made, there was still no actual cancellation of the
contract. Consequently, petitioner Leao may still reinstate the contract by updating the account during the grace
period and before actual cancellation.

2B 16-17 SALES AND LEASE Page 372


13. PADILLA VS PAREDES

G.R. No. 124874, March 17, 2000

Quisumbing, J.:

FACTS

Petitioner and private respondents entered into a contract to sell a parcel of land in San Juan, La Union. At the time,
the land was untitiled, although respondent spouses were paying the taxes thereon. Petitioner undertook to register
the property under the respondents' names. He was to pay P50,000 as downpayment, and the remainder within 10
days of the decree of registration. On Dec. 27, 1989, the court ordered the issuance of a title in favor of Adelina
Paredes. Petitioner failed to pay the full purchase price after the period set. On Apr. 17, 1990, respondents offered to
sell half the land instead in exchange for the payments already made. If petitioner did not agree, they would rescind
the contract. Petitioner did not accept the proposal. Instead, he offered to pay the rest of the purchase price, plus
interests and attorney's fees, which respondents did not accept. On May 14, 1990, petitio ner instituted an action for
specific performance, alleging his substantial compliance with the original contract. The lower court ruled in favor
of petitioner, saying that his breach was only a casual breach that did not warrant recission. The CA reversed this
ruling.

ISSUE

Whether or not private respondents are entitled to recission under Article 1191 of the Civil Code

HELD

Yes. Petitioner argues that the the contract was one of a sale, and not a contract to sell. However, he only argued this
in his reply to the respondents, and not originally. Furthermore, petitioner himself admitted that it was him and his
counsel who prepared the contract in the first place. Article 1592, which the petitioner relies on, applies only in
contracts of sale, not a contract to sell as in this case. Furthermore, Article 1191 only speaks of existing obligations.
The obligation of the spouses has not yet arisen, because the suspensive condition of payment by the petitioner has
not yet been fulfilled.

2B 16-17 SALES AND LEASE Page 373


14. Sps. ENRIQUE and CONSUELO LIM vs.THE HONORABLE COURT OF APPEALS, Sps.
TERESITA and OSCAR GUEVARRA, Sps. MARCOS and ANITA ORLINO, Sps. ROMULO and
CONSUELO ORLINO and Sps. FELIX and DOLORES ORLINO, respondents.

G.R. No. 85733 February 23, 1990

Cruz, J.

FACTS

The subject of this controversy is a parcel of land consisting of 1,101 square meters located in Diliman, Quezon City
originally owned by Felix, Manuel and Maria Concepcion Orlino, who mortgaged it to the Progressive Commercial
Bank as security for a P100,000.00 loan on July 1, 1965. The loan not having been paid, the mortgage was
foreclosed and the bank acquired the property as the highest bidder at the auction sale. The mortgagee thereafter
transferred all its assets, including the said land, to the Pacific Banking Corporation (PBC).

On May 22, 1975, the Orlinos, and their respective spouses (hereinafter referred to as the private respondents), who
had remained in possession of the land, made a written offer to PBC to repurchase the property. The bank, in turn,
sent a letter confirming the agreement of repurchase.

One year later, on November 2, 1978, PBC advised the private respondents that if the transaction was not finalized
within 30 days, it would consider the offer of other buyers. The record does not show any further development until
June 8, 1979, when the private respondents requested PBC to allow them to secure a certified true copy of its
Torrens certificate over the land for purposes of its survey and partition among them preparatory to the actual
transfer of title to them. PBC granted the request subject to the condition that title would remain with it until the
execution of the necessary deed of conveyance.

Two years later, PBC reminded the private respondents of its letter of November 2, 1978, but again no action was
taken to deliver to it the stipulated consideration for the sale. Finally, on May 14, 1980, PBC executed a deed of sale
over the land in favor of the herein petitioners, the spouses Enrique and Consuelo Lim, for the sum of P300,000.00.

private respondents filed a complaint in the Regional Trial Court of Quezon City against the petitioners and PBC for
the annulment of the deed of sale on the ground that the subject land had been earlier sold to them. In its judgment
for the plaintiffs, the court held that both PBC and the spouses Lim had acted in bad faith when they concluded the
sale knowing that "there was a cloud in the status of the property in question." The decision was affirmed in toto by
the respondent court.

ISSUE

Was the transaction between private respondents and PBC, as embodied in the letter of November 9, 1977, a
contract to sell or a contract of sale?

2B 16-17 SALES AND LEASE Page 374


HELD

Contract to sell. There was no immediate transfer of title to the private respondents as would have happened if there
had been a sale at the outset. The supposed sale was never registered and TCT No. 218661 in favor of PBC was not
replaced with another certificate of title in favor of the private respondents. In their letter to PBC on June 8, 1979,
they acknowledged that title to the property would remain with the bank until their transaction shall have been
finalized. In response, PBC reiterated the same condition. No less important, the consideration agreed upon by the
parties was never paid by the private respondents, to convert the agreement into a contract of sale. In fact, PBC
reminded them twice on November 2, 1978, and on April 8, 1980 to comply with their obligations. They did
not. Their default was not, as the respondent court described it, "a slight delay" but lasted for all of three years and in
fact continued up to the rendition of the decision in the trial court. As payment of the consideration was a positive
suspensive condition, title to the subject property never passed to the private respondents. Hence, the property was
legally unencumbered and still belonged to PBC on May 14, 1980, when it was sold by the bank to the petitioners.

2B 16-17 SALES AND LEASE Page 375


15. LAURENCIO RAMEL VS DANIEL AQUINO

G.R. No. 133208 ; July 31, 2006

Puno, J.:

FACTS:
Daniel Aquino obtained a loan from Development Bank of the Philippines (DBP) and mortgaged a land in Cordon,
Isabela with a TCT in his name. The foreclosure date on the Property was about to become due. Since Aquino did
not have enough money to pay the loan and save the entire land from foreclosure, he decided to sell part of the
property to Laurencio Ramel. They entered upon an Oral Contract of Sale for 8.20 hectares for the total sum of
P110,700. Ramel would assume the mortgage obligation in the amount of P85,543 and the balance of about P25,000
shall be paid to Aquinos on installment. Ramel paid to the respondents P24,800 almost completing the balance .
However, Ramel was only able to settle P23,097 of the P85,543 loan from DBP. For fear of losing the entire land to
foreclosure, Aquino paid the loan and orally rescinded the contract of sale with Ramel. Ramel filed for a complaint
for Specific Performance arguing that Art. 1592 of the Civil Code requires a judicial or notarial rescission in the
Sale of Real Property. In its answer in the Complaint for Specific Performance, the Aquinos claim Rescission as an
ANSWER to the complaint.

ISSUE:

Did the Aquinos observe Art. 1592 in claiming rescission as an ANSWER to the Complaint?

HELD:
Yes. We held in the case of Luzon Brokerage Co., Inc. v. Maritime Building Co. Inc. that even a cross -claim found
in the Answer filed in the trial Court constitutes judicial demand for rescission that satisfies the requirements of
Article 1592. We rule that the respondents satisfied Art. 1592 when they raised rescission as a defense in their
Answer. To be sure, petitioners learned of respondents intention to rescind even before they filed their Answer. The
Aquinos intention to rescind was revealed when they fully -settled the Mortgage with DBP. Rescission was already
proper then since the obligation of the buyer to settle the mortgage obligation was breached.

2B 16-17 SALES AND LEASE Page 376


16. VALARAO VS CA

G.R. No. 130347 March 3, 1999

Panganiban, J

FACTS

On September 4, 1987, spouses Abelardo and Gloriosa Valarao, thru their son Carlos Valarao as their attorney-in-
fact, sold to [Private Respondent] Meden Arellano under a Deed of Conditional Sale a parcel of land situated in the
District of Diliman, Q.C.,for the sum of P3,225,000.00 payable in successive monthly in stallment. They further
agreed that should the buyer fail to pay three successive installments, the "conditional sale" will automatically be
rescinded w/o necessity of judicial action and the payments made by the vendee shall be forfeited in favor of the
seller herien petitioners.

Three years later, private respondent having paid a total of 2,028,000.00 failed to pay the amount due on october and
november. But on december 30 he tried to pay amount due on october, november and december through petitioners
maid (who accepted their previous payments on several occasions) but who refused to receive allegedly because she
was ordered not to accept.

Buyer reported the matter to the barangay but it was unavailing since the sellers never appeared on the arranged
meeting with the lupon. When buyer had contacted seller through phone, the latter told him that he would no longer
accept the payments being offered. Buyer filed for petition for consignagion. On the other hand seller enforced the
"automatic rescission" as a consequence declaring the "deed of conditional sale" null and void and all payments as
well as improvement made be forfeited. Seller denied the attempt of buyer to tender payments and ordered buyer to
vacate thr premises.

RTC ruled in favor of seller, rescinding the contract.

CA reversed applying 1529 which states that even though there be stipulation of automatic rescission, buyer may
still pay balance before judicial or notarial demand by the seller. The demand for rescission made by the seller is
only through letter.

ISSUES

(1) Whether or not the rescission of the "Conditional Sale' by the buyer was proper.
(2) Whether or not 1529 is applicable in the case

HELD

No to both.

2B 16-17 SALES AND LEASE Page 377


There is a contract to sell not one of sale.

ART. 1592. In the sale of immovable property, even though it may have been stipulated that upon failure to pay

the price at the time agreed upon the rescission of the contract shall of right take place, the vendee may pay, even
after the expiration of the period, as long as no demand for rescission o f the contract has been made upon him either
judicially or by notarial act. After the demand, the court may not grant him a new term.

This rule is applicable to sales not on contract to sell as in this case where the title of the property is reserved to the
seller until after the last payment of the installments due. The term used by the parties "conditional sale" is overruled
by their intention. Also Even after the execution of the Deed of Conditional Sale, the Torrens Certificate of Title
remained with and in the name of the vendors.

From the foregoing, it is clear that petitioners were not justified in refusing to accept the tender of payment made by
private respondent on December 30 and 31, 1990. Had they accepted it on either of said dates, she would have paid
all three monthly installments due. In other words, there was no deliberate failure on her part to meet her
responsibility to pay. The Court takes note of her willingness and persistence to do so, and, petitioners cannot now
say otherwise. The fact is: they refused to accept her payment and thus have no reason to demand the enforcement of
the automatic forfeiture clause. They cannot be rewarded for their own misdeed.

Because their maid had received monthly payments in the past, it is futile for petitioners to insist now that she could
not have accepted the aforementioned tender of payment, on the ground that she did not have a special power of
attorney to do so. Clearly, they are estopped from denying that she had such authority. Under Article 1241 of the
Civil Code, payment through a third person is valid [I]f by the creditors conduct, the debtor has been led to believe
that the third person had authority to receive the payment.

Application of the Maceda Law

In any event, the rescission of the contract and the forfeiture of the payments already made could not be effected,
because the case falls squarely under Republic Act No. 6552, otherwise known as the Maceda Law. Section 3 of
said law provides:

SEC. 3. In all transactions or contracts involving the sale or financing of real estate on installment payments,
including residential condominium apartments but excluding industrial lots, commercial buildings and sales to
tenants under Republic Act Numbered Thirty-eight hundred Forty-four as amended by Republic Act Numbered
Sixty-three hundred eighty-nine, where the buyer has paid at least two years of installments, the buyer is entitled to
the following rights in case he defaults in the payment of succeeding installments:

2B 16-17 SALES AND LEASE Page 378


(a) To pay, without additional interest, the unpaid installments due within the total grace period earned by him,
which is hereby fixed at the rate of one month grace period for every year of installment payments made: Provided,
That this right shall be exercised by the buyer only once in every five years of the life of the contract and its
extensions, if any.

(b) If the contract is cancelled, the seller shall refund to the buyer the cash surrender value of the payments on the
property equivalent to fifty percent of the total payments made and, after five years of installments, an additional
five percent every year but not to exceed ninety percent of the total payments made: Provided, That the actual
cancellation of the contract shall take place after thirty days from receipt by the buyer of the notice of cancellation or
the demand for rescission of the contract by a notarial act and upon full payment of the cash surrender value to the
buyer.

Down payments, deposits or options on the contract shall be included in the computation of the total number of
installments made.

Hence, the private respondent was entitled to a one-month grace period for every year of installments paid, which
means that she had a total grace period of three months from December 31, 1990. Indeed, to rule in favor of
petitioner would result in patent injustice and unjust enrichment. This tribunal is not merely a court of law, but also a
court of justice.

2B 16-17 SALES AND LEASE Page 379


17. DIEGO VS DIEGO

G.R. No. 179965, February 20, 2013

Del Castillo, J.:

FACTS:

In 1993, petitioner Nicolas P. Diego (Nicolas) and his brother Rodolfo, respondent herein, entered into an oral
contract to sell covering Nicolass share, fixed at P500,000.00, as co-owner of the familys Diego Building situated
in Dagupan City. Rodolfo made a down payment of P250,000.00. It was agreed that the deed of sale shall be
executed upon payment of the remaining balance of P250,000.00. However, Rodolfo failed to pay the remaining
balance. Meanwhile, the building was leased out to third parties, but Nicolass share in the rents were not remitted to
him by herein respondent Eduardo, another brother of Nicolas and designated administrator of the Diego Building.
Instead, Eduardo gave Nicolass monthly share in the rents to Rodolfo. Despite demands and p rotestations by
Nicolas, Rodolfo and Eduardo failed to render an accounting and remit his share in the rents and fruits of the
building, and Eduardo continued to hand them over to Rodolfo.

ISSUE:

Whether or not the contact entered into between the parties is a contract of sale.

HELD:

YES. The stipulation to execute a deed of sale upon full payment of the purchase price is a unique and
distinguishing characteristic of a contract to sell. It also shows that the vendor reserved title to the property until full
payment.

There is no dispute that in 1993, Rodolfo agreed to buy Nicolass share in the Diego Building for the price of
P500,000.00. There is also no dispute that of the total purchase price, Rodolfo paid, and Nicolas received,
P250,000.00. Significantly, it is also not disputed that the parties agreed that the remaining amount of P250,000.00
would be paid after Nicolas shall have executed a deed of sale. This stipulation, i.e., to execute a deed of absolute
sale upon full payment of the purchase price, is a unique and distinguishing characteristic of a contract to sell. In
Reyes v. Tuparan, this Court ruled that a stipulation in the contract, where the vendor promises to execute a deed of
absolute sale upon the completion by the vendee of the payment of the price, indicates that the parties entered into a
contract to sell. According to this Court, this particular provision is tantamount to a reservation of ownership on the
part of the vendor. Explicitly stated, the Court ruled that the agreement to exec ute a deed of sale upon full payment
of the purchase price shows that the vendors reserved title to the subject property until full payment of the purchase
price.

2B 16-17 SALES AND LEASE Page 380


ON REDEMPTION

1. ALBERTO VS DE LOS SANTOS

C.A. No. 17, May 24, 1948

Feria, J.

FACTS

The plaintiff's action is to compel the defendants to accept the sum of money offered by the former for the
redemption of a parcel of land which, according to the allegations in the complaint, was sold on execution sale by
one of the defendants, as sheriff of Bulacan, to the other defendants on March, 1941. In the complaint it is averred
that "the period of redemption of said property sold at public auction should have expired on March 22, 1942,
according to the certificate of sale, but because of the war declared on December 8, 1941 by the Japanese Empire
against the United States of America, and the Commonwealth Government of the Philippines was under the
sovereignty of the latter, the period of redemption was extended or interrupted.

ISSUE

Whether or not the period of redemption was extended or interrupted.

HELD

NO.

The period within which a property sold on execution may be redeemed is not a limitation of action.
Because, as we have said in Rivero vs. Rivero (G.R. No. L-578, April 30, 1948), promulgated recently, "the
fundamental reason underlying status providing for suspension or extension of period of limitation is the legal or
physical impossibility for the interested party to enforce or exercise in time his right of action"; and the war did not
make it impossible for the herein plaintiff to exercise his right of redemption.

ART. 1176. If the creditor to whom tender of payments has been made should refuse without reason to
accept it, the debtor may relieve himself of liability by the consignation of the thing due.

The same effect shall be produce by consignation alone when made by the absence of the creditor, or if the
latter should be incapacitated to accept the payment when it is due, or when several persons claimed to be entitled to
received it, or when the muniments of the obligation have been lost or mislaid.

2B 16-17 SALES AND LEASE Page 381


2. FAUSTO ROSALES v. VICENTE REYES and JUAN ORDOVEZA

G.R. No. 8162. October 10, 1913

Trent, J

FACTS:

On July 29, 1902, Rivera (stating that he was of age) sold a parcel of land to herein respondents for 800 pesos unde r
pacto de recto, on the condition that the repurchase could not be made until after three years from the date of the
contract of sale. On May 29, 1903, Rivera sold his right to repurchase to Rosales for 1,075 pesos. He also stated
in the document evidencing this sale that he is 23 years of age. Rosales alleges that in January, 1908, he tendered
800 pesos to Reyes and Ordoveza with the request that the land be surrendered to him in accordance with the
contract entered into between them and Rivera in 1902, but that they refused to accept the money and comply
with his request.

Respondents contend that the right to repurchase had expired before Rosales attempted to exercise it and that the
complaint is defective in that it does not allege that the redemption price was judicially deposited upon the refusal of
the defendants to surrender the property.

ISSUE

Upon what basis must the duration of the right to repurchase be calculated?

HELD

The contract of 1902 provided that the right to repurchase could not be exercised within three years from
the date of the contract. A stipulation in the contract providing that the right to repurchase is suspended for a certain
time is undoubtedly a benefit to both the vendor and the purchaser. To the latter it affords a basis upon which he
may plan his management and use of the property with some accuracy during the time it is in his possession, as he is
no danger of being suddenly ousted by the vendors confronting him with the redemption price and demanding the
surrender of the property. And for the security thus afforded to the purchaser in the enjoyment of the property he
will be more inclined to pay a greater sum for it than he would in the absence of such a provision, thereby benefiting
the vendor.

In the present case, the only stipulation of the parties with reference to the right to repurchase was that it
could not be exercised within three years from the date of the sale. Had it not been for this condition, it is evident
that the right would have expired four years from the date of the sale.

It would seem that the vendor should be allowed four years from the expiration of the time within
which the right to redeem could not be exercised, or in the event that four years would extent the life of the
contract beyond ten years, the balance of the ten year period, on the ground that the vendors, where the right
to redeem is not thus suspended and no express agreement as to length of time during which it may be

2B 16-17 SALES AND LEASE Page 382


exercised is made, are also allowed four years. This construction, it must be conceded, is the most logical and
just.

The right to repurchase is to be calculated from the day upon which that right may be freely exercised by
the vendor, subject, of course, to the ten-year limitation of the law. The starting point for calculating it (the
redemption period) we understand is always the date of the contract, since, although the Code only so states in the
first of the two said cases, in the second it is expressly prohibited that the period shall exist more than ten years, and
it is clear that it would last longer if it were agreed, for example, that it would not begin to run until a certain time
had elapsed after the date of the contract. This agreement, in so far as it might imply an extension of ten years, we
believe would be null as being contrary to the manifest spirit of the law. We are of the opinion that the effect of
the express stipulation or agreement in the contract which we have been discussing was to extend the life of
the contract to seven years from the date of its execution.

The provisions of section 465 of Act No. 190 (The judgment debtor, or redemptioner, may redeem the
property from the purchaser, at any time within twelve months after the sale, on paying to the purchaser the amount
of his purchase..) and Article 1518 of the Civil Code (vendor cannot exercise the right of redemption without
returning to the vendee the price of the sale) do not make any provision for cases where the possessor of the
property refuses to accept the redemption price and surrender the property. This court, in Brusas v. Infante, where a
judgment debtor sought to redeem property sold under execution and the purchaser refused to accept the price paid
for it and surrender the property, held that the offer having been proven, it was not necessary for the judgment
debtor to deposit the redemption price.

Again, in the Chattel Mortgage Law, Act No. 1508, a chattel mortgage is defined as a conditional sale, "the
condition being that the sale shall be void upon the seller paying to the purcha se a sum of money or doing some
other act named. If the condition is performed according to its terms, the mortgage and sale immediately become
void and the mortgagee is thereby divested of his title." A chattel mortgage is in many respects similar to a sa le
under pacto de retro, and a tender of performance is sufficient. But the settled rule is that an offer of the money,
where the sum required is fixed and certain, is sufficient, and that it is unnecessary to deposit it.

The settled rule is that a bona fide offer of the of the redemption price, where that is certain and fixed, is
sufficient to preserve the vendors right of action in cases where the offer is refused.

Note (cited cases) : In Fructo v. Fuentes: Without intending to hold that the vendor of land under a pacto de retro
does not lose his right to repurchase the same on the day of the maturity of the contract, yet where, as in the present
case, at the time of the maturity of the contract, he makes a diligent effort to repurchase, as was done in the present
case, and fails by reason of circumstances over which he has no control, we are of the opinion and so hold that he
does not lose his right to repurchase his land, by reason of his failure to repurchase on the day of maturity.

In Retes v. Suelto: When a person having the right under a contract of pacto de retro makes a bona fide
offer to repurchase, in accordance with the agreement and tenders the necessary amount of money, he has done all
the law requires of him to preserve his right and to entitle him to the possession of the property.

2B 16-17 SALES AND LEASE Page 383


3. DOROTEO DE LA CRUZ VS RAFAEL RESURRECION

G.R. No. L-9304, April 28, 1956

Bautista Angelo, J.

FACTS:

On January 28, 1949 Doroteo de la Cruz and his wife Julia Robles sold to defendants under pacto de retro a parcel of
coconut land. The document was registered in the Office of the Register of Deeds. It was expressly agreed upon that
the land may be redeemed within a period of three years from the date of the sale.

On September 2, 1951, Doroteo de la Cruz went to the house of defendant to tell him that he wanted to redeem the
land and, upon being asked if he had the money with him, De la Cruz answered in the negative. Thereupon,
Resurreccion tried to dissuade him to redeem the land offering him a premium of P400, to which De la Cruz replied
that he would consult his wife. Thereafter, De la Cruz left promising to return later with his wife, but he never
returned, and on October 17, 1951 he deposited with the clerk of court the sum of P1,350 with the request th at he
notify defendants of the deposit.

On November 5, 1951, Plaintiffs instituted this action to recover the parcel of land sold under pacto de retro to
defendants and to have the latter execute the necessary deed after receiving from the clerk of court the money
consigned by plaintiffs to cover the payment of redemption.

Defendants aver that plaintiffs failed to comply with the requirements of the law to effect redemption within the
requirements of the law to effect redemption within the period stipulated thus they have irrevocably acquired the
ownership of the land.

Defendants contend that plaintiffs have forfeited their right to redeem because, while they made a deposit of the
redemption money with the clerk of court, they however failed to give not ice of their intention which the laws
requires. Plaintiffs on the other hand, contend that they had substantially complied with the requirements of the law
regarding consignation, and, even if they, failed the tender of payment they made had the effect of preserving their
right to the redemption

ISSUE:

Whether plaintiffs can still redeem the land even if they made the offer to redeem within the period stipulated,
considering that they had made the consignation not strictly in accordance with the requiremen ts of the law.

2B 16-17 SALES AND LEASE Page 384


HELD:
Yes. In the case of Rosales vs. Reyes and Ordaveza, the court ruled that a bona fide offer of the redemption price,
where that is certain and fixed, is sufficient to preserve the vendor's right of action in case where the offer is
refused. In this case, Doroteo de la Cruz offered to redeem the land from defendants on September 2, 1951, much
prior to the expiration of the period of redemption , and when defendants tried to dissuade him to redeem the land
upon payment of a premium of P400, which De la Cruz interpreted as a refusal to allow the redemption, plaintiffs
lost no time in depositing the repurchase money and instituted the present action to enforce their right. These steps
are more than necessary in contemplation of law to preserve their right of redemption. Whether therefore all the
requirements of the law to make a valid consignation were complied with or not, it would appear to be of no
consequence, the important thing being that the tender of payment be made within the period stipulated.

2B 16-17 SALES AND LEASE Page 385


4. BIENVENIDA MACHOCA ARCADIO VDA. DE CRUZO, ELENA MACHOCA ARCADIO VDA.
DE PINTON, INOCENTA MACHOCA ARCADIO VDA. DE PLIEGO, ISIDORA MACHOCA ARCADIO
DE PLIEGO (Deceased) represented by daughter Natividad Pliego de Ceballos and ARISTON "RICARDO"
MACHOCA ARCADIO (Deceased) represented by daughter Virginia Arcadio de Evangelista: Represented
by INOCENTA MACHOCA ARCADIO VDA. DE PLIEGO VS HON. GLICERIO V. CARRIAGA, JR.,
FRANKLIN ANG and MELECIO SUAREZ (Deceased) represented by the surviving spouse, Pilar de los
Reyes

G.R. Nos. 75109-10; June 28, 1989

Regalado, J

FACTS

Lot No. 1131 of the Misamis Cadastre, subject matter of this case, was originally registered in the name of Gabina
Machoca, as her paraphernal property. Petitioners herein are the children of the late spouses Leonardo Arcadio and
said Gabina Machoca.

On February 4, 1954, Gabina Machoca mortgaged Lot No. 1131 for P 425.00 to private respondent Franklin Ang
and delivered to him her aforesaid certificate of title in connection therewith.

On October 4, 1954, Gabina again borrowed an additional sum of P 175.00 from Ang as a result of which her total
obligation to the latter was in the sum of P 600.00. Petitioners claim that on the same date, Ang caused the
preparation of a deed of sale over the subject lot to which document Gabina Machoca, being illiterate, affixed her
thumb-mark in the belief that this second instrument was similar to the deed of mortgage executed by her on
February 4, 1954. later on, Gabina was informed by her children, herein petitioners, that the second document was
not a deed of mortgage but a contract of sale.

On the following day, Gabina went back to Ang and demanded the reformation of the aforesaid instrument. Ang,
instead of reforming the instrument, prepared a deed of agreements. Pursuant to the provisions of said deed of
agreement, Gabina's right to repurchase the property was to expire on October 4, 1957, that is, three years from
October 4, 1954 when the deed of sale was executed.

In 1955, However, Ang caused the registration of the deed of sale resulting in the issuance of Transfer Certificate of
Title No. T-161 for the same property in the name of Franklin Ang. On June 24, 1963, no redemption having been
made, Ang sold said Lot No. 1131 to herein private respondent Melecio Suarez who then obtained Transfer
Certificate. Gabina Machoca died on April 21, 1966 leaving herein petitioners as her only heirs.

Later on, a case for unlawful detainer with damages was filed against the petitioners. The t rial court rendered
judgement in favor of the herein respondent. The judgement became final and executory with a writ of execution
and an order of demolition issued by the city court.

Furthermore, during the pendency of unlawful detainer case, petitioners filed a complaint, with the same court
involving Lot No. 1131, against Franklin Ang, Bonifacio Longayan, Melecio Suarez and Pilar de los Reyes, for
"removal of clouds of title and declaring title of defendants as null and void or cancelled, or reconveyanc e and
damages." The complaint was dismissed for failure to prosecute. Petitioners twice moved for the reconsideration of
the order but such was denied. No appeal having been made, the order of dismissal became final.

2B 16-17 SALES AND LEASE Page 386


Finally, the same petitioners filed a case with the Regional Trial Court Ozamiz City, for conventional redemption
and damages against herein private respondents over the same subject lot. Upon motion of the defendants therein,
the complaint was dismissed by the court on February 26, 1986 on th e ground of res judicata.

Hence, this petition assailing said dismissal order.

ISSUE

Whether or not the petitioners can still exercise the right to redeem Lot No. 1131.

HELD

No.

There is no question that the order of dismissal rendered in the prior act ion for removal of the clouds to the title had
become final for failure of herein petitioners to appeal the same after their motions for reconsideration were denied.
Furthermore, while the dismissal was for failure to prosecute, it had the effect of an adjudication on the merits, and
operates as res judicata, since the court did not direct that the dismissal was without prejudice.

It must be remembered that after the execution of the deed of sale on October 4, 1954, a second document was made
wherein Franklin Ang undertook to resell the property, if Gabina Machoca elects to redeem the same, within three
years from the date of the deed of sale. With respect, therefore, to the last transaction entered into by the parties,
there were two documents involved, one of which is the deed of sale and the other, the right to repurchase. However,
We find and so hold that there is no pacto de retro sale in this case, within the contemplation of Art. 1601 of the
Civil Code.

We have similarly held in a prior case that an agreement to repurchase becomes a promise to sell when made after
an absolute sale because where the sale is made without such an agreement, the purchaser acquires the thing sold
absolutely. Hence, there having been an absolute sale of the land, respondent Ang was acting well within the ambit
of his now inviolable right to register the land in his own name, notwithstanding the unexpired stipulated period of
redemption in the deed of agreement.

Granting, for the sake of argument, that the transaction actually involves a pacto de retro sale. petitioners failure to
exercise their right of redemption within the stipulated period dictates that the instant petition must necessarily fail.
As it is, their silent acquiescence for an inexplicable length of time worked greatly to their disadvantage. Not only
did petitioners fail to repurchase the property within the stipulated period but they continued to sleep on their rights
even beyond the allowable statutory period for the enforcement of such right of redemption. The y are now barred by
laches.

The contract, not being contrary to law, morals and public policy, is binding and enforceable against Gabina
Machoca and her successors in interest. Petitioners cannot now be heard to claim otherwise after having been remiss
in their obligations. They are further estopped from asserting that the parties intended differently, contrary to what
the written contracts provide, in violation of the parol evidence rule.

2B 16-17 SALES AND LEASE Page 387


5. VDA DE ZULUETA VS OCTAVIANO

G.R. Nos. 75109-10 June 28, 1989

Regalado, J

FACTS

Lot No. 1131 was originally registered in the name of Gabina Machoca, as her paraphernal property, under Original
Certificate of Title No. 682. Petitioners herein are the children of the late spouses Leonardo Arcadio and said Gabina
Machoca. On February 4, 1954, Gabina Machoca mortgaged Lot No. 1131 for P 425.00 to private respondent
Franklin Ang and delivered to him her aforesaid certificate of title in connection therewith. On October 4, 1954,
Gabina again borrowed an additional sum of P 175.00 from Ang as a result of which her total obligation to the latter
was in the sum of P 600.00. Petitioners claim that on the same date, Ang caused the preparation of a deed of sale
over the subject lot to which document Gabina Machoca, being illiterate , affixed her thumb-mark in the belief that
this second instrument was similar to the deed of mortgage executed by her on February 4, 1954. When Gabina went
home, her children, herein petitioners, informed her that the second document was not a deed of mortgage but a
contract of sale. On the following day, October 5, 1954, Gabina went back to Ang and demanded the reformation of
the aforesaid instrument. Franklin Ang, instead of reforming the instrument, prepared a deed of agreements which,
by reason of its importance, is herein quoted in full:

City of Ozamiz October 5, 1954

DEED OF AGREEM ENT

KNOW ALL MEN BY THESE PRESENTS:

That I, FRANKLIN ANG, of legal age, married and with residence and postal address at Gango, City of Ozamiz,
Philippines, VENDEE in the Deed of Sale executed by the Vendor, GAVINA MACHOCA, as recorded by Notary
Public Manuel C. Manago in Doc. No. 284, Page No. 58, Book No. 1, Series of 1954, hereby grants and obligates
himself (sic) to resell the property therein sold within a period of three (3) years from and after the date of the said
instrument, for the same price of SIX HUNDRED PESOS ( P 600.00 ), Philippine Currency, to the said VENDOR:
PROVIDED, however, That if the Vendor shall fail to exercise her right to redeem as herein granted within the
stipulated period, then this conveyance shall be deemed to be absolute and irrevocable.

IN WITNESS WHEREOF, the party herein hereto have (sic) set his hands (sic) at Ozamiz City, Philippines, on this
5th day of October, 1954.

(SGD) FRANKLIN ANG

As early as June 10, 1955, Ang caused the registration of the deed of sale, resulting in the subsequent cancellation of
Original Certificate of Title No. 682 and the consequent issuance of Transfer Certificate of Title No. T -161 for the
same property in the name of Franklin Ang. On June 24, 1963, no redemption having been made, Ang sold said Lot
No. 1131 Melecio Suarez who then obtained Transfer Certificate of Title No. T-945 therefore in his name. Gabina
Machoca died on April 21, 1966 leaving herein petitioners as her only heirs.

ISSUE

2B 16-17 SALES AND LEASE Page 388


Whether or not the heirs of Machoca can still exercise the right to redeem Lot No. 1131. A corollary issue is whether
or not the private deed of agreement has converted the deed of sale into an equitable mortgage.

HELD

It must be remembered that after the execution of the deed of sale on October 4, 1954, a second document was made
wherein Franklin Ang undertook to resell the property, if Gabina Machoca elects to redeem the same, within three
years from the date of the deed of sale. With respect, therefore, to the last transaction entered into by the parties,
there were two documents involved, one of which is the deed of sale and the other, the right to repurchase. However,
We find and so hold that there is no pacto de retro sale in this case, within the contemplation of the Civil Code
which provides:

Art. 1601. Conventional redemption shall take place when the vendor reserves the right to repurchase the thing sold,
with the obligation to comply with the provisions of Article 1616 and other stipulations which may been agreed
upon.

In Villarica, et al. vs. The Court of appeals, et al., We had the occasion to interpret this provision of law, to wit:

The right of repurchase is not a right granted the vendor by the vendee in a subsequ ent instrument, but is a right
reserved by the vendor in the same instrument of sale as one of the stipulations of the contract. Once the instrument
of absolute sale is executed, the vendor can no longer reserve the right to repurchase, and any right there after
granted the vendor by the vendee in a separate instrument cannot be a right of repurchase but some other right like
the option to buy in the instant case.

We have similarly held in a prior case that an agreement to repurchase becomes a promise to sell when made after
an absolute sale because where the sale is made without such an agreement, the purchaser acquires the thing sold
absolutely.

Clearly, therefore, an option to buy or a promise to sell is different and distinct from the right of repurchase which
must be reserved by the vendor, by stipulation to that effect, in the contract of sale.

Hence, there having been an absolute sale of the land, respondent Ang was acting well within the ambit of his now
inviolable right to register the land in his own name, notwithstanding the unexpired stipulated period of redemption
in the deed of agreement.

Granting, for the sake of argument, that the transaction actually involves a pacto de retro sale. petitioners failure to
exercise their right of redemption within the stipulated period dictates that the instant petition must necessarily fail.

It will readily be noted that the deed of agreement specifically provided: "That if the Vendor shall fail to exercise her
right to redeem as herein granted within the stipulated period, then this conveyance shall be deemed to be absolute
and irrevocable."

Furthermore, the inadequacy of the price does not on that account alone support the conclusion that the land was not
sold to private respondent Ang, since the parties entered into a conventional, and not a forced, sale of the property
and both parties were in a position to form an independent judgment of the transaction. From the legal viewpoint,
even if the property was sold for a comparatively low price, but the seller did nothing about it for a number of years,
the contract of sale is not invalid. Besides, in a contract of sale with right of repurchase, the price is usually less than
in absolute sales since in the former the vendor expects to reacquire or redeem the propert y sold, hence the
inadequacy of the price is not an overriding determinant to set aside the sale. The same rationale obtains where, as in
this case, there was a separate agreement to resell the property to the original vendor.

2B 16-17 SALES AND LEASE Page 389


Finally, it is of essence of a contract of sale with pacto de retro that the vendee shall immediately acquire title to and
possession of the land sold, subject only to the vendor's right of redemption. With much more reason does this hold
true where a deed of absolute sale was merely complemented by a subsequently executed and separate agreement of
resale.

2B 16-17 SALES AND LEASE Page 390


6. SPS. TITO ALVARO and MARIA VALELO VS SPS. OSMUNDO TERNIDA and

JULITA RETURBAN, COURT OF APPEALS

G.R. No. 166183 January 20, 2006

FACTS

Respondent-spouses Osmundo Ternida and Julita Returban are the owners of the contested property, an 8,450 sq. m.
parcel of non-irrigated riceland situated at Barangay Labney, San Jacinto, Pangasinan. On May 26, 1986, Julita
mortgaged the land to the spouses Salvador de Vera and Juanita Orinion. According to Julita, she was made to sign a
Deed of Pacto de Retro Sale with Salvador. As worded, the document provided that Julita has three years from the
date of the execution of the document to repurchase the land. After a year, Salvador executed a Deed of Transfer of
Mortgage in favor of the spouses Jose Calpito and Zoraida Valelo. Julita was also asked to sign a Deed of Sale with
Right to Repurchase. Later on, she was informed by Jose Calpito that they have transferred the mortgage to the
spouses Tito Alvaro and Maria Valelo, herein petitioners. Julita claimed that petitioners asked her to sign a
document that she believed was a mortgage document but later on turned out to be a Deed of Absolute Sale over the
contested property. When Julita tried to redeem the property from the petitioners, the latter refused and claimed that
they had purchased the property and were in fact issued Tax Declaration No. 2747.Consequently, respondents filed a
complaint for Annulment of Deed of Sale Documents and Tax Declaration No. 2747 with the Regional Trial Court
of Dagupan City. After trial on the merits, the trial court dismissed the complaint for lack of cause of action.
Respondents filed a motion for reconsideration which was however denied. On appeal, the Court of Appeals
reversed the decision of the trial court.

ISSUE

Whether or not there was an absolute sale over the riceland

HELD

NO, The Petition lacks merit. An equitable mortgage is defined as one which although lacking in some formality ,
or form or words, or other requisites demanded by a statute, nevertheless reveals the intention of the parties to
charge real property as security for a debt, and contains nothing impossible or contrary to law. For the presumption
of an equitable mortgage to arise, two requisites must concur: (1) that the parties entered into a contract denominated
as a sale; and (2) that their intention was to secure an existing debt by way of a mortgage. Consequently, the
nonpayment of the debt when due gives the mortgagee the right to foreclose the mortgage, sell the property and
apply the proceeds of the sale to the satisfaction of the loan obligation.

We find no merit in petitioners contention that in the Deed of Absolute Sale executed between them and
Julita, the latter totally conveyed her ownership over the disputed property. We have consistently decreed that the
nomenclature used by the contracting parties to describe a contract does not determine its nature. The decisive factor
is the intention of the parties to the contract as shown by their conduct, words, actions and deeds prior to, during and
after executing the agreement.

2B 16-17 SALES AND LEASE Page 391


While there is no single conclusive test to determine whether a deed absolute on its face is really a simple
loan accommodation secured by a mortgage,[19] however, the Civil Code enumerates several instances when a
contract is clothed with the presumption that it is an equitable mortgage, to wit:

Article 1602. The contract shall be presumed to be an equitable mortgage, in any of the following cases:

XXX

(6) In any other case where it may be fairly inferred that the real intention of the parties is that the
transaction shall secure the payment of a debt or the performance of any other obligation.

It is an established rule that the presence of even one of the circumstances set forth in Article 1602 is
sufficient to declare a contract of sale with right to repurchase an equitable mortgage. Thus, under the wise, just and
equitable presumption in Article 1602, a document which appears on its face to be a sale absolute or with pacto de
retro may be proven by the vendor or vendor-a-retro to be one of a loan with mortgage. In such case, parol evidence
becomes competent and admissible to prove that the instrument was in truth and in fact given merely as a security
for the payment of a loan. And upon proof of the truth of such allegations, the court will enforce the agreement or
understanding in consonance with the true intent of the parties at the time of the execution of the contract.

Applying the foregoing considerations to the instant case, we find that the true intention of the parties in the
execution of the Deed of Absolute Sale was never to convey the o wnership of the disputed property but merely to
secure the loan obtained by Julita.

2B 16-17 SALES AND LEASE Page 392


7. SPOUSES CESAR R. ROMULO and NENITA S. ROMULO vs. SPOUSES MOISES P. LAYUG, JR., and
FELISARIN LAYUG

G.R. No. 151217 September 8, 2006

Tinga, J

FACTS

On April 11, 1996, petitioners Spouses Cesar and Nenita Romulo filed a verified Complaint for Cancellation of
Title, Annulment of Deed of Absolute Sale and Contract of Lease with Damages against respondents Spouses
Moises and Felisarin Layug.

Petitioners averred in their complaint that sometime in 1986, they obtained from respondents a loan in the
amount of P50,000.00 with a monthly interest of 10%, which subsequently ballooned to P580,292.00. To secure the
payment of the loan, respondents allegedly duped petitioners into signing a Contract of Lease and a Deed of
Absolute Sale covering petitioners house and lot located at Phase II, BF Homes, Sucat, Paraaque and covered by
Transfer Certificate of Title (TCT) No. S-71528. The Deed of Absolute Sale purportedly facilitated the cancellation
of petitioners title on the house and lot and the issuance of TCT No. 20489 in the name of respondents. Thus,
petitioners prayed for the nullification of the Deed of Absolute Sale, the contract of lease and TCT No. 20489, and
the award of moral and exemplary damages.

Respondents denied petitioners allegations. In their Answer, they vouched for the validity of the Deed of
Absolute Sale, particularly as having been voluntarily executed by the parties for the purpose of extinguish ing
petitioners indebtedness to respondents. As consideration of the sale, respondents allegedly paid the amount of
P200,000.00 in addition to the writing off of petitioners obligation to them. That they allowed petitioners to occupy
the house and lot as lessees thereof was founded on the trust they reposed on petitioners, claimed respondents.

Prior to the filing of Civil Case No. 96-0172, respondent Moises Layug, Jr. ("Moises") filed Civil Case No. 9422, an
action for ejectment, against petitioners to compel the latter to vacate the house and lot allegedly sold by petitioners
to Moises and subsequently rented out by him to petitioners. Moises alleged that petitioners violated the terms of the
Contract of Lease when the latter failed to pay any rental or exercise their option to repurchase the house and lot and
refused to vacate the property despite demand.

The Metropolitan Trial Court (MeTC) of Paraaque dismissed the complaint for lack of cause of action.
The RTC of Paraaque, likewise dismissed Moises appeal based on its finding that the parties did not intend to
enter into a lease agreement.The Court of Appeals denied Moises petition for review on the ground of late filing.
Upon elevation to this Court, Moises petition for review on certiorari was denied with finality by this Court.

On June 21, 1999, the trial court rendered judgment in favor of petitioners. Respondents elevated the matter
to the Court of Appeals, questioning, among others, the trial courts finding that the contract between petition ers and
respondents was an equitable mortgage.The Court of Appeals reversed and set aside the RTC Decision, mainly on

2B 16-17 SALES AND LEASE Page 393


the ground that petitioners failed to present sufficient evidence to prove their allegation that their signatures to the
Deed of Absolute Sale were obtained fraudulently. Their motion for reconsideration rebuffed, hence this appeal by
certiorari under Rule 45 of the 1997 Rules of Civil Procedure.

ISSUE

Whether or not the transaction between the parties constitutes an equitable mortgage.

HELD

YES, Whether or not the parties intended an equitable mortgage is a factual issue. As a general rule, factual review
is beyond the province of this Court. One of the exceptions to the rule is exemplified by the instant case where the
factual findings of the RTC and Court of Appeals are contradictory.

The trial court based its declaration that an equitable mortgage was intended by the parties on the finding
that petitioners remained in possession of the house and lot even after the property was supposedly sold to
respondents. The trial court also gave evidentiary weight to the decisions of the MeTC and RTC dismissing the
action for ejectment in Civil Case No. 9422, where both courts found that petitioners neither vacated the property
nor paid any rental even after the execution of the Deed of Absolute Sale. The Court of Appeals disagreed and
declared that an absolute sale was contemplated by the parties based on the express stipulations in the Deed of
Absolute Sale and on the acts of ownership by responden ts subsequent to its execution.

In this case, Articles 1604 and 1602 of the Civil Code come into play. The articles provide that when the
parties to a contract of sale actually intended such contract to secure the payment of an obligation, it shall be
presumed to be an equitable mortgage. For the presumption of equitable mortgage to arise, two requisites must be
satisfied, namely: that the parties entered into a contract denominated as a contract of sale and that their
intention was to secure an existing debt by way of mortgage.

Under Article 1604 of the Civil Code, a contract purporting to be an absolute sale shall be presumed to be
an equitable mortgage should any of the conditions in Article 1602 be present. To stress, the existence of any one
of the conditions under Article 1602, not a concurrence, or an overwhelming number of such circumstances,
suffices to give rise to the presumption that the contract is an equitable mortgage. It must be emphasized too,
however, that there is no conclusive test to determine whether a deed absolute on its face is really a simple loan
accommodation secured by a mortgage. In fact, it is often a question difficult to resolve and is frequently made to
depend on the surrounding circumstances of each case. When in doubt, co urts are generally inclined to construe a
transaction purporting to be a sale as an equitable mortgage, which involves a lesser transmission of rights and
interests over the property in controversy.

The Court has not hesitated to declare a purported contract of sale as an equitable mortgage even when only
one of the enumerated circumstances under Article 1602 is proved. In the case at bar, petitioners remained in

2B 16-17 SALES AND LEASE Page 394


possession of the house and lot even after the execution of the Deed of Absolute Sale. Moreover, they remained
in possession of the property for more than the reasonable time that would suggest that petitioners were mere lessees
thereof. For one, it took respondents more than five years from the time of the execution of the Deed of Absolute
Sale and the Contract of Lease to file the action for ejectment. Within this period, petitioners neither paid any rental
nor exercised the option to buy purportedly the leased property from respondents. Incidentally, in the decisions of
the MeTC and the RTC in the separate action for ejectment, both lower courts observed that when petitioners were
made to sign a blank document, which turned out to be a Contract of Lease of their house and lot, they were of the
belief that the blank document would serve only as guaranty for the payment of their obligation to respondents.

The claim that petitioners possession of the house and lot was by sheer tolerance of respondents is
specious. Respondents could not explain why they allowed petitioners more than five years to loo k for another place
to transfer. These circumstances only support the conclusion that the parties never really intended to transfer title to
the property. Under paragraph 2 of Article 1602, where the purported vendor remains in possession of the property
subject of the sale and it can be inferred that the true intention of the parties was to secure an existing debt, the
transaction shall be deemed an equitable mortgage.

Under paragraph 1 of Article 1602, where the purchase price is inadequate, a contract o f sale is also
presumed to be an equitable mortgage. Based on respondents evidence, petitioners property was valued at
P700,000.00 but the assailed Deed of Absolute Sale stated a consideration of only P200,000.00. Contrary to the
appellate courts declaration that the inadequacy of the purchase price is not sufficient to set aside the sale, the Court
finds the same as clearly indicative of the parties intention to make the property only a collateral security of
petitioners debt. The Court is not convinced that petitioners would allow the sale of their residential property for
even less than half of its market value.

The appellate court ruled that petitioners failed to rebut the presumption of the genuineness and due
execution of the questioned Deed of Absolute Sale. Based on the examination of the assailed instrument and the
Contract of Lease and the testimonies of the parties, the Court cannot sustain respondents claim that petitioners
offered to sell their house and lot in satisfaction of their indebtedness. As observed by the trial court, the Contract of
Lease appears to have been signed sometime in November 1988 or before the execution of the Deed of Sale.
Respondents were unable to explain why they had leased the property to petitioners before its s upposed purchase by
respondents. Furthermore, the records disclose that it was only after the institution of the ejectment case did
petitioners learn about the cancellation of their title to the property although under the assailed Deed of Absolute
Sale, petitioners were obliged to bear the expenses of its execution and registration. These circumstances lend
credence to petitioners claim of the surreptitious manner by which respondents made them sign certain documents
without completely disclosing the real import thereof.

2B 16-17 SALES AND LEASE Page 395


8.DINO VS JARDINES

G.R. No. 145871 January 31, 2006

FACTS:

Petitioner Leonides C. Dio filed a petition for Consolidation of Ownership with the RTC of Baguio City alleging
that on January 31, 1987, respondent Jardines executed in her favor a Deed of Sale with Pacto de retro over a parcel
of land with improvements which amounted to P165,000.00. It was stipulated that the period for redemption would
expire in six months or on July 29 1987 such period expired but neither respondent nor a ny of her legal
representatives were able to redeem or repurchase the subject property; as a consequence, absolute ownership over
the property has been consolidated in favor of petitioner. Jardines countered that the true contract of the parties was
that of a loan and the deed with pacto de retro sale was a mere security to such loan. The amount of the property was
around half a million and respondent averred that it was unthinkable for her to sell the property for only P165,000.00
In fact, the loan was even covered by interest at the rate of 9% to be paid monthly. The court rendered its decision
declaring the contract as one of deed of sale with right to repurchase or pacto de retro and that petitioner acquired
whatever rights Jardines had over the parcel of land, and she now became owner of the same. However, upon appeal
to the Court of Appeals, the judgment was reversed with the finding that the contract was one of Equitable Mortgage
and not one of Pacto de Retro.

ISSUE

Whether or not the contract was one of Pacto de Retro or an Equitable Mortgage.

HELD

Held for Jardines. The transaction was an equitable mortgage. Article 1602 of the Civil Code enumerates the
instances when a purported pacto de retro sale may be considered an equitable mortgage, to wit: A rt. 1602. The
contract shall be presumed to be an equitable mortgage, in any of the following cases:

(1) When the price of a sale with right to repurchase is unusually inadequate;

(2) When the vendor remains in possession as lessee or otherwise;

(3) When upon or after the expiration of the right to repurchase another instrument extending the period of
redemption or granting a new period is executed;

(4) When the purchaser retains for himself a part of the purchase price;

(5) When the vendor binds himself to pay the taxes on the thing sold;

(6) In any other case where it may be fairly inferred that the real intention of the parties is that the transaction shall
secure the payment of a debt or the performance of any other obligation.

In any of the foregoing cases, any money, fruits, or other benefit to be received by the vendee as rent or otherwise
shall be considered as interest which shall be subject to the usury laws. (Emphasis sup plied) In Legaspi vs. Ong, the
Court further explained that: The presence of even one of the abovementioned circumstances as enumerated in

2B 16-17 SALES AND LEASE Page 396


Article 1602 is sufficient basis to declare a contract of sale with right to repurchase as one of equitable mortgage. As
stated by the Code Commission which drafted the new Civil Code, in practically all of the socalled contracts of sale
with right of repurchase, the real intention of the parties is that the pretended purchase price is money loaned and in
order to secure the payment of the loan, a contract purporting to be a sale with pacto de re tro is drawn up.

2B 16-17 SALES AND LEASE Page 397


9. CIRELOS VS HERNANDEZ

G.R. No. 146523 June 15, 2006

FACTS:

On March 10, 1993, spouses Aniceto and Thelma Cirelos (petitioners) filed a complaint for Breach of
Contract, Annulment of Sale and Damages before the RTC, against spouses William G. Hernandez and Rosemarie
Zafe (respondents). Petitioners alleged that they are the registered owners of a house and lot located at No. 10
Kennedy Drive, Tandang Sora, Quezon City.

On March 5, 1991, Thelma Cirelos (Cirelos ) obtained a loan in the amount of P450,000.00 from respondent
William Hernandez (Hernandez), a money lender, and as security she executed a Deed of Real Estate Mortgage in
favor of Hernandez covering the said property. Hernandez asked Cirelos to sign a blank bond paper containing
nothing but her name which Hernandez said will be converted into promissory note.

On February 15, 1993, petitioners received a letter from respondents asking them to vacate the property
because respondents already own the same. Cirelos learned that there was already a Deed of Sale in favor of
respondents annotated at the back of the title of their property and a Release of Real Estate Mortgage. Cirelos also
discovered that the blank paper she signed, which Hernandez said will only be used as promissory note, was
converted into a Deed of Absolute Sale. Moreover, the Deed did not have the consent of Aniceto, husband of
Cirelos, and the Release of Real Estate Mortgage is fictitious as petitioners have not yet paid their loan.

In their answer, the respondents countered that Cirelos did not sign any blank paper neither did they require
her to do so and the execution of the Release of Real Estate Mortgage and Deed of Absolute Sale was out of the free
will and volition of petitioners who could no longer pay the loan plus interest.

The trial court dismissed the petition and ruled in favor of the respondents. The Court of Appeals affirmed
the decision of the trial court.

ISSUE

Whether or not the contract is one of equitable mortgage

HELD

NO

In the case at bar, petitioners claim that the agreement between them is one of equitable mortgage for three
reasons, that there was inadequacy of the price, they remained in possession of the property, and their intention was
only that of mortgage and not sale.

Petitioners claim that there was inadequacy of the price is not supported by the evidence on record. They
did not present any proof that the fair market value of the real property in the area at the time of the transaction were
much higher than the selling price of the lot in question. Mere allegation that the price paid is inadequate, without
more evidence, does not make a case favorable to petitioners.

2B 16-17 SALES AND LEASE Page 398


Petitioners also claim that since they remained in possession of the property, the presump tion of equitable
mortgage should govern. However, the respondents presented proof that as early as September 5, 1991, or more than
a month from July 30, 1991, the date of the deed of sale, they already sent a letter to petitioners asking them to
vacate the premises. Thus, the respondents had shown by preponderance of evidence that the possession of
petitioners of the subject property after the date of the absolute sale is without the consent of respondents.

While there is a presumption in favor of equitable mortgage, such presumption, however, is not conclusive
and may be rebutted by competent and satisfactory proof to the contrary.

2B 16-17 SALES AND LEASE Page 399


10. RAMOS VS DIZON

498 SCRA 17

FACTS

Respondents are the owners of an undivided one-half portion of a parcel of land with an area of about 89.35
square meters located in Limay Street, Manuguit Subdivision, Tondo, Manila. on 1 February 1988, respondent
Domingo executed a Special Power of Attorney (SPA) authorizing Elpidio Domingo to sell one -half portion of said
parcel of land; that Elpidio, acting pursuant to the provisions of the SPA sold, with a right to repurchase within five
months, one-half of the land to petitioner; and that respondent Domingo failed to redeem or repurchase the disputed
land within the five-month period provided for under the Deed of Sale Under Pacto de Retro, thus, ownership over
the subject land was consolidated in petitioner.

Respondent Domingo filed an Answer/Opposition [4] to the Petition alleging that the SPA was executed for
the purpose of enabling Elpidio to secure a loan of P150,000.00 by using Domingos share in the land as security.
The proceeds of the loan was supposed to be used for the construction of a duplex residential house to be supervised
by Elpidio. However, Elpidio obtained a loan of P350,000.00 and used a substantial portion thereof for his personal
advantage and benefit. As Elpidio had exceeded his authorit y, Domingo claimed that he revoked the SPA through
several letters and by a formal notice of revocation sent by his counsel. As for the pacto de retro sale, Domingo
maintains that the same was simulated as Elpidio had already obtained a loan totaling P350,000.00 from petitioner
as evidenced by a Real Estate Mortgage executed by the two of them. In any case, he claims that the pacto de retro
sale should be treated as an equitable mortgage which cannot be enforced through a petition for consolidation of
ownership.

ISSUE:

Whether the contract between petitioner and Elpidio was actually one of equitable mortgage and not a
pacto de retro sale.

HELD:

Yes, the contract is one of equitable mortgage. Under Article 1602 of the Civil Code, the contrac t of sale
will be presumed to be an equitable mortgage in any of the following cases:

(1) When the price of a sale with right to repurchase is unusually inadequate;

(2) When the vendor remains in possession as lessee or otherwise;

2B 16-17 SALES AND LEASE Page 400


(3) When upon or after the expiration of the right to repurchase another instrument extending the period of
redemption or granting a new period is executed;

(4) When the purchaser retains for himself a part of the purchase price;

(5) When the vendor binds himself to pay the taxes on the things sold;

(6) In any other case where it may be fairly inferred that the real intention of the parties is that the
transaction shall secure the payment of a debt or the performance of any other obligation.

In the case at bar, it was disclosed by Elpidio that up to the time when he took the witness stand on 20
September 1994, he still maintained possession of the two-door apartment and that he was still collecting rent from
the tenant occupying one of the units. This despite the lapse of a considerable length of time from 7 January 1989
the date when the five-month repurchase period stipulated in the pacto de retro sale was supposed to have lapsed.
Had the agreement between petitioner and Elpidio been a pacto de retro sale, we fail to see any logic in her allowing
Elpidios continued possession of the structure and collection of the rent payments therefrom over such a long period
of time. As the essence of a pacto de retro sale is that title and ownership of the property sold are immediately
bestowed upon the vendee a retro, subject to the resolutory condition of repurchase by the vendor a retro within the
agreed period, petitioner should have immediately enforced her right to the rental payments. Failure on her part to
do so casts doubt as to the true nature of the transaction she entered into with Elpidio.

Moreover, it does not escape our attention that according to Elpidio, the amount of P350,000.00 st ated in
the Deed of Sale Under Pacto de Retro is the same amount as that covered by the Real Estate Mortgage and the two
promissory notes signed by him. There was therefore no separate consideration received by him from the execution
of the pactode retro sale apart from the proceeds of the earlier loans he obtained from petitioner. This undoubtedly
gives credence to respondents position that the pacto de retro sale was but a security for the loans extended by
petitioner.

2B 16-17 SALES AND LEASE Page 401


11. AGUILAR VS AGUILAR

G.R. No. 141613; Devember 16,2005

Sandoval-Gutierrez, J. :

FACTS

On October 28, 1993, Petitioner Senen and Respondent Virgilio Aguilar purchased a house and lot located in
Paraaque City, Metro Manila for the benefit of their father, Maximiano Aguilar. On Feb ruary 23, 1970, they
executed a written agreement stipulating that their shares in the house and lot would be equal; and that Senen would
live with their father on condition that he would pay the Social Security System (SSS) the remaining loan obligation
of the former owners.

In 1974, their father died. Virgilio then demanded that Senen vacate the house and that the property be sold, the
proceeds to be divided between them. Senen refused to comply with Virgilios demand.

On January 12, 1979, Respondent Virgilio filed a complaint with the Court of First Instance of Rizal at Pasay City
for specific performance for Petitioner Senen to sell the property. The trial court ruled in favor of Respondent
Virgilio, petitioner being declared in default for failing to appear.
The Court of Appeals reversed the judgment but the Supreme Court reinstated the CFIs decision.

On March 27, 1995, Senen filed with the Regional Trial Court, Branch 260, Paraaque City, an action for legal
redemption against Virgilio and another brother, Angel, docketed as Civil Case No. 95-039. In his complaint, Senen
alleged that while he knows that Virgilio sold his share of the property to Angel in January 1989, however, he
(Senen) was not furnished any written notice of the sale. Consequently, as a co-owner, he has the right to redeem the
property.

In an Order dated June 27, 1997, the trial court dismissed Civil Case No. 05-039 on the ground of laches, holding
that Senen incurred a delay of seven (7) years before asserting his right to redeem t he property in question. The
Court of Appeals affirmed the assailed Order of the trial court. Hence, the instant petition for review on certiorari.

Petitioner, now contends that there being no written notice to him of the sale by the vendee or vendor, th e thirty-day
redemption period as provided in Article 1623 has not prescribed.

ISSUE

Whether or not Petitioner Senens complaint for legal redempton is barred by laches.

HELD

Yes.

With respect to redemption by co-owners, in case the share of a co-owner is sold to a third person, the governing law
is Article 1620 of the Civil Code which provides:

2B 16-17 SALES AND LEASE Page 402


ART. 1620. A co-owner of a thing may exercise the right of redemption in case the shares of all the other co -owners
or of any of them are sold to a third pers on. If the price of the alienation is grossly excessive, the redemptioner shall
pay only a reasonable rate.
Should two or more co-owners desire to exercise the right of redemption, they may only do so in proportion to the
share they may respectively have in the thing owned in common.

Article 1623 of the same Code also provides:

ART. 1623. The right of legal pre-emption or redemption shall not be exercised except within thirty days from the
notice in writing by the prospective vendee, or by the vendor, as the case may be. The deed of sale shall not be
recorded in the Registry of Property, unless accompanied by an affidavit of the vendee that he has given written
notice thereof to all possible redemptioners.
The right of redemption of co-owners excludes that of adjoining owners.

From the above provisions, the following are the requisites for the exercise of legal redemption: (1) There must be a
co-ownership; (2) one of the co-owners sold his right to a stranger; (3) the sale was made before the partition of the
co-owned property; (4) the right of redemption must be exercised by one or more co -owners within a period of thirty
days to be counted from the time that he or they were notified in writing by the vendee or by the co -owner vendor;
and (5) the vendee must be reimbursed for the price of the sale.

In this case, the sale took place in January 1989. Petitioner admits that he has actual knowledge of the sale.
However, he only asserted his right to redeem the property in March 1995 by filing the instant co mplaint. Both the
trial court and the Appellate Court ruled that this was seven (7) years late.

The old rule is that a written notice of the sale by the vendor to his co -owners is indispensable for the latter to
exercise their retracto legal de comuneros. However, the Supreme Court has relaxed thr wrritten notice requirement
in Si v. Court of Appeals: that a co-owner with actual notice of the sale is not entitled to a written notice for such
would be superfluous. The law does not demand what is unnecessary .

Petitioner has actual knowledge of the sale of Virgilios share to Angel in 1989. As provided by Article 1623, he has
thirty days from such actual knowledge within which to exercise his right to redeem the property. Inexplicably,
petitioner did not take any action. He waited for seven (7) years before filing his complaint. Definitely, such an
unexplained delay is tantamount to laches.

2B 16-17 SALES AND LEASE Page 403


12. SPOUSES RAYOS VS DONATO REYES

G.R. No. 150913 February 20, 2003

Bellosillo, J

FACTS

Spouses Tazal owned 3 parcels of unregistered land in Pangasinan which were sold on 1 September 1957 to
respondents predecessor-in-interest, Reyes, with right to repurchase within two 2 years from date thereof. The
vendee paid the purchase price and all expenses incident to their reconveyance. After which, the vendee a retro took
physical possession of the properties and paid the taxes thereon. However, two of the lands sold were resold by
Tazal in favor of petitioners predecessor-in-interest Rayos without availing of the right to repurchase the properties.
The right of redemption in favor of spouses Tazal expired without the right being exercised by either the Tazal
spouses or the vendee Rayos. Tazal, in the attempt to repurchase the properties from Reyes, asserted that the dee d of
sale that they entered into, with right of repurchase, was actually an equitable mortgage and offering the amount of
P724.00 to pay for the alleged debt. Thus, consignation arises. However, Reyes refused the tender of payment and
asserted that their agreement was not an equitable mortgage. Tazal filed a complaint for the declaration of the
contract as a contract of equitable mortgage. He also prayed for an order requiring defendant Mamerto Reyes to
accept the amount of P724.00 which he had deposited with the trial court as full payment for his debt, and canceling
the supposed mortgage on the three parcels of land with the execution of the documents of reconveyance in his
favor. The trial court however denied the contentions of Tazal but it said that Tazal can still redeem the three lands
within thirty days from finality of judgment. This could be done by paying to Reyes the purchase price and all
expenses to execute the reconveyance. Reyes appealed the Decision to the CA which was also appealed in the
Supreme Court. When Mamerto Reyes died in 1986, petitioner-spouses Teofilo and Simeona Rayos wrested
physical possession of the disputed properties from Reyess heirs. The case became final and executory, and the
lands were not repurchased within the 30-day period. Reyes executed an affidavit adjudicating the ownership of the
parcels of land to themselves and declared the properties in their names for assessment and collection of real estate
taxes. The deed of sale was then registered in the Register of Deeds . Respondent filed a complaint for damages and
recovery of ownership and possession and the administration of the 3 parcels of land in dispute against herein
petitioner-spouses Rayos. Neither petitioners nor their predecessors -in-interest Tazal and Rayos repurchased the
properties before buying them or when the judgment in the Civil Case became final and executory. However, Rayos
averred that the consignation of P724.00 had the full effect of redeeming the properties from respondents and their
predecessor-in-interest, and that respondents were estopped by laches because Reyes as their predecessor-in-interest
did not oppose the sale to Rayos and to petitioner-spouses Rayos.

ISSUE

Whether or not there is a valid tender of payment by consignation, predicate d upon the argument that the sale is an
equitable mortgage and not a contract of sale.

HELD

2B 16-17 SALES AND LEASE Page 404


The Court held that there is no valid tender of payment by consignation, as it was conditional. The payment made
was to evade the consequence of the expiration of the redemption period stipulated. It was also provided by the
payor that the payment was by virtue of an equitable mortgage that they entered into, not in a contract of sale with a
right of redemption. Because of the conditional payment, Reyes has the righ t to refuse the payment, it being
conditional to the contention that the sale was actually an equitable mortgage.

2B 16-17 SALES AND LEASE Page 405


13. LEON BORLAZA VS GREGORIO RAMOS

G.R. No. L-3433, July 16, 1951

Padilla, J

FACTS

This is a petition for a writ of certiorari to review a judgment rendered by the Fifth Division of the
Court of Appeals affirming that of the trial court.The Court of Appeals found that on 26 December 1936, for and in
consideration of P200, Severino Pile, in his behalf and of his brother and sister, executed a purporting to convey two
adjoining parcels of land in Lilio, Laguna, described therein, to Isidro Borgonia and Gregorio Ramos, both taking
possession of the northern and southern parts of the said parcels of land, respectively. Sub sequently, for and in
consideration of P100 paid to Isidro Borgonia the Piles repurchased and took possession of the northern part of the
adjoining parcels of land. Sometimes later, or on 27 March 1943, the Piles sold the two parcels of land to the
spouses Leon Borlaza and Leoncia Borgonia. In May 1943, the latter tendered payment of P100 to Gregorio Ramos
to repurchase the southern part of the two adjoining parcels of land, but the latter refused giving a reason for his
refusal the expiration of the time for repurchase. In view of the refusal, the spouses Borlaza and Borgonia deposited
the sum of P100 with the clerk of the court of First instance of Laguna on May 1943. Gregorio Ramos, on the other
hand, executed an affidavit of consolidation of his title to the southern part of the two adjoining parcels of land on
21 June, the same year. An action was brought by the spouses Leon Borlaza and Leoncia Borgonia praying that the
deed of sale with the right reserved by the vendors to repurchase the two parcels of land be held to be an equitable
mortgage; and that Gregorio Ramos, the vendee, be compelled to accept the sum of P100 deposited in the clerk's
office, as the consideration for the repurchase of that part of the two adjoining parcels of land held by him, an d to
Deliver the possession thereof to the plaintiffs. After trial, the court of first instance of Laguna held that the
instrument was a sale with the right to repurchase reserved by the vendors, that there was no agreement or
stipulation as to the time for repurchase; that title to the southern part of the two adjoining parcels of land was
consolidated in Gregorio Ramos, the vendee, upon the expiration of four years from the date of the execution of the
instrument, the plaintiffs having lost the right top repurchase it. This is the judgment affirmed by the Court of
Appeals.

ISSUE

Whether or not the deed purporting to sell two parcels of land in question is in fact a sale with the right to
repurchase, as the instrument shows on its face, and not an equitable mortgage, as claimed by the petitioners.

HELD

The SC held that the deed was in fact a sale with the right to repurchase, as the instrument shows on its
face, and not an equitable mortgage. The SC upheld the finding of CA that plaintiff Leon Borlaza, ha ving acted not
only as notary public before whom the execution of the deed of sale with the right to repurchase was acknowledged
but also having drawn up the said deed, was estopped from claiming and asserting that the document was different
or distinct from what it purports to be. Furthermore, it affirmed that that there is no express agreement as to the time
within which the repurchase may and should be made. Such being the case, the time within which such repurchase
may and should be made is four years. The stipulation on this point reads, as follows:

2B 16-17 SALES AND LEASE Page 406


That from now on and until the repurchase has not as yet (been) done, the Party in the Second Party, . . .
shall be entitled to the possession, ownership and usufruct of the real properties described above a nd to all
improvements herein contained, all of which are free from encumbrances and charges whatsoeve

In the instant case the phrase "from now and until the repurchase has not as yet (been) done" is a stipulation by
which the vendors reserved for themselves the right to repurchase the two parcels of land sold and is not an express
agreement as to the time within which such repurchase may or should be made. There being no express agreement
as to such time for repurchase, the same must be four years, as pro vided for in Article 1508, paragraph 1, of the
Civil Code.

2B 16-17 SALES AND LEASE Page 407


14. SOLID HOMES VS CA

G.R. No. 117501, July 8, 1997.

Panganiban, J.

FACTS

On June 4, 1979, Solid Homes executed in favor of State Financing (Center, Inc.) a Real Estate Mortgage on its
properties, which included the VV Soliven Towers, in order to secure the payment of a loan of P10,000,000.00
which the former obtained from the latter. Solid Homes applied for and was granted several additional loans by State
Financing, and to secure its payment, Solid Homes executed Amendments to Real Estate Mortgage whereby the
credits secured by the first mortgage were increased to P13,011,082.00.

Upon maturity, State Financing made repeated demands upon Solid Homes for the payment thereof, but the latter
failed to do so. Hence State Financing filed a petition for extrajudicial foreclosure of the mortgages abovementioned
with the Provincial Sheriff of Rizal. The mortgaged properties and the improvements existing thereon were set for
public auction sale in order to satisfy the full amount of the mortgage indebtedness, the interest thereon, and the fees
and expenses incidental to the foreclosure proceedings.

Before the scheduled public auction sale, mortgagor Solid Homes induced State Financing to forego with the
foreclosure. State Financing agreed to suspend the foreclosure, subject to the terms and conditions they agreed upon.
They executed a document entitled MEMORANDUM OF AGREEMENT/DACION EN PAGO. Among the terms
and conditions that said parties agreed upon were:

In the event (Solid Homes) fails to comply this document shall automatically operate to be an instrument of
dacion en pago without the need of executing any document to such an effect

Solid Homes failed to pay State Financing 60% of the principal obligation within 180 days from the signing of the
Memorandum. Hence the document automatically operated to be an instrument of dacion en pago. However, Solid
Homes now contend that the (Memorandum) is null and void because the essence thereof is that State Financing, as
mortgagee creditor, would be able to appropriate unto itself the properties mortgaged by Solid Homes which is in
contravention of Article 2088 of the Civil Code.

Solid Homes then proposed a repayment scheme, to which State Financing offered a counter-proposal requiring
Solid Homes to make an initial payment of P10 million until 22 May 1984 and the balance payable within the
remaining period to repurchase the properties as provided for under the (Memorandum) otherwise, it will have to
vacate and turn over the possession of said properties to State Financing.

In return, Solid Homes sent a letter containing a copy of the written offer made by C.L. Alma Jose & Sons, Inc. to
avail of Solid Homes right to repurchase the V.V. Soliven Towers II pursuant to the terms of the Dacion En Pago.
The letter also contained a request that the repurchase period under said Dacion En Pago which will expire on June
27, 1984 be extended by sixty (60) days to enable Solid Homes to comply with the con ditions in the offer of Alma
Jose & Sons, Inc. referred to, and thereafter, to avail of the one year period to pay the balance.

However, a day before the expiry date of its right to repurchase, Solid Homes filed the present action against
defendants State Financing seeking the annulment of said (Memorandum) and the consequent reinstatement of the
mortgages over the same properties. RTC held that the Memorandum of Agreement/Dacion En Pago was valid and
binding which did not violate the prohibition against p actum commisorium under Art. 2088 of the Civil Code and

2B 16-17 SALES AND LEASE Page 408


was a true sale with right of repurchase, and not an equitable mortgage. It also granted Solid Homes the opportunity
to exercise its right to repurchase the properties within thirty (30) days from the finality of the Decision

CA upheld the RTC in applying NCC Article 1606, paragraph 3. Solid Homes was not in bad faith in filing the
complaint for the declaration of nullity of the Memorandum. There is statutory basis for petitioners claim that an
equitable mortgage existed since it believed that (1) the price of P14 million was grossly inadequate, considering
that the building alone was allegedly built at a cost of P60 million in 1979 and the lot was valued at P5,000.00 per
sqm and (2) it remained in poss ession of the subject properties. Furthermore, NCC Article 1607 abolished automatic
consolidation of ownership in the vendee a retro upon expiration of the redemption period by requiring the vendee to
institute an action for consolidation where the vendor a retro may be duly heard. If the vendee succeeds in proving
that the transaction was indeed a pacto de retro, the vendor is still given a period of thirty days from the finality of
the judgment within which to repurchase the property.

However, CA ordered Solid Homes to deliver possession of the subject properties to the private respondent, statting
that in a sale with pacto de retro, the vendee shall immediately acquire title over and possession of the real property
sold, subject only to the vendors right of redemption.

ISSUE

Is the failure to annotate the vendor a retros right of repurchase in the certificates of title of the real estate properties
subject of dacion en pago conclusive evidence of the vendee a retros malice and bad faith, entitling the f ormer to
damages?

HELD

No, the vendee a retro is not legally bound to cause such annotation.

ISSUE

In a sale with pacto de retro, is the repurchase price limited by Article 1616 of the Civil Code?

HELD

No.

Petitioner argues that the total redemption price is in contravention of Art. 1616 of the Civil Code. We do not,
however, find said legal provision to be restrictive or exclusive, barring additional amounts that the parties may
agree upon. Said provision should be construed together with Art. 1601 o f the same Code which provides as
follows:

Art. 1601. Conventional redemption shall take place when the vendor reserves the right to repurchase the thing sold,
with the obligation to comply with the provisions of article 1616 and other stipulations which may have been agreed
upon.

2B 16-17 SALES AND LEASE Page 409


It is clear, therefore, that the provisions of Art. 1601 require petitioner to comply with the other stipulations of the
Memorandum of Agreement/Dacion en Pago it freely entered into with private respondent. Contracts have the for ce
of law between the contracting parties who may establish such stipulations, clauses, terms and conditions as they
may want, subject only to the limitation that their agreements are not contrary to law, morals, customs, public policy
or public order -- and the above-quoted provision of the Memorandum does not appear to be so.

OTHER ISSUE

Possession of the Subject Properties During the Redemption Period

In a contract of sale with pacto de retro, the vendee has a right to the immediate possession of the property sold,
unless otherwise agreed upon. It is basic that in a pacto de retro sale, the title and ownership of the property sold are
immediately vested in the vendee a retro, subject only to the resolutory condition of repurchase by the vendor a retro
within the stipulated period.

Assailed CA decision is affirmed with the modification that the redemption price shall not include the registration
and other expenses incurred by State Financing Center, Inc. in the issuance of new certificates of title in its name, as
this was done without the proper judicial order required under Article 1607 of the Civil Code.

2B 16-17 SALES AND LEASE Page 410


15. CUA VS VARGAS

G.R. No. 156536 October 31, 2006

Azcuna, J.

FACTS:

A parcel of residential land was left behind by the late Paulina Vargas. A notarized extrajudicial settlement
was executed by and among the heirs partitioning the said property. However, only 4 of them signed the document.
The other 4 did not sign. The extrajudicial settlement was later on published in the Catanduanes Tribune for 3
consecutive weeks.

Herein petitioner Cua bought the property from the signatories of the documents. The other co -heirs were
not notified before the alleged partition, although it was published. One of the co -heirs only learned about the
partition and sale when the house was demolished. Claiming that she was unaware, she tried to redeem it but Cua
refused. Hence, an action for annulment of extrajudicial settlement and legal redemption of the lot was filed. The
MTC, however dismissed the complaint stating that the transaction occurred after the partition, hence, the co -heirs
could validly dispose of their shares. Moreover, the MTC found that although there was a failure to strictly comply
with the requirements under Article 1088 of the Civil Code for a written notice of sale to be served upon
respondents by the vendors prior to the exercise of the former's right of redemption, this deficiency was cured by
respondents' actual knowledge of the sale, which was more than 30 days before the filing of their complaint, and
their consignation of the purchase price with the Clerk of Court, so that the latter action came too late. The RTC
affirmed the decision but the CA reversed it.

Petitioner now claims that the transaction was valid because the publication of the EJ settlement served as a
constructive notice to the co-heirs. He also claims that he is a possessor in good faith.

ISSUE:

Whether or not petitioners contentions are correct.

HELD

NO.

The procedure outlined in Section 1 of Rule 74 is an ex parte proceeding. The rule plainly states, however,
that persons who do not participate or had no notice of an extrajudicial settlement will not be bound thereby. It
contemplates a notice that has been sent out or issued before any deed of settlement and/or partition is agreed upon
(i.e., a notice calling all interested parties to participate in the said deed of extrajudicial settlement and partition), an d
not after such an agreement has already been executed as what happened in the instant case with the publication of
the first deed of extrajudicial settlement among heirs.

The publication of the settlement does not constitute constructive notice to the heirs who had no knowledge
or did not take part in it because the same was notice after the fact of execution. The requirement of publication is
geared for the protection of creditors and was never intended to deprive heirs of their lawful participation in the
decedent's estate. In this connection, the records of the present case confirm that responde nts never signed either of
the settlement documents, having discovered their existence only shortly before the filing of the present complaint.

2B 16-17 SALES AND LEASE Page 411


This is not to say, though, that respondents' co -heirs cannot validly sell their hereditary rights to third
persons even before the partition of the estate. The heirs who actually participated in the execution of the
extrajudicial settlements, which included the sale to petitioner of their pro indiviso shares in the subject property, are
bound by the same. Nevertheless, respondents are given the right to redeem these shares pursuant to Article 1088 of
the Civil Code. The right to redeem was never lost because respondents were never notified in writing of the actual
sale by their co-heirs. Based on the provision, there is a need for written notice to start the period of redemption,
thus:

Should any of the heirs sell his hereditary rights to a stranger before the partition, any or all of the co -heirs
may be subrogated to the rights of the purchaser by reimbursing him for the price of the sale, provided
they do so within the period of one month from the time they were notified in writing of the sale by
the vendor. (Emphasis supplied.)

It bears emphasis that the period of one month shall be reckoned from the time that a c o-heir is notified in
writing by the vendor of the actual sale. Written notice is indispensable and mandatory, actual knowledge of the sale
acquired in some other manner by the redemptioner notwithstanding. It cannot be counted from the time advance
notice is given of an impending or contemplated sale. The law gives the co -heir thirty days from the time written
notice of the actual sale within which to make up his or her mind and decide to repurchase or effect the redemption.

It should be kept in mind that the obligation to serve written notice devolves upon the vendor co -heirs
because the latter are in the best position to know the other co -owners who, under the law, must be notified of the
sale. Considering, therefore, that respondents' co-heirs failed to comply with this requirement, there is no legal
impediment to allowing respondents to redeem the shares sold to petitioner given the former's obvious willingness
and capacity to do so.

2B 16-17 SALES AND LEASE Page 412


16. LEE CHUY REALTY CORP VS CA

G.R. No. 104114, December 4, 1995

Bellosillo, J.

FACTS

A land in Meycauayan, Bulacan, covered by OCT No. 0-5290 is disputed by petitioner Lee Chuy Realty
Corporation (LEE CHUY REALTY) and private respondent Marc Realty and Development Corporation (MARC
REALTY). Originally the property was co-owned by Ruben Jacinto to the extent of one-sixth and Dominador,
Arsenio, Liwayway, all surnamed Bascara, and Ernesto Jacinto who collectively owned the remaining five -sixths.
On 4 February 1981 Ruben Jacinto sold his one-sixth pro-indiviso share to LEE CHUY REALTY. The sale was
registered on 30 April 1981. On 5 May 1989 the Bascaras and Ernesto Jacinto also sold their share to MARC
REALTY. The sale was registered on 16 October 1989. LEE CHUY REALTY claims that it was never informed of
the existence of the sale between MARC REALTY and the Bascaras and Jacinto, and that on the contrary it was
only upon inquiry from the Register of Deeds of Bulacan that the sale was brought to its attention. MARC REALTY
insists that LEE CHUY REALTY was verbally notified of the sale and was in fact given a copy of the deed of sale.
On 13 November 1989 LEE CHUY REALTY filed a complaint for legal redemption against MARC REALTY and
consigned in court a manager's check for 614,400. MARC REALTY insisted that the complain t be dismissed for
failure to state a cause of action there being no allegation of prior valid tender of payment nor a prior valid notice of
consignation. On 26 December 1990 the trial court ruled in favor of LEE CHUY REALTY. However, the CA
reversed it.

ISSUE

Whether or not the filing of the action with consignation is equivalent to a formal offer to redeem.

HELD

Arts. 1620 and 1623 of the Civil Code on legal redemption provide:

Art. 1620. A co-owner of a thing may exercise the right of redemption in cas e the shares of all the other co-owners
or of any of them are sold to a third person. If the price of the alienation is grossly excessive, the redemptioner shall
pay only a reasonable one.

Art. 1623. The right of legal pre-emption or redemption shall not be exercised except within thirty days from the
notice in writing by the prospective vendor, or by the vendor, as the case may be. The deed of sale shall not be
recorded in the Registry of Property unless accompanied by an affidavit of the vendor that he ha s given written
notice thereof to all possible redemptioners.

The formal offer to redeem is not a distinct step or condition sine qua non to the filing of the action in Court for the
valid exercise of the right of legal redemption. What constitutes a condition precedent is either a formal offer to
redeem or the filing of an action in court together with the consignation of the redemption price within the
reglementary period.

2B 16-17 SALES AND LEASE Page 413


17. FELIPA FERIA, CARLOS F. CRUZ, CLARA DE LA CRUZ, LEONILA DE LA CRUZ, PETRONILA
DE LA CRUZ, ROSARIO DE LA CRUZ, FAUSTINO DE LA CRUZ AND RUFINO DE LA CRUZ vs.
GERONIMO T. SUVA

G.R. No. L-5515 April 24, 1953

Labrador, J

FACTS

Plaintiff conveyed to the defendant on April 18, 1938, by way of sale with the right to repurchase, a parcel of land,
together with its improvements, executing to that effect a deed of sale with pacto de retro. On September 14, 1944,
plaintiffs, who were the vendors a retro, brought an action in the Court of First Instance of Nueva Ecija alleging the
existence of said pacto de retro sale, and praying that they be allowed to redeem the property, as the defendant had
agreed to re-sell the property after the harvest of 1943-1944. The defendant in that previous action, who is also the
defendant herein, admitted the existence of the contract of pacto de retro, but denied that he had ever allowed the
plaintiffs to make a redemption of the property in the year 1944, after the expiration of the period fixed for the
repurchase, as claimed by the plaintiffs. Trial having been h ad on the above issue, the Court of First Instance of
Nueva Ecija found that the original term fixed for the redemption was not extended and, therefore, dismissed the
complaint. The case was appealed to the Court of Appeals, but this court affirmed the jud gment of the trial court. It
found that the transaction was a true sale with right to repurchase, that there was no extension of the period of
redemption, and that no tender of the purchase was in due time. It, therefore, affirmed the judgment in so far as the
denial of the right to redeem was concerned, although it modified it in the sense of making the sale ineffective with
respect to the share of two co-owners of the property, who had not participated in the sale.

The present action was filed on October 14, 1951, exactly one day after the judgment rendered by the Court of
Appeals in the previous action had become final.

ISSUE

Whether or not the vendor a retro may repurchase the subject property

HELD

No. The last paragraph of said article 1606 provides that "the vendor may still exercise the right to repurchase within
thirty days from the time final judgment was rendered in civil action on the basis that the contract was a true sale
with right to repurchase." One of the members of the Code Committee, commenting on the aforequoted provision,
says:

Paragraph three of the article is a new provision formulated by the Code Commission. It is intended to
cover suits where the seller claims that the real intention was a loan with equitable mortgage, but
the court decides otherwise.

2B 16-17 SALES AND LEASE Page 414


The refusal of the Court of Appeals to grant the plaintiffs -appellants the privilege of redemption under the above-
quoted provision of article 1606 of the New Civil Code was based on the fact that the nature and character of the
transaction as a pacto de retro sale or as an equitable mortgage was never in question before the trial court, the
transaction being admittedly one of sale with pacto de retro.

But even if the decision of the Court of Appeals be interpreted in the sense indic ated by appellants, i.e., that the
transaction is not covered by the provisions of article 1606 of the New Civil Code because it took place before the
latter took effect, and the action thereon brought before the said code took effect, a claim which we do not now
decide being unnecessary, the said decision would still bar the present action. The finality of decisions of court is not
dependent upon their correctness, but upon the expiration of the period fixed by the rule therefor. Evidently, as
indicated in appellee's brief, the question now raised was actually passed upon adversely by the Court of Appeals in
a resolution on a motion for reconsideration, and also by this Court in G.R. No. L-5025. From whatever point the
decision of the Court of Appeals may be considered, it is evident that the present action is barred thereby.

2B 16-17 SALES AND LEASE Page 415


WHEN SALE PRESUMED EQUITABLE MORTGAGE

1. CIRIACA SANTOS VS TEODORICA DUATA

G.R. No. L- 20901, August 31, 1965

Bengzon JP, J

FACTS

Apolonio del Mundo and his brother, Dalmacio, were lessees of a parcel of land in the Lolomboy Friar Lands Estate
situated at Polo, Bulacan. On November 11, 1908 Apolonio del Mundo sold his rights to the land to the spouses
Pedro Duata and Epifania Aguilar for the sum of P40.00.

Since then until the present time, said spouses and later their daughter, Teodorica Duata, have adverse, peaceful and
uninterrupted possession of the land.

On March 23, 1933 Ciriaca Santos, Petrona Gaanan and Epifania Aguilar purchased Lot No. 37 from the Burea u of
Lands with the previous agreement that Petrona Gaanan would receive one -half of the lot and Ciriaca Santos and the
Duata spouses would share equally the other half.

The portion allocated to the Duata spouses was the very same land over which Apolonio del Mundo conveyed to
them his rights in 1908.

On August 3, 1955 Teodorica Duata, the daughter of the Duata spouses, instituted an action in the Court of First
Instance of Bulacan against Ciriaca Santos for the reconveyance of one-fourth of Lot No. 37.

In her answer Ciriaca Santos denied the ownership of the Duata spouses over one -fourth of Lot No. 37 and alleged
that assuming the land in question belonged to the Duata spouses, the same had already been sold to her by Epifania
Aguilar.

Ciriaca Santos died on February 8, 1957 and was substituted by her sole heir Juana Prodon as defendant.

Trial court found that the spouses Pedro Duata and Epifania Aguilar owned one -fourth of Lot No. 37; that Epifania
Aguilar sold the one-fourth interest to Ciriaca Santos on September 25, 1938 with right to repurchase the same
within one year from said date; that since the land was not repurchased within the time stipulated, ownership over
the same became vested in Ciriaca Santos.

The Court of Appeals considered the transaction between Ciriaca Santos and Epifania Aguilar an equitable
mortgage rather than a pacto de retro sale.

ISSUE

The sole issue is whether Ciriaca Santos and Epifania Aguilar in executing the document marked Exhibit 3 intended
a mortgage or sale with pacto de retro.

2B 16-17 SALES AND LEASE Page 416


HELD

The above-quoted stipulation, standing alone, would show a transfer by Epifania Aguilar to Ciriaca Santos of one -
fourth of Lot No. 37 by way of sale with pacto de retro. The coetaneous actuations of Epifania Aguilar and Ciriaca
Santos, however, indicate the contrary. For, despite execution of Exhibit 3, Epifania Aguilar remained in possession
of the land in the concept of an owner. She and her daughter continued paying the land tax a burden attached to
ownership on the property. Previous, simultaneous and subsequent acts of the parties are properly cognizable
indicia of their true intention. The attendance of such circumstances raises the presumption under Article 1602 of
the New Civil Code that Exhibit 3 is an equitable mortgage

Ciriaca Santos, however, maintains that mere possession of the land and payment of land taxes due thereon by Duata
would not warrant presumption that Exhibit 3 is an equitable mortgage. Accordingly, she contends that there must
be a "concurrence of an overwhelming number of circumstances" before the presumption would arise. To this
proposition we do not agree. Article 1602, when it expressly states "in any of the following cases," contemplates the
existence of any of the circumstances enumerated therein.

Article 1602 is a new provision in the Civil Code designed primarily to curtail the evils brought about by contracts
of sale with right of repurchase, such as the circumvention of the usury law and pactum commissorium. It
particularly envisions contracts of sale with right of repurchase where the real intention of the parties is that the
pretended purchase price is money loaned, and in order to secure the payment of the loan a contract purporting to be
a sale with pacto de retro is drawn up

Note:

The pertinent portion of Exhibit 3 reads:

II. Na, alang-alang sa halagang ISANG DAAN AT LIMAMPU'T APAT NA PISO (P154.00) na tinanggap ko at
ibinayad sa akin ni Bb. Ciriaca Santos, filipina, may sapat na gulang, dalaga at naninirahan sa Maykawayan,
Bulacan, K. P., sa pamamagitan ng kasulatang ito ay aking ipinagbibili ng biling mabibiling muli ang ika apat na
bahagi ng nasabing lupa sa itaas nito sa nasabi ng Bb. Ciriaca Santos sa ilalim ng kasunduang gaya ng mga
sumusunod:

(a) Na bibilhin kong muli sa halagang P154.00 and nasabing pag-aari ko, sa loob ng isang taon simula ngayon;

(b) Na, sakaling hindi ko mabiling muli ang nasabing lupa sa loob ng isang taon simula sa fechang ito ay magiging
lubusang pag-aari na ni Bb. Ciriaca Santos and nasabing pagaari, na hindi na kakailanganin pa ang panibagong
kasulatan ng bilihang lubusan.

ART. 1602. The contract shall be presumed to be an equitable mortgage, in any of the following cases:

(1) When the price of a sale with right to repurchase is unusually inadequate;

(2) When the vendor remains in possession as lessee or otherwise;

(3) When upon or after the expiration of the right to repurchase another instrument extending the period of
redemption or granting a new period is executed;

(4) When the purchaser retains for himself a part of the purchase price;

2B 16-17 SALES AND LEASE Page 417


2. ANGEL VILLARICA and NIEVES PALMA GIL DE VILLARICA vs.CTHE COURT OF
APPEALS, JULIANA MONTEVERDE, GAUDENCIO CONSUNJI and JOVITO S. FRANCISCO

G.R. No. L-19196 November 29, 1968

Capistrano, J

FACTS

On May 19, 1951, the spouses Villarica sold to the spouses Consunji a lot situated in the City of Davao for the price
of P35,000. The public instrument of absolute sale and the vendors' TCT were delivered to the vendees. On May 25,
1951, the spouses Consunji executed another public instrument whereby they granted the spouses Villarica an option
to buy the same property within the period of one year for the price of P37,750. The spouses Consunji registered the
absolute deed of sale, in consequence of which the TCT in the names of the sp ouses Villarica was cancelled and a
new TCT was issued in the names of the spouses Consunji. In February 1953, the spouses Consunji sold the lot to
Jovito S. Francisco for the price of P47,000 by means of a public instrument of sale which was registered in view of
which the TCT in the names of the spouse Consunji was cancelled and a new TCT in the name of Jovito S.
Francisco was issued.

The spouses Villarica brought an action in the Court of First Instance of Davao against the spouses Consunji and
Jovito S. Francisco for the reformation of the instrument of absolute sale into an equitable mortgage, contending
that: (1) the price of P35,000 was unusually inadequate; (2) the vendors remained in possession of the property sold;
(3) the period of one year for repurchase granted was extended for one month; and (4) the vendors pay the taxes on
the land sold.

ISSUE

Whether or not the contract is an equitable mortgage.

HELD

NO.

(1) The price of P35,000 was not even inadequate. The land sold was assessed for tax purposes at P8,870 effective
1950. It was purchased by the spouses Villarica from the Philippine Alien Property Custodian in October, 1950, for
the price of P20,000. The price of P35,000 stated in the instrument of absolute sale was the market price of the lo t in
1951.

(2) The vendors did not remain in possession of the land sold as lessees or otherwise. On their request in order to
help them in the expenses of their children in Manila, the vendors were merely allowed by the vendees to collect the
monthly rents of P300 for five months up to October, 1951, on the understanding that the amounts so collected
would be charged against them. But thereafter the vendees were the ones who collected the monthly rents from the
tenants. It follows that the vendors did not remain in possession of the land as lessees or otherwise.

(3) The Consunjis as new owners of the lot granted the Villaricas an option to buy the property within the period of
one year from May 25, 1951 for the price of P37,750. Said option to buy is different and distinct from the right
of repurchase which must be reserved by the vendor, by stipulation to that effect, in the contract of sale. This
is clear from Article 1601 of the Civil Code, which provides:

2B 16-17 SALES AND LEASE Page 418


Conventional redemption shall take place when the vendor reserves the right to repurchase the thing sold, with the
obligation to comply with the provisions of article 1616 and other stipulation which may have been agreed upon.

The right of repurchase is not a right granted the vendor by the vendee in a s ubsequent instrument, but is a
right reserved by the vendor in the same instrument of sale as one of the stipulations of the contract. Once the
instrument of absolute sale is executed, the vendor can no longer reserve the right to repurchase, and any
right thereafter granted the vendor by the vendee in a separate instrument cannot be a right of repurchase
but some other right like the option to buy in the instant case.

(4) The taxes paid by the vendors were back taxes up to the time of the sale on May 19, 1951. The vendors had the
obligation to pay the back taxes because they sold the land free of all liens and encumbrances. The taxes due after
the sale were paid by the vendees.

2B 16-17 SALES AND LEASE Page 419


3. PONCIANO T. MATANGUIHAN, and EUSTAQUIA M. MATANGUIHAN, vs. COURT OF
APPEALS, HERMINIO PARAN substituted by REYNALDO, JOSEPH, RONNIE, all surnamed PARAN;
ERLINDA PARAN-GONZALES, FLORA PARAN-LESCANO, represented by REYNALDO PARAN,

G.R. No. 115033. July 11, 1997

Davide Jr, J

FACTS

This is a complaint filed on February 5, 1987 by spouses Ponciano T. Matanguihan and Eustaquia
Matanguihan against Herminio Paran to recover possession of a residential house and lot located at
Poblacion Mataas na Kahoy with an area of 1,130 square meters covered by Tax Declaration No. 0473.

- October 13, 1983, spouses Ponciano Matanguihan and Eustaquia Matanguihan and spouses Herminio and
Fortunata Paran entered into an agreement denominated as Kasulatan ng Bilihang Lupang Mabibiling Muli (Pacto
de Retro) covering a residential house and lot owned by spouses Herminio and Fortunata Paran paid by spouses
Ponciano and Eustaquia Matanguihan.Said agreement is a public document having been notarized by Notary
Public.The agreement provides that:

o It will last only for three months until the spouses can purchase the property back and if not purchased within that
duration, the agreement will become absolute.

- The three-month period stipulated in the agreement lapsed without the defendant being able to repurchase the
property. Plaintiffs, upon requests of defendant, granted the latter, not only one but several extensions.

- Defendants still failed to repurchase the property. Thereafter, plaintiffs demanded from defendant possession
of the property which the latter refused. The plaintiff caused the transfer of the Tax Declaration No. 0473 (Exh. B
and B-1).

- The plaintiffs also caused the registration of the said property in the primary entry of the Register of Deeds of
Batangas. Plaintiffs now seek to enforce the aforesaid agreement.

- The defendant claims that the subject agreement entitled Kasulatan ng Bilihang Mabibiling Muli (Pacto de
Retro) is intended by the parties to be an equitable mortgage.

- TC ruled in favor of petitioners, finding that the contract is clearly a pacto de retro sale.

- CA reversed; finding that the contract is an equitable mortgage.

ISSUE

Whether CA erred in ruling that the sale is an equitable mortgage?

HELD:
NO. CA DID NOT ERR IN RULING THAT THE KASULATAN NG BILIHANG MULI (PACTO DE RETRO) IS
AN EQUITABLE MORTGAGE.

2B 16-17 SALES AND LEASE Page 420


Our law on contracts provides inter alia that in order to judge the intention of the contracting parties, their
contemporaneous and subsequent acts shall be principally considered. [12] Accordingly, there are instances where the
form and stipulations of a contract must give way to reflect the true intention of the parties.

This is best illustrated in the instances where contracts of sale, whether absolute, or one where the vendor reserves
the right to repurchase the thing sold or a sale pacto de retro, are presumed to be an equitable mortgage. These
instances are governed by Articles 1602, 1603 and 1604 of the Civil Code.

ART. 1602. The contract shall be presumed to be an equitable mortgage, in any of the following cases:

1. When the price of the sale with right to repurchase is unusually inadequate;

2. When the vendor remains in possession as lessee or otherwise;

3. When upon or after the expiration of the right to repurchase another instrument extending the period of
redemption or granting a new period is executed;

4. When the purchaser retains for himself a part of the purchase price;

5. When the vendor binds himself to pay the taxes on the thing sold;

6. In any other case where it may be fairly inferred that the real intention of the parties is that the transaction shall
secure the payment of a debt or the performance of any other obligation.

First. Sometime in October 1983, Reynaldo Paran, son of the deceased Herminio Paran, was strapped for cash,
needing the amount of P100,000.00 to answer for several expenses. He was advised by his late father to mortgage
the lot in order to secure the needed amount. Thus, p etitioners extended to Herminio Paran the loan of P100,000.00
payable within three months. Thereafter, on 7 October 1983, a deed of sale covering the lot and residential house
erected thereon was executed. The consideration appearing in the deed amounted t o P118,000.00 representing the
principal of P100,000.00 and the interest of P18,000.00 at the rate of 6% per month. Due to his pressing need to
obtain funds, Reynaldo allowed his parents to sign the deed knowing fully well that it did not reflect the real
intention of the parties.

Second. It is undisputed that the alleged vendors and their successors -in-interest remained in actual physical
possession of the disputed property as if they were still the absolute owners thereof, without an agreement for
maintenance expenses, much less, rental payments.

Third. Petitioners declared the property in their names for taxation purposes only on 13 November 1985, and paid
the taxes thereon as evidenced by Realty Tax Receipt No. 157998, [20]only on 7 May 1986.

Fourth. Petitioners allowed various extensions of the redemption period. These extensions of the redemption period
are indicative of an equitable mortgage, as expressly enumerated in Article 1602(3) of the Civil Code.

Fifth. Petitioners failed to consolidate their title over the disputed property that was allegedly sold even after the
expiration of the period to redeem. (this eroded their claim of title(

2B 16-17 SALES AND LEASE Page 421


Sixth. The circumstances show that buyer has no intention to buy and the seller has no intention to sell. Most
revealing of the want of intention to sell is the fact that the money proceeds of the alleged sale was partly used for
the construction of the very house purportedly sold. On the other hand, petitioner Ponciano Matanguihans
declaration that he never entered the premises of the disputed property in order to inspect it, unequivocally revealed
an absence of intent to buy. A buyer of sound mind would not purchase anythin g without first inspecting the thing to
be bought.

2B 16-17 SALES AND LEASE Page 422


4. ALFREDO N. AGUILA, JR Vs HONORABLE COURT OF APPEALS and FELICIDAD S. VDA. DE
ABROGA

G.R No. 127347 November 25, 1999

Mendoza, J.:

FACTS

Private respondent Felicidad s, Abrogar and her late husband, Ruben M. Abrogar, registered owners of a house and
lot, entered into a Memorandum of Agreement (MOA) on April 18, 1991, with A.C. Aguila & Sons, Co., a
partnership engaged in lending activities. The agreement provided:

(1) That the SECOND PARTY [A.C. Aguila & Sons, Co.] shall buy the above-described property from the FIRST
PARTY [Felicidad S. Vda. de Abrogar] XXX

(2) The FIRST PARTY is hereby given by the SECOND PARTY the option to repurchase the said property within a
period of ninety (90) days from the execution of this MOA effective April 18, 1991, XXX

(3) In the event that the FIRST PARTY fail to exercise her option to repurchase the said property within a period of
ninety (90) days, the FIRST PARTY is obliged to deliver peacefully th e possession of the property to the SECOND
PARTY within fifteen (15) days after the expiration of the said 90 day grace period; XXX

(8) Should the FIRST PARTY fail to exercise her option to repurchase the property within ninety (90) days period
above-mentioned, this memorandum of agreement shall be deemed cancelled and the Deed of Absolute Sale,
executed by the parties shall be the final contract considered as entered between the parties and the SECOND
PARTY shall proceed to transfer ownership of the property above described to its name free from lines and
encumbrances.

On the same day, they executed a deed of absolute sale, dated June 11, 1991, wherein private respondent sold
the subject property to A.C. Aguila & Sons, Co., for P200,000.00. In a special po wer of attorney (SPA) dated the
same day, private respondent authorized petitioner Alfredo Aguila, the manager of A.C. Aguila & Sons, Co, to cause
the cancellation of TCT No. 195101 and the issuance of a new certificate of title in the name of A.C. Aguila and
Sons, Co., in the event she failed to redeem the subject property as provided in the MOA.

Private respondent failed to redeem the property within the 90-day period. Hence, pursuant to the SPA,
petitioner caused the cancellation of TCT No. 195101 and the issuance of a new certificate of title.

Upon the refusal of private respondent to vacate the subject premises, A.C. Aguila & Sons, Co. filed an
ejectment case against her. In a decision, dated April 3, 1992, the Metropolitan Trial Court ruled in favor of A.C.
Aguila & Sons, Co. on the ground that private respondent did not redeem the subject property before the expiration
of the period provided in the MOA. Private respondent appealed first to the Regional Trial Court, then to the Court
of Appeals, and later to this Court, but she lost in all the cases.

2B 16-17 SALES AND LEASE Page 423


Private respondent then filed a petition for declaration of nullity of a deed of sale with the Regional Trial
Court, Branch 273, Marikina on December 4, 1993. She alleged that the signature of her husband o n the deed of sale
was a forgery because he was already dead when the deed was supposed to have been executed on June 11, 1991.

It appears, however, that private respondent had filed a criminal complaint for falsification against petitioner
with the Office of the Prosecutor of Quezon City which was dismissed in a resolution, dated February 14, 1994. On
April 11, 1995, Branch 273 of RTC-Marikina rendered its decision dismissing the petition.

On appeal, the Court of Appeals reversed holding that the transact ion between plaintiff-appellant and
defendant-appellee is indubitably an equitable mortgage. The purchase price for the alleged sale with right to
repurchase is unusually inadequate. The disputed MOA specifically provides that plaintiff-appellant is obliged to
deliver peacefully the possession of the property to the SECOND PARTY within fifteen (15) days after the
expiration of the said ninety (90) day grace period. Otherwise stated, plaintiff-appellant is to retain physical
possession of the thing allegedly sold. The apparent vendor, plaintiff-appellant herein, continued to pay taxes on the
property sold. Since the real intention of the party is to secure the payment of debt, now deemed to be repurchase
price: the transaction shall then be considered to be an equitable mortgage.

Since the real intention of the party is to secure the payment of debt, now deemed to be repurchase price, the
transaction shall then be considered to be an equitable mortgage. Being a mortgage, the transaction entered into by
the parties is in the nature of a pactum commissorium which is clearly prohibited by Article 2088 of the New Civil
Code. Article 2088 of the New Civil Code reads:

ART. 2088. The creditor cannot appropriate the things given by way of pledge or mortgage, or dispo se of them. Any
stipulation to the contrary is null and void.

Clearly, the agreement entered into by the parties is in the nature of pactum commissorium. Therefore, the
deed of sale should be declared void as we hereby so declare to be invalid, for being violative of law.

ISSUE

Whether Alfredo Aguila is not the real party in interest but A.C. Aguila & Co., against which this case should have
been brought?

HELD

YES

Rule 3, 2 of the Rules of Court of 1964, under which the complaint in this case was filed , provided that every action
must be prosecuted and defended in the name of the real party in interest. A real party in interest is one who would
be benefited or injured by the judgment, or who is entitled to the avails of the suit. This ruling is now embo died in
Rule 3, 2 of the 1997 Revised Rules of Civil Procedure. Any decision rendered against a person who is not a real
party in interest in the case cannot be executed. Hence, a complaint filed against such a person should be dismissed
for failure to state a cause of action.

2B 16-17 SALES AND LEASE Page 424


Under Art. 1768 of the Civil Code, a partnership has a juridical personality separate and distinct from that of each of
the partners. The partners cannot be held liable for the obligations of the partnership unless it is shown that t he legal
fiction of a different juridical personality is being used for fraudulent, unfair, or illegal purposes.In this case, private
respondent has not shown that A.C. Aguila & Sons, Co., as a separate juridical entity, is being sued for fraudulent,
unfair, or illegal purposes. Moreover, the title to the subject property is in the name of A.C. Aguila & Sons, Co. and
the Memorandum of Agreement was executed between private respondent, with the consent of her late husband, and
A. C. Aguila & Sons, Co., represented by petitioner.

Hence, it is the partnership, not its officers or agents, which should be impleaded in any litigation involving property
registered in its name. A violation of this rule will result in the dismissal of the complaint. We cannot understand
why both the Regional Trial Court and the Court of Appeals sidestepped this issue when it was squarely raised
before them by petitioner.Our conclusion that petitioner is not the real party in interest against whom this action
should be prosecuted makes it unnecessary to discuss the other issues raised by him in this appeal.

2B 16-17 SALES AND LEASE Page 425


5. ERLINDA SAN PEDRO vs. RUBEN LEE and LILIAN SISON

G.R. No. 156522. May 28, 2004

Ynares-Santiago, J

FACTS

Petitioner initiated a suit against the respondent s pouses which prayed for: (1) a declaration that the document
entitled Kasulatan ng Ganap na Bilihan ng Lupa is an equitable mortgage and not a sale; (2) the reconveyance of
the property subject of the Kasulatan ng Ganap na Bilihan ng Lupa; and (3) damages.

The respondents version of the events paints a portrait of an unscrupulous couple, usuriously taking
advantage of her financial straits to enrich themselves. Petitioner claims that she desperately needed money to
support her childrens college education. Petitioner was able to secure a loan in the amount of P105,000.00, with
interest of P45,000.00, or a total indebtedness of P150,000.00. As security for this loan, she agreed to mortgage a
17,235-square meter parcel of agricultural land located at San Juan, Balagtas, Bulacan, covered by Transfer
Certificate of Title (TCT) No. T-290387.This transaction took place in the office of Atty. Venustiano Roxas, where
she met Lee for the first time.

San Pedro claims that Atty. Roxas and Lee coerced her to sign the Kasulatan ng Ganap na Bilihan ng Lupa
and that the document was executed merely as written evidence of the loan and mortgage. She alleges that she told
her that the document was just a formality, with the assurance that respondents would never enforce the con tract
against her. She readily agreed because she believed in good faith that the spouses were tunay na tao. She further
claims that she continued in possession of the parcel of land through her tenant, Federico Santos, and continued to
receive her landowners share of the harvest from 1985 until 1995.

When petitioner attempted to pay the real property tax on the subject agricultural land, she was surprised
that the property had already been transferred to the names of respondents. She also learned that TCT No. T-290387
had been cancelled and TCT No. RT-41717 (T-305595) had been issued in the name of Ruben Lee .

After saving enough money to pay her indebtedness, San Pedro attempted to redeem her mortgage. She
approached Ruben Lees brother, Carlito, offering to pay her debt, but she was continually rebuffed. Nine years after
the contract was executed, she initiated this suit to recover title to the subject property.

Respondents, on the other hand, present an entirely different version of events. They claim that the sale of
the property in question was brokered by their mutual acquaintance and broker, Philip dela Torre. Spouses Lee and
Sison are engaged in the real estate business, and believed that San Pedros agricultural property would be a good
investment. Thus, they negotiated for the purchase of the property and petitoner accepted their offer and agreed to
sell the land.

Respondents requested that petitioner execute an affidavit of non -tenancy and a written power of attorney
authorizing respondents to pay the capital gains taxes and expenses on the registration of the property in their name.

The trial court rendered a decision in favor of petitioner, declaring the contract between petitioner and
respondents as one of mortgage and not of sale, and ordering the reconveyance of the property and the payment of
damages.On appeal, the Court of Appeals reversed the trial court, and rendered a decision in favor of respondents.

Hence, this appeal.

2B 16-17 SALES AND LEASE Page 426


ISSUE

Whether or not the contract in question is an equitable mortgage o r a deed of absolute sale?

HELD

The document appears on its face to be a contract of sale, and contains the following clause:

Na dahil at alang-alang sa halagang ISANG DAAN AT LIMAMPUNG LIBONG PISO


(P150,000.00), Salaping Pilipino, na ngayong araw na ito ay ibinayad sa akin at tinanggap ko
naman ng buongkasiyahang-loob bilang husto at ganap na kabayaran ni RUBIN T. LEE, may
sapat na gulang, Pilipino, kasal kay Lilian Sison at naninirahan sa 230 MacArthur Highway,
Karuhatan, Valenzuela, Metro Manila, aking IPINAGBIBILI, ISINASALIN at INILILIPAT ng
ganap at patuluyan at walang anumang pasusubali o pananagutan, ang lahat at boo [sic] kong
karapatan at pagmamay-ari at pamumusesyon sa nabanggit na lagay ng lupa at mga kaunlaran o
mejoras na dito ay makikita o nakatirik o matatagpuan sa nasabing RUBIN T. LEE at sa kanyang
mga tagapamana o kahalili.

It is well-settled that the presence of even one of the foregoing circumstances in Art 1602 is sufficient to
declare a contract as an equitable mortgage, in consonance with the rule that the law favors the least transmission of
property rights. For the presumption of an equitable mortgage to arise under Article 1602, two requisites must
concur: (1) that the parties entered into a contract denominated as a sale; and (2) that their intention was to secure an
existing debt by way of a mortgage.

After a careful review of the records of the case, we find no cogent reason to disturb the ruling of the Court
of Appeals.

In this case, it was incumbent upon San Pedro to adduce sufficient evidence to support her claim of an
equitable mortgage. Petitioner relies on paragraphs 1, 2, 5 and 6 of Article 1602. Upon an examination of the
evidence, we find insufficient basis to conclude the existence of any of the grounds she relied upon.

Anent alleged inadequacy of the purchase price, petitioner presented two witnesses who testified as to the
market values of real estate in the subject locale. Neither of these witnesses, however, was able to conclusively
demonstrate that the purchase price of the property was grossly inadequate.

Absent any evidence of the market value of the locale as of the date of the contract, it cannot be concluded
that the price at which the property was sold, or about P8.70 per square meter, was grossly inadequate. Me re
inadequacy of price would not be sufficient. The price must be grossly inadequate, or purely shocking to the
conscience. Since the property in question could have been worth as little as P20.00 per square meter in 1994, the
price of P8.70 per square meter nine years earlier, in 1985, does not seem to be grossly inadequate. Indeed,
respondents Declaration of Real Property No. 10786, for the year 1987, shows the market value of the property to be
only P34,470.00 for that year.

As a third ground for the establishment of the purported equitable mortgage, petitioner argues that
paragraph 5 of Article 1602 is present. Again, petitioner presented no proof that she, as vendor of property, bound
herself to pay taxes on the thing sold.

2B 16-17 SALES AND LEASE Page 427


Finally, petitioner relies on Article 1602, paragraph 6, which applies to any other case where it may be
fairly inferred that the real intention of the parties is that the transaction shall secure the payment of a debt or the
performance of any other obligation.

In contrast, respondents witnesses all testified as to the existence of a contract of sale between her and
respondent Ruben Lee. Pertinently, Philip dela Torre, who brokered the sale, and Atty. Venustiano Roxas, who
prepared the contract in question, were both unequivocal as to the nature of the contract. These two witnesses,
whose impartiality was not impugned, both affirmed the sale of the subject property. Furthermore, respondents
presented documentary evidence which shows that the contract was indeed a sale

The Kasulatan ng Ganap na Bilihan ng Lupa unequivocally states the absolute sale of the property covered
by Transfer Certificate of Title No. T-290387. Being a notarized document, it carries the evidentiary weight
conferred upon duly executed instruments provided by law, and is entitled to full faith and credit upon its face.

2B 16-17 SALES AND LEASE Page 428


6. HEIRS OF LATE SPOUSES AURELIO AND ESPERANZA BALITE VS RODRIGO LIM

G.R. No. 152168, December 10, 2004

Panganiban, J

FACTS:

Spouses Aurelio and Esperanza Balite were the owners of a parcel of land, located at Catarman, Northern
Samar, When Aurelio died intestate [in 1985, his wife], Esperanza Balite, and their children (petitioners) inherited
the property and became co-owners thereof, with Esperanza.

Esperanza became ill and was in dire need of money for her hospital. She, through her daughter, Cristeta,
offered to sell to Rodrigo Lim, her undivided share for the price of P1,000,000.00. Esperanza and Rodrigo agreed
that, under the Deed of Absolute Sale, it will be made to appear that the purchase price of the property would be
P150,000.00, although the actual price agreed upon by them for the property was P1,000,000.00.

They also executed, on the same day, a "Joint Affidavit" under which they declared that the real price of the
property was P1,000,000.00, payable to Esperanza in instalments. Only Esperanza and two of her children, namely,
Antonio and Cristeta knew about the said transaction.

Thereafter, Rodrigo took actual possession of the property and introduced improvements t hereon. He
remitted to Esperanza and Cristeta sums of money in partial payments of the property.

Subsequently, Rodrigo secured a loan from the Rizal Commercial Banking Corporation in the amount of
P2,000,000.00 and executed a "Real Estate Mortgage" over the subject] property as security therefor.

Gaspar, Visitacion, Flor, Pedro and Aurelio, Jr. learned of the sale and they wrote a letter to the Register of
Deeds of Northern Samar, saying that they were not informed of the sale of a portion of the said prop erty by their
mother nor did they give their consent thereto, and requested to: hold in abeyance any processal or approval of any
application for registration of title of ownership in the name of the buyer of said lot, which has not yet been
partitioned judicially or extrajudicially, until the issue of the legality/validity of the above sale has been cleared.

Esperanza signed a letter addressed to Rodrigo informing the latter that her children did not agree to the
sale of the property to him and that she was withdrawing all her commitments until the validity of the sale is finally
resolved.

ISSUE

Whether or not the Sale was that of an Equitable Mortgage

HELD

No, Deed of Sale not an Equitable Mortgage! Petitioner further posits that even assuming that the d eed of sale is
valid it should only be deemed an equitable mortgage pursuant to Articles 1602 and 1604 of the Civil Code, because
the price was clearly inadequate. They add that the presence of only one of the circumstances enumerated under
Article 1602 would be sufficient to consider the Contract an equitable mortgage. We disagree.

For Articles 1602 and 1604 to apply, two requisites must concur: one, the parties entered into a contract
denominated as a contract of sale and, two, their intention was to secure an existing debt by way of mortgage.

2B 16-17 SALES AND LEASE Page 429


Indeed, the existence of any of the circumstances enumerated in Article 1602, not a concurrence or an

overwhelming number thereof, suffices to give rise to the presumption that a contract purporting to be an absolute
sale is actually an equitable mortgage.

In the present case, however, the Contract does not merely purport to be an absolute sale. The records and
the documentary evidence introduced by the parties indubitably show that the Contract is, indeed, one of absolute
sale. There is no clear and convincing evidence that the parties agreed upon a mortgage of the subject property.

Furthermore, the voluntary, written and unconditional acceptance of contractual commitments negates the
theory of equitable mortgage. There is nothing doubtful about the terms of, or the circumstances surrounding, the
Deed of Sale that would call for the application of Article 1602. The Joint Affidavit indisputably confirmed that the
transaction between the parties was a sale.

When the words of a contract are clear and readily understandable, there is no room for construction.
Contracts are to be interpreted according to their literal meaning and should not be interpreted beyond their obvious
intendment. The contract is the law between the parties.

Notably, petitioners never raised as an issue before the trial court the fact that the document did not express
the true intent and agreement of the contracting parties. They raised mere suppositions on the inadequacy of the
price, in support of their argument that the Contract should be considered as an equitable mortgage.

2B 16-17 SALES AND LEASE Page 430


7. NARCISA AVILA, assisted by her husband Bernardo Avila, Spouses JANUARIO N. ADLAWAN
and NANETTE A. ADLAWAN, NATIVIDAD MACAPAZ, assisted by her husband EMILIO MACAPAZ,
FRANCISCA N. ADLAWAN and LEON NEMEO VS Spouses BENJAMIN BARABAT and JOVITA
BARABAT

G.R. No. 141993 March 17, 2006

Corona, J

FACTS:

A parcel of land in Cebu is registered in the name of Anunciacion Bahena vda de Nemeo. Upon her deat h,
ownership was transferred to her five children, the petitioners in this case. These heirs built respective houses on the
lot. Respondent Barabat leased a portion of the house owned by Avila, his wife moved in with him after their
marriage. Avila relocated to CDO and decided to sell her hose and share in the lot. Respondents agreed to buy it,
evidenced by a private document. Respondents stopped paying rent and took possession of the property as owners
and assumed the payment of realty taxes. The spouses received a letter informing them that Avila sold the house and
share in the lot to the spouses Adlawan. The spouses demanded that Avila execute a public document evidencing the
sale of the property to them, but Avila refused.

Respondents filed complaint for quieting of title with the RTC. RTC rendered judgment in favor of respondents and
nullified subsequent sale to respondents. Petitioners appealed to the CA. CA affirmed the decision of the RTC in
toto. They filed a motion for reconsideration but it was denied.

ISSUE:

Whether it was an equitable mortgage between Avila and the Spouses Barabat

HELD: NO. Art 1602 and 1604 provides for contracts of equitable mortgage. For these articles to apply, 2 requisites
must concur: (1) the parties entered into a contract denominated as a contract of sale and (2) their intention was to
secure an existing debt by way of mortgage. The private document evidenced a contract of sale. In the absence of
evidence as to the fair market value of a parcel of land at the time of its sale, we cannot reasonably conclude that the
price at which it was sold was inadequate. Art 1620 and 1623, the right of redemption of co -owners cannot apply in
this case because at the time of conveyance, the co-ownership had already been extinguished by partition. By their
own admission, petitioners already segregated and took possession of their respective shares in the lot. Their
respective shares were therefore physically determined, clearly identifiable and no longer ideal. Thus, the co -
ownership had been legally dissolved. With that, petitioners right to redeem any part of the property from any of
their former co-owners was already extinguished. CA decision affirmed

2B 16-17 SALES AND LEASE Page 431


ON ASSIGNMENT OF CREDITS AND OTHER INCORPOREAL RIGHTS

1. THE MANILA BANKING CORPORATION, VS ANASTACIO TEODORO, JR. and GRACE ANNA
TEODORO

G.R. No. L-53955 January 13, 1989

Bidin, J

FACTS:

On April 25, 1966, defendants, together with Anastacio Teodoro, Sr., jointly and severally, executed in favor of
plaintiff a Promissory Note (No. 11487) for the sum of P10,420.00 payable in 120 days, or on August 25, 1966, at
12% interest per annum. Defendants failed to pay the said amount inspire of repeated demands and the obligation as
of September 30, 1969 stood at P 15,137.11 including accrued interest and service charge. On May 3, 1966 and June
20, 1966, defendants Anastacio Teodoro, Sr. (Father) and Anastacio Teodoro, Jr. (Son) executed in favor of plaintiff
two Promissory Notes (Nos. 11515 and 11699) for P8,000.00 and P1,000.00 respectively, payable in 120 days at
12% interest per annum. Father and Son made a partial payment on the May 3, 1966 promissory Note but none on
the June 20, 1966 Promissory Note, leaving still an unpaid balance of P8,934.74 as of September 30, 1969 including
accrued interest and service charge.

The three Promissory Notes stipulated that any interest due if not paid at the end of every month shall be added to
the total amount then due, the whole amount to bear interest at the rate of 12% per annum until fully paid; and in
case of collection through an attorney-at-law, the makers shall, jointly and severally, pay 10% of the amount over-
due as attorney's fees, which in no case shall be leas than P200.00. It appears that on January 24, 1964, the Son
executed in favor of plaintiff a Deed of Assignment of Receivables from the Emergency Employment
Administration in the sum of P44,635.00. The Deed of Assignment provided that it was for and in consideration of
certain credits, loans, overdrafts and other credit accommodations exten ded to defendants as security for the
payment of said sum and the interest thereon, and that defendants do hereby remise, release and quitclaim all its
rights, title, and interest in and to the accounts receivables.

In their stipulations of Fact, it is admitted by the parties that plaintiff extended loans to defendants on the basis and
by reason of certain contracts entered into by the defunct Emergency Employment Administration (EEA) with
defendants for the fabrication of fishing boats, and that the Philip pine Fisheries Commission succeeded the EEA
after its abolition; that non-payment of the notes was due to the failure of the Commission to pay defendants after
the latter had complied with their contractual obligations; and that the President of plaintiff Bank took steps to
collect from the Commission, but no collection was effected.

For failure of defendants to pay the sums due on the Promissory Note, an action against Spouses Teodoro (son and
his wife) was filed. On April 17, 1972, the trial court rendered its judgment adverse to defendants.

ISSUE:

Whether or not the assignment of receivables has the effect of payment of all the loans contracted by the spouses?

2B 16-17 SALES AND LEASE Page 432


HELD:

NO. It is evident that the assignment of receivables executed by appellants on January 24, 1964 did not transfer the
ownership of the receivables to appellee bank and release appellants from their loans with the bank incurred under
the three promissory notes.

The Deed of Assignment provided that it was for and in consideration of certain cred its, loans, overdrafts, and their
credit accommodations in the sum of P10,000.00 extended to appellants by appellee bank, and as security for the
payment of said sum and the interest thereon; that appellants as assignors, remise, release, and quitclaim to assignee
bank all their rights, title and interest in and to the accounts receivable assigned. It was further stipulated that the
assignment will also stand as a continuing guaranty for future loans of appellants to appellee bank and
correspondingly the assignment shall also extend to all the accounts receivable; appellants shall also obtain in the
future, until the consideration on the loans secured by appellants from appellee bank shall have been fully paid by
them.

Definitely, the assignment of the receivables did not result from a sale transaction. The deed of assignment was
intended as collateral security for the bank loans of appellants, as a continuing guaranty for whatever sums would be
owing by defendants to plaintiff.

2B 16-17 SALES AND LEASE Page 433


2. BENJAMIN RODRIGUEZ VS CA

G.R. No. 84220, March 25, 1992

FACTS

Petitioner Rodriguez alias Uy Tian Kiu is a businessman from Cebu City whose business, includes the importation
of various commodities from Hongkong which he occasionally ordered from Allied Overseas Commercial Co., Ltd.,
a Hongkong corporation. The Managing Director of Allied Overseas Commercial Co., Ltd. is Lin Ping Huang, a
close friend of private respondent Lucman. Petitioner Rodriguez, as a result of business transactions with the
Hongkong Corporation, accumulated an indebtedness owed to Allied Overseas in the amount of HK $418,729.60
which had at that time in 1968 an exchange value of P540,553.00.Upon demand for payment by the Hongkong
Corporation, the petitioner issued a pay-to-cash check dated September 11, 1970 covering the indebtedness. The
check was, however, dishonored for lack of funds, the account having been closed two months earlier.Subsequently,
the Allied Overseas Commercial Co., Ltd., through its Managing Director, Lin Ping Huang, assig ned its credit to the
private respondent. The contract was evidenced by a Deed of Assignment duly executed before Philippine Consular
officials in Hongkong. The assignee then filed a suit to collect the indebtedness. The trial court ruled in its favor.
Rodriguez thereafter appealed before the CA. The CA however dismissed the same.

ISSUE

Whether or not the decision of the CA is not in accord with Article 1301 of the NCC which requires consent in
subrogation

HELD

There is no merit in petitioners contention that the obligation herein is subrogation instead of assignment of credit.

The basis of the complaint is not a deed of subrogation but an assignment of credit whereby the private respondent
became the owner, not the subrogee of the credit since the assignment was supported by HK$ 1.00 and other
valuable considerations.

The questioned deed of assignment is neither one of the subrogation nor a power of attorney as the petitioner
alleges. The deed of assignment clearly states that the private respondent became an assignee and, therefore, he
became the only party entitled to collect the indebtedness. As a result of the Deed of Assignment, the plaintiff
acquired all rights of the assignor including the right to sue in his own name as the legal assignee. Moreover, in
assignment, the debtor's consent is not essential for the validity of the assignment (Art. 1624 in relation to Art. 1475,
Civil Code), his knowledge thereof affecting only the validity of the payment he might make (Article 1626, Civil
Code).

Article 1626 also shows that payment of an obligation which is already existing does not depend on the consent of
the debtor. It, in effect, mandates that such payment of the existing obligation shall already be made to the new
creditor from the time the debtor acquires knowledge of the assignment of the obligation.

The law is clear that the debtor had the obligation to pay and should have paid from the date of notice whether or not
he consented.

2B 16-17 SALES AND LEASE Page 434


The petitioner further assails the consideration given for the deed of assignment which is stated as "HK$ 1.00 and
other valuable considerations."

A valuable considerations, however small or nominal if given or stipulated in good faith is, in the absence of fraud,
sufficient. A stipulation in consideration of $1 is just as effectual and valuable a consideration as a larger sum
stipulated for or paid (Penaco v. Ruaya, 110 SCRA 46 [1981]; Ascalon vs. Court of Appeals, 158 SCRA 542,
[1988]). It is not clear what considerations led to the assignment but they must have been sufficiently valuable to the
assignor in view of the amount involved.

Hence, by virtue of the deed of assignment whose existence and legality remains unrebutted, the respondent
acquired all the rights of the assignor including the right to sue in his own name as the legal assignee. The contract
was not executed merely to enable the foreign corporation to sue in the Philippines because even without the
assignment, the foreign corporation can also sue in the Philippines for isolated transactions even if not licensed to
engage in business in this country.

NOTE

The Deed of Assignment reads:

That WE, the ALLIED OVERSEAS COMMERCIAL CO., LTD., a commercial association duly organized and
registered in the company's registry of the Crown Colony of Hongkong with offices at No. 5-7 Des Voeux Road,
West, 1st Floor, Hongkong, represented in this instance by its Managing Director duly authorized by a Board
resolution, for and in consideration of HK$ 1 and other valuable considerations, have on this date assigned, ceded,
transferred and conveyed by way of irrevocable assignment and transferred to Hadji Esmayaten Lucman, Esq., of
legal age, Filipino citizen, and a resident of No. 95-I, A. Lake St., San Juan, Province of Rizal, Republic of the
Philippines, our outstanding and collectible credit due and owing us by and from Benjamin Uy Rodriguez alias Uy
Tian Kiu of Cebu City, Republic of the Philippines, in the total amount of HK$ 418,729.60 or its equivalent in the
Philippine Currency, for said Hadji Esmayaten Lucman to collect and secure from the aforesaid debtor, Benjamin
Uy Rodriguez alias Uy Tian Kiu the aforesaid amount in any manner, including court proceedings if necessary, in
accordance with the provisions of existing laws in the jurisdiction of the Republic of the Philippine s.

We, as creditors assignors of the aforesaid debt, have on this date notified formally the debtor named herein of this
full assignment of the aforesaid credit.

2B 16-17 SALES AND LEASE Page 435


3. PNB VS GLORIA VDA DE ONG ACERO

GR No. L-69255 February 27, 1987

Narvasa, J.

FACTS

Savings account of Isabela Wood and Construction Development Corporation in the amount of P2,000,000 opened
with PNB on March 9, 1979 was subject to conflicting claims. One from the respondents who based their claims on
the execution of a partial judgment rendered by CFI at Quezon City on Nov. 18, 1979. The partial judgment ordered
Isabela to pay respondents the amount of P1,532,000.07, notice of garnishment was sent to PNB on January 9, 1980.
On the other hand, PNB claims that the deposit was made by Isabela as collateral to a debt in favor of PNB where
Isabela bound itself to pay and to deliver its Paranaque property as mortgage, failure to comply to which will allow
PNB to apply the deposit to the loan.

PNB moved for the reconsideration of the court Order in favor of Aceros but was denied. PNB moved for another
reconsideration which was granted. The trial part ruled in favor of PNB saying that there was a valid assignment of
credits to the latter. PNB claims that when Isabela opened an account with it, PNB become indebted to Isabela,
however, due to the subsequent breach by Isabela of their credit agreement the latter also become the debtor of PNB.
There being compensation, the credit was beyond the reach of the Aceros.

Aceros appealed to IAC which sustained them.

ISSUE

Whether automatic compensation has taken place thus making the credit beyond the reach of the Aceros.

HELD

There was no legal compensation between Isabela and PNB.

Article 1278 of the Civil Code states that compensation shall take place when two persons, in their own right, are
creditors and debtors of each other. This can take place when all the requisites under Article 1279 are met.

In the case at bar, PNB failed to prove that it is actually a creditor of Isabela. Evidence only show the possibility that
a letter of credit was opened for Isabela by PNB but it does not show that this was actually availed of or that the
goods covered thereby were actually shipped. Neither did PNB present the necessary documents to prove its claim.

An alternative theory that the credits were assigned to PNB by Isabela to be applied to an existing debt will not
prosper, too. The being no indebtedness by Isabela in favor of PNB there is no compensation to speak of or any

2B 16-17 SALES AND LEASE Page 436


mutual set-off. Even assuming that an assignment was made it was preceded by the notice of garnishment in favor of
the Aceros, thus, PNB can no longer claim the subject credit.

2B 16-17 SALES AND LEASE Page 437


4. SERVICEWIDE SPECIALISTS VS CA

G.R. No. 116363 December 10, 1999

Ynares-Santiago, J

FACTS
Spouses Ponce bought on installment a Holden Torana vehicle from C.R. Tecson Enterprise, and they executed a
promissory note and a chattel mortgage on the car (registered mortgage). Later, C.R. Tecson Enterprise executed a
Deed of Assignment of the promissory note and mortgage to petitioner Filinvest Credit Corporation with consent of
Spouses Ponce. In 1976, Spouses Ponce executed a sale with assumption of mortgage with Conrado R. Tecson. In
1978, Filinvest assigned all rights over the promissory note and chattel mortgage to petitioner.

Petitioner filed a replevin with damages against the Spouses Ponce for failure to pay the installments. In their
defense, the couple stated that they are no longer liable for they have sold and transferred the car to Conrad Tecson,
who they later impleaded as third party defendant.

ISSUES

(1) Whether or not consent of the debtor-mortgagor is necessary before assignment of credit by creditor-
mortgagee, such that he can no longer be held liable if not notified
(2) Whether or not consent of the creditor-mortgagee is necessary before the debtor-mortgagor alienates the
property to a third person

HELD

(1) No, only notice to the debtor of the assignment of credit is required; his consent is not required. Only notice
to but not the consent of the debtor-mortgagor was necessary to bind the latter.
(2) Yes, consent of the creditor-mortgagee to the alienation of the mortgaged property is necessary in order to
bind the said creditor. The sale with assumption of mortgage made by the Spouses is tantamount to
novation (substitution of debtors) which would require the consent of Filinvest Credit Corp in order for
such alienation to bind him, and his successor petitioner Servicewide Specialist Inc who is deemed to have
stepped into latters shoes. Therefore, the Supreme Court held that Spouses Ponce are still liable to pay
petitioner

2B 16-17 SALES AND LEASE Page 438


5. C & C COMMERCIAL CORPORATION VS PNB

G.R. No. 42449, July 5, 1989

Cortes, J

FACTS:

C & C Commercial Corporation (now Asbestos Cements Products Phil. Inc., hereinafter referred to as
ACPPI) opened seven letters of credit with the Philippine National Bank to import machines and equipment for its
plant. Since C & Cs obligations were not paid, PNB instituted a collection suit with a prayer for prelimin ary
injunction against ACPPI. Instead of proceeding with the collection suit, PNB agreed to enter into a Voting Trust
Agreement on March 5, 1969 to protenct PNBs interest in ACPPI. Thereafter, PNB and its subsidiary or affiliate,
the National Investment Development Corporation, as the trustees named under the said agreement, immediately
proceeded to take over the management of ACPPI. On August 27, 1973, the accounting firm of Sycip, Gorres and
Velayo, after examining the management and operations of ACPPI for the first three years under the Voting Trust
Agreement, submitted a report finding that the PNB/NIDC management of ACPPI was a complete and disastrous
failure. In view of this report, ACPPI then filed a complaint in CFI Rizal for the termination of the Voting Trust
Agreement with a prayer for an award of damages by reason of the grossly negligent or incomplete management of
PNB leading to ACPPI to suffer huge losses. In the meantime, DBP executed a deed of assignment in favor of PNB
whereby the former assigned to the latter its rights and interests under the promissory notes and deeds of real estate
mortgages executed by ACPPY in for of DBP. On March 11, 1974, PNB filed a petition for foreclosure sale. ACPPI
then filed a motion to nullify the extrajudicial foreclosure proceedings which was later denied.

ISSUE:

Whether or not PNB may foreclose the properties which were assigned to them by the DBP.

HELD:

YES, there is no doubt that foreclosure can proceed as these were secured by appropriate mortgages. Moreover,
contrary to petitioners pretensions, the validity of the assignment of the mortgage credit by DBP to PNB is beyond
question. Article 1624 of the Civil Code provides that an assignment of credits and other incorporeal rights shall be
perfected in accordance with the provisions of Article 1475 which in turn states that the contract of sale is
perfected at the moment there is a meeting of the minds upon the thing which is the object of the contract and upon
the price. The meeting of the minds contemplated here is that between the assignor of the credit and his assignee,
there being no necessity for the consent of the debtor, contrary to petitioners claim. It is sufficient that the
assignment be brought to his knowledge in order to be binding up on him. This may be inferred from Article 1626 of
the Civil Code which declares that the debtor who, before having knowledge of the assignment, pays his creditor
shall be released from the obligation.

2B 16-17 SALES AND LEASE Page 439


6. RAUL SESBREO VS HON. COURT OF APPEALS, DELTA MOTORS CORPORATION and
PILIPINAS BANK

G.R. No. 89252. May 24, 1993

Feliciano, J

FACTS:

On 9 February 1981, petitioner Raul Sesbreo made a money market placement in the amount of P300,000.00 with
the Philippine Underwriters Finance Corporation (Philfinance), Cebu Branch the placement, with a term of thirty-
two (32) days, would mature on 13 March 1981. Philfinance, also on 9 February 1981, issued the following
documents to petitioner:

(a) the Certificate of Confirmation of Sale, without recourse, No. 20496 of one (1) Delta Motors
Corporation Promissory Note (DMC PN) No. 2731 for a term of 32 days at 17.0% per annum

(b) the Certificate of Securities Delivery Receipt No. 16587 indicating the sale of DMC PN No. 2731 to
petitioner, with the notation that the said security was in custodianship of Pilipinas Bank, as per
Denominated Custodian Receipt (DCR) No. 10805 dated 9 February 1981 and

(c) postdated checks payable on 13 March 1981 (i.e., the maturity date of petitioners investment), with pet itioner
as payee, Philfinance as drawer, and Insular Bank of Asia and America as drawee, in the total amount of
P304,533.33.

On 13 March 1981, petitioner sought to encash the postdated checks issued by Philfinance. However, the checks
were dishonored for having been drawn against insufficient funds.

Petitioner made Demand Letters for physical delivery of the note to no avail.

Petitioner also made a written demand on 14 July 1981 upon private respondent Delta for the partial satisfaction of
DMC PN No. 2731, explaining that Philfinance, as payee thereof, had assigned to him said Note to the extent of
P307,933.33. Delta, however, denied any liability to petitioner on the promissory note, and explained in turn that it
had previously agreed with Philfinance to offset its DMC PN against Philfinance issued in favor of Delta.

As Petitioner failed to collect his investment and interest he filed an action for damages with RTC Cebu which
dismissed the case.

The CA denied the appeal holding that if there is anyone that appears liable for the travails of plaintiffappellant, it is
Philfinance.

ISSUE:

Whether or not petitioner is a holder who can sue on the instrument and demand or receive payment by virtue of his
assignment.

HELD:

2B 16-17 SALES AND LEASE Page 440


Yes. Petitioner admits that DMC PN No. 2731 was non negotiable but contends that that Note had been validly
transferred, in part, to him by assignment and that as a result of such transfer, Delta as debtormaker of the Note, was
obligated to pay petitioner the portion of that Note assigned to h im by the payee Philfinance.

Defendant claims that the assignment of DMC PN No. 2731 by Philfinance was without Deltas consent, if not
against its instructions; and assuming (arguendo only) that the partial assignment in favor of petitioner was valid,
petitioner took that Note subject to the defenses available to Delta, in particular, the offsetting of DMC PN No. 2731
against Philfmance PN No. 143 A.

A nonnegotiable instrument may, obviously, not be negotiated; but it may be assigned or transferred, abse nt an
express prohibition against assignment or transfer written in the face of the instrument:

The words not negotiable, stamped on the face of the bill of lading, did not destroy its assignability, but the sole
effect was to exempt the bill from the s tatutory provisions relative thereto, and a bill, though not negotiable, may be
transferred by assignment the assignee taking subject to the equities between the original parties.

DMC PN No. 2731, while marked nonnegotiable, was Deltas complaint that the partial assignment by
Philfinance of DMC PN No. 2731 had been effected without the consent of Delta, we note that such consent was not
necessary for the validity and enforceability of the assignment in favor of petitioner.Deltas argument that
Philfinances sale or assignment of part of its rights to DMC PN No. 2731 constituted conventional subrogation,
which required its (Deltas) consent, is quite mistaken. Conventional subrogation, which in the first place is never
lightly inferred,must be clearly established by the unequivocal terms of the subtituting obligation or by the evident
incompatibility of the new and old obligations on every point. Nothing of the sort is present in the instant case not at
the same time stamped nontransferrable or non assignable. It contained no stipulation which prohibited
Philfinance from assigning or transferring, in whole or in part, that Note.

We turn to Deltas arguments concerning alleged compensation or offsetting between DMC PN No. 2731 and
Philfinance PN No. 143A. It is important to note that at the time Philfinance sold part of its rights under DMC PN
No. 2731 to petitioner on 9 February 1981, no compensation had as yet taken place and indeed none could have
taken place. The essential requirements of compens ation are listed in the Civil Code as follows:

Art. 1279. In order that compensation may be proper, it is necessary:

(1) That each one of the obligors be bound principally, and that he be at the same time a principal creditor
of the other;

(2) That both debts consist in a sum of money, or if the things due are consumable, they be of the same
kind, and also of the same quality if the latter has been stated;

(3) That the two debts are due;s

(4) That they be liquidated and demandable;

(5) That over neither of them there be any retention or controversy, commenced by third persons and
communicated in due time to the debtor.

The assignment to petitioner was made on 9 February 1981 or from forty nine (49) days before the co terminal
maturity date, that is to say, before any compensation had taken place. Further, the assignment to petitioner would
have prevented compensation from taking place between Philfinance and Delta, to the extent of P304,533.33,
because upon execution of the assignment in favor of petitioner, Philfinance and Delta would have ceased to be

2B 16-17 SALES AND LEASE Page 441


creditors and debtors of each other in their own right to the extent of the amount assigned by Philfinance to
petitioner. Thus, we conclude that the assignment effected by Philfinance in fav or of petitioner was a valid one and
that petitioner accordingly became owner of DMC PN No. 2731 to the extent of the portion thereof assigned to him.

Petitioner notified Delta of his rights as assignee after compensation had taken place by operation of la w because the
offsetting instruments had both reached maturity. It is a firmly settled doctrine that the rights of an assignee are not
any greater than the rights of the assignor, since the assignee is merely substituted in the place of the assignor and
that the assignee acquires his rights subject to the equities i.e., the defenseswhich the debtor could have set up
against the original assignor before notice of the assignment was given to the debtor.

Petitioner could, in fine, have notified Delta at any time before the maturity date of DMC PN No. 2731. Because
petitioner failed to do so, and because the record is bare of any indication that Philfinance had itself notified Delta of
the assignment to petitioner, the Court is compelled to uphold the defense o f compensation raised by private
respondent Delta. Of course, Philfinance remains liable to petitioner under the terms of the assignment made by
Philfinance to petitioner.

Private respondent Pilipinas must respond to petitioner for damages sustained by him arising out of its breach of
duty. By failing to deliver the Note to the petitioner as depositorbeneficiary of the thing deposited, Pilipinas
effectively and unlawfully deprived petitioner of the Note deposited with it.

2B 16-17 SALES AND LEASE Page 442


LEASE

NATURE AND EFFECTS OF A CONTRACT OF LEASE

1. INMA ROHDE SHOTWELL SHOTWELL vs. MANILA MOTOR CO., INC. and CHARTERED
BANK OF INDIA, AUSTRALIA AND CHINA,

G.R. No. L-7637, December 29, 1956

FACTS

William Rhode, now deceased, father of the plaintiff and the Manila Moto r Co., Inc., entered into a contract of lease
for 14 years. Thereafter, the Manila Motor Co., Inc. assigned, with the express consent of William H. Rohde (Exh.
B), said lease to the defendant Chartered Bank of India, Australia and China under the following reservations: 1)
Chartered Bank of India, Australia and China has the option at any time to undertake and assume the said liabilities
and obligations of Manila Motors; 2) That this assignment shall in no way absolve or release the Manila Motor Co.,
Inc. from the obligations and liabilities it has contracted in said lease.

Inma Rohde Shotwell, successor-in-interest of the late William H. Rohde, the lessor, sued the lessee and the
assignee in the CFI Manila to recover from them allegedly due and unpaid rent als from 1 January 1942 to 30
November 1945

The petitioner contends that the destruction of the buildings has not terminated the lease contract because not only
the buildings but also the land on which they had been erected formed part of the consideration or causa of the
contract of lease.

ISSUE

Whether or not the destruction of the building of the leased property caused the termination of the lease

HELD

The Supreme Court affirmed the ruling of the Court of Appeals.

The Court of Appeals found that there were buildings on the land when the contract of lease (Exhibit A) was entered
into between William H. Rohde and the Manila Motor Co. Inc., and that the reason the latter entered into such
leased contract was "that these buildings were substantial and the improvements were suitable for the business in
which we are going into, selling and repairing automobiles;" and both the trial and appellate Courts are of the
opinion "that said defendant would not have entered into the contract were it not for the suitableness of those

2B 16-17 SALES AND LEASE Page 443


buildings which it could and did use in its automobile business, and that the conditions of the lease would have been
different if there were no such buildings thereon."

The destruction of the buildings during the battle for liberation in February 1945 terminated the lease contract.
Article 1568 of the Civil Code provides:

If the thing leased should be lost or either of the contracting parties fails to comply with his undertaking, the
provisions of Articles 1182 and 1183 and of Articles 1101 and 1124 respectively shall be observed.

Article 1182 of the same Code provides:

An obligation which consists in the delivery of a determinate thing shall be extinguished if such thing should be lost
or destroyed without fault on the part of the debtor and before he is in default (mora).

The buildings and the land constitute an invisible unit and the destruction of the buildings has extinguished the
obligation or terminated the lease contract. The stipulation on the rebuilding of the destroyed improvemen ts is
potestative on the part of the lessee. The latter, is not bound to do so.

2B 16-17 SALES AND LEASE Page 444


2. FLORENTINO vs. SUPERVALUE, Inc.

GR No. 172384 Sep. 12, 2007

Chico-Nazario, J.:

FACTS

Petitioner is sole proprietor of Empanada Royale, engaged in the retail sale of empanada in malls in Metro Manila.
Respondent is engaged in the leasing of stalls in SM Malls throughout the country. On Mar. 9, 1999, they executed
contracts of lease over cart-type stalls in SM North Edsa and SM Southmall, and a store space in SM Megamall.
Each contract was for 4 months and renewable by the parties. These contracts were renewed to last until Mar. 31,
2000. On Feb. 4, 2000, petitioner received 2 letters from respondent, saying that petitioner violated two provisions
of the contract, by selling a new variety of empanada called mini-embutido, and by closing earlier than mall hours
due to non-delivery or delay in the delivery of stocks. Despite petitioner's explanation, respondent chose not to
renew the contract. Respondent took possession of and confiscated the personal belongings of the petitioner. Despite
petitioner's demands for a return of the property, and for a return of the P192,000 paid as security deposit,
respondent did not comply. On Aug. 17, 2000, petitioner filed an action for Specific Performance, Sum of Money
and Damages. Respondent argued that it was justified because petitioner committed several breaches of contract,
and that it was only exercising its retaining lien. The RTC of Makati decided in favor of petitioner, while the CA
modified the judgment, holding that the respondent had a right to the forfeiture of the deposit. Moreover, the CA
held that the respondent cannot be made to reimburse the petitioner for improvements made, because they were done
without the respondent's consent.

ISSUE

Whether or not respondent can be made to reimburse petitioner for improvements made on the space leased.

HELD

No, respondent cannot be held to reimburse. Article 1678, which mentions that the lessee is entitled to be
reimbursed for of the value of the improvements made, shall be read in consonance of Articles 448 and 546 of the
same Civil Code. Said articles require that the lessee be a builder in good faith. In this case, the petitioner cannot be
considered a builder in good faith, because lessees cannot be considered builders in good faith. To consider a lessee
so would allow him to improve the landlord out of his property.

2B 16-17 SALES AND LEASE Page 445


3. BONIFACIO NAKPIL vs. MANILA TOWERS DEVELOPMENT CORPORATION

G.R. No. 160867, September 20, 2006

Callejo Sr, J

FACTS

A 14-storey building owned by Cheong Kiang Ang was leased to about 200 Filipino Chinese tenants including
Bonifacio Nakpil who leased a unit for a law office. The tenants of the building later formed the House International
Building Tenants Association, Inc. (HIBTAI). The property was mortgaged to GSIS and later foreclosed and sold to
Manila Towers Dev. Corp. (MTDC). The HIBTAI protested and alleged that they had priority to buy the property.
They refused to pay their rentals.

The City Engineer wrote to MTDC to repair the building because the defects might endanger the lives of tenants.
But before MTDC could make the necessary repairs, the HIBTAI filed a complaint against GSIS for injunction and
damages. Court dismissed the complaint.

8 years later, the building administrator wrote to the City Engineer of Manila to conduct ocular inspection again to
determine its safety. The letter mentioned that, as far back as 1981, the City Engineer and Building Official had
ordered the building condemned after inspection. He stated that when MTDC was about to make repairs, the tenants
filed several suits and during the pendency of the case, made illegal structures without MTDCs consent.

Upon finding that the building structure is in danger, the City Building Official wrote a letter to the building
administrator, ordering him to cause the tenants to vacate the building and undertake the necessary repairs and
rehabilitation of the building. MTDC did not respond. Notices were sent to the tenants, giving them fift een (15) days
within which to vacate the building to give way to its general repair.14 However, at the time, Atty. Nakpil was in the
United States for medical treatment, and his secretary was left behind to take care of the law office.

The President of HIBTAI, Ong, and the Brgy. Captain filed for a TRO to enjoin respondents from conducting
repairs. A group of men entered the building and started doing repairs as per the order of the City Mayor but was
later temporarily suspended because of the TRO.

Upon his arrival in the Philippines, Atty. Nakpil filed a complaint against the MTDC, seeking for actual, moral, and
exemplary damages, attorney's fees, litigation expenses, costs of suit and other reliefs because his room was
destroyed and his things stolen or scattered. He said he had been renting and complying with the conditions of the
lease since 1965. The MTDC violated his right as lessee to the possession of the premises, unlawfully depriving him
of said possession without any lawful authority or court ord er. The RTC dismissed this.

2B 16-17 SALES AND LEASE Page 446


HELD

Whether or not the MTDC is liable for actual, moral and exemplary damages to Nakpil.

HELD

No. What is evident in the present case is that the disturbance on the leased premises on July 19, 1996 was actually
done by the employees under the City Engineer of Manila and the City Building Official on orders of the City
Mayor without the participation of the MTDC. It bears stressing that the City Building Official is authorized and
mandated under Section 214 of the National Building Code to order the repair, maintenance or demolition of the
building found or declared to be dangerous or ruinous, depending upon the degree of danger to life, health, safety
and/or well-being of the general public and its occupants.

The City Building Official was tasked merely to repair/rehabilitate the building and not to demolish the same and
cause the placement eviction of the tenants. Neither did respondent abandon the leased premises. Admittedly, the
MTDC failed to make the necessary repairs in the building despite requests of the City Building Official as early as
June 29, 1981 and July 10, 1981. However, the MTDC cannot be faulted for such failure. No less than the HIBTAI
or its members prevented MTDC from instituting the necessary repairs. Even Villanueva, Nakpil's witness, admitted
that HIBTAI objected to the orders of the City Building Official for the repair of the building.

2B 16-17 SALES AND LEASE Page 447


4. LUZ J. HENSON vs. THE INTERMEDIATE APPELLATE COURT, ELY FUDERANAN and
LUISA COMMENDADOR

G.R. No. 72456 February 19, 1987

Guiterrez, JR.,J.

FACTS

Luz Henson is an owner of a building in Ermita, Manila wherein she leases out office spaces. The lessee in this case
was designated as Sto. Nio Travel and Tour Agency, a sole proprietorship duly organized un der the laws of the
Philippines represented by herein respondent ELY FUDERANAN, the president and general manager. Henson
received the sum of 8000 Php as a reservation deposit for a certain apartment in her building for which she issued
a receipt to Fuderanan reiterating that the reservation is only good up to May 15, 1980 at 4PM, failure to sign the
Lease Contract, pay the required three months advance rental and 3 months guarantee deposits, the reservation
would be forfeited. Monthly rental is at 2000 php.

On the same day, Henson and Fuderanan entered into the LEASE CONTRACT which has the following
stipulations:

1. The contract shall have a duration of one year commencing from May 15, 1980 and shall be deemed
renewed on a month to month basis, unless one month before the expiration of the contract, one party
informs the other in writing of his desire not to be bound anymore. In case of termination, the guarantee
deposit will be forfeited and lessee shall be lialble for damages.
2. Lessee agrees to pay the rentals at 2000 Php a month, due and payable without need of further demand and
notice at the lessors office or residence. 6000 pesos shall be paid in advance as rentals for the first two
months. Rentals are payable monthly in advance
3. Upon execution of the contract, the lessee will deposit with the lessor 6000 pesos, three months rental. This
deposit shall answer for any damages, losses, breakage, utilities destroyed, etc.

Pursuant to the lease contract, Fuderanan paid Henson the amount of 6000 pesos in cash as deposit for rentals, water
service, and four keyes, 1660 pesos in cash and 4640 in a PDC. The PDC was later replaced by another PDC of
Luisa Commendador which was dishonored due to insufficiency of funds. However, the MINISTRY OF TOURISM
disapproved the request of Fuderanan to transfer their office to the premises owned by Henson as the place failed to
meet the minimum 50 sqm. Requirement of the ministry. Fuderanan informed Henson in writing that they had to
vacate the leased premises due to the disapproval. Henson notified Fuderanan in writing of the dishonor of
Commendadors check and asked them to make good their dishonored check. However, Fuderanan, in turn, replied
by stating that they had to rescind the lease contract and requested the refund of th e amounts they paid by way of
advance and deposit rentals less the amount rental due. Henson filed an action against Fuderanan to recover the
value of the dishonored check.

ISSUE

2B 16-17 SALES AND LEASE Page 448


Whether or not Ely Fuderanan should pay the rentals stipulated in the contract notwithstanding the disproval of the
Ministry of Tourism

HELD

YES. Contracts are respected as the law between the contracting parties. The Lease Contract executed by Henson
and Fuderanan remains the law as between them. In litigations involving the ad judication of rights and obligations
between the lessor and lessee, the lease contract shall govern. The disputed lease contract is plain and unequivocal in
its terms. The stipulations are expressed in clear and explicit language that leaves NO DOUBT as to the intention of
the contracting parties. Nowhere is it provided in the contract that the fulfillment of the terms and conditions of the
lease depend on an act of a third party (final action of the Ministry of Tourism). Neither is there any indication fro m
the evidence presented that, would justify either of the contracting parties to impugn the lease contract they
executed. Contracts are to be interpreted according to their literal meaning when the terms and conditions are clear
and leave no doubt as to the intention of the contracting parties. The predicament of Fuderanan is entirely of his own
making. He should have ascertained all the rules and requirements for the operation of a travel agency before it even
started to look for premises to house its office. Henson had absolutely nothing to do with Fuderanans violating the
requirements.

2B 16-17 SALES AND LEASE Page 449


5. CERTIFIED CLUBS, INC. v COURT OF APPEALS, INSTITUTIONAL FOOD SERVICES, INC

GR No. L 47695 May 7, 1987

Melencio Herrera, J.

FACTS:

In this Petition for Review on Certiorari, petitioner seeks a reversal of the Decision of respondent Court of
Appeals affirming, with modification, the judgment of the Court of First Instance of Manila, Branch XIX, awarding
damages to private respondent.

Petitioner, Certified Clubs, Inc., is the lessee of the building known as the VIP Building, located at the comer of
Plaza Ferguson and Roxas Boulevard, Manila. Private respondent, Institutional Food Services, Inc., runs a restaurant
business known as the "Alta Vista Restaurant and Supper Club."

Two sub-lease agreements were entered into between petitioner and private respondent, the first on June
16, 1965 for the seventh floor, and the second in May, 1968 for the sixth floor of the VIP Building, "with complete
conditioning equipment." Both agreements had the same stipulations. Private respondent, as sub -lessee, had also
agreed to comply with all the terms and conditions of the main Lease Agreement.

On April 17, 1970, however, private respondent terminated the lease and filed a complaint predicated on
the following causes of action: "(1) defendant failed to comply with its obligation to pro - vide an adequate air
conditioning system for the leased premises, as a consequence of which. plaintiff suffered actual damages in th e
amount of P10,345.15 representing the cost of electric fans, the rental of window type air conditioners, and the cost
of acquisition and installation of a package air conditioning system; (2) the construction of the windows and the
ceilings of the leased premises is defective such that rain water leaked into said premises causing damage to
plaintiff's furniture and rugs in the amount of P11,739.23; (3) as a consequence of the repeated breakdown in the air
conditioning system and the leaking of water into the leased premises, plaintiff's customers refrained from
patronizing its restaurant-thus, plaintiff failed to realize net profits amounting to not less than P300,000.00; (4)
termination of the lease contracts well short of the stipulated period due to defendant's non-compliance with their
terms resulted in plaintiff's failure to fully recover the cost of improvements it introduced in the leased premises,
amounting to P91,653.56; and (5) defendant acted in gross and evident bad faith in refusing to comply with its
plainly valid, just and demandable obligation-compelling plaintiff, by way of protecting itself and its interest, to
retain the services of counsel for a fee and incur other expenses of litigation incident to the suit.". Petitioner denied
liability invoking the provisions of the Lease Agreements and filed a counterclaim. Private respondent denied
liability on the Counterclaim. After the trial court rendered its judgment, both petitioner and private respondent
appealed said decision. The CA affirmed the decision appealed from with modification on the unpaid rentals and
utility charges.

Aggrieved, petitioner comes before the Court. He argues that the awards to the private respondent were in utter
disregard of the pertinent terms and stipulations of the Lease Agreements, as well as the established
jurisprudence on the matter; that private respondent had agreed "to shoulder and pay for any and an costs of
repairs and maintenance of the building and parts thereof, including the elevators and the air condi tioning system;"
in other words, that private respondent is equally responsible for the use and preservation on the leased premises
under the express terms of the contract, which is the law between the parties .

2B 16-17 SALES AND LEASE Page 450


ISSUE

Whether or not private respondent is equally responsible for the use and preservation on the leased
premises as expressly stated in the terms of the contract

HELD

NO. The Court ruled that considering the factual findings of both the Trial Court and respondent Appellate
Court that the defects in the airconditioning system, windows, roof and ceilings arose from faulty construction,
and that petitioner had neglected its duty to make the necessary repairs, which findings are not within the
domain of this Court to review barring the applicability of recognized exceptions, it has to be held that the
exceptions provided for in the contract rather than its general provisions determine the respective liabilities of the
parties herein so that petitioner must be held answerable for damages. This, under the basic principle that those who
in the performance of their obligation are guilty of negligence or delay, and those who in any manner contravene the
tenor thereof, are liable for damages (Article 1170, Civil Code).

It is true that paragraph 5 of the Sub-lease Agreement on repairs, one of the provisions on which petitioner relies,
specifically provides that the sub-lessee assumes all obligations for maintenance and operation as if the sub -lessee is
owner. One of the exceptions to the assumption of duties and obligations, however, is where the thing "required to
be done" arises from "faulty construction." It was apparent that the central air. conditioning was faultily constructed,
hence the frequent breakdowns that private respondent experienced, somethin g that it could ill afford considering
the nature of its restaurant business. It was, therefore, incumbent on petitioner to have made the necessary repairs
and to have maintained the airconditioning unit in running and satisfactory condition. it must not b e lost sight of
either that under the Agreements, it was petitioner which was obliged to provide the leased premises with complete
airconditioning facilities. In fact, the acceptance of the leased premises by private respondent was pre -conditioned
on "the airconditioning with compressor is running sufficient to cool the entire 7th floor." And while paragraph 4 of
the sub-lease agreement provides that improvements made on the building by the sub -lessee shall remain and
become the property of the sub-lessor as a further consideration of the lease, the package-type airconditioners were
not permanent fixtures of the building, witness the fact that private respondent's creditor was able to remove them
from the premises and repossess them. The water leakage, too, must be traced to faulty construction and petitioner's
neglect to make prompt repairs. Although under the "free and harmless" clause in paragraph 12 of the Agreements,
the sublessor was not to be liable to the sub-lessee for "any loss, damage, liability or expense" arising from "water . .
. . rain . . . . or leak from or into the premises or from any cause whatsoever," the same provision requires the
sublessor, "in case such failure shall arise from or be due to negligence on the part of the sublessor, repa irs shall be
made as soon as possible by sublessor to prevent any possible damage caused by the same."

Petitioner's negligence and failure to effect the necessary repairs notwithstanding consistent, complaints on the part
of private respondent for it to do so, were found as a fact by both the Trial Court and the Appellate Court. The fact
of faulty construction is further supported by petitioner's own admission in its Answer that it is the subject of
litigation between itself and the architect of the building. Besides, even without the exception provided for, it is the

2B 16-17 SALES AND LEASE Page 451


legal duty of the sublessor to make all the necessary repairs on the leased premises and to maintain sub -lessee in the
peaceful and adequate enjoyment of the lease as provided under Art. 1654 of the New Civil Code.

As to the other leasehold improvements introduced by private respondent in the amount of P91,653.56 with the
expectation to recover their costs through continued operation of its restaurant business, respondent Appellate Court
did not err either in ordering petitioner to pay the value thereol Petitioner contends that these improvements became
part of the leased premises pursuant to paragraph 4 of the lease agreements. As it is, however, private respondent
was compelled to terminate the lease prematurely for failure of petitioner sublessor to comply with its
obligation of adequately maintaining the leased premises for the use intended. Nor can petitioner be held to
be entitled to rentals for the unexpired period of the lease. Said rentals are governed by Article 1659 of the Civil
Code which provides that if the lessor or the lessee should not comply with the obligations set forth in articles 1654
and 1657, the aggrieved party may ask for the rescission of the contract and indemnification for damages, or only
the latter, allowing the contract to remain in force.

By its act of terminating the contract, private respondent had chosen rescission of the contract and
indemnification for damages. The early termination of the sub-lease by private respondent was for a valid
and legal cause particularly, the breach of petitioner of its contract with private respondent. In fact, the
evidence discloses that private respondent, on September 2, 1969, was already compelled to "hold in abeyance the
payment of (its) rentals until its legitimate complaints shag have been acted upon, to insure the continuity of its
operations." Private respondent had also complied with the required 2-month written notice of termination provided
for in paragraph 1 of the Sub-lease Agreements.

2B 16-17 SALES AND LEASE Page 452


6. IMELDA, LEONARDO, FIDELINO, JOSEFINA, ANITA, AZUCENA, and SISA, all surnamed
SYJUCO vs. COURT OF APPEALS and FILIPINAS BANK

G.R. No. L-80800 April 12, 1989

Paras, J.:

FACTS

Filipinas Bank leased from the Syjucos 1,387 sq. m of land. Par. 6 of the Contract allows the lessee Filipinas Bank
to sublease any part of the premises without prior permission from the lessors Syjucos. Filipinas Bank subleased to
Eugenio Trinidad, with a stipulation that Trinidad as sublessee can n o longer sublease the premises to any other
person. Also it is to be exclusively used by Trinidad in his City Food and fruit Terminal Store and cannot be used for
any other purpose without permission of Sub-lessor. In-spite of the prohibition, Trinidad sub-leased the premises to
12 persons who constructed stalls thereon. The Syjucos after the expiration of the lease filed an action for unlawful
detainer against FIlipinas Bank, which the Bank accepted as an ending of the contract. The Bank then filed a
separate action for the Sublessee Trinidad to vacate the premises. The RTC held that Fililpinas shall be liable for
P25,000/month or a total of P150,000 to the Syjucos for the duration of the lease. Also, in case the FIlipinas Bank
cannot pay, the sublessee Trinidad shall be subsidiarily liable. To comply with the termination of the lease, Filipinas
Bank surrendered the key to the premises with the Syjucos. However, the Syjucos argue that the FIlipinas has not
yet complied with its obligation to return the premises since the fruit stalls are still in the premises. Filipinas argues
that it has already complied with its obligation since the stall vendors do not derive their rights from Filipinas since
the sublease made by Trinidad is a breach of contract.

ISSUE

Did Filipinas Bank comply with its obligation to return the premises leased?

HELD

Yes. The sublease executed by Trinidad was in violation of the sublease contract between him and Filipinas Bank.
The sublease contract by Trinidad and the Stall vendors is void. Therefore, there is no juridical relation between the
stall vendors and FIlipinas Bank. Technically, the Stall owners are squatters occupying the land without any right.
There is no need for the land-owner Syjucos to file a separate ejectment suit against the sub-lessees, since a
judgment against the lessee (Filipinas Bank), is binding against the sub -lessees. Considering that the Bank already
surrendered possession of the premises, it could no longer be held liable for the occupancy of the stall owners. The
reasonable rent from the time of Filipinas Banks surrender of the premises shall be the primary liability of the stall
owners.

2B 16-17 SALES AND LEASE Page 453


7. WHEELERS CLUB INTERNATIONAL, INC. vs. JOVITO BONIFACIO, JR.

G.R. No. 139540, June 29, 2005

Carpio, J

FACTS
Rosario, Romeo, Virgilio, Generoso, Andres, Jovito, Jose (all surnamed Bonifacio), Zenaida B. Lafiguera, Corazon
B. Calub, and Ma. Cristina B. De Guzman are the registered co -owners of a parcel of land with improvement

The co-owners comprised the Board of Directors of J & R Bonifacio Development Corporation (JRBDC).
There are basically three parties here.
1. The co-owners (lessor)
2. BDAI represented by jaime bonifacio (lessee)
3. Wheelers (sublessee)

Co-owners undertook a lease development agreement with lessee and the latter is authorized to renovate, manage,
develop and sublease the property. The term of which is 5 years. On the same day, lessors executed general power of
attorney in favor of jaime as the representative of BDAI.

Co-owners demanded BdAI to submit accounting records of all income from the property, BDAI in turn, demanded
that the co-iwners furnish it with reciepts and records of cash and check advances made by BDAI.

2 years later the co-owner terminated the authority given to jaime for failure to submit an accounting of the income
from the property. Co-owners appointed Jovito as the new administrator of the property through a board resolution.

Jovito wrote to wheelers claiming that the co-owners did not authorize the contract of lease between BDAI and
wheelers. He gave them 10 days to vacate.

Wheelers continued to pay BDAI from feb to sep 1997. Jovito with co -owners demanded payment from feb to oct
1998. Then jovito filed a complaint for unlawful detainer against wheeler.

RTC ruled in favor of wheelers giving merit to the sub-lease agreement between BDAI and lessee. CA reversed

ISSUE
Whether the co-owners have a cause of action for unlawful detainer against Wheelers for non -payment of rentals
and expiration of the term of the Lease Agreement

HELD

No,
In an unlawful detainer, the possession of the defendant is inceptively lawful but it becomes illegal because of the
termination of his right to possess the property under the contract. Hence by instituting the action the lessors admit
that wheelers possession of property is lawful at the beginning. Meaning they recognize the legality of wheelers
possession in the beginning.

2B 16-17 SALES AND LEASE Page 454


The question now is, when did Wheelers possession of the Property become without legal basis to justify the
complaint for unlawful detainer?

In his complaint for unlawful detainer, Jovito claimed that Wheelers disregarded its obligation to pay rentals to the
co-owners from February to October 1997. However, Wheelers obligation to pay the rentals arose from its Contract
of Lease with BDAI. Wheelers did not have a separate lease agreement with Jovito or the other co -owners. Wheelers
continued possession of the Property was by virtue of the Contract of Lease it executed with BDAI. There is no
privity of contract between Wheelers and Jovito or the other co-owners. Since there was neither a written nor verbal
lease agreement between the co-owners and Wheelers, Jovito is mistaken in claiming that the lease contract between
the co-owners and Wheelers is on a month-to-month basis.

What is clear from the records is that the present case involves a sublease arrangement. In a sublease arrangement,
there are two distinct leases: the principal lease and the sublease. These two juridical relationships co -exist and are
intimately related to each other but nonetheless distinct from one another. The lessees rights and obligations vis --vis
the lessor are not passed on to the sublessee.

A careful review of the Lease Development Agreement between JRBDC and BDAI reveals that the co -owners are
the actual lessors of the Property, not JRBDC. In addition, the co -owners are the registered owners of Property.
BDAI, in turn, subleased the Property to Wheelers. Therefore, the co -owners, except only in the instances specified
in the Civil Code, are strangers to the Contract of Lease between BDAI and Wheelers.

Since the co-owners are strangers to the Contract of Lease between BDAI and Wheelers, Wheelers has no right or
authority to pay the sublease rentals to the co-owners as lessors since the rentals are payable to BDAI as lessee-
sublessor. Wheelers was, therefore, under no obligation to pay Jovito or the co -owners the rentals.

Moreover, although Article 1652 of the Civil Code permits the lessor to proceed against the sublessee for rent due
from the lessee, this is only on a subsidiary liability basis.There must be a judgment cancelling the lessees principal
lease contract or ousting the lessee from the premises before the sub -lessee becomes subsidiarily liable. As this
Court explained in Duellome v. Gotico.

The sub-lessee is not liable to the lessor under Article 1652 upon mere demand by the lessor on the sub -lessee. The
sub-lessee is primarily liable to his sub-lessor and only a court order can extinguish or modify this primary liability
if the sub-lessor contests the pre-termination of the principal lease by the lessor. In the present case, there is no
judgment cancelling BDAIs Lease Development Contract or ousting BDAI from the Property.

As things stand, BDAI is the sub-lessor of the Property. BDAIs sub-lease agreement with Wheelers is within the
five-year term of BDAIs principal lease with the co-owners. Until the expiration of the five-year term of BDAIs
principal lease, the sub-lease agreement between BDAI and Wheeler remains valid, unless the sub -lease agreement
is judicially annulled in the proper case,or unless there is a judgment cancelling BDAIs principal lease with the co -
owners or ousting BDAI from the Property.Moreover, no lease agreement exists between the co -owners and
Wheelers. Therefore, Jovitos claim that the term of the alleged lease agreement between the co -owners and
Wheelers has expired has no legal basis.

2B 16-17 SALES AND LEASE Page 455


8. JOSIE GO TAMIO vs. ENCARNACION TICSON

G.R. No. 154895, November 18, 2004

FACTS

"The Roman Catholic Archbishop of Manila (RCAM) is the owner of an apartment unit originally leased to Mr.
Fernando Lopez Lim. After the demise of Mr. Fernando Lim, his children became the occupants thereof. One of
them, Valentine Lim requested respondent Encarnacion Ticson, for financial assistanc e in order to purchase the
apartment unit from RCAM. In exchange, Valentine Lim executed a waiver in favor of respondent.

"On June 15, 1996, respondent executed a contract of lease in favor of petitioner, on the basis of the waiver from
Valentine Lim respecting the apartment unit, for a period of three (3) months. After signing the contract and paying
the rentals, petitioner discovered that the apartment was actually owned by RCAM.

"Meanwhile, after the expiration of the three (3) month lease, respondent demanded petitioner to vacate the premises
for the use of the former's family members. Petitioner failed to comply, giving rise to the instant case for unlawful
detainer.

MTC ruled in favor of petitioner but RTC reversed and ordered petitioner to pay respond ent P86,000.00 as rental
arrearages from September 1996 to June 1997 and from July 1997 to December 1997 at a monthly rate of P5,000.00
and P6,000.00 respectively, and dismissed petitioner's counterclaim for lack of merit.

Meanwhile, on March 3, 1998, petitioner entered into a Contract of Lease over the same property with RCAM for a
term of one year, commencing from January 1, 1998 to December 31, 1998. In that Contract, petitioner assumed to
pay the rent corresponding to her use and occupation of the property prior to its execution; that is, from June 1, 1996
to December 31, 1997.

ISSUE

Whether or not petitioner should be held liable to pay respondent the amo unt of P86,000.00 representing the alleged
rental arrearages from September 1996 to December 1997

HELD

The assignment of a lease by the lessee involves a transfer of rights and obligations pertaining to the contract; hence,
the consent of the lessor is necessary. Article 1649 of the Civil Code is explicit:

"Art. 1649. The lessee cannot assign the lease without the consent of the lessor, unless there is a stipulation to the
contrary."

In the instant case, RCAM never assented to the assignment of the lease. This is apparent from the December 11,
1997 letter of its counsel, Atty. Socrates R. Rivera, stating that Fernando Lim was no longer its tenant for his failure
to pay the rentals as of August 1988. As a rule, this letter may not necessarily result in the cessation of Mr.
Fernando's right to possess the leased premises. Under the law, mere nonpayment of rentals without the lessor's

2B 16-17 SALES AND LEASE Page 456


demand to pay and vacate is not sufficient to oust the lessee from the leased premises.The letter, however,
demonstrates the lessor's lack of consent to the assignment.

To allow respondent to receive from petitioner rental arrearages for the period September 1996 to December 1997,
notwithstanding the latter's agreement with the owner to pay rent for her occupancy of the property, would constitute
unjust enrichment at the expense of petitioner.

Prior to the March 3, 1998 Contract, petitioner and respondent were technically "strangers" to the property; both
were unlawfully withholding its possession from the owner. Petitioner cannot therefore be faulted in assuming to
pay a reasonable value for her occupancy of the property as a sign of good faith. On the other hand, nonpayment of
rentals of respondent to RCAM -- notwithstanding her receipt from petitioner of the rental covering the term of the
sublease contract -- is indicative of bad faith.

Having assumed to pay the rentals to RCAM, petitioner should no longer be required to pay rental arrearages to
respondent. To do so would be to sanction unjust enrichment in favor of respondent and to cause unjust poverty to
the petitioner. A double burden would be imposed upon the latter, because she would be paying twice for her use of
the same premises for the same period of time.

2B 16-17 SALES AND LEASE Page 457


9. COCA-COLA BOTTLERS PHILIPPINES, INC. v. CA

G.R. No. 100957 January 27, 1994

Romero, J.

FACTS:

A certain Manaloto, Coca-Cola Sales Supervisor, negotiated the lease of the premises with respondent
Bautista. The result of the negotiations was a contract of lease executed between the Bautistas as lessors and Coca -
Cola as lessee for a period of ten (10) years from October 1, 1982 to September 30, 1992, renewable for a five-year
term subject to the mutual agreement of the parties. Since the land leased was partially under water, Coca -Cola filled
it up the with ten (10) trucks of filling materials before constructing a warehouse and sales office.

Upon the onset of the rainy season, the sales office and the warehouse were sinking, mud was coming up
from the ground and that asphalt pavement was being wiped out. As a result, their trucks were stuck to the ground.
As such, Coca-cola remedied the situation by availing the services of one construction firm but the site continued to
sink. They also contracted with another firm which conducted a test of the soil and estimated the cost for site
development to almost 1 million pesos but they did not guaranty that should the recommended measures be
followed, the soil would stop sinking. Coca-cola then left the premises.

Sometime in December 1983, private respondents received a letter from Coca -Cola informing of the
company's intention to terminate the lease agreement. After he met with the managers of Coca -Cola and with the
heirs of Paciano Bautista, Cesar Bautista wrote Coca-Cola that its proposal of terminating the contract was not
acceptable. For failure of Coca-Cola to pay the rentals on the leased property after December 1983, the Bautistas
filed a complaint against it with the lower court for specific performance and damages.

The trial court ordered Coca-Cola to pay back rentals plus interest. Coca-Cola elevated the case to the
Court of Appeals, praying for reimbursement of the cost of improvements and the expenses it had incurred to keep
the site from sinking. The Court of Appeals held that there was no duty on the part of private respondents to fill up
the place so that petitioner could properly use it for the purpose intended. That was the obligation of the petitioner.

ISSUE:

Whether or not petitioner was denied its rights to use and enjoy the thing leased under Art. 1643, hence justified in
pre-terminating the contract of lease.

HELD:

2B 16-17 SALES AND LEASE Page 458


NO.

The Appellate Court, having found that petitioner was fully aware of the nature and condition of the land as
it, in fact, had even filled it up and sought a lower rental price from private respondents, held that the latter were not
in bad faith when they entered into the contract of lease with petitioner. Petitioner insists that private respondents, as
lessors, have the obligation to render the subject premises fit for the use intended, and in the face of their denial to
comply with their duty to make the necessary repairs, their refusal to accede to petitioner's demand to pre -terminate
the contract of lease was unjustified.

Petitioner's authorized representatives made ocular inspections of the land in 1981 and several times
thereafter until the contract of lease was executed on October 11, 1982. After satisfying themselves that the subject
land would serve their purpose, or, put differently, that the land was indeed fit for the use intended, petitioner
entered into the aforesaid contract of lease. It constructed a warehouse and other structures and used a number of
route trucks, haulers and other vehicles, including a forklift. Petitioner took steps to remedy the situation and did not
make any demand on respondents to make the "necessary repairs" thereon. This shows that the petitioner did not
consider it the duty of respondents at the time, to solve the problem. Evidently, it tried to do everything it could to
continue the lease until it became financially prohibitive to do so.

It was clearly the intention of the parties, upon entering into the contract of lease, for petitioner to simply
obtain a site, "a flat surface," for the establishment of a Malabon sales office. This was understood and agreed upon
by private respondents. Thus, in accordance with the said contract, petitioner constructed on the subject land its sales
office, warehouse, water tank and gasoline tank. Under these circumstances, private respondents were under no
obligation to make the necessary repairs in order to keep the land suitable for the purpose for which it had been
intended. Had it been private respondents who constructed the structures on their land and leased the same to
petitioner, the situation would have been different. They would have then the oblig ation to deliver the said structures
in such a condition as to render them fit for the use intended. However, as lessee/builder, petitioner had the
responsibility to make sure that the foundations of its buildings and the structures were firm and secure. It should
have tested the underground soil before and not after the construction.

2B 16-17 SALES AND LEASE Page 459


10.PEDRO T. BERCERO versus CAPITOL DEVELOPMENT CORPORATION

G.R. No. 154765 March 29, 2007

Austria-Martinez, J

FACTS: On January 31, 1983, Respondent corporation leased its commercial building and lot to R.C. Nicolas
Merchandising, Inc., (R.C. Nicolas) for a 10-year period or until January 31, 1993 with the option for the latter to
make additional improvements in the property to suit its business and to sublease portions thereof to third parties.
R.C. Nicolas converted the space into a bowling and billiards center and subleased separate portions thereof to
several persons, including petitioner whose sublease contract with R.C. Nicolas was for a three -year period or until
Aug 16, 1988. Meanwhile, for failure to pay rent, respondent filed an ejectment case against R.C. Nicolas before the
MeTC, also impleading the sub-lessees of R.C. Nicolas as parties -defendants. During the pendency of case, several
sub-lessees including petitioner, entered into a compromise settlement with respondent, wherein the sub -lessees
recognized: 1) respondent as the lawful and absolute owner of the property; 2) that the contract between respondent
and R.C. Nicolas had been lawfully terminated because of the latters non-payment of rent; 3) that the sub-lessees
voluntarily surrendered possession of the premises to respondent; 4) that the sub -lessees directly executed lease
contracts with respondent considering the termination of leasehold rights of R.C. Nicolas.

Thereafter, petitioner entered into a lease contract with respondent for a three -year period, from August 16,
1988 to August 31, 1991. On October 21, 1988, respondent and petitioner, as well as several other sub -lessees of
R.C. Nicolas, filed a Joint Manifestation and Motion manifesting to the MeTC that they entered into a compromise
settlement and moved that the names of the sub-lessees as parties-defendants be dropped and excluded.

On November 14, 1988, R.C. Nicolas filed a complaint for ejectment and collection of unpaid rentals
against petitioner before the MeTC, which rendered a Decision in favor of R.C. Nicolas and ordered the eviction of
petitioner from the leased premises. Pending appeal of petitioner to the RTC, who directed the issuance of a writ of
execution pending appeal since petitioner failed to file a bond and periodically deposit the rentals due during the
pendency of the appeal. Sometime in November 1990, petitioner was evicted from the leased premises, hence, he
made repeated demands on respondent for the restoration of his possession of the commercial space leased to him to
no avail.

Thus, on March 24, 1992, petitioner filed a complaint against the respondent before the RTC, who rendered
its decision in favor of petitioner. The RTC held that respondent miserably failed to comply with its obligation under
Article 1654 of the New Civil Code due to its apathy and failure to extend any assistance to the petitioner and was,
therefore, liable for the restoration of petitioners possession and the payment of actual damages corresponding to
lost profit, cash, generator, and other items petitioner lost due to the eviction, as well as moral and exemplary
damages and attorneys fees.

2B 16-17 SALES AND LEASE Page 460


On appeal, CA set aside the decision of the RTC branch, ruling that although respondent as lessor failed to
ensure the peaceful possession of petitioner as its lessee in the subject premises, the latter is not entitled to damages
since he was aware of the facts which led to his ouster from the subject premises; and that petitioner was well aware
that respondent had a 10-year lease contract with R.C. Nicolas which was subject of an ejectment suit that was still
pending litigation when petitioner executed a lease contract with respondent.

Petitioner contends that the principle of estoppel is inapplicable because he dealt with respondent in good
faith and relied upon the latters representations that the lease of R.C. Nicolas was already terminated at the time he
contracted with the latter; that respondent assured him that it had a valid and legal right to enter into a new lease
contract with him; that he is entitled to damages since respondent did not even lift a finger to protect him when R.C.
Nicolas filed an ejectment case against him; and that res pondent acted in utter bad faith when it still refused to
restore his possession after he was evicted in November 1990, notwithstanding that his lease contract with
respondent was valid until August 31, 1991.

Respondent counters that the CA correctly applied the principle of estoppel since petitioner voluntarily
entered into a lease agreement with respondent despite full knowledge that the latters lease with R.C. Nicolas over
the subject premise had yet to be judicially terminated; and that petitioner knew that at the time he contracted with
respondent, he still had existing obligations to R.C. Nicolas relating to their sub -lease agreement.

ISSUE

(1) Whether or not respondent was obliged to restore his possession following his eviction from the premises ;
(2) Whether or not the lease contract between respondent and petitioner is valid

HELD

NO; NO

Under Article 1654 (3) of the New Civil Code, the lessor is obliged to maintain the lessee in the peaceful
and adequate enjoyment of the lease for the entire duration of the contract. It is the duty of the lessor to place the
lessee in the legal possession of the premises and to maintain the peaceful possession thereof during the entire term
of the lease. The obligation of the lessor arises only when acts, termed as legal trespass, disturb, dispute, object to, or
place difficulties in the way of the lessees peaceful enjoyment of the premises that in some manner or other cast
doubt upon the right of the lessor by virtue of which the lessor himself executed the lease, in which case the lessor is
obligated to answer for said act of trespass. The lessee has the right to be respected in his possession and should he
be disturbed therein, he shall be restored to said possession by the means established by the law or by the Rules of
Court.Possession is not protection against a right but against the exercise of a right by ones own authority. However,
void are all contracts in which the cause or object does not exist at the time of the transaction.

In the present case, the lease contract between petitioner and respondent is void for having an inexistent
cause - respondent did not have the right to lease the property to petitioner considering that its lease contract with
R.C. Nicolas was still valid and subsisting, albeit pending litigation. Having granted to R.C. Nicolas the right to use
and enjoy its property from 1983 to 1993, respondent could not grant that same right to petitioner in 1988. When

2B 16-17 SALES AND LEASE Page 461


petitioner entered into a lease contract with respondent, the latter was still o bliged to maintain R.C. Nicolas
peaceful and adequate possession and enjoyment of its lease for the 10-year duration of the contract. Respondents
unilateral rescission of its lease contract with R.C. Nicolas, without waiting for the final outcome of the e jectment
case it filed against the latter, is unlawful. A lease is a reciprocal contract and its continuance, effectivity or
fulfillment cannot be made to depend exclusively upon the free and uncontrolled choice of just one party to a lease
contract. Thus, the lease contract entered into between petitioner and respondent, during the pendency of the lease
contract with R.C. Nicolas, is void.

There is no merit to petitioners claim of good faith in dealing with respondent. Good faith is ordinarily used
to describe that state of mind denoting honesty of intention, and freedom from knowledge of circumstances which
ought to put the holder upon inquiry; an honest intention to abstain from taking any unconscientious advantage of
another, even through technicalities of law, together with absence of all information, notice, or benefit or belief of
facts which render the transaction unconscientious. Being privy to the pendency of the ejectment case involving the
leasehold rights of R.C. Nicolas since he was impleaded as a party-defendant in said ejectment case, petitioner
cannot feign innocence of the existence thereof. Petitioner was fully aware that R.C. Nicolas had a lease contract
with respondent which was subject of a pending litigation.

It is well-settled that parties to a void agreement cannot expect the aid of the law; the courts leave them as
they are, because they are deemed in pari delicto or in equal fault. No suit can be maintained for its specific
performance, or to recover the property agreed to be so ld or delivered, or the money agreed to be paid, or damages
for its violation, and no affirmative relief of any kind will be given to one against the other. Each must bear the
consequences of his own acts They will be left where they have placed themselves since they did not come into
court with clean hands.

2B 16-17 SALES AND LEASE Page 462


11. JOSE L. MADAMB A vs. SALVADOR ARANETA, ETC., ET AL.

G.R. No. L-12017, August 28, 1959

Concepcion, J

FACTS:

On August 17, 1926, Madamba filed with the Bureau of Lands an application fo r a lease contract on an
agricultural public land in Isabela. The contract of lease was awarded to Madamba on September 14, 1935.
Beginning 1945 there was failure to pay the stipulated rents so said contract was, on October 2, 1953, cancelled by
respondent Salvador Araneta, as Secretary of Agriculture and Natural Resources.

Petitioner admits nonpayment of the stipulated rentals from 1945 to 1953, but maintains that the same did
not justify the cancellation of the lease contract for his aforementioned omiss ion was allegedly due to the usurpation
of portions of the leased property by third persons and the Government has failed as a lessor (under Article 1658),
despite several demands made by him, to keep him, as lessee, in peaceful possession of said property .

ISSUE

Whether the Government should have been held liable as lessor to keep petitioner in peaceful possession of said
property

HELD

No. Article 1664 provides that: The lessor is not obliged to answer for a mere act of trespass which a third
person may cause on the use of the thing leased; but the lessee shall have a direct action against the intruder.

There is a mere act of trespass when the third person claims no right whatever.

In the case at bar, the disturbance in plaintiff's possession, with respect to small portions of the leased property, was
admittedly caused by mere intruders, who acted without any color of title. The disturbance in the possession of
petitioner herein was the product of an "act of mere trespass," for which, "the lessor sha ll not be liable" or "shall not
be obliged to answer". Plaintiff was not entitled to shift to the Government, as lessor, the task of suing the intruders,
which the law explicitly imposes upon him, as lessee.

2B 16-17 SALES AND LEASE Page 463


12. LL AND COMPANY DEVELOPMENT AND AGRO-INDUSTRIAL CORPORATION vs. HUANG
CHAO CHUN AND YANG TUNG FA

G.R. No. 142378. March 7, 2002

Panganiban, J

FACTS

The petitioner alleged that respondents violated their amended lease contract over a 1,112 square meter lot
it owns when they did not pay the monthly rentals thereon. It also alleged that the amended lease contract already
expired on September 16, 1996 but respondents refused to surrender possession thereof plus the improvements made
thereon, and pay the rental arrearages despite repeated demands.

The amended lease contract was entered into by the parties sometime in August, 1991. The same amended
the lease contract previously entered into by the parties on August 8, 1991.

One of the terms and conditions in the contract is the following: The term of this lease is FIVE (5) YEARS
from the effectivity of said lease, and with the option to renew, specifically shall commence from September 15,
1991 and shall expire on September 16, 1996, and maybe adjusted depending upon the ejectment of tenants.

The MTC and RTC favored respondents. The RTC likewise agreed that the Contract of Lease entered into
by the parties could be extended unilaterally by the lessees for another five years or until September 16, 2001, on the
basis of justice and equity. It also held that the parties had a reciprocal obligation: unless and until petitioner
presented the increased realty tax, private respondents were not under any obligation to pay the increased monthly
rental.

Court of Appeals affirmed in toto the lower courts judgment.

Hence, this Petition.

ISSUES

(1) Whether the court could still extend the term of the lease, after its expiration.

(2) Is refusal of the lessor to accept or collect rentals a valid reason for non -payment of rentals[?]

HELD

(1) No.

In general, the power of the courts to fix a longer term for a lease is discretionary. Such power is to be
exercised only in accordance with the particular circumstances of a case: a longer term to be granted where equities
demanding extension come into play; to be denied where none appear -- always with due deference to the parties
freedom to contract. Thus, courts are not bound to extend the lease.

2B 16-17 SALES AND LEASE Page 464


In the case before us, the Contract of Lease provided for a fixed period of five (5) years -- specifically from
September 16, 1991 to September 15, 1996. Because the lease period was for a determinate time, it ceased, by
express provision of Article 1669 of the Civil Code, on the day fixed, without need of a demand.

Furthermore, the extension of a lease contract must be mad e before the term of the agreement expires, not
after. Upon the lapse of the stipulated period, courts cannot belatedly extend or make a new lease for the parties,
even on the basis of equity. Because the Lease Contract ended on September 15, 1996, without the parties reaching
any agreement for renewal, respondents can be ejected from the premises.

(2) No. Mere failure to pay rentals does not make possession unlawful, but when a valid demand to
vacate the premises is made by the lessor, the lessees continued withholding of possession becomes unlawful. Well-
settled is the rule that the failure of the owners/lessors to collect or their refusal to accept the rentals is not a valid
defense.

Respondents justify their nonpayment of rentals on the ground that petitioners refused to accept their
payments based on Article 1256 of the Civil Code.

Based on the foregoing, respondents should have deposited in a bank or with judicial authorities the rent
based on the previous rate. Moreover, the mere subsequent paymen t of rentals by the lessee and the receipt thereof
by the lessor does not, absent any other circumstance that may dictate a contrary conclusion, legitimize the unlawful
character of the possession. The lessor may still pursue the demand for ejectment.

Respondents were ordered to vacate the said premises and to pay the accrued rentals.

2B 16-17 SALES AND LEASE Page 465


ON IMPLIED NEW LEASE (Articles. 1670, 1682 and 1687)

1. AREVALO GOMEZ CORPORATION vs.ANDERS LAO HIAN LIONG, doing business in the
name and style of "TIONGSON BAZAAR" and The Honorable SALVADOR J. VALDEZ, JR.

G.R. No. 70360, March 11, 1987

Cruz, J

FACTS

On December 1, 1964, the petitioner through its Vice-President, Renato Arevalo, and respondent Andres
Lao HIAN Liong, executed a "Contract of Lease" covering the petitioner's property at Magsaysay Avenue, Baguio
City, for a term of fifteen years, effective September 1, 1964. The monthly rental was fixed at P2,450.00 but in
addition to this the respondent agreed to construct on the interior portion of the land le ased a three-story building of
strong materials without right to reimbursement from the petitioner. The cost of the building was to be not less than
P150,000.00, of which the sum of P45,000.00 would be contributed by petitioner.

Prior to the expiration of the lease on August 31, 1979, and for some time thereafter, the parties entered
into negotiations to fix a new rental but could not come to any agreement. In the end, on October 2, 1979, the
petitioner served on the respondent a written notice to vacate th e leased premises in view of the termination of their
contract. When the respondent refused to comply, the petitioner filed a complaint for ejectment against him in the
City Court of Baguio City.

Applying Article 1670 of Civil Code, the trial court held in favor of the defendant as follows:

In the case on hand, it is admitted that the 15-year lease contract between the parties expired on August 31,
1979. However, the defendant has continued occupying the leased premises thereafter and even to this day. And it
was only on October 2, 1979, or after more than 15 days after the expiration of the original contract of lease, that he
was given the requisite notice to vacate. It is, therefore, abundantly clear that under the law, an implied new lease
had already set in when the plaintiff commenced its action for ejectment on November 19, 1979.

The petitioner contended that the original lease had not been impliedly renewed but automatically expired
on August 31, 1979. The Regional Trial Court of Baguio City affirmed the implied renewal of the lease.

ISSUE

Whether or not the RTC judge, as well as the trial judge, erred in holding that there was implied renewal or tacita
reconduccion despite the refusal of respondent Liong to agree to the increased rental demanded by p etitioner prior to
the expiration of the contract of lease and despite the refusal of petitioner to accept payment of rentals from
respondent Liong after the expiration of the Contract of Lease.

HELD

2B 16-17 SALES AND LEASE Page 466


Not a case of implied lease.

It is not disputed that the original lease contract between the parties was only for fifteen years expiring on
August 31, 1979. The private respondent nonetheless continued occupying the leased premises beyond that date and
it was only on October 2, 1979, that he was formally served with notice to vacate. What is in issue then is whether
such continued occupancy was with or without the implied acquiescence of the petitioner.

The applicable provisions of Civil Code are the following:

Article 1669. If the lease was made for a determinate time, it ceases upon the day fixed, without the need of
a demand.

Article 1670. If at the end of the contract the lessee should continue enjoying the thing leased for fifteen
days with the acquiescence of the lessor, and unless a notice to the contrary by either party has previously been
given, it is understood that there is an implied new lease, not for the period of the original contract, but for the time
established in Article 1682 and 1687. The other terms of the original contract shall be revived.

Under the second article, an implied new lease or tacita reconduccion will set in if it is shown that: (a) the
term of the original contract of lease has expired; (b) the lessor has not given the lessee a notice to vacate; and (c)
the lessee continued enjoying the thing leased for fifteen days with the acquiescence of the lessor. This acquiescence
may be inferred from his failure to serve a notice to quit.

The service of an express notice to quit is not the only way to prevent the implied renewal of the lea se.
Demanding a higher rental is also a manifestation of non -acquiescence if the lessee does not accept the rate
demanded. In other words, failure of agreement on the new conditions of the lease results in an automatic notice to
vacate upon the expiration of the original lease.

Article 1670 applies only where, before the expiration of the lease, no negotiations are held between the
lessor and the lessee resulting in its renewal. Where no such talks take place and the lessee is not asked to vacate
before the lapse of fifteen days from the end of the lease, the implication is that the lessor is amenable to its renewal.

Weeks before the deadline for the notice to vacate, the petitioner had already communicated to the
respondent its intention to increase the rental. This increase had to be accepted by the respondent if he wanted the
lease to be renewed.

2B 16-17 SALES AND LEASE Page 467


2. ANIANO TORRES and JOSEFINA TORRES vs. THE HON. COURT OF APPEALS, HON.
REGIONAL TRIAL COURT OF NEGROS ORIENTAL, BRANCH 34, and ADELA B. FLORES

G.R. No. 92540 December 11, 1992

FACTS

Private Respondent Adela B. Flores leased ten parcels of land to the spouses Aniano and Josefina Torres, for a
specified term of four agricultural years commencing from 1985 and at stipulated rentals payable in piculs of su gar.
Before the expiration of the contracts, the parties agreed on their renewal under the original terms, subject to the
condition that the lessees would deliver to Flores the amount of P50, 000.00 not later than February 15, 1989. Flores
said she would need the money for her projected trip abroad. Flores claims that petitioner failed to comply with the
aforementioned condition and that she subsequently informed them that she was taking over her property upon the
expiration of the contracts. When her subsequent demands for the surrender of her property were disregarded, she
sued the petitioners for illegal detainer. The petitioners submitted that the contracts had been validly renewed
because they had complied with the above-mentioned condition. As affirmative defense, they contended that the
lease had been continued under the rule of tacita reconduccion and that the complainant was estopped from denying
that she had granted them an option to renew the contracts. After trial under the Rule on Summary Procedu re, Judge
Teopisto L. Calumpang held that the contracts had not been validly renewed because the Torreses had failed to
deliver the amount of P50,000.00 in cash to Flores as promised. This finding was affirmed on appeal to the Regional
Trial Court of Dumaguete City by Judge Rosendo B. Bandal, Jr. On appeal to respondent court, the petitioners
argued that (a) what was subject to the condition was the execution of the renewed written contract of lease; (b)
granting that the condition was to be applied to the principal contract, this was complied with as evidenced by the
subsequent acts of plaintiff in withdrawing P50,000.00 worth of piculs of sugar; and (c) the appellee was placed in
estoppel and guilty of bad faith because of the previously referred withdrawa l. The Court of Appeals sustained the
findings of the lower courts that the contracts had not been validly renewed. Hence, the appeal.

ISSUE

Whether or not the original contracts of lease entered into between the petitioners and the private respondent hav e
been validly renewed.

HELD

NO, the contracts were not validly renewed. It is clear from the documentary evidence submitted by both parties
that the two checks paid to Flores by the spouses Torres were in the separate amounts of only P1,686.15 and
P9,729.74. They were indisputably far short of the required payment of the P50,000.00 rentals on the land by the
petitioners as a condition for the renewal of the lease. Hence, Flores as lessor was not obliged to extend the
contracts. The invocation of tacita reconduccion is futile. The facts of this case as found by the lower courts clearly
show that there was no implied renewal but instead an express termination of the contracts of lease. This is evident
from the letters of Flores to the petitioners advising them on February 17, 1989, that she would take over her
property upon the end of agricultural year 1989 and on March 8, 1989, that they should not undertake any new
cultivation of the leased lands and demanding again that they vacate the same. This demand h ad earlier been made
by Flores's counsel on February 20, 1989.

2B 16-17 SALES AND LEASE Page 468


The Civil Code provides:

Art. 1670. If at the end of the contract, the lessee should continue enjoying the thing leased for fifteen days
with the acquiescence of the lessor, and unless a notice to the contrary by either party has previously been given, it is
understood that there is an implied new lease, not for the period of the original contract but for the time established
in Art. 1682 and 1687. The other terms of the original contract shall be revived.

There was no acquiescence on the part of the lessor to the petitioners' continued stay in her property. On the
contrary, she expressly informed them that she was not renewing the lease and in fact later demanded that they
vacate her property. The private respondent's acceptance of the rentals beyond the original term did not signify that
she had agreed to the implied renewal of the lease. The simple reason is that the petitioners remained in possession
of the subject lands and, regardless of the outcome of their case, had to pay rentals to the private respondent for the
use of her property.

2B 16-17 SALES AND LEASE Page 469


3. JOVEN YUKI, JR.VS WELLINGTON CO

G.R. No. 178527, November 27, 2009

Del Castillo, J

FACTS

Mr. Joseph Chua was the registered owner of a parcel of land, together with a commercial building erected thereon,
situated at the corner of Espaa and Instruccion Sts., Sampaloc, Manila. In 1981, he leased a portion of the building
to petitioner who put up a business therein under the name an d style "Supersale Auto Supply." The contract of lease
between Mr. Chua and petitioner had a term of five years but was not reduced into writing. Thereafter, the lease was
renewed through a series of verbal and written agreements, the last of which was a written Contract of Lease
covering the period of January 1, 2003 to December 31, 2003 at a monthly rental of P7,000.00.

However, in November 2003, Mr. Chua informed petitioner that he sold the property to respondent
Wellington Co and instructed petitioner to thenceforth pay the rent to the new owner.

After the expiration of the lease contract, petitioner refused to vacate and surrender the leased premises.
Consequently, respondent, being the new owner of the property pursuant to a contract of sale executed b etween the
company and the previous owner, filed a Complaint for unlawful detainer before the MeTC of Manila praying that
petitioners possession of subject premises be declared unlawful and that petitioner be ordered to vacate it. He also
sought reasonable compensation for the use of the property until such time that it is surrendered to him.

Before MeTC:

Petitioner denied having been served with copies of the alleged notice of sale and notice to vacate.
Petitioner also asserted that since he was not notified by the former owner of the sale, he was deprived of his
preemptive rights. Moreover, respondent has no cause of action against him because respondent is not the true
owner of the property but merely acts as a representative of persons whom respondent refused to disclose. Further,
petitioner argued that there was an implied renewal of lease considering that a) he did not receive a notice to vacate,
b) the two months deposit and one month advance payment he gave to Mr. Chua were never returned to him, a nd c)
respondent accepted his payments for the months of January and February 2004.

Petitioner also asserted that his property rights would be violated if he is evicted because he has been
operating his business in the premises for more than 20 years and h as established goodwill in the area. He thus
proposed that he be compensated the amount of not less than P1 million or be allowed to dispose of his stocks within
a reasonable period of time, before he vacates the premises.

MeTC ruled in favor of the respondent ordering petitioner to vacate the property and to pay the reasonable
compensation for the use and occupancy.

Before RTC:

The RTC reversed the decision and dismissed the unlawful detainer case. It found no proof on record that
petitioner actually received the notice to vacate, thereby making the Complaint fatally defective. The RTC likewise

2B 16-17 SALES AND LEASE Page 470


opined that the resolution of the case hinges on the existence of implied new lease, a question which is incapable of
pecuniary estimation and, therefore, beyond the MeTCs jurisdiction.

Before the CA:

Aggrived by the decision, respondent filed with the CA a Petition for Review under Rule 42 of the Rules of
Court assailing the RTC Decision.

The CA ruled in favor of the respondent and set aside the decision of the RTC.

Before the Supreme Court:

Petitioner contends that the Petition for Review filed by the respondent with the CA is procedurally
infirmed because while respondent attached to the petition the parties respective position papers, he failed to attach
to said position papers the annexes thereto, thus, such should have been outrightly dismissed.

Petitioner also contends that the CA grievously erred in reversing the Decision of the RTC. He maintains
that the RTC correctly held that the key issue to be resolved in this case is the existence of an implied new lease, a
matter which is incapable of pecuniary estimation and, therefore, beyond the MeTCs jurisdiction.

Petitioner likewise claims that the RTC correctly held that there was no sufficient evidence on record that
he received the alleged notice to vacate. While he admits that a notice to vacate is no longer necessary when the
ground for unlawful detainer is the expiration of the lease, proof that he actually received said notice is still
important in this case in view of his allegation of implied new lease. Citing Article 1670 of the Civil Code,
petitioner contends that if at the expiration of the contract of lease the lessee continued to enjoy the leased property
for 15 days with the acquiescence of the lessor, there is an implied new lease. In this case, the determination of
whether or not his continued stay in the leased premises is with the acquiescence of the lessor hinges on whether or
not he received the notice to vacate. And, as correctly found by the RTC, he did not receive any notice to vacate.

ISSUES:

(1) Whether or not Section 2 of Rule 42 of the ROC requires that all the pleadings and documents filed
before the lower courts must be attached as annexes to the petition? NO

(2) Whether or not an implied new lease exist in this case, thus its beyond the jurisdiction of the MeTC?
NO

(3) Whether or not petioner received any notice to vacate? YES

(4) Whether or not petitioner was deprived of his preemptive rights because he was not notified of the
intended sale? NO

HELD

2B 16-17 SALES AND LEASE Page 471


(1) NO, Section 2 of Rule 42 does not require that all the pleadings and documents filed before the lower
courts must be attached as annexes to the petition. Aside from clearly legible duplicate originals or true copies of
the judgments or final orders of both lower courts, it merely requires that the petition be accompanied by copies of
pleadings and other material portions of the record as would support the allegations of the petition. As to what these
pleadings and material portions of the record are, the Rules grants the petitioner sufficient discretion to determine
the same. This discretion is of course subject to CAs evaluation whether the supporting documents are sufficient to
make out a prima facie case. Thus, Section 3 empowers the CA to dismiss the petition where the allegations
contained therein are utterly bereft of evidentiary foundation. Since in this case the CA gave due course to
respondents Petition for Review and proceeded to decide it on the merits, it can be fairly assumed that the appellate
court is satisfied that respondent has sufficiently complied with Section 2 of Rule 42.

In the case at hand, the CA rollo reveals that the annexes to the position papers can be found somewhere
else in the petition. The annexes to the parties respective position papers are the same annexes attached to the
Complaint and the Answer. These are more than substantial compliance with the requirements of the rules. Indeed,
if we are to apply the rules of procedure in a very rigid and technical sense as what the petitioner suggests in this
case, the ends of justice would be defeated.

(2) NO, the argument is bereft of merit. The allegation of existence of implied new lease or tacita
reconduccion will not divest the MeTC of jurisdiction over the ejectment case. Jurisdiction of the court in ejectment
cases is determined by the allegations pleaded in the complaint and cannot be made to depend upon the defenses set
up in the answer or pleadings filed by the defendant. This principle holds even if the facts proved during trial do not
support the cause of action alleged in the complaint. In connection with this, it is well to note that in unlawful
detainer cases the elements to be proved and resolved are the facts of lease and expiration or violation of its terms.

Here, no interpretative exercise is needed to conclude that respondent has complied with such requirement.
In respondents Complaint, he specifically alleged that:

(1) The former owner, Mr. Chua, and petitioner entered into a contract of lease;

(2) Subsequently, respondent purchased the leased premises from Mr. Chua and became the owner thereof;

(3) Thereafter, the lease contract between Mr. Chua and petitioner expired; and

(4) Petitioner refused to vacate the premises despite the expiration and non-renewal of the lease.

Besides, the Court does not agree with the RTC that the MeTC does not have jurisdiction to resolve the
issue of existence of implied new lease in the unlawful detainer case. Tacita reconduccion refers to the right of
the lessee to continue enjoying the material or de facto possession of the thing leased within a period of time
fixed by law. During its existence, the lessee can prevent the lessor from evicting him from the disputed premises.
On the other hand, it is too well-settled to require a citation that the question to be resolved in unlawful detainer
cases is, who is entitled to de facto possession.

Therefore, since tacita reconduccion is determinative of who between the parties is entitled to de facto
possession, the MeTC has jurisdiction to resolve and pass upon the issue of implied new lease in unlawful detainer

2B 16-17 SALES AND LEASE Page 472


case. In Mid-Pasig Land Development Corporation v. Court of Appeals, the Court that the MeTC is clothed with
exclusive original jurisdiction over an unlawful detainer case even if the same would entail compelling the plaintiff
therein to recognize an implied lease agreement.

(3) YES, The Court finds that there was a valid demand to vacate. Under Article 1670, an implied new
lease will set in if it is shown that:

(a) the term of the original contract of lease has expired;

(b) the lessor has not given the lessee a notice to vacate; and

(c) the lessee continued enjoying the thing leased for 15 days with the acquiescence of the lessor.

This acquiescence may be inferred from the failure of the lessor to serve notice to vacate upon the lessee.

In the instant case, however, the MeTC and the CA correctly found that there was a valid demand to vacate.
Prior to the sale of the property by previous owner Joseph Chua to herein plaintiff, defendant was formally notified
by the previous owner in a letter of his intention to sell the property but herein defendant failed to exercise his pre -
emptive right to purchase the property.

Thus, the subject premises was sold to plaintiff who became the registered owner Plaintiff, as new
owner/vendee, informed defendant through a letter, even prior to the expiration of the contract that he will be
needing the premises thus the contract will not be renewed or no contract will be executed, and directed defendant to
vacate the premises by January 1, 2004. The said notice was sent by registered mail and by personal service. The
notice sent by registered mail was returned to sender for failure of the defendant to claim the s ame at the post office.
Despite notice given to him, defendant failed to vacate and a formal demand letter was served to him personally
which he refused to acknowledge that he received the same. Moreover, the defendant failed to show that the portion
being occupied by him which is the subject matter of this case is covered by another lease contract.

Moreover, there is no cogent reason to disturb the finding of said courts. Under the rules, if the addressee
refuses to accept delivery, service by registered mail is deemed complete if the addressee fails to claim the mail
from the postal office after five days from the date of first notice of the postmaster. Further, the absence of personal
service of notice to vacate in this case could only be attributed to petitioners unexplainable refusal to receive the
same.

The formal demands to vacate sent to petitioner, coupled with the filing of an ejectment suit, are categorical
acts on the part of respondent showing that he is not amenable to another renewal of th e lease contract. Therefore,
Respondent did not acquiesce to petitioner's continued possession of subject premises.

(4) NO, the right of first refusal, also referred to as the preferential right to buy, is available to lessees
only if there is a stipulation thereto in the contract of lease or where there is a law granting such right to them
(i.e., Presidential Decree No. 1517 (1978), which vests upon urban poor dwellers who merely lease the house where

2B 16-17 SALES AND LEASE Page 473


they have been residing for at least ten years, preferential right to buy the property located within an area proclaimed
as an urban land reform zone). Unlike co-owners and adjacent lot owners, there is no provision in the Civil Code
which grants to lessees preemptive rights. Nonetheless, the parties to a c ontract of lease may provide in their
contract that the lessee has the right of first refusal.

In this case, there is nothing in the Contract of Lease which grants petitioner preferential right to buy
the subject premises. Moroever, there is no applicable law which vests upon him priority right to buy the
commercial building subject matter of this case. In fact, aside from the sweeping statement that his preferential right
to buy was violated, petitioner failed to cite in his pleadings any specific provis ion of a law granting him such right.
In other words, petitioner failed to lay the basis for his claim that he enjoys a preferential right to buy.

And even assuming that he has, the same will not prevent the ejectment case filed by the respondent from
taking its due course. A contract of sale entered into in violation of preemptive right is merely rescissible and the
remedy of the aggrieved party whose right was violated is to file an appropriate action to rescind the sale and
compel the owner to execute the necessary deed of sale in his favor. An action for unlawful detainer cannot be
abated or suspended by an action filed by the defendant-lesseee to judicially enforce his right of preemption.

2B 16-17 SALES AND LEASE Page 474


4. YOLANDA ROSELLO-BENTIR, SAMUEL PORMIDA and CHARITO PORMIDA vs.
HONORABLE MATEO M. LEANDA, in his capacity as Presiding Judge of RTC, Tacloban City, Branch 8,
and LEYTE GULF TRADERS, INC.

G.R. No. 128991, April 12, 2000

FACTS

On May 15, 1992, respondent Leyte Gulf Traders, Inc. (herein referred to as respondent corporation) filed a
complaint for reformation of instrument, specific performance, annulment of conditional sale and damages with
prayer for writ of injunction against petitioners Yolanda Rosello -Bentir and the spouses Samuel and Charito
Pormida. The case was docketed as Civil Case No. 92-05-88 and raffled to Judge Pedro S. Espina, RTC, Tacloban
City, Branch 7. Respondent corporation alleged that it entered into a contract of lease of a parcel of land with
petitioner Bentir for a period of twenty (20) years starting May 5, 1968. According to respondent corporation, the
lease was extended for another four (4) years or until May 31, 1992. On May 5, 1989, petitioner Bentir sold the
leased premises to petitioner spouses Samuel Pormada and Charito Pormada. Respondent corporation questioned the
sale alleging that it had a right of first refusal. Rebuffed, it filed Civil Case No. 92-05-88 seeking the reformation of
the expired contract of lease on the ground that its lawyer inadvertently omitted to incorporate in the contract of
lease executed in 1968, the verbal agreement or understanding between the parties that in the event petitioner Bentir
leases or sells the lot after the expiration of the lease, respondent corporation has the right to equal t he highest offer.

In due time, petitioners filed their answer alleging that the inadvertence of the lawyer who prepared the
lease contract is not a ground for reformation. They further contended that respondent corporation is guilty of laches
for not bringing the case for reformation of the lease contract within the prescriptive period of ten (10) years from its
execution.

ISSUE

Whether the complaint for reformation of instrument has prescribed.

HELD

Yes, the petition has merit. The remedy of reformation of an instrument is grounded on the principle of
equity where, in order to express the true intention of the contracting parties, an instrument already executed is
allowed by law to be reformed. The right of reformation is necessarily an invasion or limita tion of the parol
evidence rule since, when a writing is reformed, the result is that an oral agreement is by court decree made legally
effective. Consequently, the courts, as the agencies authorized by law to exercise the power to reform an instrument,
must necessarily exercise that power sparingly and with great caution and zealous care. Moreover, the remedy, being
an extraordinary one, must be subject to limitations as may be provided by law. Our law and jurisprudence set such
limitations, among which is laches. A suit for reformation of an instrument may be barred by lapse of time. The
prescriptive period for actions based upon a written contract and for reformation of an instrument is ten (10) years
under Article 1144 of the Civil Code. Prescription is intended to suppress stale and fraudulent claims arising from
transactions like the one at bar which facts had become so obscure from the lapse of time or defective memory. In
the case at bar, respondent corporation had ten (10) years from 1968, the time when the contract of lease was
executed, to file an action for reformation. Sadly, it did so only on May 15, 1992 or twenty -four (24) years after the
cause of action accrued, hence, its cause of action has become stale, hence, time-barred.

2B 16-17 SALES AND LEASE Page 475


According to respondent corporation, there was an agreement between the parties to extend the lease
contract for four (4) years after the original contract expired in 1988, then Art. 1670 would not apply as this
provision speaks of an implied new lease (tacita reconduccion) where at the end of the contract, the lessee continues
to enjoy the thing leased "with the acquiescence of the lessor", so that the duration of the lease is "not for the period
of the original contract, but for the time established in Article 1682 and 1687." In other words, if the extended period
of lease was expressly agreed upon by the parties, then the term should be exactly what the parties stipulated, not
more, not less. Even if the supposed 4-year extended lease be considered as an implied new lease under Art. 1670,
"the other terms of the original contract" contemplated in said provision are only those terms which are germane to
the lessee's right of continued enjoyment of the property leased . The prescriptive period of ten (10) years provided
for in Art. 1144 applies by operation of law, not by the will of the parties. Therefore, the right of action for
reformation accrued from the date of execution of the contract of lease in 1968.

Even if we were to assume for the sake of argument that the instan t action for reformation is not time-
barred, respondent corporation's action will still not prosper. Under Section 1, Rule 64 of the New Rules of Court,
an action for the reformation of an instrument is instituted as a special civil action for declaratory relief. Since the
purpose of an action for declaratory relief is to secure an authoritative statement of the rights and obligations of the
parties for their guidance in the enforcement thereof, or compliance therewith, and not to settle issues arising from
an alleged breach thereof, it may be entertained only before the breach or violation of the law or contract to which it
refers. Here, respondent corporation brought the present action for reformation after an alleged breach or violation
of the contract was already committed by petitioner Bentir. Consequently, the remedy of reformation no longer lies.

2B 16-17 SALES AND LEASE Page 476


5. DIZON vs. MAGSAYSAY

G.R. No. L-23399 May 31, 1974

FACTS:

On April 1, 1949, Ambrosio Magsaysay, registered owner of land located in Sampaloc, Manila, and the late
Bernardo M. Dizon executed a written contract of lease over a portion of the parcel of land which the latter had been
occupying as lessee since 1937 and on which he had constructed a residential house as well as a six-lane bowling
alley. The two-year term of the lease contract expired on April 1, 1951 without the parties' having expressly renewed
their agreement. Dizon, however, continued to occupy the leased premises, paying the same monthly rental of
P100.00, while Magsaysay accepted the same.

On March 3, 1953, the counsel of Magsaysay formally advised Bernardo Dizon of the termination of the
existing lease at the end of that month. On March 24, Dizon learned that as early as February 19, 1953 there were
negotiations for the sale of the entire lot to Nicanor Padilla, which negotiations were concluded on March 7, 1953
with the execution of an absolute deed of sale.

When Dizon learned of the sale he communicated with Magsaysay and Padilla, inviting their attention to
paragraph 9 of the original written lease contract which gave him the preferential right to purchase the land under
the same conditions as those offered by other buyers. On March 25, 1953 he actually commenced suit against
Magsaysay and Padilla in the Court of First Ins tance of Manila. Dizon prayed that the deed of sale between
Magsaysay and Padilla be declared null and void, that they be ordered to sell the land to him and to pay him
damages and attorney's fees.

The trial court rendered judgment, dismissing the complain t. On appeal to the Court of Appeals, the decision was
affirmed.

ISSUE:

Whether or not Dizon had a preferential right to purchase the disputed property.

HELD:

NO

The Court of Appeals held that "the other terms of the original contract" which are reviv ed in the implied
new lease under Article 1670 of the Civil Code are only those terms which are germane to the lessee's right of
continued enjoyment of the property leased. It is based on the presumption that when the lessor allows the lessee to
continue enjoying possession of the property for fifteen days after the expiration of the contract he is willing that
such enjoyment shall be for the entire period corresponding to the rent which is customarily paid, in the case at bar,
up to the end of the month because the rent was paid monthly. The presumption covers the other terms of the
contract related to such possession, such as the amount of rental, the date when it must be paid, the care of the
property, the responsibility for repairs, etc. but no such presumption may be indulged in with respect to special
agreements which by nature are foreign to the right of occupancy or enjoyment inherent in a contract of lease.

2B 16-17 SALES AND LEASE Page 477


Paragraph 2 of the contract of lease between Bernardo and Magsaysay, states that it was rene wable for the
same period of two years (upon its expiration on April 1, 1951). This stipulation embodied the agreement of the
parties with respect to renewal of the original contract, and there was nothing in it which was incompatible with the
existence of an implied new lease from month to month under the conditions laid down in Article 1670 of the Civil
Code. However, incompatibility existed with respect to any implied revival of the lessee's preferential right to
purchase, which expired with the termination of the original contract. The express agreement of the parties should
govern and not the legal provision relied upon by the petitioner.

2B 16-17 SALES AND LEASE Page 478


6. VICENTE J. SANTI vs. HON. COURT OF APPEALS, HEIRS OF AUGUSTO A. REYES, JR.,
represented by ALEXANDER REYES,

G.R. No. 93625 November 8, 1993

Nocon, J

FACTS:

Esperanza Jose was the registered owner and in absolute possession of a parcel of land in Cavite City. She
leased a portion of the property unto spouses Eugenio Vitan and Beatriz Francis co for a period of 20 years
"automatically extended" for another 20 years on which the lessees constructed a cinema house. Sometime in 1962,
the lessees sold all their rights, interest and participation over the cinema house together with the leasehold rig hts on
the lessor's property unto Augusto A. Reyes, Jr. and a new contract of lease was entered between the new owner and
Esperanza Jose for a period of 20 years, said period of lease being "extendable" for another period of 20 years. In the
interim, Esperanza Jose sold all her rights and participation over the parcel of land to Vicente J. Santi. When the
lease expired, plaintiff wrote Alexander Reyes as representative of Augusto Reyes, Jr., who had died, informing him
of the termination of the lease and demanding peaceful turn-over of possession. Defendant refused on the ground
that after consulting his lawyer, Atty. Gregorio R. Familiar the latter informed him that the lease was automatically
extended for another 20 years. In view of defendant's refusal to vacate plaintiff filed a routine complaint against
Alexander Reyes with the office of Barangay Captain of Barangay 34, "Lapu -lapu" of the City of Cavite; and no
settlement having been reached the Barangay Captain issued a certification to file action.

The trial court rendered judgment for the petitioner.

Petitioner contends that to extend the lease contract for another 20 years requires a subsequent agreement
between the parties as the phrase "being extendable" meant "capable of being extended."

On the other hand, private respondents argue that the terms of the lease contract are clear and that the same
should be automatically extended upon the expiration of the first 20 years.

The court a quo, however, gave merit to the contention of herein private respon dents and held that to enter
into new negotiations to extend the contract would, therefore, be superfluous and unnecessary, an idle ceremony, for
the lease contract already contains all that is necessary for the extension thereof. The suggestion to enter into new
negotiations run counter to the lease contract for, as already said, everything necessary for its renewal or extension
has been agreed upon. All that was left to abide by the lease contract.

ISSUES

(1) Whether the contract of lease contained automatic extension of lease?


(2) Whether an implied lease was created?

HELD:

The phrase "automatically extended" did not appear and was not used in the lease contract subsequently
entered into by Esperanza Jose and Augusto Reyes, Jr. for the simple reason that the lessor does not want to be
bound by the stipulation of automatic extension as provided in the previous lease contract.

2B 16-17 SALES AND LEASE Page 479


If the intention of the parties were to provide for an automatic extension of the lease contract, then they
could have easily provided for a straight forty years contract instead of twenty.

We find the trial court's decision more in accord with the true intention of the.

The law in point is Article 1670 and 1687 of the Civil Code.

The law provides that if after the end of the lease contract, the parties continue to enjoy the thing leased, an
implied lease is created for the period mentioned in Article 1687, hence herein private respondents may continue to
occupy the leased premises provided such is with the permission and consent of herein petitioner-lessor. Since the
lease contract provided for a monthly rental of P220 to be paid by the lessee upon the expiration of the first twenty
years, the latter shall be bound by such amount wh ich shall be paid by herein private respondents to petitioner-
lessor.

2B 16-17 SALES AND LEASE Page 480


7. JOSE ITURALDE, Plaintiff-appelle vs. ANTONIO GARDUO, Defendant-appellant

G.R. No. L-2997, January 16, 1908

Mapa, J

FACTS

The defendant alleges that the rental of 1 peso and 50 cent s per annum cannot be either increased or
decreased and that he cannot be ejected so long as he punctually pays the rent according to the condition stipulated
in the contract.

The court below held that in accordance therewith the duration of the lease was left to the will of the
defendant and in consequence thereof the complaint was dismissed, reserving to the plaintiff the right to commence
another action to obtain the designation of the period for the termination of the lease.

It appears that the defendant or his ancestors had built a house valued at 550 pesos, and planted fruit trees
on the land in questions. For this reason, the lower court adjudged that such fact impresses upon the contract the
character of an indefinite term and implies long duration.

ISSUE

Whether or not the lease is for an indefinite period due to the improvements introduced by the defendant or his
ancestors

HELD

No.

The duration of lease contracts depends on what may have been stipulated by the parties at the time when
the same were entered into, and not on the more or less importance of the improvements introduced or effected by
the tenant on the leased property. Neither does the circumstance that the fruit trees require 8 to 11 years to yield the
first crop support the claim of the defendant.

Thus, the decision of the lower court was reversed and set aside and the defendant is directed to return to
the plaintiff the land in controversy, and pay to him the unpaid rents in the sum of 1 peso and 50 cents as rent for the
year 1902 and the sum of 9 pesos for each of the succeeding years, beginning with 1903, until the time when the
judgment entered in this suit shall be executed.

2B 16-17 SALES AND LEASE Page 481


8. HERNANIA LANI LOPEZ VS GLORIA UMALE-COS ME

G.R. No. 171891. February 24, 2009

Puno, J

FACTS

Respondent Gloria Umale-Cosme is the owner of an apartment building at 15 Sibuyan Street, Sta. Mesa Heights,
Quezon City, while the petitioner is a lessee of one of the units therein, paying a monthly rent of P1,340.00 as of
1999. On April 19, 1999, respondent filed a complaint for unlawful detainer against petitioner on the grounds of
expiration of contract of lease and nonpayment of rentals from December 1998. In her answer, petitioner denied that
she defaulted in the payment of her monthly rentals, claiming that respondent did not collect the rentals as they fell
due in order to make it appear that she was in arrears. Petitioner also alleged that she had been depositing her
monthly rentals in a bank in trust for respondent since February 1999. The RTC held that the lessee may not be
ejected on the ground of termination of the period until the judicial authorities have fixed such period, since it ruled
that the contract of lease did not have a definite perio d. On appeal, respondent stated that Article 1673 (1) of the
Civil Code provides that the lessor may judicially eject the lessee when the period agreed upon, or that which is
fixed for the duration of leases under Articles 1682 and 1687, has expired. Article 1687 of the same Code provides
that if the period for the lease has not been fixed, it is understood to be from year to year, if the rent agreed upon is
annual; from month to month, if it is monthly; from week to week, if the rent is weekly; and from da y to day, if the
rent is to be paid daily.

ISSUE

Whether or not the lessee may be ejected

HELD

Yes. The nonpayment of the rent warrants the ejectment of the lessee. The lessee may not interpose as a defense that
there was no definite period on the lease, because it is well-settled that where a contract of lease is verbal and on a
monthly basis, the lease is one with a definite period which expires after the last day of any given thirty -day period.
The rentals being paid monthly, the period of such lease is deemed terminated at the end of each month. Thus,
respondents have every right to demand the ejectment of petitioners at the end of each month, the contract having
expired by operation of law. Without a lease contract, petitioner has no right of possess ion to the subject property
and must vacate the same. Respondents, thus, should be allowed to resort to an action for ejectment before the MTC
to recover possession of the subject property from petitioner.

2B 16-17 SALES AND LEASE Page 482


ON THE BULK SALES LAW

1. THE PEOPLE OF THE PHILIPPINES vs. FELIPE MAPOY and R. M. MAIPID

G.R. No. L-48336, September 21, 1942.

FACTS

Defendants were charged with violation of the Bulk Sales Law in that they mortgaged all of their stock of
goods, etc., without any notice to Daido Boeki Kaisha, Ltd., one of the offended parties, to which they were
indebted in the sum of P2,568.85. They pleaded guilty and its sentenced by the Court of First Instance of Manila to
pay a fine of P100, and the costs, and to indemnify Daido Boeki Kaisha, Ltd., jointly and s everally in the sum of
P2,568.85, with subsidiary imprisonment in case of insolvency

ISSUE

Whether or not the sum of P2,568.85, which was already existing when the mortgage was signed resulted from the
defendants violation of the Bulk Sales law.

HELD

No. The Supreme Court held that it was error for the trial court to consider said indebtedness as a liability
arising from the crime charged, and to order defendants to indemnify Daido Boeki Kaisha, Ltd., in the sum of
P2,568.85, with subsidiary imprisonment in case of insolvency.

Inasmuch as under section 4 of the Bulk Sales Law, the mortgaged in question was fraudulent and void,
and there being no proof that the mortgaged goods have disappeared, the same are still subject to attachment for the
satisfaction of creditors' lawful claims against the defendants. Daido Boeki Kaisha, Ltd., may still bring a separate
civil action against defendants herein for the collection of any indebtedness that may be due from defendants, and if
the latter will not pay the judgment in such civil case, the goods involved in the instant case may be seized and sold.
Therefore, the obligations of defendants to pay Daido Boeki Kaisha , Ltd., the sum of P2,568.85, which was already
existing when the mortgage was signed, was not the result of the violation of the Bulk Sales Law, nor was it affected
by said violation.

2B 16-17 SALES AND LEASE Page 483


2. STROCHECKER vs. RAMIREZ

G.R. No. 18520 September 26, 19S22

Romualdez, J

FACTS

In the lower court there were three mortgagees each of whom claimed preference. They were the mortgages
of Fidelity & Surety Co, Ildefonso Ramirez and Concepcion Ayala. The latter's claim was rejected by the trial court,
and from that ruling she did not appeal.

There is no question as to the priority in time of the mortgage in favor of the Fidelity & Surety Co. which
was executed on March 10, 1919, and registered in due time in the registry of property, that in favor of the appellant
being dated September 22, 1919, and registered also in the registry.

The appellant claims preference on these grounds: (a) That the first mortgage above-mentioned is not valid
because the property which is the subject-matter thereof is not capable of being mortgaged, and the description of
said property is not sufficient; and (b) that the amount due the appellant is a purchase price, citing article 1922 of the
Civil Code in support thereof, and that his mortgage is but a modification of the security given by the debtor on
February 15, 1919, that is, prior to the mortgage executed in favor of the Fid elity & Surety Co.

ISSUE

Whether or not the mortgage in favor of the Fidelity & Surety Co should be given preference.

HELD

Yes. With regard to the nature of the property mortgaged, which is one -half interest in the business, such
interest is a personal property capable of appropriation and not included in the enumeration of real properties in
article 335 of the Civil Code, and may be the subject of mortgage. All personal property may be mortgaged. (Sec. 2,
Act No. 1508.)

Turning to the second error assigned, numbers 1, 2, and 3 of article 1922 of the Civil Code invoked by the
appellant are not applicable. Neither he, as debtor, nor the debtor himself, is in possession of the property
mortgaged, which is, and since the registration of the mortgage has been, legally in possession of the Fidelity &
Surety Co. (Sec. 4, Act No. 1508; Meyers vs. Thein, 15 Phil., 303.)

In no way can the mortgage executed in favor of the appellant on September 22, 1919, be given effect as of
February 15, 1919, the date of the sale of the drug store in question. On the 15th of February of that year, there was
a stipulation about a persons security, but not a mortgage upon any property, and much less upon the property in
question.

Moreover, the appellant cannot deny the preferential character of the mortgage in favor of the Fidelity &
Surety Co. because in the very document executed in his favor it was stated that his mortgage was a second
mortgage, subordinate to the one made in favor of the Fidelity & Surety Co.

2B 16-17 SALES AND LEASE Page 484


3. HPS SOFTWARE AND COMMUNICATION CORPORATION and HYMAN YAP vs.
PHILIPPINE LONG DISTANCE TELEPHONE COMPANY (PLDT), ET AL.

G.R. No. 170217, December 10, 2012

Leonardo-De Castro, JS

FACTS

PAOCTF filed with the court two applications for issuance of search warrant for Theft of Telephone Services and
unauthorized installation of telephone communication equipments by HPS Software Communications Corporation
following the complaint of PLDT. HPS is engaged in the business of International Simple Resale or unauthorized
sale of international long distance calls. Subsequently the search warrants were issued thus HPS sought to quash the
search warrants on the grounds that it did not refer to a specific offense, that they were general warrants and were
wrongly implemented. The RTC granted the motion. Aggrieved, PLDT filed a petition for certiorari with the CA
which reversed the RTC ruling.

ISSUE

Whether or not international simple resale is considered a criminal act of theft.

HELD

Yes. The business of providing telecommunication or telep hone service is personal property which can be the object
of theft under Article 308 of the Revised Penal Code. Business may be appropriated under Section 2 of Act No.
3952 (Bulk Sales Law), hence, could be the object of theft:

"Section 2. Any sale, transfer, mortgage, or assignment of a stock of goods, wares, merchandise,
provisions, or materials otherwise than in the ordinary course of trade and the regular prosecution of the business of
the vendor, mortgagor, transferor, or assignor, or any sale, transfer, mortgage, or assignment of all, or substantially
all, of the business or trade theretofore conducted by the vendor, mortgagor, transferor or assignor, or a ll, or
substantially all, of the fixtures and equipment used in and about the business of the vendor, mortgagor, transferor,
or assignor, shall be deemed to be a sale and transfer in bulk, in contemplation of the Act. x x x."

2B 16-17 SALES AND LEASE Page 485

Вам также может понравиться